Você está na página 1de 772

RHEUMATOLOGY, ORTHOPEDICS AND SPORTS:

1.Sarcoidosis:
Usually affects young people. Prevalence is high in African Americans, Puerto Ricans,
Irish and Scandinavians. It mainly involves the lungs, peripheral and mediastinal lymph
nodes, liver and skin (e.g erythema nodosum; painful and erythematous subcutaneous
nodules, distributed bilaterally over the lower extremeties), although the nervous
system, salivary glands, heart and other organs may also be involved. The presence of
peripheral nerve disease (e.g facial palsy), skin lesions , cardiac or hepatic compromise
and other systemic manifestations is highly suspicious for this condition.
Some of the dermatologic manifestations of sarcoidosis are papules, scar infiltration
and subcutaneous nodules.
Upto 90% of the pts have radiographic evidence of hilar enlargement. This makes Chest
X ray a good screening test to evaluate the presence of this disease.The Chest X ray may
reveal bilateral lymphadenopathy(mediastinal enlargement) or pulmonary infiltration.
Definitive diagnosis of the sarcoidosis can be made from the biopsy of easily accessible
lesions, including:
=>Any palpable lymph node
=>Subcutaneous nodules except erythema nodosum
=>Enlarged parotid
=>Lacrimal gland
If there is no easily accessible lesion, fiberoptic bronchoscopy with transbronchial biopsy
is the procedure of choice.
Nerve and liver biopsies are not recommended , unless all other approaches have failed.

2.Hemochromatosis:
=>Hepatomegaly
=>Diabetes mellitus type-2
=>Hypogonadotrophic hypogonadism leading to decreased Libido
=>Arthropathy can be the first manifestation of hemochromatosis.This usually involves
the second and third metacarpophalangeal joints, although the presentation may be a
polyarthritis involving the knees, ankles and shoulders.A common accompanying
symptom is morning stiffness which lasts less than thirty minutes.X ray of the hand
shows subchondral cysts, sclerosis, osteopenia and hook like osteophytes.Aspiration is
usually performed and microscopic examination of the synovial fluid may reveal the
presence CPPD crystals (pseudogout) in 50% of cases.These crystals may be visualized
under polarized light and have a rhomboid shape and positive birefringence.
In addition to the usual clinical presentation and radiologic findings, the arthropathy
of hemochromatosis is diagnosed by obtaining evidence of iron overload in the pt.
Screening may be performed by obtaining the transferrin saturation, serum iron, and
ferritin levels. Confirmation is obtained through liver biopsy. Arthroscopy or synovial
biopsy is not needed to diagnose the disease.
Arthropathy of Hemochromatosis does not respond to hemochromatosis therapy.

3.Gout crystals are negatively birefringent on polarized microscopy.

4.RA involves the proximal interphalangeal joints, metacarpophalangeal joints, as well


as the wrist.The morning stiffness lasts more than an hour.Subchondral erosions are
seen on X rays.
Rheumatoid factor can be used as a screening test for RA, while ESR is useful to follow
the course of pts with rheumatic conditions.

5.Polymyalgia Rheumatica And Giant Cell Arteritis:


Polymyalgia rheumatica (PMR) affects pts aged >50 years and is characterized by pain in
the large muscles of the shoulders & pelvic girdle. The physical examination z frequently
unremarkable, with little or no significant muscle or joint tenderness. Pts may have mild
stiffness with active range of motion (ROM) but passive ROM is usually normal. Signs of
inflammation in the joints are minimal to absent. Headache, jaw pain, visual loss or
temporal tenderness suggests presence of giant cell arteritis (GCA), which often
accompanies the PMR.
The diagnosis of PMR & GCA z primarily clinical & ESR shd be obtained for confirmation
and is highly sensitive. In pts age >50 with a new, localized headache or tenderness of
the temporal artery, ESR >50 mm/h is 91% specific for GCA.
Pts with possible Giant cell arteritis shd receive high dose glucocorticoids and be
considered for temporal artery biopsy.

6.Polymyositis may occasionally mimic PMR but it generally characterized by muscle


weakness with little or no associated pain.

7.Developmental dysplasia of Hip (DDH) :


A condition characterized by an abnormal formation of the hip joint that prevents the
femoral head from seating properly in the acetabulum. Risk factors include female
gender, breech delivery and positive family history. Physical examination reveals
asymmetry of inguinal folds and shorter affected leg. An evaluation of the affected leg
shd always include Barlow test, which attempts to dislocate an unstable hip & produces
a click. The diagnosis is made with an U/S in the first 6 months of life. DDH is treated
mainly by maintaining the hip in a flexed , abducted position for 1-2 months. This is

most commonly accomplished by Pavlik Harness. Treatment shd be managed by an


orthopedic surgeon.

8.Anterior cruciate ligament tear:


Pts with partial or full thickness tears complain of a popping sensation in the knee at the
time of injury, followed by the development of rapid onset hemarthrosis & a feeling of
joint instability with weight bearing.
Pts with ACL tears will show laxity at the knee with the tibia able to be pulled forward
relative to the femur. Two such maneuvers-the Lachman test and the anterior drawer
test-are highly sensitive and specific for ACL injuries. The diagnosis is usually confirmed
on MRI. Treatment consists of Rest, Ice, Compression and Elevation (RICE), though
surgery may or may not be required depending upon the individual cases.

9.Medial collateral ligament injury:


Commonly occurs following injuries in which the knee is struck from the lateral side with
the foot planted. MCL injuries will show tenderness at the medial knee along the joint
line and there may be laxity when the lower leg is gently forced into abduction with the
knee stationary (valgus stress test).

10.Pts with a meniscal tear may report a subacute or chronic locking or catching
sensation in the knee but often do not appreciate the extent of injury when it occurs.
Effusions, when they occur, typically develop slowly.

11.Patellofemoral pain syndrome is associated with chronic overuse rather than acute
trauma. Pts report pain over the anterior knee, which can be reproduced by extendng
the knee while compressing the patella (Patellofemoral compression test).

12.Rupture of the popliteal cyst/ Bakers cyst may cause posterior knee pain. This is a
common condition in older pts (associated with osteoarthritis of the knee) but is
uncommon in young adults.Pt may have pain and swelling at the posterior knee and
calf resembling deep venous thrombosis.

13.Sarcoidosis: The biopsy of tissues for confirmation of diagnosis is not required in an


asymptomatic pt with the typical hx and chest x ray findings. If indicated,
biopsy shd be performed in the most easily accessible lesions or organs

14.Although pts with pulmonary sarcoidosis typically reveal a restrictive pattern on PFTs
with reduction in diffusion capacity for Carbon monoxide, they may also have normal
PFTs.

15.Carpal tunnel syndrome:


Pain and numbness in the wrist and palmar surface of the first three fingers associated
with repetitive movements of the wrist is typical for carpal tunnel syndrome.Occurs
due to involvement of the Median nerve in the carpal tunnel

16.Back pain:
Concept of recovery expectation is an important predictor of work outcome for
pts with occupational back pain. Education to improve pts understanding of natural
history and prognosis may improve the likelihood of returning to work. Pts with
positive expectations regarding their recovery have generally better functional
outcomes than pts with negative expectations. Pts recovery expectation can be
influenced by physician-patient relationship & measured using specific standardized
evaluation tools. The implications are that, depending on diagnosis, physicians can

provide pts with information and education about prognosis in back pain which can
positively influence a pts recovery expectation, increasing the odds that the pt will
return to work.
Stress, job satisfaction & depression may adversely affect the pts perception of pain.
For a given somatic pain level, pts with concurrent affective symptoms experience a
greater level of pain-related distress than those without. This is often referred to as
the affective component of pain. These psychological issues have significant impact
on the quality of life but have not been shown to be predictive of work outcome.
There is little evidence that analgesic use predicts the functional outcomes or return
to work.

17.Nerve Injuries:
>Femoral Nerve Injury;
=>weakness of quadriceps leading to inability of extension of knee against resistance
=>Sensory loss over the anterior and most of the medial aspect of the thigh
=>Sensory loss over the medial shin
=>Sensory loss over the arch of the foot due to dysfunction of the saphenous nerve
(branch of the femoral nerve).
=>Decreased/Absent Knee Jerk
=>Hip adduction is spared (function of obturator nerve)
>Sciatic nerve Injury (due to traumas such as hip dislocation, fracture or replacement.
wayward buttock injections, compression by external sources e.g prolonged bed rest
or any deep seated mass in the pelvis e.g hematoma);
=>Weakness of Hamstrings
=>Ankle jerk is unobtainable
=>Hip flexion, extension, abduction, adduction and knee extension are all normal

=>Sensations over the medial calf and arch of the foot are preserved.
>Obturator nerve injury;
=>Weakness in leg adduction
=>Sensory loss over a small area in the medial thigh
>Common peroneal nerve injury ( injury is usually located at the knee on the lateral
aspect of the fibular head );
=>Acute foot drop accompanied by weakness in foot dorsiflexion and eversion
=>Paresthesias and/or sensory loss over the dorsum of foot and lateral shin
(superficial peroneal nerve territory)

18.Shoulder pain:
A careful history and physical examination can determine the cause of shoulder pain in
the majority of pts. Most of the pts presenting with shoulder pain have an intrinsic
shoulder disorder. Lateral shoulder or deltoid pain aggravated by reaching or lifting the
arms up is the classic pattern of pain seen in pts with rotator cuff tendonitis or tear,
impingement syndrome, and frozen shoulder. Rotator cuff tendonitis and tear are the
two most common causes of chronic shoulder pain in the middle aged and older pts. A
rotator cuff tear is likely if the pain is accompanied by weakness or loss of strength
during external rotation or abduction of the shoulder. Frozen shoulder refers to a
stiffened glenohumeral joint and is most commonly caused by rotator cuff tendonitis. It
is suspected when the lateral pain pattern is accompanied by stiffness and a decreased
range of motion in abduction and external rotation.

19.Anterior shoulder pain is seen is pts with acromioclavicular or glenohumeral joint


osteoarthritis and biceps tendonitis.

20.Posterior shoulder pain is the least common pain pattern and is usually a referred
pain from the cervical spine. The common causes are nerve impingement due to disc
herniation and spinal stenosis at the cervical spine level.

21.Poorly localized pain is usually due to referred pain from the neck, bony structures,
or nerve compression.

22.Rotator cuff tendonitis usually manifests as lateral shoulder pain aggravated by


movements requiring abduction and external rotation of the shoulder.

23.Gout:
>NSAIDs (i.e Indomethacin) or Colchicine are the first line agents for an acute attack of
gout.
>The angiotensin receptor blocker Losartan is a good choice for hypertensive pts who
also have hyperuricemia and gout since it has a modest uricosuric effect.
>Allopurinol is a xanthine oxidase inhibitor which decreases the uric acid production and
can be considered as prophylaxis.
>Following medications shd be avoided in gout;
=>Furosemide, as it can trigger hyperuricemia
=>Thiazides, as they raise the serum uric acid levels.
=>Low dose aspirin decreases the renal urate production & thus is not recommended
for pts with gout. NSAIDs are preferred to aspirin in the treatment of gout and are
typically used during acute flares or when uric acid lowering therapy is initiated in
order to prevent an acute flare.

24.Avascular necrosis:
>Avascular necrosis/Osteonecrosis/Osteochondritis dessicans usually occurs in the
anterolateral femoral head, although it can also affect the humeral head, femoral
condyles, vertebrae, proximal tibia and small bones of the hands and feet.
>Most common causes in 90% of cases are Corticosteroids AND Excessive Alcohol
>Other important causes r SLE, Sickle cell disease, Antiphospholipid antibody syndrome,
chronic renal insufficiency and hemodialysis, following renal transplant, HIV infection,
Caissons disease, Gauchers disease.
>Those who are at greatest risk are SLE pts on chronic high dose Corticosteroid therapy
(more than 15-20 mg/day)
>MRI scanning is the most sensitive imaging modality for diagnosing osteonecrosis of
the femoral head.
>The main modalities of treatment include conservative therapy, core decompression,
osteotomy and joint replacement;
=>Core decompression offers a chance to preserve the native joint in the early
stage I or II disease (positive radiographs w/o femoral head collapse)
=>Total hip replacement is the therapy of choice for stage 4 disease of the femoral
head (flattening of the femoral head with joint space narrowing)
=>Conservative therapy is usually ineffective in halting the progression of disease.

25.Carpal tunnel syndrome:


Carpal tunnel syndrome is the most common nerve entrapment disorder characterized
by numbness, tingling and pain in the thumb and first three fingers. Eventually, atrophy
of the hand musculature can develop. The diagnosis is suspected when 1) the patient
complains of paresthesias, hypesthesia, pain or numbness in an area of the hand served
by the median nerve and 2) there are physical findings of median nerve compression

such as pain or paresthesias after compression of the median nerve at the wrist, tapping
of median nerve (Hoffman-Tinnel test) or 30-60 seconds of acute wrist flexion (Phalen
maneuver). Atlhough the sensitivity and specificity of these tests are rather low , the hx
and physical findings will make the diagnosis of carpal tunnel syndrome more likely.
Initial modalities of treatment are NSAIDs, Splinting and Corticosteroids injections.
When standard methods of treatment fail, it z important to focus on occupational rehab
as a means of limiting the symptom severity. Studies have shown that pressure within
the carpal tunnel rises significantly with a number of acitvities, including wrist flexion &
extension, making a fist, holding items, pushing and finger work such as key pushing and
typing. Its therefore importnt to minimize these tasks as much as possible in this patient
population. In addition, the periodic performance of brief wrist & hand exercises known
to reduce intratunnel pressure shd also be recommended.

26.Sarcoidosis:
African Americans are at elevated risk.Its a disorder of unknown etiology characterized
by formation of noncaseating granulomas in lymph nodes, lungs and other organs.Skin
involvement is very common in sarcoidosis, with possible manifestations including a
maculopapular rash, scattered nodular lesions, erythema nodosum, or a variety of
atypical lesions.
Activated marcophages in sarcoid granulomas produce 1-alpha-hydroxylase, which
converts 25-hydroxy vit-D to 1,25-dihydroxy Vit-D , leading to an increase in the GIT
absorption of calcium. In hypercalcemia due to sarcoidosis , PTH is suppressed (<20)
and increased urinary calcium excretion.
Hypercalcemia secondary to sarcoidosis responds quickly to glucocorticoids.
Glucocorticoids decrease calcium by inhibiting the production of calcitriol, thereby
decreasing GI absorption of calcium. Osteoclastic inhibition may play a minor role.

27.Hypercalcemia caused by overproduction of 1,25-dihydroxy Vit-D can also occur in


other granulomatous diseases, such as fungal infections, berylliosis, tuberculosis, and
lymphomas.

28.Calcitonin decreases the serum calcium by decreasing the bone resorption.

29.Cinacalcet is a calium sensing receptor agonist, which decreases serum calcium by


decreasing PTH levels in pts with hyperparathyroidism (primary, secondary & tertiary)
Cinacalcet is approved in pts with secondary hyperparathyroidism due to renal failure
and primary hyperparathyroidism due to parathyroid cancer.

30.Loop diuretics decrease serum calcium by increasing the urinary excretion of calcium

31.Bisphosphonates decrease the serum calcium by inhibiting osteoclastic bone


resorption. IV bisphosphonates very effective in hypercalcemia of primary hyperparathyroidism and PTH related protein overproduction by malignant tumors.

32.Amputated body parts:


In general, all amputated body parts shd be retrieved and brought to the emergency
dept. Handling of the amputated part shd be as limited as possible, with care taken to
prevent it from drying or being soaked in liquid.
For transportation purposes, the amputated part shd be wrapped in sterile gauze,
moistened with saline and placed in a sealed, sterile plastic bag.The bag shd be placed
in a chilled container to best preserve it and reduce warm ischemia. The goal temp. is
1-10 C and to maintain that temperature, ice shd be mixed with saline or sterile water

(50/50 mixture) and the digit containing closed plastic bag shd be placed in the mixture.
Amputated parts generally tolerate cold ischemia time (i.e temp of 1-10 C , goal of 4 C)
better than warm ischemia time (i.e temp range b/w body & ambient temperature).Due
to their small mass, digits can tolerate longer ischemia times than other body parts.
However, the body part shd not be cooled below 1-10 C to avoid inducing frostbite.

33.SLE:
Look for a young reproductive age female with constitutional symptoms such as fatigue,
arthralgias / myalgias, weight loss and facial rash ( butterfly rash ), commonly seen after
sun exposure. Fever is a common manifestation. Pleuritis can occur which can manifest
as decreased breath sounds and chest pain. Pts may have a positive family history of
SLE or other connective tissue disorders. Look for pancytopenia.
Testing for ANA is positive but is sensitive, not specific. Anti-dsDNA is specific for SLE.
Anti-Smith antibodies is also specific but we prefer Anti-dsDNA because Anti-dsDNA is
more sensitive (66-95%) than Anti-Smith antibodies (25%).
Anti-dsDNA antibodies can be used to follow disease activity in SLE pts and is associated
wth the development of lupus nephritis-immune complexes containing these antibodies
r actually seen within the glomeruli of pts wth lupus nephritis.Rising titers of anti-dsDNA
antibodies cn be helpful in confirming that the underlying lupus is the cause of a clinical
problem as opposed to infection, medication toxicity or unrelated disease.
Anti-Smith antibodies do not correlate with disease activity in SLE pts and may remain
positive in pts with clinically improving SLE when anti-dsDNA antibodies have returned
to the normal range.
Hydroxychloroquine is an anti-malarial agent that is particularly effective at improving
arthralgias, serositis and cutaneous symptoms. Hydroxychloroquine may also prevent
the future kidney and CNS damage. Low dose prednisone at 5-15 mg/day is helpful in

short term in improvng the pts symptoms until the hydroxychloroquine takes full effect.
Higher doses of steroids or other immunosuppressants are typically reserved for pts wth
more severe , solid organ damage.
A combination of prednisone and cyclophosphamide is reserved for pts with more
serious manifestations (e.g lupus nephritis , CNS involvement and vasculitis).
P.S: Dont use Rituximab (anti-B cell antibody) becx it hz been reportd to be asscociated
with progressive multifocal leukoencephalopathy (PML).

34.Anti-Centromere antibodies-> CREST syndrome and scleroderma

35.Anti-mitochondiral antibodies->Primary biliary cirrhosis. Levels are usually stable in a


given patient over time.

36.Anti Ro/SSA antibodies->Sjogrens syndrome, photosensitivity rash of SLE and


other cutaneous manifestations of SLE as well as interstitial lung disease and
congenital heart block.

37.Gout:
Gout occurs due to intraarticular deposition of monosodium urate crystals (MSU). Risk
factors for gout include volume depletion, certain medications (e.g diuretics), high
protein or high fat diet, increased alcohol consumption, and recent surgery or trauma.
Pts usually develop acute onset of severe joint pain with erythema, swelling and limited
range of motion. Although most cases (>80%) occur in the 1st toe or knee, gout can also
occur in othr joints (e.g ankle, wrist, finger, shoulder, olecranon bursa). Pts with chronic
hyperuricemia may also develop tophi (nodular deposits of MSU crystals in soft tissues
with associated inflammatory changes).

Even if it is highly suspected clinically, gout shd be confirmed with arthrocentesis. The
synovial fluid usually contains MSU crystals that are needle shaped and negatively
birefringent under polarizing light. In contrast, pseudogout has positively birefringent
rhomboid shaped crystals. Joint fluid analysis & culture can also rule out septic arthritis,
which can coexist with acute gout.
In pts unable to have arthrocentesis, gout can be conditionally diagnosd with the classic
clinical findings (e.g acute onset, 1st toe involvement, significantly elevated uric acid
levels, tophi). However, clinical features are not as sensitive or specific as synovial fluid
analysis. Many pts can have a normal or even low uric acid during an acute gouty attack.
NSAIDs are the first line therapy & are effective in >90% of pts with acute gouty attack.
Non selective agents with a short half life (e.g indomethacin, ibuprofen) are usually
preferred as they are inexpensive and effective in most pts. COX-2 inhibitors are also
used as alternative therapy in some pts. Pts shd continue the drug until 1-2 days after
symptom resolution (a total of 5-7 days in most cases).
Oral colchicine , an alternative therapy in pts with contraindications to NSAIDs, is used
in acute or chronic kidney disease, heart failure, or peptic ulcer disease, or in those
currently taking anticoagulation. Colchicine is most effective if given within 24 Hours of
symptom onset. It shd be given as reduced dose in elderly. And is avoided in severe liver
or renal disease and in pts taking another P-450 inhibitor.
Intraarticular steroids can be used in acute gout for pts with one or two inflamed joints
and contraindications to NSAIDs or colchicine. In addition, steroids are usually given
after definitively diagnosing gout by arthrocentesis. Or or intramuscular steroids are
preferred for pts who cannot take NSAIDs, colchicine or intraarticular steroids.
Allopurinol is avoided during the attack so as to not to aggravate the flare, rather its
used after the symptom resolution to prevent recurrent episodes.

BOTTOM LINE: Serum uric acid confirms acute gout by showing monosodium urate
crystals that r negatively birefringent & needle-shaped under polarized
light. Serum uric acid does not accurately diagnose acute gout as many
pts can have normal or even low uric acid during an acute gouty attack.
NSAIDs are the first line therapy and are effective in >90% of pts with
acute gout. Oral colchicine is an alternate therapy in pts with contraindications to NSAIDs. Intraarticular steroids can be used in acute gout for
pts with one or two inflamed joints and contraindications to NSAIDs or
Colchicine.

38.For the noncontact athlete with a clavicle fracture, a gradual return to usual activities
may be allowed in approximately 4-6 weeks after the injury.
For contact athletes, more time is required becx the risk of refracture is greater.

39.Heberdens nodules are arthritic swellings of distal interphalangeal joints. These are
typically associated with osteoarthritis (OA) & are more common in females. Genetic
factors play an important role in the development of Heberdens nodules.
Bouchards nodules are the arthritic swellings of proximal interphalangeal joints.
These can be present in both OA and rheumatoid arthritis.

40.Xanthomatous nodules are associated with hereditary hyperlipidemic syndromes.


Tendinous forms of these nodules frequently involve the dorsum of the hands and
the Achilles tendon.
Xanthelasma is the deposition of cholesterol in the periorbital skin and is associated
with hereditary hyperlipidemic syndrome.

41.Fibromyalgia:
Characterized by widespread musculoskeletal pain, fatigue,disordered sleep and
cognitive/mood disturbances. Pts tend to have normal physical examination except for
point tenderness in multiple areas.Inflammation, joint swelling and muscle weakness
are absent.
Its mainly a clinical diagnosis and has no specific lab findings, though limited lab testing
shd be done to exclude common conditions that can mimic FM such as anemia (CBC),
inflammatory markers (ESR, CRP) and hypothyroidism (TSH).

42.Shoulder pain:
>Adhesive capsulitis/Frozen shoulder:
Adhesive capsulitis (AC) also known as frozen shoulder syndrome can be idiopathic or
secondary to underlying conditions such as rotator cuff tendinopathy (most common),
subacromial bursitis, paralytic stroke, diabetes mellitus or humeral head fracture. The
glenohumeral joint loses its normal distensibility due to chronic inflammation, fibrosis
and contracture of the joint capsule.
AC typically begins with shoulder pain that is often worse at night. Pts then develop
gradual shoulder stiffness (with or without pain) that limits their ability to flex, abduct
or rotate the humerus.The diagnosis is confirmed clinically with >50% reduction in both
passive & active ROM in > 2 planes (especially external rotation & abduction) compared
to the unaffected shoulder. Imaging (X-ray, U/S, MRI) is not required for diagnosis but
may occasionally be needed to rule out other etiologies of shoulder pain.
Most cases can be managed with resting the shoulder combined with gentle range of
motion (ROM) exercises, such as pendulum type swinging of hand held weights and
walking the fingers up a wall. Pts with significant limitation in function or those who do
not improve after 2-3 months of ROM exercises can receive a glucocorticoid injection

into the joint; this may also be combined with saline distension of the joint space.
BOTTOM LINE: Adhesive capsulitis is due to chronic inflammation, fibrosis & contracture
of joint capsue.It can be idiopathic or scndry to rotator cuff tendinopathy
,subacromial bursitis,paralytic stroke, diabetes, or humeral head fracture.
Pts have shoulder stiffness with a reduction in both passive and active
range of motion.
>Subacromial bursitis typically occurs with heavy lifting or repetitive movements,
esp with the arm raised above shoulder level. Pts can develop pain on abduction
of the shoulder and restricted active ROM. However, passive ROM shd be intact.
>Rotator cuff tendinopathy with a partial or complete tear may cause pain with mildly
(partial) or severely (complete) decreased active ROM. However, unless there is
concurrent adhesive capsulitis , passive ROM is not affected.
>Rotator cuff impingement or tendinopathy presents with pain with abduction and
external rotation and subacromial tenderness. There is normal range of motion with
positive impingement tests (e.g Neer, Hawkins).
>Rotator cuff tear is similar to rotator cuff impingement/tendinopathy, however there
is also weakness with external rotation. It occurs mostly in people age >40.
>Biceps tendinopathy presents with anterior shoulder pain with lifting, carrying or
overhead reaching
>Glenohumeral osteoarthritis is uncommon and is usually caused by trauma. There is
gradual onset of anterior or deep shoulder pain and decreased active and passive
abduction and external rotation.

43.Psoriatic arthritis usually affects the distal interphalangeal joints of the fingers and
toes, although larger joints may be affected. Psoriatic arthritis is usually associated
with destructive changes in X-rays.

44.Slipped capital femoral epiphysis: (SCFE)


This condition is more common in African American children followed by Hispanics. Boys
are affected more often than girls and typically present 10 and 16 years of age. Twenty
percent of those affected have bilateral involvement at presentation; an additional 20 to
40% will have bilateral involvemnt within 18 months of the initial SCFE. There z a strong
associated with obesity. In children younger than 10 years, it z associated wth metabolic
endocrine disorders (hypothyroidism, panhypopituitarism, hypogonadism, renal osteodystrophy, growth hormone abnormalities). SCFE is considered chronic if it has been
present more than 3 weeks and acute if it hz been present for 3 weeks or less. It z called
stable if the patient can bear weight & unstable if the patient cannot ambulate.Unstable
SCFE is associated with more complications , including avascular necrosis of the femoral
head (AVN). SCFE is diagnosed with X ray of the pelvis and bilateral hips. The underlying
cause is a widened epiphyseal growth plate, due to abnormal cartilage maturation and
endochondral ossification.The treatment z surgical,requiring immediate internal fixation
with a single screw. Delay in treatment of more than 24 Hrs leads to increased AVN,SCFE
progression from stable to unstable and high risk of future degenerative arthritis.
Prophylactic contralateral fixation of the unaffected hip is not routinely done in the US,
except in pts with endocrine abnormalities.
The pts presents with externally rotated leg in SCFE.

46.Scaphoid fracture:
Scaphoid fractures are the most common carpal bone fracture that commonly occurs
while falling onto an outstretched hand. The fracture is typically caused by forceful
dorsiflexion at the wrist to >95%. The scaphoid can be forced against the dorsal part of
the distal radius while falling, creating a fulcrum for injury. Pts usually have possible
swelling, decreased grip strength,pain on the radial aspect of the wrist in the anatomical

snuffbox & minimally decreased range of motion (unless dislocated fracture z present).
Next step in management would be to confirm the diagnosis with plain X rays of wrist
in full pronation and ulnar deviation to better expose the scaphoid. Initial X rays can be
negative if the fracture is compressed or minimally displaced.
If scaphoid fracture is suspected but plain radiographs are negative, then additional
imaging might be required because X rays can take 2 weeks to show the fracture.
Do either of the following ;
=>MRI (or CT) of the wrist
=>Repeat radiographs in 7-10 days
=>Bone scan in 3-5 days
MRI is the most sensitive test and allows for immediate assessment of a fracture and
associated soft tissue injury.
Pts shd be referred to an orthopedic surgeon if they are found to have associated tilt of
the lunate , a scaphoid fracture displaced >1 mm , nonunion during the follow-up,
osteonecrosis, or scapholunate dissociation.
Pts who do not meet surgical indications & have a nondisplaced fracture shd be placed
in a short-arm thumb spica cast (with the wrist deviated radially and neutrally fixed);
follow up X rays shd be done in 2 weeks intervals to monitor healing.
Scaphoid fractures that are not treated appropriately or in a timely fashion can result in
avascular necrosis and increase the risk nonunion. The arterial blood supply to the
scaphoid enters in the distal pole and travels to the proximal pole. Scaphoid fractures
(particularly at the waist and proximal end of the scaphoid) can compromise this blood
supply , causing avascular necrosis & increase the risk of nonunion in upto 10% of cases.
As a result, proximal fractures of the scaphoid require longer immobilization (upto 12
weeks) for adequate healing. Other factors that cause nonunion include delay of care,
an unstable fracture fragment and delay in surgical treatment when indicated.

47.Colles fracture is caused by falling onto an outstretched hand. Its a dorsally angulated
or displaced distal radius fracture that is typically associated with visible angulation
proximal to the wrist joint (dinner fork deformity). Lateral radiographs can confirm
Colles fracture diagnosis.
Acute Carpal tunnel syndrome is a possible early complication in pts with a displaced
or comminuted distal radial (Colles) fracture that has been treated with multiple
reductions or splinted in extreme wrist flexion.
Complex regional pain syndrome (CRPS) can occur as a complication of a distal radial
fracture and in pts with with wrist fracture splinted with palmar flexion >15%. Early
mobilization and Vit-C use have been recommended to reduce the incidence of CRPS.

48.A hamate fracture can occur while falling onto an outstretched hand, usually near
the hook of the hamate (Hamulus) containing the ulnar artery and nerve. Pain and
swelling of a hamate occurs in the hypothenar eminence and ulnar aspect of wrist.

49.A radial styloid fracture (Hutchinson or chauffeur fracture) typically occurs due to a
direct blow to the styloid while falling onto an outstretched hand with ulnar deviation
and supination. This results in a fracture dislocation of the radial styloid and either the
lunate or scaphoid.

50.Dupuytren contracture is characterized by progressive fibrosis of the palmar fascia of


unknown etiology. Its associated with the diabetes mellitus , exposure to repetitive
vibration to the hands, CRPS, alcohol consumption and occasionally, malignancy.

51.Malunion is commonly seen in physeal fractures of the distal forearm in children (less

common in adults) due to injury of the growth plate and growth arrest. This causes
growth disparity between the radius and ulna and results in possible wrist deformity.

52.Anklyosing spondylitis:
Its a seronegative spondyloarthropathy (Negative ANA and RA)
>How do you diagnose AS?
The three important clinical criteria for the diagnosis of AS are ;
a.Presence of low back pain and stiffness for more than a three month duration that
improves with exercise or activity.
b.Limitation of range of motion of the lumbar spine
c.Limitation of chest expansion relative to the normal values

>What to do once you are clinically sure about AS?


Plain X rays of the sacroiliac joint for evidence of sacroiliitis is the next best step in the
management. Other abnormalities might be seen are erosions of ischial tuberosity and
iliac crest.Early in the course of disease, squaring of the vertebral bodies on plain X ray
may also suggest the diagnosis.
With High suspicion, and negative X rays, we might consider MRI or CT scan.
HLA-B27 is a sensitive test, not specific. Its presence wont make a diagnosis, but its
absence would rule out AS.

>How do you monitor disease activity?


Plain radiographs are used to monitor the disease progression;
a.Anteroposterior and lateral view of the lumbar spine
b.Lateral view of the cervical spine
c.Pelvic radiograph including the sacroiliac joints and hip

Acute phase reactants such as ESR can also be used to monitor the disease activity.

>What are the extraarticular manifestations?


=>Eyes: Anterior uveitis, cataracts, cystoid macular edema. Uveitis presents with
acute onset onset unilateral eye pain, photophobia, and blurring of vision.
It is sometimes the initial manifestation of AS.
=>Lungs: Restrictive lung disease due to costovertebral joint motion limitation and
development of apical pulmonary fibrosis. Counsel them on smoking
cessation to prevent early deterioration of lungs function.
=>Valvular: Aortic regurgitation and mitral valve prolapse.
=>Testes: Increased incidence of varicocele.
=>GIT: Non specific ileal and colonic mucosal ulcerations.
=>Other: Atlantoaxial subluxation causing spinal cord compression, cauda equine
syndrome, IgA nephropathy, and secondary amyloidosis presenting as
nephrotic syndrome.

>Should AS pts do exercise?


AS pts and all pts with rheumatologic disorders shd be encouraged to perform aerobic
exercise to improve the functional status.
Regular aerobic exercises such as swimming, walking and bicycling improve the joint
stability, muscle strength and overall functional status w/o an increase in the disease
activity.These also help to prevent bone loss due to immobility & use of medications.
An initial evaluation by physicl therapist shd be done to determine the exercise regime
tailored to the individual tolerance, extent and severity of disease and other coexisting
medical conditions.

>What is the prognosis of AS?


Most pts with AS do well and have no functional or employment disabilities. There is
no increased overall mortality or reduced life expectancy.
Prolonged standing at work or exposure to cold conditions are risk factors for diability.

53.Hip fractures in elderly:


>Consists of intracapsular fractures (Femoral head and neck) and extracapsular fractures
(Intertrochanteric and subtrochanteric)
>Femoral neck fractures typically present with pain w/o significant ecchymoses & have
higher risk of avascular necrosis.
>Extracapsular fractures usually have significant ecchymoses and are at higher risk for
displacement.
>Hip fractures in elderly shd be managed with surgery within 48 Hours In pts who are
stable and ambulatory and is associated with lower mortality and a lower risk of
pressure ulcers and pneumonia. Most require either arthroplasty or open reduction
and internal fixation.
>Non operative management is reserved fr pts who are non-ambulatory, have advanced
dementia, have end-stage terminal illness, have an old non displaced or impacted
fracture or are unstable with major comorbidities.
>Surgery may be delayed briefly (up to 72 Hours) to stabilize acute life threatening
conditions (Acute coronary syn, decompensated CHF, Significant arrhythmias, severe
valvular disease).
>Preoperative cardiac stress testing prior to non-cardiovascular surgery is generally
advised only if it would be indicated in the absence of surgery.

54.Complex regional pain syndrome: (CRPS)


This syndrome was previously known as reflex sympathetic dystrophy, algodystrophy,
causalgia, Sudeck atrophy, transient osteoporosis, and acute atrophy of bone. Usually
occurring after an injury, CRPS presents with pain out of proportion to the injury,
temperature change, edema, and abnormal skin color. Type-I CRPS (90% of CRPS cases)
occurs w/o a definable nerve lesion, while type-II occurs with a definable nerve lesion.
The pathogenesis is likely due to an injury causing increased sensitivity to sympathetic
nerves, an abnormal response to and sensation of pain and increased neuropeptide
release causing burning pain to light touch (allodynia).
Typically CRPS occurs in 3 stages. Stage 1 includes burning pain, edema and vasomotor
changes in a limb after injury. Stage 2 includes progression of edema, skin thickening
and muscle wasting. Stage 3 is the most severe and includes limited range of motion
and bone demineralization on X ray. Diagnosis can be confirmed by either autonomic
testing that measures increased resting sweat output or MRI that looks for the above
changes. The treatment of CRPS is regional sympathetic nerve block or intravenous
regional anesthesia.
BOTTOM LINE: Complex regional pain syndrome shd be suspected in pts with recent
injury who present with burning pain, edema, skin changes, & decreased
range of motion.

55.Colles fracture:
It is a distal radius fracture that would show up on the X ray as shortening and dorsal
displacement of the distal radius. Its common in older patients following a fall onto an
outstretched hand (FOOSH). Osteoporosis is a major risk factor for Colles Fracture and
pts with this fracture following a ground level fall shd have a bone density measurement
to assess for osteoporosis.

Pts with FOOSH related trauma shd be screened for additional injuries. In particular, at
least half of pts with a Colles fracture will have concurrent ulnar styloid fracture. Other
associated injuries might include Scaphoid fracture, wrist sprain, supracondylar humerus
fracture and a variety of potential injuries to the elbow and shoulder. Pts with Colles
fracture are also at risk for acute carpal tunnel syndrome.
Acute management of Colles fracture is dependent on the displacement & angulation
of the fracture as well as any additional injuries. Most can be managed wth conservative
measures such as Sugar tong splinting, with or w/o closed reduction. More severe
injuries , especially those with significant displacement or angulation >15-20 degrees
may require urgent orthopedic consultation for possible surgical intervention.

56.A fall with the wrist in flexion may lead to hyperflexion in addition to a ventrally
displaced distal radius fracture (Smiths fracture). These fractures have a higher risk
of complications and usually require orthopedic consultation.

57.Fractures of the 4th and 5th metacarpals are frequently caused by punching with a
closed fist (boxers fracture).

58.Digital arteries, nerves and veins run on the sides, while the flexor tendons run on
the anterior surface of the phalanges. Attempting to grab a knife with a hand is likely
to injure the tendons than arteries, veins or nerves due to their relative, vulnerable,
anatomic location.

59.Hemochromatosis:
This disease is characterized by liver dysfunction, central hypogonadism, diabetes
mellitus, arthropathy and skin pigmentation. It can cause arthritis at the second and

third metacarpophalangeal joints; typical hook like osteophytes may be seen on X ray.
The combination of diabetes mellitus and skin pigmentation is referred to as bronze
diabetes. All the clinical manifestations are due to excess iron deposition and cellular
destruction. Serum iron are the preferred screening method. A fasting transferrin
saturation over 50% is very suspicious of hemochromatosis. Liver biopsy is the gold
standard for diagnosis. Checking for the familial hemochromatosis gene (C282Y and
H63D) is very useful for identifying family members who are at risk for developing
hemochromatosis.

60.Patellofemoral pain syndrome (PFPS): (runners knee)


Is the most common cause of knee pain in young adults. It is much more common in
women than men, likely due to the greater angle between main axis of the quadriceps
group and the generally vertical axis of the patellofemoral groove. Pts with PFPS have
poorly localized pain at the anterior knee that is worsened by activities that involve
quadriceps contraction such as squatting or ascending and descending stairs. The course
is usually subacute to chronic. The diagnosis of PFPS is primarily clinical and may be
challenging as objective findings are often minimal. The patellofemoral compression test
(pain elicited by extending the knee while compressing the patella) and reproduction of
pain with squatting are highly suggestive. Imaging tests such as X rays and MRI of the
knee are usually normal and are generally needed only if the diagnosis is in doubt or the
pt fails to improve as expected.
Management of PFPS z primarily biomechanical-exercises to stretch and strengthen the
thigh muscles- & avoiding activities that aggravate the pain. NSAIDs r often prescribed
but frequently are not helpful. Persistenc z required on part of the patient as resolution
of symptoms may take weeks to months.

61.Anserine bursitis is a common cause of medial knee pain. Pts have localized pain and
tenderness at the medial aspect of the knee joint distal to the joint line.Symptons are
usually acute or episodic.

62.Osgood schlatter syndrome causes pain at the insertion of the patellar tendon at the
anterior tibial tubercle. It occurs almost exclusively in adolescent and preadolescent
patients frequently following a rapid growth spurt. Pts will have localized tenderness
at the tibial tubercle.

63.Patellar tendonitis (Jumpers knee) z most common in pts who participate in Jumping
sports (e.g volleyball, basketball). Pain is episodic and localized to the inferior patella
& patellar tendon. Tenderness at the inferior margin of the patella is a typical finding.

64.Prepatellar bursitis (housemaids knee) is a common cause of anterior knee pain in


pts who work extensively on their knees. Unlike PFPS, the pain of prepatellar bursitis
is typically acute and highly localized with visible swelling anterior to the patella.
Prepatellar bursitis is frequently complicated by secondary infection (septic bursitis)
due to Staph. aureus.

65.Preoperative evaluation:
>Smoking cessation atleast 4 weeks prior to surgery decreases the risk of postoperative
pulmonary complications.
>Routine preoperative pulmonary function tests or ABGs do not help predict or change
postoperative complications.
>Preoperative PFTs are recommended;
=>Prior to lung resection to estimate postoperative lung volumes.

=>To optimize preoperative COPD control if baseline clinical status cannot be


determined.
=>To evaluate the cause of dyspnea or exercise intolerance (e.g cardiac disease vs
deconditioning)
>The 6-minute walk test and cardiopulmonary exercise testing are useful risk assessmnt
tools in pts undergoing lung resection surgery or various forms of cardiac surgery.
>What are the risk factors for postoperative pulmonary complications?
->Age >50
->Severe COPD
->Current cigarette smoking
->Obstructive sleep apnea
->Pulmonary HTN
->Congestive heart failure
->Poor general health
->Metabolic factors (e.g serum albumin < 3 g/dL)

66.Back pain due to nerve root irritation presents as brief and shooting pain & radiates
along the corresponding nerve distribution. It can be provoked by forward bending,
coughing, or straining and it diminishes on lying down. A positive straight leg raising
test at a 60 degrees angle or less is characteristic.

67.Spinal stenosis is responsible for neurogenic claudication. The pain in spinal stenosis
increases with extension of the spine and decreases with the flexion of spine.

68.

69. NSAIDs and colchicine are the first line treatments for acute gout flares but are
contraindicated in pts with renal failure. Instead, corticosteroids shd be used in these
pts and can be injected locally in pts with a monoarticular arthritis or given orally in
pts with multiple joint involvement.

70.The classic dashboard injury results from a posteriorly directed force on the anterior
aspect of proximal tibia with the knee in a flexed position. This results in disruption of
the posterior cruciate ligament. A similar injury can be seen in an athlete who falls on
a flexed knee with the foot in plantar flexion. It presents with rapid onset swelling but
does not result in a pop and unstable knee as in ACL injury.

71.Medial meniscus injury:


Mensical injuries principally occur after a twisting injury to the knee with one foot fixed
to the ground (e.g with sudden turning while running). The medial meniscus is more
commonly injured as compared to the lateral meniscus. A bucket handle tear is most
common type of medial meniscus tear. Pts generally complain of pain and swelling of
the knee and a popping sensation at the time of injury. In medial meniscus injuries,
tenderness is generally felt along the medial side of the knee. Effusion with meniscus
injuries takes about 24 Hrs to form and is often not significantly bloody, unlike ACL or
osteochondral injuries. Locking of knee joint on extension is generally seen. McMurrays
sign is snapping felt with the tibial torsion and the knee flexed at 90 degrees.

72.Medial and lateral collateral ligament are injured when the line of force strikes from
the side of the joint. An abduction injury with torsional component causes a medial
collateral ligament tear in most of the instances. On examination, the knee joint is
swollen due to the effusion, with tenderness over the medial aspect of the knee. As
the medial collateral ligament resists valgus angulation at the knee, injury to this
ligament leads to increased angulation of the affected knee on valgus stress. A valgus
stress test is generally performed with the knee flexed at 20-30 degrees.

73.Lumbar spinal stenosis:


It is a degenerative disease that frequently affects the elderly population. It usually
appears during sixth decade of life and is very unusual before that age. The associated
pain characteristically disappears/decreases upon sitting down, increases with spine
extension & decreases with flexion or bending forward (e.g when using a grocery cart).
The encroaching of osteophytes at the facet joints, hypertrophy of the ligamentum
flavum, & protrusion of intervertebral discs results in narrowing of the spinal canal.
In some pts, gait disturbance is so prominent that they complain of having spaghetti
legs or walking like a drunken sailor. The preservatn of pedal pulses helps distinguish
the disease from vascular claudication.
MRI is the investigative procedure of choice for suspected lumbar spinal stenosis.
Surgical decompression through a laminectomy is an option when other therapies fail.

74.Fever and leukocytosis can occur in gout.

75.Cyclosporine causes hyperuricemia in more than 50% of the pts by decreasing the
renal urate excretion. Joint aspiration will determine the definitive diagnosis of acute
monoarthritis in majority of the pts.

76.Positively birefringent rhomboid shaped crystals occurs with pseudogout/CPPD


deposition disease. Knee joint is most commonly involved. CPPD may present like
rheumatoid arthritis and osteoarthritis.

77. A WBC count of more than 1,00,000 cells/mm3 in the synovial fluid is indicative of
acute septic arthritis.

78.Calcium phosphate crystals are small but they aggregate with each other to form
large, coin like particles. Calcium phosphate deposition usually involves the shoulder
joint and causes a large, cool effusion, joint destruction, and periarticular calcification
(aka Milwaukee shoulder).

79.Calcium oxalate crystals are bipyramidal in shape and have variable birefringence.
Calcium oxalate induced arthritis z usually seen with endstage renal disease; however
,the crystals are usually not seen with ESRD pts who have successfully undergone
kidney transplantation as the creatinine clearance improves.

80. NSAIDs and Cyclosporine decrease the vasodilatory prostaglandins and concurrent
use of both can compromise the renal blood flow in post-transplant period.

81.Probenecid is mainly uricosuric agent and is of no use when creatinine clearance


drops as in renal failure or post transplant pts.

82. Allopurinol;
=>Dont use during acute attacks of gout
=>Once its started, Azathioprine dose needs to be reduced by 50-70% in post
transplant pts, to avoid toxicity of azathioprine.

83.Acute monoarticular arthritis in renal failure & post-transplant pts is best treated by
increasing dose of systemic steroids or by injection of intraarticular glucocorticoids.
The use of Allopurinol, probenecid, NSAIDs and Colchicine is not recommended for
such pts.

84.The characteristic Chest X ray finding in Sarcoidosis is the presence of bilateral hilar
adenopathy, with or w/o an associated right paratracheal lymph node enlargement.
In the absence of histologic diagnosis, the presence of typical clinical examination
(erythema nodosum) and radiologic findings (bilateral hilar adenopathy) is a reliable
indicator for the diagnosis of sarcoidosis.
An elevated ESR, an increased ACE levels and hypercalcemia can all be seen in pts
with sarcoidosis. However, these nonspecific abnormalities can be seen with other
medical conditions as well.

85.DVT prophylaxis in Hip fractures:


Pts with Hip fractures are at high risk for DVT at the time of fracture & subsequent bed
rest. Because of this thromboembolic prophylaxis has been recommended for all pts
with hip fractures. Current guidelines recommend using fondaparinux, low molecular
wt heparin, adjust dose warfarin or low dose unfractionated heparin. Of these agents,
LMWH is the most cost effective and preferred agent. Many physicians also prefer
fondaparinux as a first line agent , despite being expensive.
It is currently recommended to treat preoperatively upon admission with either LMWH
or low dose unfractionated heparin. Both of these are short acting agents & have been
shown to have relatively low perioperative bleeding risk while significantly lowering the
risk of DVT. Anticoagulation shd then be continued for at least ten days (up to 35 days)
after surgery depending on the pts risks for thrombosis.
BOTTOM LINE: Low molecular weight heparin is considered the prophylactic therapy of
choice for preventing deep vein thrombosis in pts with hip fractures and
shd be started on admission, even if the pt is scheduled for surgery. This
can be stopped 12 hours before the surgery.

86.->Warfarin is 2nd line for DVT prophylaxis.


->Aspirin is used for DVT prophylaxis when both Heparin and warfarin are
contraindicated
->Compression stockings prevent calf vein thrombosis but not pelvic vein thrombosis
,so they cannot be used as sole agents for DVT prophylaxis.

87.Scleroderma renal crisis:


Scleroderma renal crisis presents with severe Hypertension and renal failure in a patient
with evidence of underlying scleroderma, such as Raynaud phenomenon and GERD.
Scleroderma (systemic sclerosis) is characterized by the abnormal deposition of collagen
in multiple organ systems. The majority of pts have underlying renal involvement, but
overt symptoms of scleroderma renal disease are seen less commonly. Thickening of
the vessel wall and narrowing of the vascular lumen in the renal arterioles result in
ischemia, which activates the renin-angiotensin system. This results is hypertension,
which is often as severe as malignant hypertension. It is therefore important to monitor
the blood pressure in scleroderma pts and the presence of hypertension shd alert the
physician to the possibility of scleroderma renal crisis. Acute renal failure is also typical
of scleroderma renal crisis and is likely secondary to a combination of severe HTN
superimposed on the abnormal renal vasculature.
ACE inhibitors are the agents of choice in the treatment of scleroderma renal crisis since
these reverse the angiotensin induced vasoconstriction. Most pts respond favorably if
these drugs are used promptly after the diagnosis and the goal shd be to reduce the
blood pressure to baseline over 72 hours. The ACE inhibitor of choice in these pts is
captopril, given that it has relatively short time to onset & is the agent with which there
is the most clinical experience for scleroderma renal crisis. ACE inhibitors also alleviate
renal failure in the majority of pts in the long term, although they may temporarily

worsen renal failure in the short term. Since ACE inhibitors result in efferent arteriole
vasodilation, there may be a mild drop in GFR and subsequent worsening of creatinine,
which is already commonly elevated in scleroderma renal crisis. Creatinine should be
monitored closely in these pts but generally any increase in creatinine is mild and does
not warrant cessation of ACE inhibitor. While ACE inhibitors are generally avoided in
most pts with acute renal failure, scleroderma renal crisis is an exception to this general
rule as long as renal function is closely monitored.
The addition of intravenous medicatns such as Nitroprusside z needed if CNS symptoms
or papilledema is present.Dont let the blood pressure drop suddenly as it can cause
renal hypoperfusion & subsequnt acute tubular necrosis.Only one dose of nitroprusside
is generally necessary to gain control of blood pressure acutely as the captopril takes
effect.

88. Calcium channel blockers may be used as second line agent in addition to captopril
in the management of chronic HTN in pts with scleroderma. CCBs may also be of
benefit in the management of Raynauds phenomenon in these pts.

89.Pts with a fracture following minor trauma such as ground level fall is likely due to
osteoporosis. Confirmation with measurement of central ( i.e spine and hip ) bone
density is recommended, preferably with Dual energy X-ray absorptiometry (DEXA)
scan.
T-score of -1 to -2.5 -> Osteopenia
T-score of < -2.5 -> Osteoporosis
T-score of < -2.5 and a prior fragility fracture-> Severe or established
osteoporosis

90.Bone scan for;


=>Osteomyelitis
=>Pagets disease
=>Bone metastasis
=>Arthritis

91. Bisphosphonates r generally the first line therapy for postmenopausal osteoporosis.
Bisphosphonates lower the fracture risk by decreasing the bone resorption.
When do you give bisphosphonates ?
T-score of -2.5 or less
Low trauma hip or vertebral fractures regardless of T-score
Elevated 10 year fracture risk calculated by Fracture risk assessment tool
(FRAX) , greater than 20% for major osteoporotic fractures and greater than
3% for a hip fracture.
In addition to bisphosphonates, postmenopausal women with osteoporosis shd also
take supplemental calcium and Vit-D.

93.Thoracic Kyphosis:
Flexible thoracic kyphosis is a common finding in adolescents. It is typically noticed by
parents or teachers who observe the adolescents sitting or standing in a slouched
position & noticed as round back. On lateral radiographs, the angle of thoracic kyphosis
is normal or slightly increased (Normal 20-40 degrees).
Thoracic kyphosis is easily correctable by voluntary efforts and on prone-extension test.
There is no evidence that flexible kyphosis leads to adverse physical effects or
permanent physical deformity.
In contrast to the flexible kyphosis which is a benign finding in many adolescents,
Scheuermann disease is a structural disorder. Structural kyphosis is not corrected

with voluntary efforts (i.e voluntary hyperextension) and a sharp angulation z commonly
seen on forward bending. The typical treatment fr structural kyphosis that is not severe
(less than 70-80 degrees) includes the use of a Milwaukee brace. In more severe cases
(significant angulation, intractable pain, neurological abnormalities), surgical correction
is used.

92.Teriparatide (PTH 1-34) is only the second line agent for osteoporosis in post
menopausal women becx of its high cost, particularly in those in whom
bisphosphonates therapy has failed.

93.Carpal tunnel syndrome:


Pts present with pain, paresthesias, or less commonly weakness in the hand. Carpal
tunnel syndrome is caused by compression of the median nerve as it passes through
the carpal tunnel. Symptoms are frequently worse at night, and may be relieved by
shaking the hands or running them under warm water.
Carpal tunnel syndrome is primarily a clinical diagnosis and there are several physical
exam maneuvers that can help confirm it. Phalens test consists of hyperflexion of both
wrists. A positive test reproduces characteristic symptoms within one minute of starting
flexion. Hyperflexion at the wrist exacerbates compression of the median nerve in the
carpal tunnel. A positive Tinel sign consists of reproduction of characteristic symptoms
upon tapping or percussing over the region of the median nerve at the carpal tunnel.
Phalens test is slightly more sensitive than Tinels test but both demonstrate a similar
specificity of about 75%. A hand elevation test is positive if symptoms are reproduced
by raising the hands over the head.
Nerve conduction studies are frequently used to confirm the diagnosis of Carpal tunnel

syndrome & frequently combined with EMG studies. Compression of the median nerve
results in loss of normal myelination. Axonal loss may also occur if the compression is
particularly severe. Nerve conduction studies will therefore demonstrate slowed
conduction along the median nerve at the level of carpal tunnel with a normal conductn
velocity proximally. Reduced amplitudes of nerve conduction can occur if there is axonal
loss. EMG is helpful in demonstrating that other nerves are not involved by detecting
denervation in the muscles typically innervated by the median nerve with relative
sparing of those that are not.
Night time splinting shd be the first line treatment for pts with mild to moderate carpal
tunnel syndrome for less than 10 months. Other conservative measures such as Injected
or oral corticosteroids shd then be attempted if night time splinting alone doesnt result
in symptom improvement. Surgical decompression shd be used in pts who failed
conservative measures & have had moderate to severe symptoms for at least 6 months.
*NSAIDs are not recommended.

94. The painful arc test refers to the presenz of shoulder pain when the arm is abducted
between 60 and 120 degrees and is characteristic of rotator cuff impingement.

95.ESR is raised in RA and X rays can show periarticular erosions.

96.Rheumatoid arthritis:
In North America and Europe, Methotrexate is commonly selected for early therapy of
active disease. In Pts who are resistant to methotrexate and corticosteroids after six
mnths of therapy are, we either switch to or add Anti-cytokine agents (e.g infliximab,
etanercept). The use of anti-cytokine therapy is associated with a higher incidence of
opportunistic infections , particularly reactivation of tuberculosis; therefore, all such

pts shd be screened for latent TB by PPD skin testing.


Other potential solutions are triple therapy with methotrexate, hydroxychloroquine and
sulfasalazine or switching to cyclosporine.
Minocycline is a tetracycline antibiotic that may be used in pts with mild disease (it
inhibits metalloproteinases and slows joint damage).
Any monoarticular arthritis in pts with RA, especially when accompanied by systemic
signs of infection (e.g fever) shd be considered as septic arthritis until and unless proven
otherwise. Joint aspiration shd be the next step becx this will help to establish diagnosis
and institute the appropriate treatment quickly.

97.Fat embolism:
Fat embolism is a clinical diagnosis. It typically manifests 24-72 Hrs after severe trauma
and is usually characterized by a triad of respiratory insufficiency, neurologic impairmnt
and a petechial rash. Other common findings include fever, tachycardia and altered
mental status. The classic petechial rash is most commonly found on the trunk, this is
typically a late manifestation and is described in fewer than 50% of cases. Petechiae
result from the occlusion of dermal capillaries by fat globules, thereby leading to
extravasation of erythrocytes. No abnormalities of the platelets function have been
documented.
Early immobilization and operative fixation of fractures reduces the chances of fat
embolism. Supportive therapy is the mainstay of therapy for clinically apparent fat
embolism.

98.Patellar tendon rupture:


The most common mechanism is a sudden and unusual quadriceps contraction with the
foot firmly planted and with the knee and with the knee in partial flexion during the

injury. Patellar tendon tear or rupture usually occurs at the osseotendinous junction
and results in excruciating pain, swelling and difficulty in bearing weight. Physical ex.
generally reveals swelling and tenderness in the anterior part of the knee. With a
complete tendon rupture, the pts are unable to perform active extension of the leg
and are unable to maintain the passively extended knee against gravity.
BOTTOM LINE: Patellar tendon rupture presents with excruciating pain, swelling in the
anterior part of the knee, and an inability to maintain passive extension
of the knee against gravity.
Early surgical intervention and repair of the patellar tendon is the treatment of choice.
Delayed treatment can lead to quadriceps muscle atrophy, contracture formation, and
limited range of motion of the knee, thereby causing early fatigue, pain and significant
disability. Early diagnosis and surgical repair usually leads to excellent recovery of knee
function and prevent long term disability.

99. The valgus and varus stress tests are used to determine the integrity of the medial
& lateral collateral ligaments, respectively. A history of knee injury, local tenderness
on the medial or lateral joint lines & an abnormal opening of the knee as compared
to the opposite side on valgus or varus stress is suggestive of ligament injury or
disruption.

100.McMurray sign-> Popping sensation on external rotation and passive extension of


the lower leg.

101.Low back pain:


Physicians shd take a comprehensive history and do a detailed physical examination for
pts presenting with low back pain.
Pts with signs and symptoms of cord compression (e.g bowel or bladder dysfunction,
saddle anesthesia, focal spinal tenderness, motor weakness, other neurologic deficits)
require urgent MRI of the back to exclude cauda equine syndrome due to infection or
tumor.
All other pts with radiculopathy (e.g sciatica) or red flags associated with increased risk
of systemic disease (e.g night time pain, age >50, unexplained weight loss, history of
malignancy) can undergo plain X ray films of the back and ESR measurement. Plain film
X-rays can identify lytic bone lesions and a significantly elevated ESR (although nonspecific) may suggest an inflammatory condition (e.g osteomyelitis, inflammatory
arthritis, epidural abscess). Pts with abnormal X-rays or ESR results shd have an MRI;
those with normal X-ray and ESR results can be managed conservatively and have an
MRI in 4-6 weeks if there is no improvement.
In the absence of cord compression, sciatica, or red flags, pts with low back pain shd be
managed conservatively with nonopioid analgesics and physical therapy.

102.Peripheral arthritis and enthesitis can occur with Anklyosing spondylitis.

103.Chronic glucocorticoid therapy:


Chronic glucocorticoid therapy increases the risk of osteoporosis by decreasing the
intestinal calcium absorption and increasing the calcium excretion in urine and also
accelerates the bone resorption. For this reason, it is important to provide the lowest
dose of steroids for shortest duration & to add vit-D as well as calcium supplementation
to the pts therapy. Based on the duration of therapy, bone densitometry is indicated

at baseline and every year onwards.

104. Bisphosphonates might be teratogenic in premenopausal women.

105. Compartment syndrome:


Common etiologies include long bone fractures, trauma without fracture (e.g crush
injuries, thermal burns, and vascular injury in extremities) and non traumatic causes
(e.g prolonged limb compression, animal venom exposure, and nephrotic syndrome).
Human muscle is divided into compartments linked by strong fascial planes that are
not flexible. Each of the above etiologies can cause either a decrease in compartment
size or an increase in compartment pressure that does not allow adequate tissue
expansion for an increase in fluid in the injured area. The rise in the intracompartmental
pressure causes decreased venous outflow, increased venous pressure, decreased
arteriovenous pressure gradient and decreased perfusion to the tissues. The end result
is tissue edema & increased interstitial pressure. In the later stages, arterial blood flow
also decreases, further worsening compartment pressure and edema.
Early symptoms for compartment syndrome include pain out of proportion to the injury
(the most sensitive and early sign), persistent deep pain and paresthesias. Early physical
signs of acute compartment syndrome include tightness of the area, muscle weakness
and pain with passive stretching of the muscles. Pts left untreated can have symptoms
progress to paralysis, muscle contracture, sensory deficits, infection and skin ulceration
tissue necrosis, possibly requiring amputation of the limb. Compartment pressures shd
be measured and pressures > 20-30 mmHg typically require fasciotomy as definitive
therapy to relieve high compartment pressure.

106.Sicca syndrome:
Presents with dry mouth (xerostomia) and dry eyes (keratoconjunctivitis sicca). Its very
common in the elderly ( prevalence may be as high as 35-36%).
The most reasonable initial approach is confirming the secretory deficiency (schirmers
test) and doing an autoantibody screen (Ro- and La- antibodies, RF, ANA etc).
Although labial salivary gland biopsy is the gold standard for diagnosing Sjogrens synd,
its typically not the first test to perform. It helps to confirm the diagnosis especially in
young pts with aggressive disease.

107.Viral arthritis:
It typically occurs after an upper respiratory infection and is usually due to Parvovirus.
Rheumatoid arthritis like symptoms occur with small joints involvement and there is a
weakly positive RF test. The symptoms duration is usually less than 6 weeks and there
is no evidence of synovitis. (The hallmarks of synovitis include soft tissue swelling,
warmth over a joint, joint effusion).
Viral arthritis is typically self limited and of short duration. Therapy is generally directed
at the relief of symptoms and maintenance of function; therefore , pts are treated with
simple analgesics and antinflammatory drugs. Physical and occupation therapy may be
employed if required to maintain or improve function.

108.Proximal muscle weakness with increased CPK levels indicates a muscle disorder.
Inflammatory myositis can lead to increased serum Ferritin and ESR levels which
are acute phase reactants.
Majority of pts with inflammatory myositis respond to systemic high dose
glucocorticoids (e.g prednisone 1 mg/kg/body weight). Occasionally, CPK levels
decline w/o any significant improvement in muscle strength. In such cases, long

term steroid use is necessary. Other immunosuppressants can be added to treat


pts with no clinical response to steroids, or to create a steroid-sparing effect.

109.Polymyalgia rheumatica: (PMR)


Characteristic features of PMR are;

Age >50
Subacute to chronic (>1 month) pain in the shoulder and hip girdles
Morning stiffness lasting >1 hour
Constitutional symptoms (fever, malaise, weight loss)
Elevated ESR (>40 mm/hr)
No other apparent explanation for symptoms

The physical examination in PMR is frequently unremarkable, wth pts indicating no focal
tenderness or pain with active or passive range of motion. Signs of inflammation in the
joints are absent. When asked to identify the location of their pain, pts typically indicate
the soft tissues, not joints.
Low dose glucocorticoids are the treatment of choice fr PMR (e.g prednisone 10-20 mg
daily). Rapid and thorough relief of symptoms is expected and failure to improve rapidly
on prednisone shd call the diagnosis into question. PMR is frequently associated with
giant cell arteritis (GCA), aka temporal arteritis. Symptoms of GCA include headache, jaw
claudication, vision loss & tenderness over the temporal artery. If GCA is suspected, pts
shd be considered for an expedited temporal artery biopsy & receive significantly higher
doses of glucocorticoids (e.g prednisone 40-80 mg or higher daily).
BOTTOM LINE: Polymyalgia rheumatica (PMR) affects pts age >50 and is characterized
by elevated ESR and pain and stiffness in the neck, shoulders & pelvic
girdle. The treatment of choice for uncomplicated PMR is low-dose
prednisone, which results in rapid relief of symptoms.

110.Flail chest:
Flalil chest is usually the result of double rib fractures in more than one site, with
paradoxical motion of the free segment of the chest wall during inspiration. It is often
accompanied by muscular spasm and pain as well as pulmonary contusions, which lead
to hypoxemia and increased work of breathing. The paradoxical or segmental chest wall
movement will typically make the diagnosis of flail chest obvious. However, many
cases are occult and gross findings may be absent due to muscle splinting, shallow
breathing or mechanical ventilation. The occult flail chest typically presents with
tachypnea, tachycardia, shallow breathing & signs of inadequate ventilatn e.g cyanosis.
There might be anterior chest wall bruises as a clue for occult flail chest.
Management includes supplemental oxygen, pain control (invasive or non invasive),
positive pressure ventilation and surgical stabilization in severe cases.

111.Achilles tendon rupture:


The Achilles tendon is composed of the gastrocnemius-soleus complex tendons and is
commonly injured when athletes abruptly increase their activity. The ruptures produces
a popping sound and results in calf pain.
The most appropriate physical examination maneuver to test for complete rupture of
the Achilles tendon is the Thompson test which has a sensitivity of 96% and specificity
of 93%. With the pt lying in the prone position, feet hanging off the table, the clinician
shd squeeze the pts calf muscles and observe for the presence of plantar flexion of the
foot. If plantar flexion is observed on calf squeeze, the test is negative and indicates
normal function of the Achilles tendon.
Absence of active plantar flexion is not as reliable as the calf-squeeze test to rule out
Achilles tendon rupture becx the pt can also use accessory muscles to actively plantar
flex and falsely reassure the clinician.

Achilles tendinopathy w/o rupture requires no further imaging. MRI is the diagnostic
modality of choice for Achilles tendon rupture and particularly useful in the setting of
high clinical suspicion with a negative Thompson test or if partial tendon tears are
suspected.

112.Plantar sensory loss is seen in pathology affecting the medial and lateral plantar
nerves and/or the medial calcaneal nerve, all of which are branches of the tibial
nerve. A defect of the tibial nerve would result in loss of both gastrocnemius-soleus
motor function and plantar sensory.

113.Complications of dry mouth in sicca syndrome include dental caries, candidiasis


in 70% of pts, and chronic esophagitis.
To reveal xerostomia in pts suspected with Sjogrens syndrome/ Sicca syndrome,
some Yes/No questions may be asked, such as Do you wake up at night feeling
dry ad then drink some water? or Do you frequently drink water to help you
swallow some dry foods?.

114.Lung involvement is present in 90% of pts with sarcoidosis. The classic presentation
is bilateral hilar adenopathy with or w/o pulmonary parenchymatous changes.

115. Reactive arthritis: (ReA)


Spondyloarthropathies are a group of disorders characterized by sacroiliitis, peripheral
asymmetric oligoarthritis, dactylitis and enthesitis.
Reactive arthritis is a spondyloarthropathy that presents with peripheral asymmetric
oligoarthritis, often associated with;

Uveitis (blurry vision)


Urethritis (discharge/burning urination)
Achilles enthesitis (Pain at ligament/tendon insertion sites)
Dactylitis
Keratoderma blennorrhagica
Circinate balanitis (painless, shallow ulcers of glans penis)

ReA usually occurs after a genitourinary (e.g Chlamydia trachomatis) or gastrointestinal


(e.g Salmonella, Shigella, Yersinia, Campylobacter) infection. No pathogen is usually
isolated on synovial fluid cultures, although signs of inflammation (elevated WBC count)
are present on fluid analysis and some studies have reported the presence of Chalmydia
DNA from synovial fluid by PCR.
HLA-B27 is present in only 30-50% of cases and is neither sensitive nor specific.
BOTTOM LINE: Reactive arthritis presents with asymmetric oligoarthritis, usually with a
hx of genitourinary or gastrointestinal infection. Enthesitis and dactylitis
may also be seen. Extra-articular manifestations such as uveitis, keratoderma blenorrhagica, and circinate balanitis occur in a minority of pts.
Synovial fluid culture has a high white cell count but culture is negative.
The incidence of reactive arthritis after infection with Chlamydia trachomatis or a
predisposing gram negative rod is higher in indiviudals who are HLA-B27 positive.

115.Medial Collateral ligament injury:


MCL tear is a common knee injury that is typically caused by a valgus (abductor) stress
test to the partially flexed knee. It typically occurs following a blow to the lateral knee
with the foot fixed but may also be seen following a severe twisting injury (e.g skiing).
MCL tears may occur in isolation or can be associated wth injury to the medial meniscus.
MCL tears are characterized by tenderness at the medial knee and laxity when the foot
is forced into abduction with the knee stationary (valgus stress test). Acute effusions/
hemarthrosis is uncommon unless there is concurrent injury to the anterior cruciate
ligament. Due to local swelling and muscle spasm, examination findings may be unclear
if the pt is examined more than 30 mins after the injury. In such cases repeat examinatn
at a later date may reveal the full extent of the injury. X ray is not needed unless there
z concern for additional bone injury & MRI is generally reserved fr pts being considered
fr surgical interventn. Pts with uncomplicated MCL tears can be managed conservatively
with RICE measures and analgesics with progressive return to activity as tolerated.
BOTTOM LINE: Medial collateral ligament injuries occur commonly when a force is
applied to the knee from the lateral to the medial direction during skiing
or contact sports. Examination shows valgus laxity. Most pts are managd
conservatively.

116.Prepatellar bursitis is most frequently caused by trauma from a fall or secondary to


prolonged friction and pressure from repetitive kneeling (housemaids knee).

117.Any change in the character or intensity of pain in a pt with chronic pain syndrome
shd be thoroughly investigated before attempting to give any pain medication or
assumption of drug seeking behavior. The primary goal of emergency physician is
to evaluate for potentially dangerous causes that, if not identified, could result in

significant morbidity and mortality.

118.Transient synovitis/Toxic synovitis:


Transient synovitis aka toxic synovitis is a common condition that causes pain in the hip,
thigh or knee in boys aged 3-10 years old. Upto 25% of children with transient synovitis
will have bilateral effusions. The disorder may be preceded by a respiratory infection,
although the ESR & WBC count are typically normal. If the clinical presentation suggests
transient synovitis and plain radiographs are unremarkable, an Ultrasound should be
performed. Ultrasonography is the preferred technique for identifying the small joint
effusions and may demonstrate widening of the joint space of the hip.This modality also
is useful in guiding aspiration of the joint, a procedure that is warranted in a febrile child
with hip effusion.
BOTTOM LINE: Transient synovitis of the hip z a common, self limited condition in young
boys that is best evaluated by Ultrasound.

119.Pagets disease:
X rays show sclerotic lesions and bowing of the femur. Technetium bone scan shows
increased uptake.
Treatment of Pagets disease is indicated when pts have intolerable pain, involvement
of weight bearing bones, neurological impairment, hypercalcemia, hypercalcinuria,
and congestive heart failure. Several bisphosphonates (alendronate, risedronate, and
pamidronate) are approved for the treatment of Pagets disease. Six months of treatmnt
wth oral alendronate & two months of treatment wth oral risedronate lead to sustained
remission in a large number of patients. IV pamidronate can be used in patients with
contraindications to use of oral bisphosphonates. Treatment wth older bisphosphonates
(etidronate) & calcitonin are not currently used for the management of Pagets disease.

120.Statins myopathy-> Elevated CPK level and normal ESR.

121.Stress fractures:
Stress fractures are a common injury in athletes (upto 15% incidence in runners) and
nonathletes who suddenly increase their activity. Besides runners, stress fractures are
common in ballet dancers, basketball and soccer players and military recruits. The risk
of stress fractures can be increased by activity relatd (e.g excessive training or improper
footwear), biomechanical (e.g weak calf muscles or high arched feet) and metabolic
(e.g demineralized bone from hormonal or nutritional diseases) factors. Stress fractures
occur due to a sudden increase in repeated tension or compression without adequate
rest, which eventually breaks the bone.
Pts usually initially develop medial tibial stress syndrome (shin splints) without tibial
tenderness. Further activity can cause progression to a complete or incomplete fracture,
resulting in tibial tenderness on palpation. A stress fracture is diagnosed clinically when
there is pain at a specific area that increases with jumping or running and is associated
with local swelling and point tenderness on examination.
Initial Plain X rays of the tibia have low sensitivity and are negative during the first few
weeks in over half of the cases. Possible abnormalities on X rays may take upto 4 weeks
to become apparent and include bone sclerosis, cortical thickening, periosteal elevation,
and visible fracture line. If the initial X-ray is negative, pts may be managed empirically
based on clinical findings. However, if a definitive diagnosis is needed, MRI is preferred
over bone scan or ultrasound as it can show the fracture line that extends through the
cortex into the medullary line. MRI can also identify ligament, muscle and cartilage
injuries. However, MRI findings may be persistently abnormal for up to 1 year after the
stress fractures has healed.
BOTTOM LINE: Tibial stress fractures are common in athletes and non athletes who

suddenly increase their physical activity. Clinical features include pain,


localized tenderness and swelling. Plain X ray is <50% sensitive for stress
fracutres, esp in the first 2-3 weeks after the onset of symptoms.
Tibial stress fractures can be managed in most cases with pneumatic splinting, reduced
weight bearing and a graduated exercise program. Crutches are often advised in the first
1-2 weeks to minimize the weight bearing. Most pts may resume full intensity exercise
within 12 weeks but the program may need to be delayed if the pt develops recurrent
pain.

122.Pagets disease of bone:


The CT scan of head shows thickened calvarium with an inhomogenous bone density
or cotton wool appearance. Next step in pts with typical CT head findings accidentally,
shd be calcium and alkaline phosphatase. The results wud show normal calcium and
raised alkaline phosphatase. If necessary, the elevated alkaline phosphatase shd be
differentiated from the hepatobiliary disease by measuring the bone specific fraction.
Pts shd also have a radionuclide bone scan to identify other involved sites.Pagets
disease is due to increased bone turnover and bisphosphonates are the mainstay
of therapy becx they decrease the bone turnover.Most pts, including those with bone
pain or involvement of high risk areas (such as skull) she be considered for treatment.
Hearing loss is a common complication of Pagets disease of bone likely due to auditory
nerve compression or involvement of the cochlea and/or cochlear capsule itself.
Treatment with calcitonin or bisphosphonate therapy may slow the progression of
hearing loss but is unlikely to reverse the loss that has already occurred. Other
interventions that have been studied include external hearing aids & cochlear implants.

123. Rheumatoid arthritis:


Clinical features of symmetric polyarthritis (warmth, swelling and tenderness) associatd
with morning stiffness of more than 30 mins for several weeks ( more than six ) are
compatible with the diagnosis of Rheumatoid arthritis. ( Arthritis in SLE is typically more
migratory and has a shorter duration of morning stiffness ).
Diagnosis is mainly clinical but presence of rheumatoid factor and anti-CCP is helpful for
confirmation. So, the best next step in high suspicion is to order RF and anti-CCP levels.
Hand X rays shd be ordered which might show periarticular osteopenia and erosions.
NSAIDs are the first line drugs in management and they are helpful for symptomatic
relief. (steroids shd be avoided, if possible ).
Aggressive use of DMARDS is particularly helpful for decreasing the disease progression
in pts with increased number of inflamed joints, functional limitation, extraarticular dx,
bone erosions on X-ray, markedly elevated rheumatoid factor or positive anti-CCP levels.
Anti-malarials (Hydroxychloroquine) , sulfasalazine and methotrexate are the DMARDs
which are most frequently used.
RF is positive in only 70-80% of RA cases, so absence of a positive test does not exclude
the diagnosis. Erosive joint disease in RA is a clear cut indication for the use of DMARDs
and methotrexate is the initial drug of choice for this purpose.
Parvovirus IgG antibodies can be prznt in the general population & are not significantly
associated with RA. Postive IgM antibodies can make a diagnosis of Parvovirus infection.

124. While both SLE & RA frequently present with morning stiffness, this generally lasts
only for minutes in SLE but usually for hours in RA. Symptoms of lupus arthritis are
frequently migratory lasting only a few days in any one location. Pts with SLE also
generally have manifestations in other organ systems, particularly the skin and
kidneys, in addition to the musculoskeletal system.

125.Rheumatoid arthritis is a clinical diagnosis. In pts with negative RF levels, but with
the classic clinical (swelling, warmth and tenderness over the PIP, MCP and wrist
joints and possibly knee joints) & radiographic presentation (Periarticular erosions
and osteopenia) , the most likely diagnosis is still Rheumatoid arthritis. Elevated
RF levels r detected in approximately 75-80% of pts at some time during the course
of their disease. Anti-CCP antibodies have some value in pts with negative RF levels
;besides, they carry a higher specificity.

126.Glucorticoids suppress the release of Gonadotropin-releasing hormone from the


hypothalamus, leading to central hypogonadism, which can aggravate the bone
loss.

127.Consent for treatment is generally obtained from a parent or legal guardian prior
to initiating medical treatment for a minor except in specific circumstances.
It can be medical circumstances such as giving emergency care, treatment of
sexually transmitted infections, substance abuse ( most states ) or prenatal care.
Consent is also not required for emancipated minor such as homeless, the minor
is parent or married, serving military, financially independent or is a high school
graduate.

128. A supracondylar fracture is a medical emergency and delayed treatment can lead
to displacement of the fracture or neurovascular compromise.

129.Asymmetric pulmonary sarcoidosis generally does not need treatment. Hilar


adenopathy associated with erythema nodosum represents a very favorable variant

of sarcoidosis that is associated with a high rate of spontaneous remission and good
prognosis. No treatment, except observation and periodic check ups is necessary.
Symptomatic pulmonary disease is usually an indication for corticosteroid therapy
in pts with sarcoidosis, especially if it is accompanied by systemic symptoms such as
severe fatigue, fever & hypercalcemia. Severe extrapulmonary manifestations such
as cardiac, ocular & neurologic involvement require high dose corticosteroid therapy

130.Anemia in pts with sarcoidosis is typically anemia of chronic disease

131.Dihydropyridine calcium antagonists, typically nifedipine and amlodipine, are


considered first line pharmacological therapy for pts with Raynaud phenomenon.
Raynaud phenomenon may be idiopathic ( also called Raynauds disease ) or
secondary to other conditions such as connective tissue diseases, vascular lesions,
medications etc. When managing pts with Raynaud phenomenon, potential
predisposing factors such as medications and environmental factors (e.g frostbite,
vibration) shd first be excluded. The presence of symptoms suggestive of systemic
disease (arthralgias, myalgias,fever and weight loss) and resistance to treatment,
warrants further evaluation; this may include ANA, RF, CBC, blood chemistry, U/A,
and measurement of complement levels.
Pts are diagnosed with primary Raynaud phenomenon when there are no historic
clues to a secondary condition, normal physical findings and no ischemic digital
lesions. No specialized studies are indicated in these pts.

132.Keratoconjucitivits sicca may present with prominent bulbar vessels and stringy
discharge from the eyes.

133.Sjogrens syndrome:
Anti-Ro-SSA and anti-La/SSB antibodies are commonly present and can be helpful to
confirm ones clinical suspicion of Sjogrens syndrome along with decreased wetting on
filter paper in Schirmer test , as no true diagnostic test is available.
B-Lymphocyte activation occurs as a component of the chronic inflammation in pts with
Sjogrens syndrome. This activation results in an increased risk of B-cell non-hodgkins
lymphoma.
Metastatic squamous cell carcinoma frequently spreads to lymph nodes in the head and
neck but is usually associated with a history of tobacco or alcohol use.

134.Radial head subluxation/Nursemaids elbow:


Occurs due to sudden pulling of the childs arms or swinging a child by the arms. It
typically occurs in children aged 6 months to 5 years. The sudden pulling force causes
the annular ligament to slide over the radial head & become entrapped in radiohumeral
joint. Children wth this injury typically hold the arm close to their body with the forearm
pronated & the elbow extended. There is no swelling or tenderness at the elbow region.
Radiographs r not necessary to make diagnosis. Hyperpronation & supination /flexion
are the 2 primary methods of reduction. In the hyperpronation method, the childs arm
z held at the elbow, pressure is applied to radial head & the forearm is hyperpronated.
In the supination/flexion method, the elbow is supported similary while the patients
forearm is quickly supinated and fully flexed at the elbow in a continuous motion.
Hyperpronation hz a significantly higher chance of successful reduction than supination,
although practitioners may use whichever method they r most comfortable performing.
Following reduction, the child shd be able to move the arm normally in 5-10 minutes.

135.Clavicular fracture:
Clavicular fractures account for about 5% of all fractures and the majority occur in the
middle third of ths S-shaped bone, its thinnest part. In a displaced fracture, the proximal
(medial) segment may move superiorly and the distal (lateral) segment inferiorly past
each other, causing shortening of bone. The skin over the clavicle may also be stretched
(tented) leading to skin necrosis, hematoma and conversion to an open fracture.
Open reduction and internal fixation is indicated for open fractures with neurovascular
injury or tenting of the skin, widely displaced fractures, significant shortening, and
comminution. Open reduction and internal fixation requires an incision to be made over
the clavicle with subsequent application of a fixation plate and multiple screws at the
fracture site.
Non displaced or minimally displaced fractures of the middle third of the clavicle are
usually managed conservatively with ice, analgesics, elbow range of motion exercises &
either a sling or figure of eight bandage.
Figure of eight bandage is a harness used in the conservative management of nondisplaced mid third clavicular fractures. It leaves the elbow and hand free for daily
activities, avoids elbow stiffness & sometimes corrects shortening. However, it requires
periodic adjustments to keep it tight and excess tightening risks complications. Most
pts prefer a sling over a figure of eight bandage.
A sling can also be used as conservative therapy in nondisplaced clavicular fractures. and
tends to provide more patient comfort than the figure of eight harness. However, the
sling does not allow the pt to have complete freedom of the elbow and hands for daily
activities. If pts do not perform regular range of motion exercises, elbow stiffness may
occur.
NSAID shd be avoided for pain control becx they can delay the healing process, rather
acetaminophen or opioids shd be used for pain control.

Claviuclar fractures;
Displaced=> Open reduction and internal fixation
Non-displaced=> Conservative (ice,analgesics,sling/figure 8 bandage)
Management is similar in children and adults.

136.Reactive arthritis:
Recent onset asymmetric oligoarthritis that is inflammatory (elevated WBC count) yet
sterile (culture negative) in a pt with a recent history of chlamydia infection is suggestive
of reactive arthritis (ReA).Sacroiliac joints may also be involved. Extraarticular symptoms
such as urethritis, uveitis, circinate balanitis and keratoderma blenorrhagicum may be
present.
ReA usually develops within 1-4 weeks of inciting infection. Thererfore, new symptoms
suspicious for ReA in a pt with a history of chalmydia shd prompt repeat testing for
infection even in the absence of obvious symptoms. The nucleic acid amplification test
(NAAT) for chlamydia is usually performed on the urine samples. Rectal screening is
recommended for men who have sex with men or in pts with a history of chlamydia
proctitis.
Circinate balanitis (painless, shallow ulcers of the glans penis) occurs in 20-40% of pts
with ReA. In some pts, it may be the only initial symptom. Circinate balanitis appears
within weeks of the primary infection. It may persist for several months before the
spontaneous resolution, unlike the painless lesions of lymphogranuloma venereum/
LGV which resolves within a few weeks or syphilis which resolves in 3-6 weeks. In
addition, lymphadenopathy presents in the secondary stages of syphilis. Circinate
balanitis may be treated with topical steroids.
Management of ReA consists of antibiotics against chalmydia infection OR nonresolving GI infection AND NSAIDs for the pain control. In cases of failure of NSAIDs,

we can consider intraarticular steroids and if that fails then we can go to systemic
steroids therapy. If systemic steroids also fail, we can give DMARDs then.
BOTTOM LINE: Reactive arthritis may develop within 1-4 weeks of chalmydia infection.
Circinate balanitis is an extraarticular manifestation of the ReA that is
painless, resolves after several months & is not associated with inguinal
lymphadenopathy. New onset ReA in pts with a history of chalmydia inf.
shd prompt repeat testng fr chlamydia, even in those who are otherwise
asymptomatic. Mangement consists of antibotics and NSAIDs.

137.AST to ALT ratio of 2:1 is seen in alcoholic liver disease.

138.An isolated elevatn of alkaline phosphatase can be seen in infiltrative liver diseases
which include malignancy, granulomatous diseases, infection & certain medications.
Malignancy can often be excluded by the absence of focal lesions on liver imaging.
Additional evidence of granulomatous disease, such as bilateral hilar adenopathy in
sarcoidosis, may be helpful In making the diagnosis.

139.Conditions predisposing to Carpal tunnel syndrome;

Diabetes
Rheumatoid arthritis
Hypothyroidism (Due to mucopolysaccharides deposition in carpal tunnel)
Obesity
Wrist trauma or fracture
Acromegaly
Pregnancy
Menopause
End stage renal disease

Most common are Diabetes, rheumatoid arthritis and Hypothyroidism.

Studies have not shown association between workplace conditions (e.g computer use)
and CTS development. However, activities involving repeated wrist flexion or extension
(e.g typing or driving) can worsen/precipitate the pain if CTS is present.

140.Phalen sign=> Pain with elbows raised & dorsal suface of hands touching each other

141. Carpal tunnel syndrome symptoms sometimes improve with flicking of wrist called
flick sign

142. Surgery is indicated for carpal tunnel syndrome after motor weakness or atrophy
of thenar eminence.

143. Although Diabetes is associated with CTS, but tight blood sugar control does not
improve the symptoms of CTS.

RENAL,URINARY SYSTEM & ELECTROLYTES1


1.Posterior urethral valves are predominantly found in males and are the most common
cause of severe obstructive uropathy in children.The abnormal development of valves
in utero can obstruct urinary flow , leading to detrusor hypertrophy and eventually,
vesicoureteral reflux and hydronephrosis.Hallmarks of PUV include a distended blader
& a weak urine steam.
VCUG is a radiographic examination of the bladder and lower urinary tract that is esp
helpful in evaluating young children for vesicoureteral reflux and posterior urethral
valves.

2.Prune belly syndrome is characterized by a multitude of renal,ureteral and urethral


abnormalities in the neonate.Obstruction and upper urinary tract dilatation are often
observed.Abdominal musculature is underdeveloped , which leads to constipation and
a weak cough.
Radiographs of kidneys, ureter and bladder are often used to make the diagnosis of
prune belly syndrome based on the classic appearance of the bowels hanging over the
lateral edge of the abdominal wall.

3.Refeeding syndrome in alcoholics:


Chronic alcoholics are phosphate depleted even though serum phosphate levels may
initially be normal.
Pts with severe alcoholism often have chronic depletion of phosphate secondary to
decreased vit-D and phosphate intake along with decreased intestinal uptake in those
with chronic diarrhea.Urinary phosphate excretion may also be increased because of
secondary hyperparathyroidism from decreased Vit-D intake as well as proximal tubular

reabsorption defect from alcohol abuse itself.Despite the depletion of phosphate,serum


levels are often maintained ( extracellular shift ) until the pt is admitted to the hospital
and fluids are initiated.
Once the pt is fed or receives intravenous fluids with glucose , insulin secretion is incrzd
which results in shifting of phosphate intracellularly and unmasking of the previously
compensated phosphate depletion.A respiratory alkalosis occurs in many of these pts as
well, which can further shift phosphate into the cells and out of the serum. The resulting
hypophosphatemia can precipitate symptomatic rhabdomyolysis (muscle weakness) in
alcoholic pts who already have baseline underlying myopathy, hence CPK levels shd be
rechecked if previously normal.

4.Secondary HTN :
>Secondary HTN shd be suspected in pts with severe HTN resistant to multiple medctns
, worsening from a previously stable state, young age of onset or malignant HTN.
>Renovascular HTN is the most common cause of secondary HTN and she be suspected
in pts with widespread atherovascular disease, elevated creatinine, unilateral renal
atrophy , recurrent flash pulmonary edema and abdominal bruit on physical exam.
>For pts with renovascular HTN, imaging evaluation with MR angiogram, CT angiogram
or Doppler U/S shd be performed.

5.Hyponatremia:
>Characterized by an excess of water in relation to the total body sodium conc.
>Caused by;
=>Excessive free water intake ( primary polydipsia )
=>Adrenal insufficiency and hypothyroidism.
=>Impaired water excretion from advanced renal failure.

=>Excessive ADH release.

6.SIADH:
>Excessive ADH secretion can be due to decreased effective circulatory volume ( true
volume depletion secondary to vomiting and diarrhea, Congestive heart failure,
cirrhosis, or overuse of diuretics) and SIADH.
>SIADH can be seen in a variety of CNS disorders, pulmonary diseases, HIV infection ,
tumors (i.e small cell carcinoma of the lung), use of drugs such as carbamazepine,
cyclophosphamide, chlorpropamine and SSRIs and in post-operative pts.
>Labs in SIADH;
=>Decreased serum osmolarity
=>Elevated Urinary osmolality
=>Urinary Na > 40 mEq/L
=>Normal Renal, Adrenal, and Thyroid function tests.

7.Pseudo hyponatremia can occur secondary to hyperlipidemia or hyperprotenmia from


any cause.

8.True volume depletion due to GI or renal causes would lead to Urine Na < 20 mEq/L

9.Primary polydipsia=> Dilute urine with urine osmolality of <100 mOsm/Kg.

10.Hypocalcemia can occur in pts undergoing major surgery and requiring extensive
transfusions.Hyperactive deep tendon reflexes may be the initial manifestation.
Muscle cramps and rarely, convulsions can also occur.

11.Hypomagnesemia may mimic hypocalcemia but is associated with alcoholism,


prolonged NG suction, diarrhea and diuretics abuse.

12.Mild hypermagnesemia results in decreased deep tendon reflexes.A severe form


causes loss of the deep tendon reflexes and muscle paralysis, thereby leading to
flaccid quadriplegia, decreased respiration and eventual apnea.

13.Hyperkalemia can occur in severe burns, crush injuries and renal insufficiency.

14. Hyperkalemia can cause nausea and vomiting.

15.Stress incontinence:
>The history, physical examination and clinical stress testing are usually sufficient to
diagnose stress urinary incontinence.In a clinical stress test, the pt is advised to relax
and give a single vigorous cough.Leakage that occurs instantaneously with coughing
is virtually diagnostic of stress urinary incontinence.
>Routine urodynamic studies not recommended unless surgical intervention is planned

16.Post-void vol. >200 cc on bladder U/S suggests the presence of detrusor ms. weaknes
or obstruction which can lead to overflow incontinence.

17.Urinary incontinence can occur with UTIs, atrophic vaginitis,stool impaction, and use
of certain drugs.

18.Analgesic-induced nephropathy:
Can present with florid nephrotic range proteinuria.This often results from NSAIDs
medications which cause a reversible decline in renal blood flow and glomerular filtratn
rate due to inhibiton of vasodilatory prostaglandin production.

19.Multiple myeloma induced proteinuria would not be detected by urine analysis.

20.Drug induced lupus does not cause renal failure.

21.Uncontrolled HTN can cause a progressive decline in GFR and chronic renal failure
but usually takes several layers.The kidneys are typically small.

22.Post-streptococcal GN would present as nephritic syndrome with RBCs and RBC casts

23.Major Abdominal or Thoracic surgery is commonly associated with hypersecretion of


ADH which is probably mediated by pain afferents.Other factors that may contribute
to the development of hyponatremia in the post-operative pts are nausea, hypotensn
and excessive use of hypotonic IV fluids.
The resulting hyponatremia can manifest as nausea and malaise as the earliest
symptoms, followed by headache, lethargy, obtundtion, seizures, coma & respiratory
arrest if sodium level drops below 115-120 mEq/L.
Hyponatremia causing seizures or other severe neurologic abnormalities shd be
initially treated aggressively with hypertonic saline (3% saline).Lower the serum
sodium at a rate of 1.5-2 mEq/L or 12 mEq/24 Hrs to avoid osmotic demyeilination

24.Female urethral diverticula:


>Can be congenital or acquired.
>Aquired diverticula are typically secondary to maternal birth trauma or instrumentatn
of the urethral tract.
>The cndition often presents with postvoid dribbling, followed by dysuria & dyspareunia
(three Ds).Pain is esp common if the diverticulum becomes infected and distended wth
pus.Hematuria, recurrent urinary tract infections and stress urinary incontinence are
also associated with urethral diverticula.
>Careful pelvic examination reveals anterior vaginal wall fullness.Compression of this
fullness may result in the discharge of urine, blood or pus from the urethra.
>Urethroscopy may be used to diagnose a urethral diverticulum but is invasive and may
have difficulty in visualizing the contents of diverticulum such as calculus or neoplasm.
Transvaginal ultrasound and MRI are both noninvasive imaging modalities that are
sensitive for the detection of urethral diverticula in the appropriate context.U/S is
preferred becx its less expensive.

25.Endometriosis:
Occurs when endometrial glands exist outside of the endometrial cavity and uterine
musculature.They present most frequently with dysmenorrhea, dyspareunia ( when the
endometriomas are located in the cul-de-sac, the fornices or the uterosacral ligament ),
dyschezia ( pain on defecation ), hematoschezia, hematuria and premenstrual or postmenstrual spotting.Endometriois can also result in infertility.In fact, 30-40 % of infertile
couples are diagnosed with this condition.The physical exam typically reveals a tender
adnexal mass and firm nodularity located in the broad ligaments, the uterosacral
ligament or in the cul-de-sac.

26.Posterior urethral valves is the most common cause of bilateral hydronephrosis in


boys.

27.Instillation of methylene blue into the bladder via catheter is of use in diagnosing
urinary incontinence secondary to vesicovaginal fistula.The test is positive if the
vaginally palced tampon is dyed blue.

28.With Q-tip test , a cotton swab is placed inside the urethra to measure the angle of
excursion when the pt strains.This test measures urethral hypermobility and is of
benefit when evaluating for urinary stress incontinence.

29.Acute urinary obstruction:


>Any case of acute urinary obstruction in which there is no evidence of pelvic or urethral
trauma or no history of urethral strictures, must be managed immediatly thru urethral
catheterization in order to decompress the bladder, alleviate the pain and avoid furthr
renal compromise.
>Suprapubic catheterization shd never be done on an empty bladder ( inc risk of bowel
perforation ) and is only needed when there is evidence of pelvic or urethral trauma
or urethral strictures or if the Foley catheterization is unsuccessful.

30.PSA can be raised in urinary retention,catheterization procedure, as well as any


vigorous rectal examination.

31.Hypertension in Kidney transplant pts:


~90-95% of Renal transplant pts would have HTN.
>Transplant renal artery stenosis;
=>Causes HTN in 10-12% of pts usually within first 2 years after the kidney transplant.
Improper surgical anastomosis is an important cause.It can also occur in acute
rejection and CMV infection.
=>Renal hypoperfusion due to renal artery stenosis stimulates the renin-angiotensinaldosterone system that in turn causes hypoperfusion and maintains the glomerular
capillary pressure and GFR.
=>you suspect renal artery stenosis as to be the cause of HTN in a kidney recipient
when you give ACEIs to the pt and the creatinine jumps.Actually in pts with bilateral
renal artery stenosis or renal artery stenosis in a solitary kidney ( e.g pts with
transplant ), administration of ACEIs lowers the angiotensin-II levels acutely, leading
to a significant decrease in GFR (by >30%) and acute kidney injury.
=>Renal artery stenosis shd also be suspected in transplant pts with resistant HTN,
flash pulmonary edema, or progressive loss of renal function.
>Transplant renal dysfunction(due to rejection/calcineurin inhibitors toxicity/recurrent
glomerular disease/obstruction/thrombotic microangiopathy);
Due to inability to excrete sodium and water.
>Corticosteroids used in transplant pts;
Stimulates sodium and water retention (inc preload and cardiac output)
>Acute and chronic Calineurin inhibitors toxicity; ( calcineurin/tacrolimus)
=>Increasing vasoconstrictor endothelin, inhibiting nitric oxide activity and increasing
sympathetic outflow.
=>Acute toxicity is detected by high Cyclosporine levels.
=>Chronic toxicity is indicated by rise of Creatinine over time

32.Stress incontinence: ( Urethral hypermobility )


>Occurs due to loss of vaginal support due to pelvic floor trauma during childbirth in
premenopausal women.
Pelvic floor exercises shd always be the first line of treatment in pts with stress urinary
incontinence.
Amitriptyline/Imipramine have alpha-agonist properties and can sometimes be used
when pelvic floor exercises do not result in significant improvement.
Surgical treatment is the last resort.
>Post-menopausal women can have stress incontinence due to hypoestrogenism that
leads to vaginal atrophy.
Local estrogen creams improves the stress incontinence remarkably in such pts.

33.Urge incontinence;
=>Responds to biofeedback
=>Anticholinergics such oxybutynin.

34.Pregnancy testing is imperative prior to UTI treatment in any sexually active female
of reproductive age, even if they claim to be using contraception becx pregnant pts
require treatments that are safe for a developing fetus ( amoxicillin/cephalexin/
nitrofurantoin)

35.Sexaully active females younger than 25 years of age are recommended to have
routine screening for gonorrhea and chlamydia yearly.

36.Nitrofurantoin;
=>Little bacterial resistance even though it has been used over the last 50 years
=>Low cure rate and more side effects than TMP-SMX and fluoroquinlones in general
=>Contraindicated at term, during labor and delivery or when the onset of labor is
imminent.

37.Gentamicin has the potential to cause total irreversible congenital deafness in fetus.

38.UTI in children:
>Most uncircumscribed boys do not develop UTI but they are at inc risk compared
to circumscribed boys.
>All children aged <24 months with a first-time febrile UTI shd be treated >7 days of
antibiotics and followed closely for any recurrence.Renal and bladder ultrasounds
shd be performed to evaluate for any anatomic abnormalities that may predispose
the child to UTI.
>VCUG is performed when;
=>There are abnormal findings on U/S ( e.g hydronephrosis,renal scarring,reflux,
obstruction)
=>recurrent UTIs
>There is no need to do urine culture if the child is improving.

39.Renal scintigraphy with DMSA ( Dimercaptosuccinic acid ) is utilized to detect renal


scarring in pts with recurrent UTIs.

40.Alcoholic ketoacidosis:
>Alcoholic ketoacidosis is characterized by impaired mentation,anion gap acidosis,

increased osmolal gap,ketonemia or ketonuria and variable blood glucose levels ( can
be high, low or normal ). High plasma glucose levels are speculated to be due to
impaired insulin secretion combined with increased insulin resistance.
NOTE: Blood glucose levels are generally higher than 250 mg/dL in pts with DKA.
>Pts need to be hospitalized for treatment;
=>Almost all alcoholics are likely to be thiamine deficient unless proven otherwise.
=>Most pts with alcoholic ketoacidosis will respond to an administratn of intravenous
dextrose containing normal saline and thiamine.Insulin is generally not required.
Dextrose leads to an increase in Insulin secretion which leads to the metabolism of
Ketone bodies to bicarbonate.

41.Glucose shd not be given in alcoholics before thiamine, otherwise it will increase the
utilization of thiamin,thus aggravating thiamine deficiency and leading to wernickes
encephalopathy.

42.Renal masses;
If the clinical suspicion for malignancy is low, U/S can be performed initially to
differentiate simple renal cysts from a solid mass.If a solid tumor is discovered
on ultrasonography, CT scan is required for staging.In cases with a high clinical
suspicion for renal malignancy, directly proceeding to CT scan of the abdomen
may be performed to evaluate the renal mass.

43.Renal cell carcinoma:


Smoking is a major risk factor for renal cell carcinoma.Pts present with flank pain,a
palpable mass and non-glomerular hematuria.Paraneoplastic syndrome i.e Polycythemia
is common due to excess erythropoietin secreted by hypernephroma and can lead to

formation of red cell mass.Systemic symptoms such as fever (usually intermittent) is


frequently accompanied by night sweats,anorexia,weight loss and fatigue.
The initial evaluation includes renal imaging ( ultrasound,CT and/or MRI ) and urine
dipstick/microscopic examination.
BOTTOM LINE: Pts with a renal mass, polycythemia,flank pain and a history of chronic
smoking are likely to have renal cell carcinoma.
Surgical management is the only chance of cure for pts with renal cell carcinoma;
=>If the renal mass is confined within the renal capsule ( stage-I ), partial nephrectomy
can be offered.
=>If the process extends through the renal capsule but not beyond Gerotas fascia
( stage-II ), radical nephrectomy is the best treatment option.

44.Ureterolithiasis:
Pts with obstructing ureterolithiasis associated with infection, acute kidney injury or
severe pain that has failed initial measures require decompression of the upper urinary
tract with percutaneous nephrostomy or retrograde ureteral stent placement .
Percutaneous nephrostomy is preferred in pts who are clinically unstable or have very
large stones.

45.Alpha blockers such as tramsulosin have been shown to facilitate stone passage in
stable pts with relatively small (<10 mm) ureteral stones.

46.Shock wave lithotripsy is used in the management of uncomplicated proximal


ureteral stones.

47.Enuresis:
>Urinary incontinence in children aged >5 years of age
>The first step would be to do urinalysis to exclude secondary causes such as UTI or
diabetes insipidus.A normal urinalysis is reassuring that the diagnosis is most likely
primary nocturnal enuresis.
>The first step in management of primary nocturnal enuresis would include behavioural
modifications and motivational therapy (e.g reward system) for 3-6 months .
Behavioural modifications include avoiding sugary/caffeinated beverages,voiding
regularly during the day & immediately before bedtime,drinking ample fluids in the
morning and minimizing fluid intake before bedtime.
>For children with ongoing bedwetting despite initial measures, enuresis alarms would
provide the best long term outcomes and low risk of relapse.
>Desmopressin can provide immediate improvement but has a high rate of relapse.

48.Contrast-Nephropathy:
>Likely to occur with iodinated contrast.
>Contrast induced nephropathy refers to AKI seen within 2-3 days of contrast exposure
and lasts about 5-7 days.
>Impaired renal function ( estimated GFR <40 mL/min/1.73 m2 or serum creatinine >1.5
mg/dL ) and volume depletion are important risk factors.
>Contrast induced renal vasoconstriction is thought to cause contrst inducd nephropthy
>Intravenous volume expansion with isotonic solution ( sodium chloride or sodium
bicarbonate ) before and after contrast exposure to prevent renal hypoperfusion is the
only intervention that has been shown consistently to reduce the risk of contrast
induced acute kidney injury.
>N-Acetylcysteine has also been used to prevent contrast induced nephropathy but has

not been shown to be superior to intravenous saline or sodium bicarbonate.


>Diuretics are held around the time of contrast study to avoid volume depletion that
increases the risk of contrast nephropathy.
>Metformin shd also be held to avoid the risk of metformin-assoicated lactic acidosis.

49.Autosomal dominant polycystic kidney disease:


>Most pts have progressive decline in renal function, the risk factors for which are
severe HTN, male sex, early age of diagnosis and early development of renal
dysfunction.So,thats why its important to keep the blood pressure under control.ACEIs
are the drugs of choice for HTN in ADPKD pts.The ideal goal for blood pressure in any
pt with CKD, including ADPKD , is 130/80 mmHg.
>~10% of pts have intracranial berry aneursyms( not cysts), regular screening for which
is not recommended except in pts with a positive family hx of subarachnoid hemrhage.
Besides, the blood pressure control would also be helpful in preventing the rupture of
berry aneurysms and subsequent intracranial bleeding.
>HTN in ADPKD pts is cause by compression of intrarenal vasculature, leading to
ischemia and hence leading to activation of renin-angiotensin system.
>The most common extrarenal manifestation is hepatic cysts.
>A higher incidence of colonic diverticula is seen ESRD pts undergoing dialysis.The
chances of rupture of these diverticula is also higher than normal population with
colonic diverticula.
>Ultrasonography is the procedure of choice for screening the asymptomatic family
members of a patient with ADPKD.
>Genetic linkage analysis for screening ADPKD is reserved for cases where U/S is -ve
and the need for definitive diagnosis is critical ( e.g screening an individual who is a
potential renal transplant donor )

50.Symptomatic Ureteral Stones:


>Stones <5 mm usually pass spontaneously.
>Stones <10 mm can be given a trial of medical therapy;
=>IV hydration
=>Pain control
=>Strain Urine
=>Alpha blockers ( e.g tramsulosin ) for 4 weeks.
Imaging may be repeated if the stone has not passed.
>If the stone is >10 mm or if there is persistent uncontrolled pain,acute renal failure or
signs of urosepsis , then urgent urologic referral is required for stone removal.
>IV antibiotics are indicated if there are signs of infection ( e.g fever, costovertebral
angle tenderness, pyuria )

51.IgA nephropathy:
Gross hematuria after 3-5 days of acute upper respiratory tract infection is a common
presentation.Characterized by IgA deposition in the renal glomerulus.Flank pain can
occur and this is possibly caused by stretching of the renal capsule.Dysmorphic RBCs
in the urine.Complement levels are normal.
Many pts have a benign course , however some male pts with HTN and urinary protein
>1 g per 24 Hrs are likely to progress rapidly.There is no definitive treatment.ACE
inhibitors and fish oil treatment have been tried.

52.Acute poststreptococcal infection:


Gap of more than 10 days between throat infection and onset of renal dx.Gross
hematuria is uncommon.Pts are typically hypertensive.urine sediment reveals RBCs
and RBC casts.Complement levels are decreased.

53.IgA nephropathy and thin membrane dx have certain similarities which may
sometimes make it extremely difficult to differentiate.For instance, these two dxs
have a benign course with virtually no progression of the renal dx.Thin membrane dx
also presents with episodic hematuria, although most episodes of hematuria in thin
membrane dx are unrelated to an upper respiratory tract infection.The best way to
differentiate is by performing a renal biopsy.

54.Gross hematuria:
The first step in evaluating hematuria is a urinalysis and urine culture.Urinalysis confirms
the presence of hematuria and can differentiate between infectious , glomerular and
extra glomerular causes.

55.Rhabdomyolysis: (RM)
In this condition, creatinine kinase (CK) levels are usually greater than 10,000 U/L.
Pts develop muscle tenderness, increased muscle tone, renal failure, hyperkalemia,
hyperphosphatemia, hypocalcemia.The latter three characteristics are due to muscle
breakdown.Cocaine or alcohol intoxication , as well as severe trauma or extreme
exertion can lead to rhabdomyolysis.Neuroleptic malignant syndrome, hypothermia,
hypothyroidism and polymyositis have also been associated with RM.
Management involves immediate adequate Intravenous isotonic saline solutions to
ensure adequate hydration, followed by alkalinization of urine.

56.Orthostatic proteinuria:
Orthostatic proteinuria means higher than normal protein excretion during the day
(when upright) but normal protein excretion at night (when supine).Orthostatic
proteinuria is the most common cause of proteinuria in adolescents (60-75%)

prevalence and rarely occurs after age 30.Possible mechanisms of proteinuria include an
exaggerated normal response to upright posture , increased glomerulr capillary pressure
due to an exaggerated angiotensin-II response with standing , subtle glomerular
abnormality (e.g focal mesangial hypercellularity and/or basement membrane thickng)
or left renal vein entrapement.
Diagnosis can be confirmed by comparing the urine protein-to-creatinine ratio in urine
samples collected in both the supine and standing positions.Another confirmatory test
is to compare protein excretion in a split 24 Hour urine collection divided between the
daytime (after morning void until bedtime) and nightime periods.Orthostatc proteinuria
is confirmed in pts with a significant elevation in daytime protein but a normal nighttime
rate.
Orthostatic proteinuria is a benign finding and requires no invasive diagnostic workup
(e.g kidney biopsy) or treatment (e.g angiotensin converting enzymye inhibitors or oral
steroids).It has an excellent prognosis and does not predict the development of
significant glomerular pathology in the future.The pt shd be reassured about the benign
nature of dx and tendency to resolve with age.The pt shd be monitored periodically and
have orthostatic proteinuria reconfirmed if pt has persistent proteinuria.

57.Positive antinuclear antibody screen and low serum complements ( C3 and C4 )


suggests lupus nephritis in pts with acute glomerulonephritis.

58.First degree relatives of children with nephrotic syndrome who do not respond to
prednisone shd undergo urine protein testing to diagnose rare genetic mutations that
affect podocyte function.

59.Ileus presents as abdominal distension due to gaseous distension of small and large

bowels.The abdominal examination shows as decreased bowel sounds. Ileus cn occur


after surgery, due to certain medcations & due to metabolic causes (e.g hypokalemia)

60.Diuretic indced hypokalemia z a common compliction of antihypertensive/congestive


heart failure therapy, esp with loop diuretics.

61.Dysfunctional bowel,as in paralytic ileus and cardiac and neurologic complications r


indications for prompt parenteral potassium replacement.

62.Opioids most commonly cause constipation.Methylnatrexone selectively blocks mu


opioid receptors on the gut mucosa w/o reversing the analgesic effect of opioids.It
is used to reverse opioid induced constipation.

63.Npehrolithiasis:
>Shd be strngly suspected in any pt with atraumatic flank pain, particularly if abdominal
tenderness is absent and urinalysis shows hematuria.
>Non-contrast helical CT scan is the gold standard for diagnosis followed by an IVP.
>U/S would miss small stones as well as ureteral stones, so dont go for that unless and
until CT is contraindicated e.g pregnant pts or in individuals where you want to avoid
radiation exposure.
>X rays miss calcium-less stones whch is the case 20% f stones which dont have calcium
,so thats again not an option.

64.Gross hematuria:
>Gross hematuria in pts with risk factors for urinary tract malignancy (e.g smoking,hx of
pelvic radiation or exposure to aniline dyes) and aged > 40 needs evaluation of upper &

lower urinary tract.


>The upper urinary tract is best visualized with contrast enhanced CT scan but this test
is relatively contraindicated in pts with underlying chronic kidney disease ( e.g raised
BUN/Cr. ). Alternatives would be renal U/S and MRI in such cases.
>The lower urinary tract is best visualized with cystoscopy .Urine cytology is an alterntve
for low risk pts.

65.Cola-colored urine following exercise:


>The D/Ds include exercise induced hematuria, myoglobinuria from rhabdomyolysis &
march hemoglobinuria from RBC trauma.All these entities would give positive result for
blood on urinalysis.
>Idenifying intact RBCs on microscopy would exclude myoglobinuria & hemoglobinuria,
leaving us with diagnosis of exercise-induced hematuria, which occurs in utpo 24% of
marathon running participants.
>Exercise indced hematuria in non-contact sports can occur due to repetitve up & down
movemnts of the blader during running.In contact sports, direct trauma to the kidneys
or bladder may result in hematuria.Shunting of blood flow towards the muscle & away
from the kidneys may also contribute.
>The diagnosis shd be considered one of exclusion even with appropriate history and
follow-up urinalysis is needed in 1 week to ensure that hematuria has resolved.

66.Post-operative urinary retention:


>Is a potential complication of surgery and anesthesia.A precipitating event ( e.g general
or epidural anesthesia ) can lead to inefficient detrusor muscle activity & acute urinary
retention.Post-operative bladder distension is also caused by aggressive intravenous
volume replacement in hypotensive pts.

>Pts present with abdominal discomfort and possible distension and tenderness to
palpation.
>Post-operative oliguria/anuria required immediate assessment with initial portable
bladder scan to assess bladder volume. Pts with significant urinary retention require
urinary catheterization to restore normal urine output and resolve or prevent hydronephrosis , tubular atrophy and renal injury.If a large amount of urine is drained from
the bladder , the catheter may be kept in place for one day, followed by voiding trials.

67.Intraabdominal hemorrhage:
Intraabdominal hemorrhage would most likely be caused by bleeding from the surgical
site.Pts may present with hypotension and/or tachycardia if there was significant blood
loss.The initial presentation would be pain and nausea , progresing to abdminal guardng
,rebound tenderness and rigidity as the amount of blood in the peritoneum increases.

68.Normal findings after surgery:


=>Not passing flatus for Hrs ( not passing urine shd make u suspect urinary retention)
=>Minimal tympany over the liver after laproscopy.

69.Hypercalcemia is not associated with specific physical findings.There may be anxiety,


depression, mild muscular weakness, constipation and peptic ulcer disease.

70.Hypocalcemic pts present with Chvosteks sign or trousseaus sign, hyperpigmentatn,


seizures, muscular weakness or hypotension.

71.Tinels sign is +ve in Carpal tunnel syndrome where repetitive tapping of the palmar
aspect of wrist with examiners thumb wud produce/inc paresthesias in median

nerves area.

71.Hyperpigmentation;
->renal dx
->liver dx
->Chronic hypocalcemia
->Addisons disease
->Some cases of hyperthyroidism

72.Kidney donation:
>No inc risk of end stage renal disease.
>increased risk of gestational complications like fetal loss, gestational diabetes,
preeclampsia and gestational HTN but overall rate is similar to general population.
>lower risk of depression than general population.

73.Calcium gluconate or calcium chloride is the first line treatment for hyperkalemia if
severe ECG manifestations ( i.e ECG changes more severe than tall peaked T waves
like prolongation of PR interval & QRS complex with loss of P-waves or eventual sine
wave pattern ) are present.Beta agonists or a combination of insulin and glucose are
then typically given to reduce serum potassium by driving potassium intracellularly.

74.Sinus bradycardia can occur with hyperkalemia.

75.Recurrent UTIs in females:


>Antibiotic prophylaxis can be considered in young female pts who have had more than
two UTIs in 6 months or more than three UTIs in a year.Prophylaxis can be used either
continuously for upto several years or solely after intercourse in women whose UTI
occurs only after sexual activity.
>Fluoroquinolones, TMP-SMX and Nitrofurantoin are the most commonly used agents.
>Further diagnostic evaluation is typically unrevealing unless there is clinical evidence
to suggest the presence of nephrolithiasis or obstruction.

76.Potassium citrate is used to alkalinize the urine and decreases the risk of stone
formation.

77.Raised BUN and Cr. ? => it cant be unilateral obstruction

78.Analgesic nephropathy:
It is seen in pts with heavy , long term use of salicylates ( aspirin ) and acetaminophen
with phenacetin or other NSAIDs.A cumulative amount of these analgesics causes
chronic tubular injury and tubulointerstitial nephritis that leads to the loss of renal fnctn
Ischemic damage to the renal papilla may occur due to vascoconstriction of the vasa
recta.Chronic necrosis of the renal papilla with calcification is typically seen.However,
sloughing of the papilla may cause hematuria and present acutely as renal colic.Urine
studies usually show pyuria, hematuria, proteinuria, and WBC casts whereas the urine
culture is typically sterile.Discontinuation of the analgesic will cause stabilization or even
improvement in renal function.
BOTTOM LINE:Papillary necrosis occurs with heavy long-term use of analgesics such as
aspirin, phenacetin, acetaminophen or other NSAIDs.Nephropathy is

usually chronic but may prznt acutely wth hematuria, pyuria,proteinuria


and renal colic.Discontinuation of the analgesc usually rzlts in stalization
or improvement in renal function.

79.UTI in men:
Urinary tract infections in men usually represent prostatitis as opposed to cystitis,
particularly if there are symptoms of urinary obstruction.????
Acutally pts with underlying BPH are predisposed to prostatitis i.e inflammatory
swelling of prostate.This would lead to the signs of obstruction ( abdominal discomfort,
palpable bladder and inability to void ) as well as manifest with signs of UTI i.e fever and
dysuria. A clue of prostatic tenderness might be there.
Avoid doing foley catheterization and prostatic massage , as they can increase the risk
of bacteremia.
Go straight to Suprapubic catheterization to relieve the obstruction and meanwhile send
the samples for antibiotic sensitivities.
Fluoroquinolones OR TMP-SMX would be good choices.

80.In pts with HTN and Chronic kidney disease, Urinary protein excretion shd be checkd
,as treatment with ACEIs/ARBs would help to reduce the proteinuria and hence the
progression of CKD if protein level is in excess of 500-1000 mg/day.The goal for B.P
control shd be 130/80 mmHg.
Secondary treatment with a diuretic in addition to the ACE inhibitor or ARB may be
necessary for some pts to meet these goals.
A non-dihydropyridine CCB is the typical choice if a third drug is required becx of its
anti-protenuric properties.

81.SLE renal disease:


>A kidney biopsy is indicated to guide the treatment is all SLE pts with renal involvement
>Type-I and Type-II;
No treatment
>Type-III and Type-IV; (Type-IV is the most frequent and severest form)
Immunosuppression
>Type-V;
Requires treatment when proliferative lesions are superimposed.
>Corticosteroids are the mainstay treatment.IV methylprednisone is usually used.
>Immunosupressive agents such as cyclophosphamide may be needed when the
response to steroids is inadequate to improve renal disease or when the disease is
aggressive.
>Renal disease in SLE is due to immune complex mediated glomerular injury.These
immune complexes are primarily composed of anti-dsDNA antibodies, and are
deposited in the mesangium or subendothelial space , where they are fixed with the
resultant influx of neutrophils and mononuclear cells.
>Complement deficiency is not the initial event and results from complement activatn
by immune complexes deposited in the glomerulus.
>Complement levels and Anti-dsDNA are used to monitor the disease activity in SLE pts
>ANA and anti-Sm antibody titers are not used for monitoring the disease activity

82.Calcium homeostasis:
>Calcium homeostasis involves blood transport as albumin bound calcium ( 45 % ),
ionized calcium ( 40% ) or calcium bound to inorganic or organic anions ( 15% ).
>The serum calcium concentration decreases by 0.8 mg/dL for every 1 g/dL drop in
serum albumin concentration.

Corrected calcium=(Measured total Calcium) + 0.8(4 g/dL + measured albumin in g/dL)


>Direct measurement of ionzed ( physiologically active calcium ) is usually mre accurate
in pts with acid base disorders ( acidosis increases ionized calcium , alkalosis decreases
ionized calcium ) , primary hyperparathyroidism, chronic kidney dx, multiple myeloma
(elevated proteins can increase corrected calcium).

83.Enuresis is more common in boys than girls.

84.Causes of secondary enuresis;


>Psychological stress
>Urinary tract infection
>Diabetes mellitus
>Diabetes insipidus
>Obstructive sleep apnea

85.Euvolemic hyposmolar hyponatremia:


Can occur due to SIADH, hypothyroidism or adrenal insufficiency.If the pt is asymptomtc
or the hyponatremia is mild , treatment is first directed at correcting the underlying
disorder ( e.g using corticosteroids or levothyroxine ).Moderate hyponatremia & SIADH
respond to water restriction.Symptomatic pts and those with severe hyponatremia may
need sodium replacement using hypertonic saline solution.

86.Pts with SIADH and mild or moderate hyponatremia can be managed with water
resitriction.

87.Demeclocycline is helpful in some chronic persistent cases of SIADH or anti-psyschotc

induced hyponatremia.

88.Kidney transplantation:
>Age < 18 is an absolute contraindication to organ donation.
>Diabetes and HTN with end organ damage (e.g left ventricular hypertrophy) are
relative contraindications.
>Rhesus matching is not important in kidney transplant as the antigen is not expressed
on kidney cell surfaces.
>Advances in immunosuppressive therapy have expanded the number of possibly
suitable kidney donors.

89.Pts with adult polycystic kidney disease are predisposed to following extrarenal
manifestations;
=>Hepatic, pancreatic, splenic and pulmonary cysts
=>Cerebral aneurysms
=>Aortic aneurysms
=>Colonic diverticula
=>Mitral valve prolapse
=>Inguinal and abdominal hernias
Before considering peritoneal dialysis as a treatment option in pts with adult polycystic
kidney disease, it z imprtant to rule out diverticulosis which can potentially complicate
peritoneal dialysis.This can be done by performing a total colonscopy.

90.Renal colic in pregnancy:


>U/S of kidneys and pelvis is recommended to evaluate renal colic in pregnant pts.
Ultrasonography is also useful for detecting secondary signs of obstruction ( e.g

hydronephrosis, hydroureter ) , though physiologic hydronephrosis in pregnancy


must be distinguished from pathological hydronephrosis secondary to obstruction.
>If renal and pelvic U/S is non-revealing, go for transvaginal U/S.
>If transvaginal U/S is non-revealing, either treat empirically and observe or go for
Magnetic resonance urogram where the diagnosis is critical.
>Low dose CT urography can be considered only in 2nd and 3rd trimesters.

91.Renovascular HTN:
Renovascular HTN is the most common cause of secondary HTN and shd be suspected
in pts with resistant HTN and signs of diffuse atherosclerosis.Such pts shd be evaluated
with non-invasive testing with renal duplex Doppler ultrasonography or CT or MR
angiography.Due to the risk of contrast induced nephropathy and nephrogenic systemic
fibrosis when performing CT and gadolinium MR angiography, these shd be avoided in
pts with renal insufficiency.

92. FOR GOD SAKE, MR. FUCKING DR.WAQASSSSSSSSSSSSSSS:


BUN is fucking raised and Cr. is raised=> fucking think of Chronic kidney dx and decide
your fucking decision accordingly. Fuck u,
every fucking time, u make it wrong asshole

PULMONARY & CRITICAL CARE


1.Even though its not common, a new pleural effusion in a pt already on anti-TB
treatment may occur and must be studied with thoracentesis , becx this can progress
despite the clinical improvement of the pt.There is no need to change therapy, unless
there is proof of a new infection or drug resistance.Some authors advocate the use of
steroids in this setting, but this is not a universally accepted practice.

2.Asthma:
>Chronic inflammatory airway disorder with increased airway responsiveness and
reversible or partly reversible airway obstruction.
>The characteristic clinical picture is recurrent episodes of wheezing, dypnea, cough
and chest tightness, esp in the night time and early morning hours.In between such
episodes , pts may be completely normal or have only minimal symptoms.
>Asymptomatic pts with asthma may have normal pulmonary function tests at the time
of their office visit.In such pts, methacholine challenge test would reveal a fall in FEV1
of greater than 20% from the initial baseline value.
>Pulmonary function tests in pts with airflow limitation due to asthma typically reveal;
=>Obstructive pattern with reduced FEV1/FVC
=>Normal or elevated TLC
=>Normal or elevated DLCO

3.Tension pneumothorax:
>Small one only need observation.
>Large ones can progress to hypotension and hypoxia , so it needs chest tube placement
>Both CT scans and CXR can be used, but CXR is preferred initial test.

>Supine CXR needs about 500 cc gas before pneumothorax can be idenitifed while
upright CXR only needs 50 cc gas , So , Upright radiograph is preferred.
>Both inspiratory and expiratory upright CXRs can be employed, but expiratory has
limitations, so inspiratory is preferred.
>Lateral decubitus CXRs can identify pneumothorax with as little ass 5 cc gas, but
clinicians still prefer upright CXRs.
BOTTOM LINE: Upright Chest radiography ( posteroanterior view ) is the initial test of
choice to confirm the diagnosis of pneumothorax.

4.Septic shock:
>The most important initial step in the management of septic shock is aggressive fluid
resuscitation to a central venous pressure of 8-12 mmHg.Vasopressors such as norepinephrine or dopamine shd be added if the pts hypotension is poorly responsive
to fluid resuscitation.
>A serum lactate measurement may be helpful to determine the sevrity of hypoperfusn
>Pts on longstanding corticosteroids often have suppression of the pituitary-adrenal
axis secondary to negative feedback.Hypotension may be exacerbated in these pts,
should they develop septic shock unless stress dose steroids are given during the acute
illness.Corticosteroids may even be necessary in pts who did not previously take
corticosteroids due to a relative adrenal insufficiency.

5.Stress induced cardiomyopathy, aka takotsubo cardiomyopathy, can occur in critical


illness.

6.Post-operative hypoxemia:
>Immediately following surgery;
Residual anesthetic effect:
Residual effects of inhaled anesthetic agents, opioids, benzodiazepines and paralytic
medications ( diaphragm paralysis ) can cause hypoxemia in post-operative pts.Pts
with comorbid obstructive sleep apnea or chronic obstructive pulmonary disease
are at increased risk.
>Immediately following surgery;
Airway obstruction/edema:
Often due to endotracheal intubation or pharyngeal laxity.Stridor is common.
>Typically early after surgery;
Bronchospasm:
Severe enough to cause hypoxemia would likely be associated with significant
wheezing and subjective dyspnea.
>1-5 days after surgery;
Pneumonia:
Ther will be fever and elevated white cell count and purulent secretions.
An infiltrate will be seen on chest X ray.
>2-5 days after surgery;
Atelectasis:
Primarily due to lack of deep breathing from surgical pain.It is often worsened by
retained respiratory secretions and is especially common after thoracic or abdominal
surgeries.
>Uncommon before 3 days;
Pulmonary embolism:
Presents wth chest pain & tachycardia.Shows little improvement to supplemental O2.

7.Asthma in pregnancy:
>Poorly contolled asthma during pregnancy is associated with an increased risk of
preterm births, low birth weight, preeclampsia and spontaneous abortions.
>During pregnancy, physiologic changes that increase respiratory drive cause a resting
alkalosis due to tachypnea.As the pt becomes fatigued, the respiratory rate begins to
decline and PaCO2 levels increase.PaCO2 >35 mmHg suggests developing respiratory
compromise in pregnant pts.
>The SaO2 levels shd be kept above 95% to maintain adequate fetal health.
>Management is similar to non-pregnant pts;
=>First try inhaled bronchodilators.
=>Go to systemic IV steroids in cases of worsening compromise.
=>In case of respiratory failure intubate the pt for mechanical ventilation.

8.Recurrent Pneumonias:
>Recurrent pneumonias in elderly smoker may be the first manifestation of
bronchogenic carcinoma which actually causes these recurrent pneumonias through
endobronchial obstruction.
>A carcinoid is another common cause of endobronchial obstruction esp in younger
and non-smoking pts.
>Endobronchial lesions cause compression of the airway lumen and prevent adequate
clearance of secretions.This causes stasis of secretions and recurrent pneumonias ,
even after successful treatment of previous episodes.
>Other causes of non-resolving pneumonia or pulmonary infiltrates are bronchoalveolar
carcinoma, lymphoma, eosinophilic pneumonia, bronchiolitis obliterans organizing
pneumonia ( BOOP ) , systemic vasculitis, pulmonary alveolar proteinosis and drugs
( Amiodarone ).

>Flexible bronchoscopy is a primary diagnostic tool to evaluate pts with persistent or


non-resolving pneumonia or pulmonary infiltrates.It provides a confirmation of the
diagnosis and definitively diagnoses the cause of the recurrent pneumonia.Enables
the physician to perform biopsy of the lesion.Helps easy visualization and low risk of
complications. Also has a better yield for diagnosing certain other infections such as
fungal infections or tuberculosis.
>A high resolution CT scan is an alternative but does not provide the definite cause of
the lesions and recurrent pneumonia.
>The best diagnostic tool for endobronchial obstructive lesions is flexible bronchoscopy.
If the question asks for next best step in management, then the answer would be CT
scan.

9.Obstructive sleep apnea: (OSA)


>There is no indication for special tests in a snoring pt w/o associated symptoms and
clinical findings of OSA ( presence of upper airway structural abnormalities,daytime
hypersomnolence, nocturnal choking, gasping, apnea with oxygen desaturation and
hypertension ).
>Predisposing factors include obesity, genetic or acquired upper airway structural
abnormalities, hypertension, smoking and alcohol and sedative-hypnotic use.
>Obesity contributes through several mechanisms including increased peri-airway fat
deposition, airway collapsibility and the hormonal and metabolic changz accompanying
obesity.
>Smoking contributes by an as-yet-unknown mechanism
>Alcohol and sedative-hypnotics predispose a patient to ,as well as increase the severity
of OSA once acquired, by decreasing upper airway muscle activity and prolonging
apnea by inhibiting the post-apneic arousal response.

>Pts who snore but have no other signs or symptoms of OSA shd be advised to lose
weight, stop smoking and avoid alcohol near bedtime.

10.Pulmonary embolism:
>The initial treatment for hemodynamically stable pts with pulmonary embolism is
anticoagulation with low molecular weight heparin or unfractionated heparin.The
intravenous heparin must be favored in pts with renal dysfunction or if there is high
concern for bleeding risk as the drug has a relatively short half-life.
>Warfarin is used in the long term treatment of pulmonary embolism but shd not be
started acutely w/o heparin already on board secondary to a theoretical tendency
towards thrombosis when the medication is first initiated.Once the INR is in therapeutc
range , generally 2-3 , the heparin can be discontinued.
>Inferior vena cava filters shd be considered in pts for whom anticoagulation is contraindicated , who have had complications of anticoagulation or where anticoagulation hz
failed.
>Intravenous fibrinolytic therapy is reserved for pts with hemodynamic instability.
>Anti-platelet therapy with aspirin or clopidogrel is not utilized in the management of
pulmonary embolism.
>Low grade fever can be present in up to 14% of pts with pulmonary embolism .The
clinical picture can be confusing for the possible presence of pneumonia given that pts
may also have pulmonary opacities from infarction or atelectasis as well as an
elevated WBC count secondary to pulmonary embolism itself (stress response).

11.Septic emboli usually present as multiple small cavitating pulmonary nodules.Septic


emboli typically affect pts with endocarditis or a history of injection drug use.

12.SIADH is associated with normovolemic hyponatremia and can be caused by small


cell carcinoma of the lung.

13.Asbestosis occurs more often in construction and shipyard workers and generally
occurs approximately 20 years after the initial exposure to asbestosis.The condition is
characterized by dyspnea on exertion, cough, chest tightness & wheezing.Pulmonary
fibrosis is evident on CXR or chest CT.

14.Mesothelioma is linked to asbestos exposure.Mesothelioma if often characterized by


dyspnea and chest pain and CXR shows nodular thickening of the pleura and/or
obscuring of the diaphragm.

15.SIADH:
>The hyponatremia that arises in the SIADH is secondary to the combination of water
retention and the loss of sodium and potassium.In chronic SIADH, the loss of sodium is
more significant than the amount of water retained.
>Water retention (with the possible inclusion of salt administration) is the preferred
means of correcting mild to moderate hyponatremia associated with SIADH.
>Furosemide lowers the urine osmolality and blunts the renal response to ADH, thereby
increasing water excretion.Since loop diuretics decrease the NaCl reabsorption in the
loop of Henle, they shd be administered concomitantly with hypertonic saline or salt
tablets.
>Demeclocycline and lithium work to blunt the response of the collecting tubule cells to
ADH , ultimately increasing the excretion of water.These drugs are indicated in pts with
persistent severe hyponatremia when water restriction , salt intake or loop diuretics
fail to resolve the condition.Although it is potentially nephrotocxic, Demeclocycline is

typically preferred to lithium bex its more effective and less toxic.

16.Thiazides may induce hyponatremia, typically within a few weeks of starting therapy.
Thiazides cause the loss of both sodium and water and when water is better replaced
than the sodium, hyponatremia ensues.

17.The primary goal of medical management of a brain dead organ donor is to maintain
a euvolemic, normotensive and normothermic state.

18.Anticoagulation wth Heparin or a low molecular wt Heparin followed by oral warfarin


is the treatment of choice for a pt with pulmonary embolism.An IVC filter placement
is recommended in pts with complications of anti-coagulation, contraindications to
anti-coagulation , or failure of anti-coagulation.

19.COPD:
Hypoxemia in COPD is primarily caused by ventilation-perfusion (V/Q) mismatch ,
secondary to emphysema and chronic bronchitis.Emphysematous destruction of the
alveolar-capillary interface reduces the surface area available for gas exchange ,creating
areas of high ventilation relative to perfusion (high V/Q) .In addition, emphysema causes
airway obstruction due to loss of elastic tissue and small bronchiolar collapse.This, along
with the patchy airway obstruction caused by the chronic bronchitis, results in areas of
reduced ventilation relative to perfusion (low V/Q).In COPD exacerbation, this airflow
limitation is further worsened by mucous plugging and bronchospasm.
The alveoli in the low V/Q regions have lower alveolar partial pressure of oxygen
compared to normal or highly ventilated regions.Similarly, blood in adjacent capillaries
also has low partial pressure of oxygen compared to normal/highly ventilated areas.

Increasing oxygen delivery to the poorly ventilated alveoli improves alveolar partial
pressure of oxygen, thereby increasing arterial partial pressure of oxygen and total
arterial blood oxygen content.This is the mechanism how Oxygen supplementation
would increase the arterial SaO2 in COPD pts i.e by increasing the oxygen delivery to
low V/Q regions. Conversely, blood traversing high V/Q regions has normal arterial
partial pressure of oxygen, which cannot be further elevated by increasing oxygen
delivery.

20.Cyanotic heart disease wont respond to O2. why ?


Arteriovenous anatomic shunts in cyanotic heart diseases bypass normal pulmonary
capillary beds.Becx blood is never delivered to the alveoli , increasing their oxygen
content will not alter systemic oxygenation.

21.U see normal DLCO in COPD ?


it means the predominant component is chronic bronchitis.

22.U see dec DLCO in COPD ?


It means predominant component is emphysema ( destruction of alveolar-capillary
membrane )

23.Bohr effect;
Inc. PCO2-> Inc H+ and Dec Hb affinity for O2 -> Inc O2 unloading

24.Haldane effect;
Inc PO2 -> Inc CO2 and H+ unloading

25.Asthma:
>Severity of airflow limitation is assessed using peak expiratory airflow (PEF)
=>PEF of 40-69% of baseline is considered mild to moderate exacerbation.These pts
shd be started on inhaled short-acting beta agonists ( SABA ) for rapid correction of
bronchospasm.
=>Systemic steroids shd be administered early in the treatment of acute asthma
exacerbation.Becx their effect is usually not apparent in the firt few Hrs, SABA
shd be used concomitantly.
=>Pts with PEF <40% shd receive high dose SABA with ipratropium.
=>Parenteral magnesium sulfate is a bronchodilator.Its is indicated when the PEF is
is <40% despite >1 hour of intensive therapy with inhaled SABA and glucocorticoids.
It has not been shown to benefit pts with mild to moderate exacerbations.

>Major side effects of SABA are tachycardia and tremors.


>Tachycardia in asthma exacerbation is due to hypoxia , which stimulates carotid
chemoreceptors to increase catecholamines secretion from the adrenal medulla.
Correction of bronchospasm to improve ventilation and hypoxia is most effectively
achieved with SABA.Therefore, pre-existing tachycardia is not a contraindication to
SABA use.
>Intravenous theophylline is not used becx its not effective and can worsen
tachyarrhythmias.
>Clues to clinical deterioration include;
=>Accessory muscle use with suprasternal retractions
=>Cyanosis
=>Diaphoresis
=>Exhaustion or extreme fatigue

=>Marked tachycardia
=>Pulsus paradoxus
=>PaCO2 >42 mmHg
=>PEF <25% baseline.
These pts shd be assessed immediately for impending respiratory failure.Ventilatory
support shd be considered early in the course of the disease.Signs of impending
respiratory failure include :
=>Confusion and drowsiness
=>Depressed respiratory drive
=>Severe hypoxemia
Pts exhibiting any of these symptoms shd be intubated immediately to secure & protect
the airways.
Inhaled beta agonists,inhaled ipratropium , intravenous glucocorticoids and intravenous
magnesium shd also be given.

26.Pulmonary embolism:
>Sudden onset SOB + Hypoxia + Tachycardia=Pulmonary embolism
>Look for a hx of prolonged immobility
>PE causes myocardial strain leading to elevated levels of troponins & does not indicate
ischemia.
>Pts often have a normal chest x-ray or have mild, non specific changes such as pleural
effusion or mild, subsegmental atelectasis.
>If PE is unlikely, then D-dimers can be performed to rule it out ( i.e <500 ng/mL with
>500 ng/mL being +ve and indication for CT angiography)
>A CT pulmonary angiogram shd be peformed immediately to evaluate for likely PE.V/Q

scan may be considered as an alternate test if the pt is unable to receive IV contrast


agents e.g renal insufficiency, contrast allergy.While results are pending , pts with signs
of a clinically significant PE shd receive oxygen and anticoagulation ( in contrast to pts
with an uncomplicated deep venous thrombosis who may defer anticoagulation until
definitive diagnosis has been made ).
>Transthoracic echocardiogram to check for signs of right sided heart strain may be
performed at the bedside in pts with rapid hemodynamic deterioration and shock
who are too unstable to be transported for CT scan.
>Elevated troponins, hypotension & hemodynamic instability indicate a poor prognosis
>Lobar consolidation is indicative of pulmonary infarcts.
>Current guidelines recommend early initiation of warfarin ( e.g same day as parenteral
therapy is started ) over delayed initiation and continuation of parenteral anticoagulatn
for a minimum of 5 days and until the INR is >2.0 for atleast 24 Hrs.
>IVC filters shd be considered in pts with low cardiopulmonary resrve due to either prior
pulmonary emboli or preexisting cardiopulmonary disease.
>A small amount of hemoptysis is normal in pts with pulmonary embolism, particularly if
pulmonary infarction has occurred.The treatment regimen need not be altered if the pt
is otherwise stable.
>Massive hemoptysis defined as 600 mL over 24 Hrs may occur in pts with large PE and
pulmonary infarction who are receiving anticoagulation.Bronchoscopy or angiographic
embolization can be considered for extreme situations.

27.Bronchiolitis:
>An upper and lower respiratory tract infection caused by respiratory syncytial virus.
>Mild upper respiratory symptoms, low grade fever and wheezing and crackles are
typical.

>In healthy children, bronchiolitis is usually a self limited illness, however, young infants
are at increased risk of respiratory distress.
>Hypoxia pts or those with respiratory distress, apnea or dehydration shd be hospitalizd
in respiratory isolation to prvent nosocomial spread of the extremely contagious virus.
Therapy generally consists of supportive measures (e.g intravenous fluids, nasal bulb
suctioning, humidified oxygen).
>Dont use inhaled bronchodilators.
>Dont use steroids ( can be considered in underlying chronic lung dx and recurrent
wheezing ).
>Dont use ribavirin ( only reserved for severe immunocompromised pts ).
>Prevention is with Palivizumab in infants with the following conditions;
=>Preterm birth <29 weeks gestation
=>Chronic lung dx of prematurity
=>Hemodynamically significant congenital heart dx
>The diagnosis is primarily clinical through history and physical examination.
>Laboratory screening z made quickly by detection of RSV antigen in nasal or pulmonary
secretions.This is reserved for infants who receive palivizumab prophylaxis to detect
breakthrough respiratory syncytial virus infection.
>Chest x ray is unnecessary becx it would show atelectasis and results in overdiagnosis
of pneumonia and unnecessary antibiotic therapy.
>Serology is also not helpful in acute illness as it can take weeks for antibody titers to
rise.Serology is also unreliable in infants aged <6 months due to passively acquired
maternal antibodies.
>Infants aged <2 months are at greatest risk of apnea and respiratory failure.Secondary
bacterial infections are relatively uncommon but acute otitis media can be seen in 20%
of children.RSV is associated with the future development of recurrent wheezing and

reactive airway disease in up to 30% of pts.

28.ARDS:
>Development of pulmonary contusions secondary to chest wall trauma is an important
etiological factor of ARDS.
>ARDS is associated with a mortality rate of 35-40%.
>Among the different strategies, mechanical ventilation that delivers lower tidal
volumes ( <6 mL/kg ) and limits plateau pressure ( <30 cm H20 ) has been shown
to be relatively more efficient than other strategies such NO inhalation, use of
prostacyclins, exogenous surfactant and steroids.

29.Postoperative complications:
>Some factors that increase the risk of post-operative complications are
=>Abdominal and thoracic surgeries
=>Hx of smoking within last 8 weeks
=>Baseline PaCO2 >45 mmHg.
=>Duration of surgery of more than 3-4 Hrs.
=>Use of general anesthesia
>Pts shd be urged to quit or stop smoking atleast 8 weeks prior to the surgery.
>Pts with COPD/chronic pulmonary disease shd be treated aggressively to optimize
their pulmonary function tests prior to the surgery.
>Pts shd be educated regarding the use of lung expansion maneuvers such as deep
breathing exercises and incentive spirometry.The goal is to increase the lung volumes
and help prevent atelectasis.
>Routine use of preoperative antibiotics is not indicated in a pt with underlying stable
chronic lung disease.

30.Pulmoanry contusion:
Pulmonary contusion is the most common lung parenchymal injury seen in pts with
blunt chest trauma.The forces associated with blunt trauma are transmitted to the lung
parenchyma and results in contusion with hemorrhage into the lung tissue.The clinical
presentation depends upon the extent of injury.The pts usually present with varying
degrees of dyspnea, tachypnea, hypoxemia and hemoptysis
Physical examination reveal decreased breath sounds over the affected areas of the
lungs.
Chest radiography generally reveals homogenous opacification of the lung fields that do
not conform to a specific anatomic segment of the lung.
Pts with pulmonary contusion are prone to develop pulmonary edema with overly
aggressive fluid resuscitation.
The full clinical and radiographic features of pulmonary contusion may take several Hrs
to develop after the initial blunt injury.Since pts are at risk for late clinical deterioration
, they shd be admitted to the hospital after the initial injury and monitored for 24-48 Hrs
for any signs of deterioration.The symptoms of mild pulmonary contusn usually resolve
within 3-5 days.In the initial phase, management is genrally conservatve wth aggressive
pulmonary toilet, supplemental oxygen, pain control and careful fluid management to
prevent worsening edema of the injured lung.In pts with significant lung parenchymal
damage, endotracheal intubation and mechanical ventilation may be necessary to
provide respiratory support until the resolution of parenchymal injury.

31.Hypophosphatemia:
Continuous glucose infusions are the leading cause of hypophosphatemia in hospitalized
pts.The pts are otherwise alcoholic or otherwise debilitated and the nadir in serum
phosphate appears in first few days after admission.Hypophosphatemia can impair the

ATP generation which is needed by the skeletal muscles to perform work and muscle
weakness can result.Respiratory muscle weakness can result that is severe enough to
prevent weaning from mechanical ventilation.In addition, phosphate deficiency reduces
cardiac contractility and chronic phosphate deficiency has been implicated as a cause of
cardiomyopathy.Phosphate depletion is also associated wth depletion of 2,3 diphosphoglycerate that results in leftward shift of oxyhemoglobin dissociation curve.As a result,
the oxygen bound to hemoglobin is less readily released to the tissues.

32.Due to sepsis and hypomagnesemia, hypocalcemia can occur in as many as 2/3rds of


ICU pts resulting in neuromuscular excitability like hyperreflexia,tetany, and seizures.
Cardiac effects include peripheral vasodilation, hypotension & prolonged QT interval.

33.Zinc deficiency is quite common in ICU pts and can occur due to multiple factors like
malnutrition,chronic renal failure, burns, prolonged illness, diarrhea, diuresis.As Zinc
is involved in DNA synthesis and lymphocyte transformations, the deficiency leads to
infections and skin rash.Reduced plasma levels can confirm the diagnosis.Daily
replacements can prevent these complications.

34.Symptomatic Intracranial Hypertension:


Pts with intracranial HTN are classically described to have HTN,bradycardia & respiratory
depression (Cushings triad).Pts initially complain of headaches, vomiting, blurred vision
and have papilledema on funduscopic examination.Further raise in ICP leads to transtentorial herniation that causes dilation of ipsilateral pupil, third cranial palsy ( outward
& downward deviation of the eyeball ) , hemiparesis , decrebrate rigidity ( adduction of
extremeties and hyperpronation ) and respiratory depression.
Securing and maintaining a patent airway is the first priority in the management of

trauma pts with symptomtic intracranial HTN.(CT scan and Mannitol actually come later)

35.Intravenous thiamine alcoholics prevents the development of wernickes


encephalopathy.

36.The use of hyperventilation to lower the intracranial HTN by causing vasoconstriction


is actually contraindicated in traumatic and stroke pts becx of the fact that vasoconstriction can actually lead to worsening of neurological injury in these pts.

37.Allergic bronchopulmonary aspergillosis: (ABPA)


>This hypersensitivity reaction is seen in pts with underlying asthma.When the airways
of such pts become colonized wth Aspergillus, the intense IgE and IgG mediated
immune response leads to the characteristic recurrent episodes of fever, malaise
,cough with brownish mucoid expectoration,wheezing and symptoms of bronchial
obstruction. There is no single diagnostic test to confirm the diagnosis of ABPA. The
diagnosis is usually made by clinical, radiographic and immunologic criteria, which
includes;
=>A history of asthma
=>Immediate skin test reactivity to Aspergillus antigen
=>Precipitating serum antibodies to Aspergillus fumigatus
=>Serum total IgE concentrations of greater than 1000 ng/mL
=>Peripheral blood eosinophilia greater than 500/mm3
=>Lung infiltrates, usually involving the upper lobes
=>Central bronchiectasis on high resolution CT-scan
>A skin prick test for Aspergillus shd be performed initially in all asthmatic pts suspected
of having ABPA.If the skin prick test is +ve, serum total IgE and precipitating serum

antibodies to Aspergillus fumigatus shd be measured.If the skin prick test is negative,
the diagnosis of ABPA is extremely unlikely.
>Oral Corticosteroids are the mainstay of therapy for pts with APBA.Treatment with
corticosteroids is effective in controlling the episodes of acute inflammation and
preventing progressive lung fibrosis.The clinical response to treatment is measured
by the reduction in serum total IgE concentration, clinical improvement and resolution
of radiographic findings.

38.Exercise induced bronchoconstriction: (EIB)


>EIB may occur in asthmatics or in pts w/o a pre-existing diagnosis of asthma.Bronchial
smooth muscle constriction may be triggered by exertion or passage of large volumes
of dry cold air causing mast cell degranulation.Pts with EIB may be otherwise
asymtpomatc
>Clinical symptoms are not sufficient to make a diagnosis of EIB & bronchoprovocation
testing with exercise or inhalation of dry, cold air is recommended.A fall in FEV1 of
>10% is positive while a fall >15% is diagnostic.
>Pre-exercise warm up may be sufficient to prevent EIB in those with only occasional
symptoms.
>Pts with persistent symptoms who exercise infrequently (less than daily)
require short-acting beta-2 agonists (SABAs) 10-20 mins before exercise.Mast cell
stabilizers (cromolyn sodium or nedocromil) may be used in pts who do not tolerate
SABAs.
>Atheletes who exercise frequently shd be started on daily inhaled corticosteroids
with SABAs as needed.

39.A frequent complication of Parkinsons disease is the development of dysphagia and


aspiration which may be silent.Undetected dysphagia may lead to weight loss and
frequent aspiration pneumonia.Aspiration often involves polymicrobial & anaerobic
organisms.Initial management includes blood and sputum cultures with initiation of
broad-spectrum antibiotics with anaerobic coverage (e.g clindamycin).
Diagnosis is confirmed with videofluorscopic swallowing study.A multidisciplinary
intervention is required to address nutrition and reduce the risk of recurrent aspiratn
pneumonia.Specific interventions typically include thickened liquids and modified
swallowing techniques.

40.Pulmonary tuberculosis is a potential cause of persistent/recurrent lung infections in


the elderly.In majority of cases (>80%), however, tuberculosis will involve the upper
/apical lung regions.

41.ARDS:
>ARDS z an inflammatory condition typically caused by infection (e.g sepsis, pneumonia)
,trauma (e.g fractures, pulmonary contusion) or other causes (e.g massive transfusion,
pancreatitis).Lung injury causes release of proteins, inflammatory cytokines and
neutrophils into the alveolar space.This leads to leakage of bloody and proteinaceous
fluid into the alveoli , alveolar collapse due to loss of surfactant and diffuse alveolar
damage.The end result is impaired gas exchange , decreased lung compliance (stiff
lungs) and increased pulmonary arterial pressure (pulmonary HTN) requiring invasive
mechanical ventilation.
Pts typically develop new or worsening respiratory symptoms during the previous week
or within one week of known clinical insult.Findings include respiratory distress,diffuse
crackles on lung examination, hypoxemia and bilateral alveolar infiltrates on chest

imaging.However, pts usually do not have findngs of fluid overload.The PaO2 decreases
,leading to an increased requirement of FiO2.As a result, PaO2/FiO2 is decreased (<300
mmHg) and the ratio indicates the degree of ARDS severity.ARDS is diagnosis of exclusn
after objective assessment (e.g echocardiography) to definitively exclude hydrostatic
pulmonary edema in pts w/o ARDS risk factors.
>Hypoxemia in ARDS is due to shunting.
>The ARDS pts need to be put on mechanical ventilation.
> =>FiO2->Set at max 60%
=>PEEP->10-15 cm H2O
=>Tidal vol.->6-8 mL/Kg to achieve plateau press <30 cm H2O
=>RR
>FiO2 and PEEP->Oxygenation
Tidal vol and RR->CO2
>Hypoxemia ? -> See if the PEEP is low, increase it then. It opens the collapsed alveoli
>Low tidal vol. ventilation ( LTVV ) allows permissive hypercapnia to maintain alveolar
hypoventilation.This permissive hypercapnia is safe for most pts except those with e
elevated intracranial pressure or a seizure disorder.
and prevents ventilator-assoicated lung injury.
>The goal in ARDS are PaO2 of 55-80 mmHg or oxygen saturation of 88-95%
>A very high plateau pressure will increase the risk of barotrauma (pneumothorax or
pneumomediastinum)

42.Increasing the PEEP can have adverse consequences in the form of worsening
hypotension due to decreasing of preload and overinflating noninvolved alveoli
leading to barotrauma.

43.CPAP is often used when weaning pts from the ventilator.It essentially provides only
PEEP w/o any ventilator support.

44.Increasing the tidal volume would help increase the minute ventilation.This would be
effective if the pt was ventilating poorly i.e elevated CO2. Same goes for Respiratory
rate (RR).

45.Acute bronchitis:
Its typically a self limited bronchial inflammation presenting with cough lasting >5 days
& upto 3 weeks.The majority of acute bronchitis cases are due to viruses (e.g influenza
A and B, parainfluenza, rhinovirus ) & only rarely is the cause bacterial (e.g mycoplasma,
Bordetella pertussis).The cough can range from nonproductive to productive of yellow,
green, or purulent sputum. However, purulent sputum does not indicate bacterial
infection.Systemic findings (e.g fever, chills ) are not common and may suggest actually
pneumonia if present.
Physical examination may show wheezing due to bronchospasm and rhonchi (that clear
with coughing) due to airway secretions & may show chest wall tenderness from muscle
strain due to coughing.Crackles indicate disease beyond the bronchioles and are usually
not present in acute bronchitis.
Diagnosis is based on clinical findings alone, without the need for further testing.Acute
bronchitis is usually due to self-limiting viral illness, current guidelines dont recommend
routine antibiotic use. Antibiotics may be considered for those with high suspicion for
mycoplasma or pertussis, but these pts tend to have a prolonged cough for many weeks.
Pts with acute bronchitis usually improve with symptomatic therapy, including NSAIDs,
aspirin, acetaminophen and/or bronchodilators (in those with wheezing).

46.Capnography:
Capnography is a non-invasive technique used to measure CO2 concentration over time
in the exhaled breath and is the most reliable method for verification of proper endotracheal tube placement.It consists of two methods of doing this. 1) Quantitative
waveform 2)colorimetric analysis

A.Quantitative waveform:
A normal capnogram has a characteristic rectangular waveform with 4 phases.A normal
waveform with all 4 phases after intubation indicates proper ETT placement whereas a
flat-line waveform typically indicates improper esophageal placement.

B.Colorimetric analysis:
A colorimetric End-tidal CO2 (EtCO2) detector consists of a litmus paper that changes
color when exposed to CO2. CO2 exhalation from an ETT properly palced in the trachea
will lead to a color change (purple to yellow) , whereas an ETT improperly placed in the
esophagus will not deliver CO2 to the detector , resulting in no color change.

NOTE:Clinical findings , including auscultation of breath sounds over both lung fields,
visualization of the ETT through the cords, the rise of the chest wall with assisted
ventilation and fogging of the ETT with exhalation, should not be used as reliable
indicators of correct placement in the trachea.

47.Respiratory syncytial virus is the most common cause of bronchiolitis.Outbreaks peak


during the winters.RSV outbreaks cause uncomplicated nasal congestn & rhinorrhea
in adults.Children aged <2 years tend to have upper and lower respiratory tract
involvement (e.g bronchiolitis).Infants age <2 months or those wth hx of prematurity

,congenital heart disease and chronic lung disease are most susceptible to life
threatening apnea.
48.Management of choking:
A.Complete obstruction;
a.child age >1;
The child is initially conscious but may be cyanotic and unable to speak, cough, or
breathe.The American Heart Association recommends that abdominal thrusts (i.e
Heimlich maneuver) be administered immediately to conscious children with
complete obstruction to dislodge the aspirated object.The rescuer shd stand behind
the child and lean the child forward.Abdominal thrusts shd be repeated until the
object is dislodged or the pt becomes unconcscious.
As an alternative American Red Cross advises 5 back blows alternating with 5
abdominal thrusts with the child leaning forward infront of the rescuer.
b.child age <1;
Infants age <1 with complete airway obstruction shd be placed face down/head
down on the rescuers arm and receive alternating back blows and chest thrusts.
This process shd be repeated until the obstruction is relieved or until the patient
becomes unconscious, when CPR shd be initiated.
B.Partial obstruction;
Children with partial obstruction are able to speak but may be coughing or gagging.
They shd be allowed to cough up the object on their own and taken to a hospital
for further evaluation.All children with suspected aspiration shd undergo bronchoscopic evaluation of the tracheobronchial tree to assess for retained foreign bodies
Note:If a pt becomes unconscious,CPR shd be started immediatly.Before rescue breaths
are administered, the mouth shd be opened and inspected for foreign bodies that
may have become dislodged.

49.Parainfluenza virus-> croup

50.Postoperative atlelectasis:
Is common after thoracoabdominal surgeries & is caused by splinting & reduced cough
due to abdominal pain, retained secretions and diminished lung compliance.It typically
occurs on 2nd 5th postoperative day and can cause significantly increased work of
breathing and hypoxemia.Obesity and obstructive sleep apnea increase the risk of
postoperative hypoxemia.

51.Suspect the post-op pneumonia in pts with fever, purulent sputum and infiltrate on
chest x ray.

52.Post-operative pulmonary embolism is uncommon before the 3rd post-op day and
occurs >1 week after surgery (esp in pts age >40)

53.Postoperative bronchospasm is characterized by wheezing, chest tightness and


prolonged expiration.It is more common in pts with underlying asthma and is often
due to durg hypersensitivity or a direct effect of certain medications (e.g opiates).
Sever bronchospasm is less likely w/o wheezing on physical examination.

54.Critically ill pts who are unstable with significant hypoxia require rapid-sequence
intubation fr ventilator support and/or airway protection.Rapid sequence intubation
involves administering rapidly acting sedatives (etomidate, propofol, midazolam)
with a paralytic agent (e.g succinyl choline, rocuronium) to facilitate emergency
intubation.Post-intubation care involves obtaining a chest x ray & arterial blood gas
and determining initial ventilator settings.

After initial stabilization of the airway, pts may be appropriately assessed for the
underlying etiology.

55.Pulmonary embolism:
>Can occur after orthopedic surgery despite the use of appropriate DVT prophylaxis
such as low molecular wt heparin.
>Findings on ECG include acute pulmonary HTN and rt ventricular (RV) strain include
right bundle branch block, atrial arrhythmias, inferior Q waves & ST-segment changes.
The presence of RV strain indicates a possible increased mortality rate in hypotensive
pts with massive PE.Echocardiography can identify signs of RV dysfunction , including
increased RV size , decreased RV contractility , presence of RV thrombus and tricuspid
regurgitation.Massive PE causes pulmonary HTN that dilates the tricuspid annulus and
causes functional tricuspid regurgitation.

56.Pulseless electrical activity and asystole:


Pulseless electrical activity (PEA) refers to an organized rhythm on cardiac monitoring
without a measurable blood pressure or palpable pulse.In contrast, asystole refers to
a complete absence of electrical and mechanical cardiac activity.PEA and asystole are
both non-perfusing rhythms and require immediate and effective CPR to maintain
adequate oxygenation and organ perfusion.
Current ACLS guidelines recommend managing PEA and asystole with continued
CPR and vasopressin therapy (intravenous atropine or vasopressin) to achieve adequate
cerebral and coronary perfusion.CPR shd be continued uninterrupted while attempts
are made to identify and treat any potential reversible causes of PEA. Atropine & cardiac
pacing are NOT recommended for the treatment of asystole or PEA.

57.Asthma exacerbation:
>A condition frequently triggered by viral infections or allergies
>Current guidelines recommend the use of ;
=>Supplemental oxygen to maintain the oxygen saturation >90%
=>Short acting inhaled bronchodilators for significant dyspnea
=>Systemic corticosteroids (no significant difference in intravenous and oral, unless
the pt cant tolerate oral intake)
=>Empiric antibiotics are not useful for treatment of asthma exacerbations in the
absence of pneumonia as most exacerbations are triggered by viral infections.
Similary chest x ray has very low yield for pathology in uncomplicated asthma
exacerbations unless symptoms persist beyond the initial treatment period.

58.A low probability V/Q scan does not rule out diagnosis of acute pulmonary embolism
in pts with a high clinical suspicion of PE.

59.After the airway has been secured by endotracheal intubation, its important to
confirm endotracheal tube placement by checking ET tube depth,chest excursion
& bilateral breath sounds & by obtaining a chest radiograph.In general, appropriate
insertion depth for endotracheal tubes is 21 cm for women and 23 cm for men.
Usually the endotracheal tubes would go wrongly into right main stem bronchus and
clinically there would be decreased chest excursions and breath sounds on the left.

60.Tension pneumothorax:
Presents with decreased or absent breath sounds and hyper-resonance to percussion on
affected side along with hypoxia.Tension pneumothorax causes marked compression &
shifting of mediastinal structures, including the heart and great vessels towards the

opposite side, causing a reduced cardiac output and hypotension.Emergency large bore
needle decompression in 2nd or 3rd intercostal space in the midclavicular line shd be
performed, followed by a chest tube thoracostomy.

61.Lung cancer screening with annual low-dose helical chest CT is recommended for pts
aged 55-80 with a >30 pack year smoking history and who currently smoke or quit
within the last 15 years.Screening is aassociated with 20% relative reduction in
mortality risk but a false positive rate of 96%.
Screening shd be terminated if the pt is age >80 OR if the pt quit smoking >15 years
OR pt has other medical problems that significantly limit life expectancy or ability/
willingness to undergo lung cancer surgery.

62.Croup(Laryngotracheobronchitis):
Croup ( laryngotracheobronchitis ) is a parainfluenza viral infection of the larynx and
tachea that peaks in the fall and winter.Croup usually affects children age <6 , with a
peak incidence around age 2.Clinical manifestations include rhinorrhea, cough and a
low grade fever , which later progresses to a harsh,barking cough, inspiratory stridor
and hoarseness.Stridor in mild croup occurs only with agitation and activity.Children
with moderate or severe croup have stridor, retractions and distress at rest.
Croup is usually diagnosed clinically.An x-ray can be ordered for uncertain or recurrent
cases and may show the steeple sign due to subglottic narrowing.Croup is generally
a self-limited illness with symptoms usually resolving within a week.Treatment for mild
croup consists mainly of humidified air to provide comfort,decrease drying of inflamed
mucosal surfaces and soften secretions.A single dose of corticosteroids can be offered
to pts with mild croup to decrease the stridor , which often peaks at night.
Corticosteroids plus nebulized racemic or L-epinephrine should be administered for

moderate/severe croup for relief of upper airway obstruction.Many pts with moderate/
severe croup will improve significantly after treatment and can be discharged home aftr
a few hours of observation.Hypoxic pts or those who have no improvement or who
worsen shd be hospitalized for observation & repeated doses of nebulized epinephrine.
BOTTOM LINE: Croup is a viral infection characterized by a barking cough and stridor.
Pts with mild croup (stridor with activity/agitation only) are treated with
humidified air and corticosteroids.Those with moderate/severe croup
(stridor at rest) shd receive corticosteroids and nebulized epinephrine.
Hospitalization is indicated for pts who fail first-line treatment.

63.CT is preferred over Chest X ray as a screening modality for lung cancer.

64.Cessation of smoking would;


=>Reduces the decline in pulmonary function
=>Reduces the risk of developing COPD
=>Reduces the risk of experiencing COPD exacerbations in those with COPD
=>80-90% reduction in lung cancer risk compared to current smokers,if not smoked for
atleast 15 years.
=>Appears to be associated with decrease in all-cause mortality.

65.Solitary pulmonary nodule: (SPN)


>Defined as a round opacity , upto 3 cm in diameter and surrounded by normal lung
tissue.
>The first step is to compare previous X rays or scans as a SPN with stable size and
appearance over 2-3 years has a low risk for malignancy and requires no further
workup.X ray findings favoring malignancy include size >0.8 cm or interval change in

size, irregular border and ground glass appearance.


>Pts with unstable lesions or no previous chest imaging require chest CT scans fr further
investigation.CT can help identify small nodules missed by X rays as it evaluates entire
lung parenchyma (apices,perihilar & costophrenic regions).Its also more sensitive than
chest X ray in identifying features concerning for malignancy and can help guide needle
biopsy of suspicious lesions.
>In pts aged <40, with nodules <0.8 cm with smooth margins, have never smoked or
have quit smoking > 15 years are at low risk and less likely to be having malignant
lesions.Such pts would require serial CT scans for nodule size and characteristics
continuous follow-up.Once the size goes to <4 mm and no features of malignancy
in the nodule, the follow-up can be terminated.
>In pts 40-60,with nodule size 0.8-2.0 cm with scalloped borders who are current
smokers or have quit smoking 5-15 years back are at intermediate risk.Such pts require
FDG-PET scans to see the nodule characteristics further.If FDG-PET scan shows risk of
malignancy, then the nodule shd be surgically excised and if FDG-PET scan shows the
nodule to be not suspicious for malignancy, then serial CT-scans shd be performed.
>In pts aged >60, with nodules >2 cm with spiculated margins(corona radiata), are
current smokers or have quit smoking <5 years have a higher likelihood of malignancy.
Such pts require surgical excision preferably by video-assisted thoracoscopy.

66.Bronchiectasis:
>Chronic daily cough with mucopurulent sputum production and recurrent infections
are characteristic of bronchiectasis.
>Damage to bronchial walls from recurrent infections (such as pneumonias) or
inflammation predisposes to repeated infections , setting up a cycle of further airway
damage and recurrent infections.

>The chronic airway inflammation and edema can lead to infiltration and rupture of
superficial blood vessels and cause hemoptysis.
>Pulmonary infections, acute or chronic, overall are the most common cause of
hemoptysis , accounting for upto 70% of the cases.

67.An obstructing endobronchial malignancy shd be suspected in pts with an extensive


smoking history who have non-resolution of a pneumonia.A chest CT can help to
diagnose such a malignancy as well as other causes of a non-resolving pneumonia
such as an abscess or empyema.

68.Pts with hypoxemia due to an acute asthma exacerbation shd be immediately


managed with oxygen inhalation, followed by administration of an inhaled beta-2
agonist , which is the mainstay of therapy.

69.Oral candidiasis in asthmatics: (Thrush)


Occurs due to inhaled corticosteroids used for asthma.
Presents as oral white plaques that can be typically removed , although at times it may
be difficult. A residual area of inflammation may be seen where the plaque has been
removed.Most pts are asymptomatic although some may complain of a strange, cottony
taste in the mouth.
In cases where its caused secondary to inhaled corticosteroids, proper technique shd be
assessed by watching the pt use the inhaler, as thrush may be a sign that the pt is using
the inhaler improperly with a large amount of medicine depositing in the oropharynx as
opposed to passing into lungs.A spacer device may be helpful if the pt z havng trouble.
The best initial treatment for oral candidiasis (thrush) is topical antifungal (e.g nystatin
suspension or clotrimazole troches), with an oral antifungal such as fluconazole used for

resistant cases. Pts shd be advised to rinse their mouth after using the inhaler.

70.Topical steroids-> aphthous stomatitis.

71.Chronic smoker + Chronic productive cough => COPD


Chronic productive cough+Mucupurulent sputum+recurrent infections=> Bronchiectasis

72.The two most important predictors of survival in COPD pts are FEV1 and age.After
adjusting for age, FEV1 remains as the single most important factor in determining
the prognosis.
Other prognostic indicators described in several studies are airway responsiveness,
cigarette smoking

73.Cough induced by expiration is an indication of airway hyperreactivity and this is a


recognized clinical clue for the diagnosis of asthma.
However,this presentation has been described in some pts with COPD, who can
have occasionally an overlapping condition with asthma.

74.Tracheomalacia presents with stridor and inspiratory airway collapse.

75.Steroids tapering :
>Discontinuation of the corticosteroids has 3 primary risks;
=>Unmasking latent symptoms of the condition being treated.
=>Steroids withdrawal syndrome (non specific symptoms such as fever, fatigue and
headache associated with normal cortisol levels)
=>Adrenal insufficiency due to suppression of HPA axis.

>The use of exogenous glucocorticoids for extended period is likely to cause significant
suppression of the HPA axis and requires a gradual taper to allow for adequate
endogenous cortisol production.
>Short term use of glucocorticoids , generally < 3 weeks, is unlikely to cause significant
suppression of the HPA axis and can be stopped rapidly without causing any significant
adrenal insufficiency.
>When the risk of HPA suppression is uncertain or there are other factors that may
trigger adrenal crisis (e.g scheduled surgery) , the HPA axis can be assessed with a low
dose ACTH stimulation test.Pts who do not have increased cortisol production in
response to ACTH will require a taper.
>Alternate day dosing of steroids is associated with less suppression of HPA axis and
reduce the need for slow tapering.
>Doses of prednisone <5 mg do not require tapering.We are uncertain with 10-20 mg
doses , so we can do ACTH stimulation test to confirm if HPA is suppressed or not.
Doses >20 mg do require tapering if used for >3 weeks.
>One published regimen for tapering the dose of steroids after long term use involves
a weekly decrease in the dose of prednisone by 2.5-5.0 mg until a daily dose of 7.5 mg
is reached.Once the pt reaches this dose, it is converted to 20-30 mg of hydrocortisone
daily;this is then gradually decreased by 2.5 mg weekly, until the pt reaches 10 mg of
hydrocortisone daily.Hydrocortisone is discontinued once the morning (pre-dose)
plasma cortisol level is >10 ug/dL.
BOTTOM LINE: The use of glucocorticoids for <3 weeks , even in relatively high doses,
does not cause significant suppression of the HPA axis.When given for
a short period, glucocorticoids can be stoppd rapidly w/o a significant
risk of adrenal insufficiency.

76.Pts with asthma who have had symptoms severe enough to warrant emergency dept
evaluation r at high risk of further exacerbations.Inhaled steroids such as fluticasone
shd be given to such pts for long term management.

77.Cystic fibrosis:
>Shd be suspected in pts with chronic lung disease, prolonged diarrhea (usually
steatorrhea due to pancreatic disease), and failure to thrive.
>Sweat chloride test is the gold standard for the diagnosis (>60 mEq/L)
>Blood levels of immunoreactive trypsin are preferred in infants, esp <2 months age due
to difficulty performing sweat chloride test in them.
>Genetic testing would only identify 10% of pts, so not recommended as such.
>Sweat chloride test is preferred over nasal potential difference measurement.

ENDOCRINE, DIABETES & METABOLISM


1.Clinical features of thyrotoxicosis with elevated T3, T4 and TSH levels, along with an
increase in serum alpha-subunits are highly suggestive of TSH-secreting pituitary
adenoma.Thyroid hormone resistance also has increased T3, T4 and TSH levels but the
serum alpha subunit levels r not elevated.Pts wth thyroid hormne resistnce syndrome
are likely to be hypothyroid.

2.Pregnant pts have increased serum T3 and T4 levels while the serum TSH level is
normal or suppressed.

3.Pts with familial dysalbuminemic hyperthyroxinemia hve increased T4 levels & normal
T3 levels.This is because abnormal albumin with a very high affinity to bind T4 but not
T3.These pts have normal TSH levels.

4.Stress hyperglycemia:
Stress hyperglycemia is defined as transiently elevated blood glucose levels in the
context of severe illness in pts w/o known diabetes mellitus.During illness and stress,
hyperglycemia result from the release of high levels of stress hormones (e.g cortisol,
catecholamines, proinflammatory cytokines).Risk factors fr stress hyperglycemia include
severe illness, temperature >39C (102.2F), sepsis, meningitis, and admission to the
intensive care unit.
In hyperglycemic critically ill adult pts, glucose containing fluids are minimized & insulin
is administered to achieve a blood glucose target of 140-160 mg/dL.
For hyperglycemic critically ill children, the optimal blood glucose target range and data
on insulin therapy are inconclusive.

No relationship btwn stress hyperglycemia & persistent abnormal glucose metabolism


or subsequent diagnosis of diabetes mellitus has been demonstrated.
Pts with actual diabetes mellitus would have classic symptoms of diabetes mellitus (wt.
loss, polyuria, polyphagia, polydipsia) and elevated HbA1c.

5.HYPOGLYCEMIA APPROACH:
>Exogenous Insulin injections would result in a low level of C-peptide.
>Both Oral hypoglycemic agents & Insulinoma would result in elevated C-peptide;
I
Next step is Hypoglycemic drug assay for oral hypoglycemic agents
I
=>If negative, its most likely an insulinoma->go for Abdominal CT-scan/Ultrasound.
=>If positive, the hypoglycemia was caused by oral hypoglycemic agent such as
sulfonylureas/meglitinide.

6.What is Whipples triad?


Symptoms consistent with hypoglycemia, low blood glucose levels and symptom
resolution after glucose administration.

7.The following principles shd guide the evaluation of hypoglycemia;


>Whipples triad must be fulfilled before a hypoglycemia workup is initiated.Many
normal individuals may have blood glucose in the normal hypoglycemic range w/o
symptoms of hypoglycemia.
>Low blood glucose in normal individuals will suppress production of insulin, C-peptide
and proinsulin.In the presence of significant hypoglycemia, normal levels of these
hormones indicate an abnormal response.

>Fingerstick blood glucose measurement in unreliable in the hypoglycemic range.Low


blood glucose recorded by a bedside glucometer must be confirmed by a standard
lab assay.
>Blood samples for biochemical testing must be taken when the pt is hypoglycemic &
before glucose administration.

8.Hyperthyroidism:
>Conditions where thyroid hormone synthesis is increased such as Graves disease and
toxic adenoma have a high radioactive iodine uptake
>Conditions where preformed thyroid hormone is released secondary to inflammation
or destruction of thyroid gland, such as thyroiditis, have a decreased radioactive iodine
uptake.
>Postpartum thyroiditis initially results in hyperthyroidism followed by hypothyroidism,
is painless, occurs following a recent pregnancy, has positive anti-TPO antibodies.
>Painless thyroiditis is essentially identical to postpartum thyroiditis with the exception
of postpartum history.

9.Focus on common causes of common medical conditions. Obesity in children and


adults most often results from over eating and not from endocrinologic conditions
such as hypothyroidism and Cushings disease.

10.Precocious pubarche:
>Precocious puberty is caused by the premature activation of HPA axis.Sequential
development of the following is present; testicular enlargement, penis enlargement,
pubic hair growth, and lastly the growth spurt.
>Precocious pseudo-puberty is caused by a gonadotropin-independent excess, typically

an excess of sex steroids.


>Late onset congential adrenal hyperplasia can cause signs of androgen excess e.g cystic
acne, coarse pubic and axillary hair, growth acceleration etc.

11.Low radioactive iodine uptake (RAIU);


=>Thyroiditis->High thyroglobulin
=>Struma ovari->High thyroglobulin
=>Exogenous thyroid hormone intake-> Low thyroglobulin

12.Exogenous thyroid hormone intake is the only known condition where there is low
RAIU and low thyroglobulin levels.The T4 levels are elevated as the pt is taking them
and the TSH level is reflexively suppressed.

13.Exogenous hyperthyroidism shd be suspected in pts who have non-enlarged thyroid


gland and admit to taking dietary supplements which they may or may not know
contain thyroid hormones.The RAIU shd be low in these pts becx the thyroid gland is
intrinsically making a lower thyroid hormone.The thyroglobulin level shd be low since
there is no inflammation or destruction of native thyroid gland, helping to distinguish
this problem from thyroiditis.

14.An elevated levels of thyroglobulin with a normal RAIU is a pattern that may be seen
in pts with papillary or follicular thyroid cancer.The thyroglobulin level can be a useful
indicator of disease recurrence in these pts.

15. Grave dx->High RAIU, High thyroglobulin


Thyroiditis->Low RAIU, High thyroglobulin
Struma ovari->Low RAIU, High thyroglobulin
Exogenous->Low RAIU, Low thyroglobulin
Papillary/follicular CA->Normal RAIU, High Thyroglobulin.

16.Excessive iodine administration can lead to thyrotoxicosis.


Coronary angiography is one example where iodine is administered and it can lead to
thyrotoxicosis.

17.Iodine-induced thyrotoxicosis occurs mainly in pts with multinodular goiter where


excess iodine administration can serve as a substrate for excessive thyroid hormone
formation.Characteristically, the radioactive iodine uptake in the thyroid gland is low.
Iodine induced thyrotoxicosis is a self-limiting disorder if the source of excess iodine is
discontinued.However, it can persist for months & is usually refractory to antithyroid
medications.
Beta blockers are used for mild symptoms & antithyroid drugs for moderate to severe
symptoms.Radioactive iodine ablation is ineffective because of low radioactive iodine
uptake.Potassium perchlorate can be used in refractory cases.

18.A toxic thyroid nodule is characterized by signs and symptoms of hyperthyroidism,


along with an increased focal uptake in the thyroid scan.

19.Pts with hypothyroidism shd be instructed to take levothyroxine on empty stomach,


preferably in the morning, separately from other medications.Common medications,
such as Calcium and iron, can impair its absorption.

20.Requirements for levothyroxine for hypothyroidism tends to be lower.

21.Central hypothyroidism as suggested by low TSH & low Thyroid hormones levels shd
prompt the investigation for other pituitary hormones levels as well and pituitary
imaging (MRI).

22.levothyroxine administration for hypothyroidism in pts with uncorrected central


adrenal insufficiency may trigger adrenal crisis by increasing metabolic demand and
clearance of glucocorticoids.

23.Normal cortisol levels are 5-20 ug/dL. A borderline or low normal suggests adrenal
insufficiency.
Once you suspect adrenal insufficiency, the first tests to do are ACTH levels and ACTH
stimulation test.
In case of primary adrenal insufficiency, the ACTH stimulation would lead to a
subnormal response, however, chronic central adrenal insufficiency can also lead to
subnormal response to ACTH stimulation test becx of atrophy of adrenal gland.
Low ACTH levels wth subnormal response would suggest Central adrenal insufficiency
while high ACTH levels with subnormal response would suggest Primary adrenal
insufficiency.
If the response to ACTH stimulation is normal and there is still a concern for central
Adrenal insufficiency, then Metyrapone test and Insulin-induced hypoglycemia tests
can be performed.

SUSPICION OF ADRENAL INSUFFICIENCY

24.Measuring reverse T3 levels (rT3) , particularly in a hospitalized pt, is sometimes


done to distinguish euthyroid sick syndrome from central hypothyroidism, as it is
elevated in euthyroid sick syndrome but low in hypothyroidism.But rT3 levels may
be normal or even elevated in some pts with mild central hypothyroidism, which
limits the clinical utility of this test.Also, there is often a substantial delay in reporting
of this test, which further limits its usefulness.

25.Thyroglobulin level is useful in the follow-up management of pts with differentiated


thyroid cancers and in the evaluation of those with factitious thyrotoxicosis.

26.Cardiovascular signs and symptoms are common in pts with hyperthyroidism. There
is increased risk for supraventricular arrhythmias (e.g atrial fibrillation, atrial flutter,
paroxysmal atrial tachycardia, with atrial fibrillation being the most common type of
supraventricular arrhythmia occurring in ~10-20% of pts with hyperthyroidism),
coronary vasospasm, myocardial ischemia/infarction leading to angina, pulmonary
HTN due to increased pulmonary artery pressure, and congestive heart failure.
These cardiovascular effects occur due to increased sympathetic activity secondary
to increase in beta adrenergic gene expression.
Beta blockers (e.g propranolol) recommended initially to ameliorate hyperadrenergic
symptoms and control heart rate until the pts become euthyroid with thionamides
and/or radioiodine.

27.Gestational diabetes mellitus:


>All pregnant pts are screened at 24-28 weeks of gestation.
>First step is 1-hour 50 g oral glucose tolerance test (OGTT).
=>If the glucose level after one hour is <140 mg / dL, GDM is ruled out.
=>If the glucose level is >140 mg/dL, then 3-hour 100 g glucose challenge test is
performed.
>3-Hour 100 g glucose challenge test is performed and there are >2 abnormal values,
then GDM is diagnosed.
=>Fasting glucose >95 mg/dL.
=>one hour glucose is >180 mg/dL
=>two hour glucose is >155 mg/dL
=>three hour glucose is >140 mg/dL
>In GDM , our target is to keep the blood glucose levels <95 mg/dL fasting and keep it
<140 mg/dL one hour post-prandial and <120 mg/dL at 2 hour post-prandial.
>The first line approach to controlling GDM would be dietary modifications.
>The second line approach would be Subcutaneous insulin (regular/lispro/aspart),
oral metformin and oral glyburide, all equally efficacious.
>Fetal macrosomia and shoulder dystocia are the most common complications of poor
maternal glycemic control as the consequent fetal hyperglycemia stimulates the fetal
hyperinsulinemia and fetal overgrowth.Fetal growth restriction is also a potential
complication as severe maternal hyperglycemia can cause uteroplacental insufficiency.
>Insulin therapy has been shown to reduce the risk of fetal macrosomia and shoulder
dystocia.
>The pregnant women are also at long term risk of developing type-1 DM, type 2 DM,
metabolic syndrome and cardiovascular disease.
>Women shd be counselled on maintaining the appropriate weight, choosing healthy

food, exercising regularly and breast feeding to minimize the future diabetes mellitus
risk.

28.Pts who are unable to tolerate hyperosmolar glucose load (e.g history of bariatric
surgery) can undergo serial fasting and postprandial blood glucose measurements
to diagnose GDM.

29.Cushings syndrome:
Young pts with diabetes, osteoporosis, hypertension and hypokalemia shd be screened
for cushings syndrome. It can also cause mood swings and metabolic alkalosis.
( HTN + Hypokalemia + Hyperglycemia + metabolic alkalosis => Cushings syndrome )
Screening for Cushings syndrome cab be performed by an overnight Dexamethasone
suppression test or measurement of 24-Hr urinary free cortisol levels.
In normal individuals , Serum cortisol is suppressed below 3 mcg/dL after administration
of 1 mg of Dexamethasone.False positive results can be obtained in pts wth alcoholism,
significant depression & malabsorption of Dexamethasone. Many physicians prefer
urinary free cortisol measurement.

30.Graves disease:
>Caused by thyroid stimulating antibodies and causes thyrotoxisis.
>In United States, radioactive iodine is the preferred definitive therapy for Graves
hyperthyroidism.However, radioactive iodine alone may cause worsening of
ophthalmopathy.Concurrent use of prednisone is less likely to worsen the
opthalmopathy. Before going for radioactive iodine ablation, antithyroid drugs are
also given to reduce the size of the gland.
>In Japan, India and many European countries, antithyroid drugs are preferred over

radioactive iodine ablation.Many pts with mild Graves hyperthyroidism will go into
permanent remission with 12-18 months of treatment with an anti-thyroid drug.
=>Methimazole is preferred over propylthiouracil (PTU) due to case reports of fatal
Hepatotoxicity with PTU.
=>PTU is used during first trimester of pregnancy and in pts unable to tolerate
methimazole (e.g skin reaction, GI upset).
=>Neither of the drug is used if pt develops agranulocytosis or severe skin reaction.
>Due to the risk of worsening opthalmopathy following radioiodine therapy, subtotal
thyroidectomy is preferred for pts with severe ophthalmopathy at baseline.Surgery is
also preferred in pts with large goiters ( particularly with obstructive symptoms ),
suspicion of thyroid cancer, or coexisting hyperparathyroidism. Thyroidectomy may be
used during pregnancy (generally in the 2nd trimester) in pts who develop significant
side effects with an antithyroid drug.
>Total T3 and free T4 levels are used to assess thyroid function soon after treatment
wth radioactive iodine (RAI). TSH levels may remain suppressed for weeks or even
months following RAI treatment and do not reliably reflect thyroid functional status.
>TSH receptor antibodies (TSIs) may be used to predict the relapse in pts with Graves
disease.

31.Diabetic Neuropathy:
Diabetic neuropathy shd be suspected in pts with diabetes who have symmetric sensory
changes in the feet that consist of injury to nerve fibers controlling pain, temperature,
vibratory and proprioceptive sense. The tuning fork test is an easy and inexpensive way
to assess for the loss of vibratory sense with diabetic neuropathy.
Aggressive glycemic control is the most important aspect of treatment for diabetic
neuropathy. However, for pts with severe associated pain, treatment for pain becomes

necessary. First line agents include duloxetine (SNRI) , pregabalin and TCAs. All of these
agents interfere with neural transmission of pain. Other agents that are frequently used
include the anticonvulsant medications gabapentin, lamotrigine, and carbamazepine.
Topical treatments such as capsaicin cream and topical lidocaine can also be considered
for some pts.Pain symptoms of diabetic neuropathy resolve within a year, so eventual
discontinuation of these medications is often possible.

32.The risk of developing type-1 diabetes in offsprings of parents with type-1 diabetes is
higher than general population. If the mother has type-1 diabetes, the risk is ~3%.If
the father has type-1 diabetes, the risk is 6%.

33.21-Hydroxylase deficiency:
The most common cause of congenital adrenal hyperplasia is 21-hydroxylase enzyme
deficiency, which is autosomal recessive.
In addition to low production of mineralocorticoids and glucocorticoids , the deficiency
of 21-hydroxylase results in an excess of its substrates.These excess substrates are
shunted towards androgen synthesis, resulting in ambiguous genitalia in girls and
precocious puberty in boys.ACTH levels are also elevated and lead to hyperplasia of
the adrenal glands, further contributing to the increased production of androgens.
Salt-wasting syndrome occurs in severe deficiencies of 21-hydroxylase and presents in
the first few weeks of life with life-threatening emesis, dehydration & shock. Laboratory
findings are hyponatremia, hyperkalemia (from lack of aldosterone) & hypoglycemia
(from lack of cortisol).
A diagnosis of 21-hydroxylase deficiency is confirmed by the excessive levels of
17-hydroxyprogesterone. Aldosterone and cortisol levels are abnormally low.
The gold standard for treatment includes chronic glucocorticoid and mineralocorticoid

treatment to maintain normal blood pressure, electrolytes, growth and weight gain and
to suppress adrenal androgens.Hydrocortisone is the most common glucocorticoid used
in infants and children.Fludricortisone is given for mineralocorticoid replacement.Infants
may require salt supplementation until they begin eating solid foods.
Virilization of external female organs can be corrected by clitoroplasty and vaginoplasty
when the pt is stable and the parents are emotionally prepared.

34.34. 21-hydroxylase deficiency=>Hypotension


11-beta-hydroxylase deficiency=>Hypertension
17-alpha-hydroxylase deficiency=>Hypertension

35.11-deoxycorticosterone and 11-deoxycortisol levels are elevated in 11-hydroxylase


deficiency causing hypertension, hypernatremia, and hypokalemia.

36.Steroid 5-alpha reductase deficiency is a rare autosomal recessive disorder that can
cause 46 XY individuals to have external female appearing genitalia, wth clitoromegaly
in some pts.Affected 46 XX individuals have normal external female genitalia.

36.Galactosemia is an autosomal recessive disorder characterized by the inability to


digest galactose due to deficiency of galactose-1-phosphate uridyl transferase.Pts are
hypoglycemic and become very ill and dehydrated due to vomiting after ingestion of
breast milk or cows milk based formula.Such infants shd receive soy formula, not
breast milk or cows milk.

37.Pheochromocytoma:
Screening for pheochromocytoma is indicated in pts with episodic symptoms
(headaches, diaphoresis and tachycardia), early age onset HTN, refractory HTN,
paroxysmal HTN, adrenal incidentiloma and idiopathic dilared cardiomyopathy.
The classic triad is Headache, Sweating and Palpitations. However, all 3 findings may
not be present and some pts may even have orthostatic hypotension, weight loss and
blurry vsision.
Measurement of 24-Hour Urinary fractionated metanephrines and catecholamine levels
is one of the preferred screening tests, however, some experts would go for plasma free
metanephrines which has higher sensitivity but lower specificity.
The urinary levels of fractionated metanephrines and catecholamines levels is affected
by several drugs (e.g TCAs and over the counter decongestants) which shd be stopped
2 weeks before the test is performed to avoid false results.
Once the biochemical diagnosis is done, go for abdominal CT/MRI which will show a
mass >4-4.5 cm in diameter.
If the abdominal imaging is negative , we can do MIBG scan (metaiodobenzylguanidine
scan) which is a functional scintigraphy with I22 labelled MIBG. MIBG resembles
norepinephrine , is taken up by adrenergic tissue and can detect tumors not detected by
CT or MRI. In addition, MIBG is also performed in pts with large tumors (>5cm diameter)
as they have higher risk for malignancy and possible extra-adrenal disease.Younger pts
and those with a familial disorder also require MIBG after initial CT or MRI as they have
higher risk of extra-adrenal and multiple tumors.
So once the diagnosis is finally confirmed, go for alpha blocker therapy for 10-14 days
and also replenish the intravascular volume with liberal fluid and salt. Once that is done,
go for surgical resection of the tumor.

Some complications are possible during/after surgery;


=>Hypotension can occur due to decreased circulatory catecholamines levels followng
tumor removal.In addition, persistent preoperative alpha blockade results in decrzd
vascular tone.Intra and postoperative hypotension usually responds well to normal
saline bolus.Pressors may be required in pts who do not respond to intravenous
fluids.
=>Hypertensive crisis cn occur due to catecholamines release secndry to endotracheal
intubation and adrenal gland manipulation.It can also occur due to spontaneous
norepinephrine release due to large tumors (> 4cm dia). Intravenous infusion of
phentolamine (alpha blocker) or sodium nitroprusside or nicardipine can be given
to these pts.
=>Hypoglycemia can also occur due to sudden insulin secretion that was previously
suppressed by catecholamines.So, the tumor removal leads to decreased
catecholamines and hence increased insulin secretion. The hypoglycemia shd be
controlled with intravenous dextrose.
=>Cardiac tachyarrhythmias can occur due to catecholamines release. Intravenous
lidocaine or esmolol can be given to such pts.

38.Some points about pheochromocytom;


=>24-Hr urinary vanillyl mandellic acid excretion has a much lower sensitivity &
specificity for biochemical diagnosis.
=>Chromogranin-A (CGA) is stored and released in from dense-core secretory granules
of neuroendocrine cells and is elevated in 80% of pts with pheochromocytoma.
However, elevated CGA levels are not specific for pheochromocytoma and can be
elevated in other neuroendocrine tumors (e.g carcinoid) or in pts with mild renal
insufficiency. CGA is sometimes used in addition to measuring catecholamine and

metanephrine levels in pts with possible drug interference to biochemical testing.

38.Common causes of thyroiditis with thyroid pain and tenderness include;


=>Subacute thyroiditis ( postviral inflammation )
=>Infections ( bacterial suppuration )
=>Radiation thyroiditis
=>Vigorous palpation or trauma induced thyroiditis.

39. Subacute thyroiditis/ de Quervains thyroiditis/ Granulomatous thyroiditis :


Is characterized by hyperthyroid symptoms with fever and a painful , diffusely enlarged
thyroid. It is caused by inflammatory infiltrate in the thyroid with susbsequent release
of preformed thyroid hormone, leadng to suppressed TSH & decreased radioactive
iodine uptake.The ESR is typically elevated. Histology will show mixed inflammatory
infiltrate consisting of lymphocytes, neutrophils, histiocytes and multinucleated giant
cells.
Subacute thyroiditis is a self limited condition.Treatment primarily consists of NSAIDs
for pain relief and beta blockers to minimize the hyperthyroid symptoms. Pts with pain
that z severe or does not respond to initial measures may require corticosteroid therapy

40. Lymphocytic infiltrate is seen in silent/painless thyroiditis and Hashimoto thyroiditis

41.In Graves disease, T3 is more elevated than T4. In pts with subacute thyroiditis, T4 is
more elevated than T3, reflecting the ratio of T4 to T3 in preformed thyroid hormonz
within the thyroid gland.

42.Graves diseas=> diffuse uptake on RAIU

Toxic multinodular goiter=>Patchy uptake


Toxic adenoma=>Hot nodule

43.Toxic multinodular goiter is seen in older pts and thyrotoxic symptoms are usually
subtle if present at all.

44.Suppurative infection of thyroid gland (infectious thyroiditis) is a rare condition


causing high grade fever and pain at the thyroid gland. Pts will often have evidence
of a thyroid abscess on examination or ultrasound. Pts with infectious thyroiditis are
usually euthyroid becx the involvement of the thyroid gland is focal. Bacterial thyroid
infections are treated with systemic antibiotics and surgical drainage may be requird.

45.Pts with primary hypothyroidism are predisposed to get other autoimmune diseases
such as pernicious anemia. Vit-B12 defeciency in pernicious anemia is due to a
deficiency of intrinsic factor secreted by stomach.Neruologic involvement in Vit-B12
deficiency is characterized by subacute degeneration of the spinal cord and peripheral
neuropathy.
Subacute combined degeneration is characterized by involvement of posterior and
lateral columns in the spinal cord, that can lead to ataxia, loss of proprioception and
vibration sensations. Severe spasticity, weakness and peripheral nerve involvement
can occur.Symptoms & signs are more prominent in the lower than upper extremeties

46.Endocrinopathy with polyneuropathy can be seen with POEMS (polyneuropathy,


organomegaly, endocrinopathy, M-proteins and skin changes) syndrome.

47.Treatment with Vit-B12 in pts with moderate to severe megaloblastic anemia can
cause hypokalemia in the first 48 Hours , which can sometimes be very severe and life
threatening. Hypokalemia results following the uptake of potassium by newly forming
RBCs.Close monitoring z required during this period. Potassium z replaced depending
upon the measured serum potassium levels. Some physicians transfuse packed red
blood cells in pts with severe megaloblastic anemia before Vit-B12 supplementation
to prevent hypokalemia.

48.Nelsons syndrome:
Bitemporal hemianopsia and hyperpigmentation following bilateral adrenalectomy for
Cushings disease z suggestive of Nelsons syndrome.The disease is caused by pituitary
enlargement due to loss of feedback by adrenal glucocorticoids.
MRI & plasma ACTH levels r required fr making the diagnosis. A pituitary microadenoma
with suprasellar extension on MRI and extremely high ACTH levels are diagnostic.
The tumor in Nelsons syndrome is aggressive and is treated by surgery and/or pituitary
radiation.
Following bilateral adrenalectomy , prophylactic pituitary radiation sometimes prevents
the development of Nelsons syndrome; however this leads to an increased risk for
hypopituitarism.Previosuly, bilateral adrenalectomy was the preferred treatment for
Cushings disease, however,wth the advent of better localization & improved techniques
of transsphenoidal surgery, primary pituitary surgery is now the preferred treatment for
Cushings disease.

49.Euthyroid sick syndrome: (ESS)


This condition, often referred to as Low T3 syndrome, z thought to result from decreasd
conversion of T4 to T3 , decreased T4 production and clearance, altered T4 protein

binding, and TSH suppression. TSH and T4 levels are often normal in ESS, although they
also fall in severe and prolonged cases and so ESS may represent a transient central
hypothyroidism rather than a true euthyroid state.
Thyroid function testing is unreliable in pts with acute illness and thyroid hormone
supplementation in ESS has not been found to improve clinical outcomes.Treatment is
deferred until abnormal thyroid function is confirmed after the pt has returned to the
baseline health.During recovery, some pts will experience a modest, transient rise in
TSH.
P.S=> Reverse T3 (rT3) is an inactive metabolite of T4.Clearance of rT3 is reduced in
severe non thyroidal illness and so levels will be elevated in ESS. rT3 is primarily
used to differentiate central hypothyroidism (low T4 leads to low rT3) from ESS
in pts with low TSH.
Euthyroid sick syndrome => High rT3
Central hypothyroidism=> low rT3

50.Subclinical hypothyroidism is characterized by normal T4 and elevated TSH. Prsence


of anti-thyroid peroxidase antibodies in such pts predicts the development of
Hashimotos thyroiditis ( chronic lymphocytic thyroiditis).

51.Incidentally discovered small lesions in the sellar region are very common. Pts with a
small mass or lesion w/o any hormonal dysfunction can be safely followed by periodic
assessment of the pituitary gland with MRI.

52.Metformin:
>What are the adverse effects?
=>GI upset (nausea, abdominal pain and diarrhea)
=>Decreased intestinal Vit-B12 absorption
=>Lactic acidosis
>The incidence of adverse effects are higher in those with renal dysfunction or hepatic
dysfunction.
>What are the contraindications?
=>Renal insufficiency
=>Hepatic dysfunction
=>Alcohol abuse
=>Sepsis
=>Congestive heart failure (especially when the Cr. is >1.5 mg/dL)
>Metformin can increase the risk of lactic acidosis when combined with large dose
intravenous iodine contrast (e.g during coronary angiography).Metformin is usually
held on the day contrast is given and restarted atleast 48 Hours later after documentng
stable renal function.

53.Aspirin can cause platelet dysfunction for >1 week and so is usually discontinued at
least 7 days before procedures with increased bleeding risk.

54.Some studies have shown that ACE inhibitors may protect against contrast induced
nephropathy by blocking renal vasoconstriction.

55.Thyroid hormones;
=>TSH-> 0.35-5.0
=>Total T4->4-11
=>Total T3->80-180

56.Amiodarone effects on thyroid hormones:


Amiodarone causes significant thyroid dysfunction due to its high iodine content, direct
effect on thyroid follicular cells and changes in thyroid hormone metabolism.
>Amiodarone can decrease the peripheral conversion of T4 to T3, resulting in low T3,
high T4 and mildly elevated TSH (<20 mU/L). The thyroid test abnormalities are most
prominent in the first 1-3 months after starting amiodarone and may become less
apparent afterwards in subsequent 3-6 months.These pts are clinically euthyroid. No
treatment is needed for such pts.
>The high iodine content in amiodarone inhibits the thyroid hormone synthesis, leading
to primary hypothyroidism (Wolff-Chaikoff effect).The treatment is levothyroxine.
>Amiodarone can also induce thyrotoxicosis (Amiodarone induced thyrotoxicosis<AIT>)
=>AIT-1 is due to increased synthesis of thyroid hormones.There will be increased
vascularity on ultrasound and decreased RAIU. The treatment is antithryoid drugs.
=>AIT-2 is due to destructive thyroiditis.There will decreased vascularity on ultrasound
and undetectable RAIU.The treatment is glucocorticoids.
P.S=>In both subtypes, TSH will be low.

57.Aspirin in high doses (>2g/day) displaces thyroid hormone from thyroid hormone
binding proteins, leading to an increased thyroid hormone effect with subsequently
decreased TSH, total T3 & toal T4. Low dose aspirin does not have a significant effect.

58.Propranolol decreases the peripheral conversion of T4 to T3, and is therefore useful


in thyrotoxic emergencies.

59.Amiodarone induced biochemical abnormalities in thyroid hormones metabolism


improve with time and no treatment is necessary. Discontinuation is not going to work
becox half life of amiodarone is more than 100 days and so wont do any good.

60.Hormone replacement therapy:


>Increased risk of venous thromboembolism, both deep venous thrombosis and
pulmonary embolism and appears to be higher in women with known coronary artery
disease.
>Has beneficial effects on lipid profile. It decreases LDL cholesterol, increases HDL
cholesterol, & increases triglycerides levels by a modest amount.Despite these modest
benefits, HRT is not effective for primary or secondary prevention of coronary artery
disease.
>Combination therapy with estrogen and progesterone associated with an increase in
the risk of coronary events, mostly in the form of non fatal myocardial infarction.
>Risk of invasive breast cancer is significantly increased with the use of combined HRT.
>Prevention of osteoporosis and control of vasomotor symptoms are now the only two
indications for combined HRT. HRT causes an increase in the bone mineral density &
reduces the incidence of vertebral and hip fractures.
>Twice the risk of dementia or mild cognitive impairment with HRT.
SUMMARY;
=>Better lipid profile
=>Decreased risk of osteoporosis
=>Increased risk of coronary events
=>Increased risk of forming blood clots
=>Increased risk of breast cancer
=>Increased risk of dementia

61.A plasma glucose level of less than 60 mg/dL is suggestive of hypoglycemia.In some
normal individuals, the blood glucose levels can reach around 45 mg/dL without
symptoms.Whipples triad is suggestive of true hypoglycemia.

62.Type-1 diabetes:
>NPH insulin;
Intermediate acting insulin with peak actions after 4-6 hours of injection & its effects
can last upto 16-18 hours.Lowering the morning dose would result in elevation of
blood glucose in the evening hours.
>Regular insulin;
=>Injection of regular insulin in the morning is responsible for controlling the blood
sugar level of diabetics during the breakfast.So, lowering the regular insulin dose
in the morning is likely to cause post-breakfast elevations in the blood sugar level
=>Lowering the dose of regular insulin before supper is likely to result in unwanted
elevations of the post-dinner blood glucose levels.
>Type-1 diabetes pts do not have endogenous basal insulin and require basal insulin by
injection of twice daily NPH insulin OR one injection of glargine insulin at bedtime.
>Acute effects of exercise on the blood glucose level are more commonly seen in pts
with type-1 than type-2 diabetes. Such hypoglycemic episodes during exercise in type-1
diabetes can be prevented by reducing the insulin dose, eating before exercising and
avoiding insulin injections in the exercising limb.

63.Thyroid status in pregnancy:


>The level of thyroxine binding globulin (TBG) increases during pregnancy, resulting in
higher levels of total T3 and total T4.In addition, human chorionic gonadotropin (hCG)
stimulates the production of thyroid hormones by binding to TSH receptors on thyroid
follicular cells.Higher productions of thyroid hormones is required during pregnancy to
saturate the higher levels of TBG & for the transplacental transfer of thyroid hormones
to the developing fetus.
>As a result of hCG induced thyroid stimulation, free T3 and free T4 are typically high

normal or borderline high, with TSH levels appropriately low normal or even mildly
suppressed.
>TSH is the best initial screening test for evaluating thyroid function during pregnancy as
the usual reference ranges for T3 and T4 studies do not reflect normal pregnancy.
>After TSH is measured, if TSH is abnormally low using the trimester specific range, then
measurement of thyroid hormonz levels will be necessary.The use of trimester specific
ranges for TSH & thyroid hormones (free & total) is crucial for accurate interpretation.
Measurement of free thyroid hormone levels using the standard analog assays may
provide inaccurate results due to artifacts produced by higher TBG and lower albumin
levels.
>In contrast to problems wth free thyroid hormone levels, measurement of total thyroid
hormone levels is reliable during pregnancy, with normal levels 1.5 times higher than
the non pregnant adult range.
>Diagnosis of thyrotoxicosis in pregnant women shd be accomplished using trimester
specific serum TSH levels and either total T4 and T3 with their reference levels adjusted
at 1.5 times the nonpregnant range or free T4 and free T3 estimations shd be used if
the trimester specific ranges are available (not available in all laboratories).
>Trimester specific ranges of TSH;
=>First trimester

-> 0.1 2.5 microU/mL

=>Second triemester-> 0.2 3.0 microU/mL


=>Third trimester

->0.3 3.0 microU/mL

>Radioactive iodine uptake scan is contraindicated in pregnant women.


MORAL OF STORY;
First is TSH , and use the trimester specific range for interpretation.
Second is T3 and T4, and use the total values by multiplying nonpregnant range
by 1.5.
Free T3 and Free T4 can be used if the trimester specific range is available in lab.

64.Gestational transient thyrotoxicosis may occur in first trimester of pregnancy due to


hCG mediated thyroid stimulation.Gestational hyperthyroidism is generally associatd
with minimal symptoms and mild biochemical hyperthyroidism, with resolution as
hCG levels decline after 12 weeks of pregnancy.Mild gestational hyperthyroidism is
not associated with adverse otucomes and does not require treatment.

65.Graves disease is the most common cause of hyperthyroidism during pregnancy


either from preexisting disease or new onset.

66.When a pts history or physical findings do not confirm a specific diagnosis, the
physician shd state in general terms what the most likely possibilities are and what
testing is necessary. When giving an opinion about a pts condition, the physician
must be careful to avoid giving false reassurance or a premature diagnosis (esp. an
unconfirmed psychiatric diagnosis) that cannot be supported by the available clinical
evidence.

67.Addisons disease/Chronic Adrenal Insufficiency:


Chronic adrenal insufficiency shd be suspected in pts with fatigue, weakness, poor
appetite, weight loss, abdominal pain, myalgias, decreased pubic and axillary hair (due
to decreased adrenal androgens), and increased pigmentation (due to co-secretion of
ACTH and MSH). Hyponatremia, hyperkalemia and hyperchloremic metabolic acidosis
are characteristic laboratory findings. A tendency for hypoglycemia can also be seen in
chronic adrenal insufficiency.
The next step in evaluation would be measurement of morning cortisol with concurrent
ACTH; low cortisol with elevated ACTH is diagnostic for primary adrenal insufficiency.If
results are equivocal, this can be followed by ACTH stimulation test (or the ACTH

stimulation test may be ordered at the same time as the morning cortisol to expedite
the evaluation).

68.Hypoaldosteronism usually presents as asymptomatic hyperkalemia with mild


metabolic acidosis.Hyponatremia is generally not seen unless there is concurrent
cortisol insufficiency.

69.Diabetes:
>Renal failure is an indication to manage a diabetic pt with insulin.Insulin is the mainstay
therapy for diabetic pts with renal failure.
>It is recommended to stop metformin and other sulfonylureas metabolized by kidneys,
in cases of renal failure to avoid toxicity and adverse effects.
>Pioglitazone, rosiglitazone, acarbose and repaglinide are metabolized through liver and
are alternatives to continue oral therapy in cases of renal failure.

70.Hashimotos thyroidits:
>High titers of antimicrosomal antibodies (anti-TPO) is diagnostic of Hashimotos
thyroiditis.Other lab tests which can support the diagnosis but are not routinely
used are : ultrasonography (thyroid enlargement and heterogeneous echo structure)
, radionuclide thyroid scan (heterogeneous uptake) and FNAB (lymphocytic infiltrate)
>Large needle aspiration biopsy may be required in pts with suspected thyroid
lymphoma. A rapid increase in thyroid size accompanied by compressive symptoms
is suggestive of thyroid lymphoma in pts with Hashimotos thyroiditis.
>Low titers of Anti-TPO antibodies in present in ~10% of general population.

71.Hypothyroidism:
>Hyperlipidemia is seen in more than 85% of pts.Although majority have isolated
elevations of LDL, combined elevations of LDL and triglyceride levels can occur.Isolated
hypertriglyceridemia is rare. Treatment with thyroid hormones can decrease the
cholesterol levels.
>Symptoms of coronary artery disease are usually masked due to deceased cardiac
contractility and hence decreased oxygen requirement of myocardium.Starting
levothyroxine may precipitate angina, so levothyroxine shd be started at small dose
and then titrated accordingly later.
>HTN occurs due to increase in peripheral vascular resistance and is more likely to occur
in pts with predisposing factors (e.g family hx, obesity).
>Ascites and enlargement of tongue occurs in long standing cases.

72.Why OCPs increase the need for levothyroxine in hypothyroid pts ?


Estrogen & estrogen like medications including OCPs increase the serum concentration
of thyroxine binding globulin (TBG) which binds the T3 and T4.In normal pts, feedback
loops promote increased endogenous production of thyroid hormones to compensate
for the increased number of bound thyroid hormone moleclules.However, in pts with
hypothyroidism, TSH feedback mechanism is unable to increase endogenous hormone
production and hence free T3 and T4 levels would fall, TSH would rise and there would
be an increased dose of levothyroxine required to maintain euthyroid state.
P.S: Androgens and glucocorticoids can cause the opposite problem since they decrease
the TBG.

73.-Cholestyramine
-Calcium carbonate

=> Decreased absorption of Levothyroxine

-Medications that inhibit


gastric acid secretion

74.-Phenytoin
-Phenobarbital

=>Increased hepatic metabolism of thyroid hormones.

-Carbamazepine
-Rifampin

75. -Glucocorticoids
-Beta blockers

=>

Inhibit the conversion of T4 to T3.

-Propylthiouracil
-Amiodarone

76.Lithium-> inhibits the creation of thyroid hormones within the thyroid gland.

77.Measurement of serum PTH is generally the first step in evaluating hypercalcemia.


Depending upon the PTH level, hypercalcemia can be divided into 2 broad categories;
parathyroid dependent (elevated PTH) & parathyroid independent (suppressed PTH).
Examples of parathyroid dependent hypercalcemia include primary or tertiary hyperparathyroidism, lithium inducd hypercalcemia & familial hypocalciuric hypercalcemia.
Parathyroid independent causes are most malignancies, vit-D toxicity, granulomatous
disease and milk alakali syndrome.

78.Hypercalcemia associated with malignancy is often characterized by rapid onset of


severely elevatd calcium levels (often >13 mg/dL).Although most pts wth malignancy
have overt clinical features of underlying cancer, hypercalcemia can be occasionally
be the presenting feature.

79.PTHrP in cancer-> causes hypercalcemia by binding to PTH receptors.

80.Immobilization induced hypercalcemia:


This is usually seen in pts with very high bone turnover (e.g adolescent pts and older pts
with pagets disease).When these pts are immoblilized, bone resorption is increased and
bone formation is decreased (uncoupling of bone turnover). The mechanism of this
uncoupling is not clearly known.Excessive bone resorption after immobilization leads to
hypercalcemia.It happens in days to weeks following immobilization.High serum calcium
levels suppress PTH secretion.Vit-D (25-OH and 1.25-OH2 D) levels are usually normal,
although 1,25-dihydroxy vitamin D levels may be lower due to the suppressed PTH
secretion (PTH stimulates 1-alpha hydroxylase activity in the kidneys , an enzyme
responsible for formation of 1,25-dihydroxy vitamin D).
Bisphosphonates are particularly usueful in decreasing the bone resorption.Their use
may preserve bone mass in pts who are immobilized for very long periods.

81.Factors that favor type 1 diabetes mellitus include rapid onset of osmotic symptoms
(polyuria, polydipsia), negative family history, Caucasian race and normal BMI.

82.Pts with type-1 diabetes shd be managed with a combination of a long acting basal
insulin (glargine/detemir) and meal time boluses of a rapid acting insulin.

83.Exenatide & liraglutinide-> GLP-1 analogs-> used in difficult to control hyperglycemia

84.Thyroid nodules shd be evaluated with TSH and thyroid ultrasound. Pts with
suspicious U/S findings (e.g large size, hypoechoic microcalcifications, internal
vascularity) or cancer risk factors (family hx, childhood radiation exposure,cervical
lymphadenopathy) shd undergo FNA. In addition, pts with normal or elevated TSH
have a higher risk of malignancy and shd be considered for FNA depending on U/S
findings.Low TSH levels suggest a hyper functioning nodule , which has a lower risk
of malignancy and shd be evaluated with thyroid scintigraphy.A hot nodule (incrzd
isotope uptake in the nodule with decreased surrounding uptake) is rarely malignant
and may be treated as benign hyperthyroidism. Decreased uptake / cold nodule is
again suspicious for malignancy and shd be evaluated with FNA.

85.Retrosternal goiter-> CT scan.

86.Thyroid cancer:
Ultrasound of the neck and cervical lymph nodes is the primary modality for initial
staging of thyroid cancer.
Pts with a small (< 1 cm) papillary thyroid tumor may be treated with thyroid
lobectomy.
Total thyroidectomy is recommended for tumors > 1 cm in diameter, tumor extension
outside of the thyroid, distant metastasis, and in pts with hx of head or neck dissection.
High dose levothyroxine is used following thyroidectomy to replace normal thyroxine
production and to suppress TSH stimulation of any potentially malignant residual tissue.
Radioactive iodine is used as adjuvant therapy following thyroidectomy for pts at risk of
metastatic disease, such as those with very large tumors, invasive tumors or lymph node

involvement.

87.Non-functioning pituitary adenoma:


It usually arises from the gonadotrophin-secreting cells (gonadotrophs) in the pituitary
gland.Normal gonadotrophs secrete LH and FSH (which are dimeric hormones consisting
of common alpha subunit and different beta subunit) but the dysfunctional cells in most
gondatroph adenomas secrete primarily just the common alpha-subunit. So thats why
we have characteristically incrzd alpha-subunits in non-functioning pituitary adenomas.
Clinical symptoms of alpha-subunit overproduction r usually minimal (non-functioning
adenoma) and the diagnosis is not apparent until the adenoma is large enough to cause
headaches or visual disturbances due to mass effect. Pts present with hypogonadism &
low gonadotropin levels. Large non-functioning pituitary adenomas that compress the
pituitary stalk may block the normal hypothalamic inhibition of prolactin secretion,
leading to a mild increase in prolactin levels.
The primary treatment modality is trans-sphenoidal surgery.

88.Pts with chronic hypoparathyroidism are usually treated with Vit-D and calcium.
After starting treatment with Vit-D and calcium, majority of pts will have an increase
in urinary calcium ( PTH usually helps in renal calcium reabsorption ) and low serum
calcium. In such pts with borderline low serum calcium & increased urinary calcium,
addition of thiazide diuretics z helpful becx it would not only decrease urinary calcium
but also helps to increase the serum calcium.

89.Vit-D ( calciferol ) is cheaper and effective & is preferred for chronic hypoparathyroid
pts.However 1,25-dihyrdoxy Vit-D (calcitriol) is rapid acting & is preferred in pts with
acute hypocalcemia. It can also be used in chronic hypoparathyroid pts refractory to

Vit-D.

90. Sulfonylurea overdose:


Sulfonylureas cause increased insulin secretion and large doses of sulfonylureas taken
accidentally or intentionally by diabetic pts can lead to hypoglycemia.
Dextrose is the first line treatment , however dextrose infusion can sometimes lead to
transient hyperglycemia that can cause reflex increase in insulin secretion further and
lead to recurrent hypoglycemia. In such cases, Octreotide , a somatostatin analog, shd
be considered as it can decrease the insulin secretion especially in pts with severe
sulfonylureas overdose.

91.Beta blockers mast the symptoms of hypoglycemia.

92.Pentazocine is a mixed opioid agonist-antagonist. It shd be avoided in opioid


dependant subjects as it can result in rapid withdrawal.

93.Sitagliptin-> dipeptidyl peptidase-IV inhibitor.

94.Exercise induced amenorrhea is due to a decrease in pulsatile secretion of LH, which


leads to a decline in estrogen production. Such pts are at risk of osteopenia and even
osteoporosis. Spontaneous fractures have been reported in these types of pts.
Treatment consists of improving the caloric intake; if this z not possible, pts r started
on hormonal replacement with oral contraceptives and supplementation with calcium
and vitamin-D.
Such pts can also develop breast and vaginal atrophy, mild hypercholesterolemia , and
infertility.

95.Pts with fasting blood glucose levels between 100 and 126 mg/dL are said to be
having impaired fasting blood glucose. Such pts are at risk of coronary artery disease
(even with normal lipid profile) and progression to overt diabetes.

96.Type-1 diabetics with blood glucose greater than 250 mg/dL shd avoid exercise becx
it can precipitate ketoacidosis.
Intense exercise programs have been known to cause progression of proliferative
diabetic retinopathy, therefore pts who have proliferative diabetic retinopathy shd
avoid certain exercises such as weight lifting which tends to increase the intravascular
pressure within the retinal vessels and predispose them to hemorrhage.

97.Gynecomastiais extremely common during puberty. Its occurrence is associated with


relative excess production of estrogens from the testis. Majority of pts with pubertal
gynecomastia improve after 18-24 months and rarely require medical or surgical
treatment

98.Pregnancy leads to increased TBG which leads to increased total T3 and total T4 but
free thyroxine is only mildly increased. This leads to increase in net thyroid hormone
requirements by 25-50%.Normal pts are able to compensate to saturate the incrzd
number f sites on TBG, but hypothyroids cant. This leads to increase in levothyroxine
requirements by 25-50% in hypothyroid pts who become pregnant.
TSH and total thyroxine shd be monitored and kept in the normal reference range for
pregnancy.

99.The use of liothyronine (T3) is not recommended during pregnancy as it can suppress
the production of T4 that can be transferred to the fetus. If a woman is using

combination T3/T4 therapy (including natural thyroid extracts) before pregnancy,


these shd be discontinued and changed to levothyroxine (T4).

100.Discontinuation of thyroid hormone replacement during pregnancy may result in


miscarriage , still birth, preeclampsia, or postpartum hemorrhage. Psychomotor and
cognitive impairment may also occur in the offspring of pts who are suboptimally
treated as placental transport of T4 is necessary for early fetal CNS development.

101.Drugs such as corticosteroids, anti-psychotics, diuretics, sympathomimetics & beta


blockers can precipitate hyperosmolar hyperglycemic non-ketotic coma. (HHS)

102.Initial management of pts wth HHS includes high flow IV fluids,continuous IV insulin
infusion and careful potassium replacement. Pts with HHS initially require very high
doses f IV insulin due to insulin resistanc inducd by stress, infections, corticosteroids
and hyperglycemia.
Subcutaneous insulin is generally started once the glucose has been corrected to
<200 mg/dL and the pt is able to eat consistently. Short acting human insulin or
insulin analogue (lispro, aspart or glulisine) shd be given for meals as well as
intermittent correctional boluses based on the level of blood glucose (sliding scale).
However, short acting insulin alone is not sufficient for glycemic control in most
hospitalized pts and basal insulin (e.g detemir/glargine) shd be added as well.

103.Amiodarone:
>Amiodarone causes thyroid dysfunction due to high iodine content.Hypothyroidism
(85%) is more common than hyperthyroidism (15%).
>Type-1 amiodarone induced thyrotoxicosis is due to activation of Graves disease.

Its treated with thionamides (methimazole or propythiouracil).Perchlorate can be used


decrease further iodine uptake by the thyroid gland.
>Thyroid function tests shd be monitored atleast every 6 months in euthyroid pts on
amiodarone.
>Its prudent to start the pt on low dose in elderly pts or those who have significant
coronary artery disease.
>Its not necessary to discontinue amiodarone if a pt becomes hypothyroid.Generally,
pts who hve amiodarone inducd hypothyroidism require a higher dose of levothyroxine
to bring their TSH within normal range becox amiodarone inhibits the conversion of T4
to its active form T3. The hypothyroidism is treated with levothyroxine.
>Amiodarone can cause thyroid dysfunction, corneal deposits, skin discoloration,
pulmonary fibrosis (lipoid pneumonitis) and liver toxicity.

104.Diabetes control in hospital setting:


>Insulin is the mainstay of therapy for treating diabetes in hospital setting and has 3
basic components:
1.Basal (intermediate or long acting <glargine/detemir/NPH>) insulin for controlling the
glucose levels between meals (necessary for all pts with type 1 diabetes and most with
type-2 diabetes)
2.Nutritional boluses (short acting <lispro/aspart/glulisine>) of insulin to control blood
glucose increase following a meal (not required if pts are not eating).
3.Correctional blouses (short acting) of insulin to decrease high glucose levels
(commonly referred to as sliding scale)
>During hospitalization, the dose of insulin shd generally be decreased by 20-30% becx
pts are eating less.However, in some pts the basal insulin may need to be increased
due to insulin resistance caused by infections, stress, inactivity and medications such

as corticosteroids.
>The American Diabetes Association suggests the following glycemic goals in hospital
settings. Premeal shd be 100-140 mg/dL , post-meal/ random shd be <180 mg/dL &
140-180 mg/dL in pts treated with insulin infusion.

105.Tight blood glucose control in pts with diabetes;


=>Decreases the risk of microvascular complications (e.g retinopathy, nephropathy)
=>Increases the risk of hypoglycemia
=>Uncertain effect on macrovascular complicatns (e.g Myocardial infarction,stroke,
peripheral arterial disease).

106.Diabetic neuropathy:
Strict blood glucose control decreases the risk of developing peripheral neuropathy
in pts with type-1 diabetes and possibly type-2 diabetes. Strict control also appears
to slow the progression of neuropathy and decreases the risk of other microvascular
complications (i.e nephropathy, retinopathy).
The effect of glycemic control in reversing establishd diabetic neuropathy z unclear.
Metabolic factors other than glycemic control such as hyperlipidemia, HTN, smoking
and obesity , may also play a role in development of neuropathy. However, there is
only limited data to show that improving non-glycemic metabolic factors prevents
or reverses diabetic neuropathy.
Symptoms of painful diabetic neuropathy will decrease spontaneously in up to 55%
of patients. However, remission is most likely when neuropathy is associated with a
significant metabolic event, such as ketoacidosis or severe weight loss following
bariatric surgery and is less likely in longstanding diabetes.

107.Neonatal screening for hypothyroidism:


Neonatal screening for hypothyroidism is performed by obtaining a small sample (few
drops) of blood from the heel pad and using a piece of filter paper to absorb the blood
sample. This test may be done within two to five days following delivery. It is performed
after the first 24 Hrs of life because there is a normal physiologic surge of TSH following
delivery. After 24 Hrs, the TSH levels gradually drop to normal levels or may remain
slightly elevated for the next few days. It is very important to avoid any delays in the
diagnosis and treatment to avoid permanent neurological deficits.
Most centers in United States use total T4 meaurement as a primary tool for screening
for neonatal hypothyroidism. If total T4 levels are low, TSH levels of the same sample is
measured and if the TSH levels are over 20 U/L , repeat testing (measurement of both
free T4 and TSH levels) is performed from a regular blood draw to confirm the diagnosis.
Some programs use TSH a s a primary screening tool.

108.Levothyroxine dose for treatment of hypothyroidism in neonate is higher than adult


dose.
109.Normal testosterone levels=> 300-1000 ng/dL

110.The earliest sign of puberty is nocturnal increase in the LH surge. This is followed by
a daytime increase in the levels of gonadotropins and increase in the testosterone
levels. On physical examination, enlargement of testis is one of the earliest signs of
onset of puberty.

111.Constitutional pubertal delay:


Constitutional pubertal delay is characterized by delayed puberty, retarded bone and a
positive family hx w/o any evidence of syndromic features or systemic disorder. Labs
usually reveal decreasd levels of LH and FSH (gonadotropins) & decreased Testosterone.
The first interventn for delayed puberty shd be reassurance and psychoscocial therapy.
Pharmacotherapy with testosterone for boys or estrogen for girls can be considered if
the pubertal delay or associated psychosocial problems are severe.
If pharmacotherapy is considered, it shd not be used until 14 yrs age in boys and 12 yrs
age in girls becx testosterone/estrogen caused decreased final adult height.

112. Gastric bypass surgery:


Gastric bypass surgery and other malabsorption syndromes lead to development
of the Vitamin-D deficiency which leads to hypocalcemia and then subsequent
hyperparathyroidism, which will bring the calcium levels to normal and lead to
hypophosphatemia due to renal excretion. Alkaline phosphatase levels will be
high on lab tests.Serum 25-hydroxy vitamin D level is a very sensitive indicator for
Vitamin-D stores and shd be measured in pts suspected of Vitamin-D deficiency. A
level <20 ng/mL is diagnostic.
To prevent the development of deficiencies following a bariatric procedure, all pts
need lifelong supplementation of vitamins (e.g B1, B2, folic acid, Vitamin-D) and
minerals (e.g iron, calcium , trace minerals). Pts shd receive atleast 2000-3000 units
of cholecalciferol (vit-D3) per day to maintain Vit-D levels of 30-50 ng/mL and some
may require even higher doses.

112.Subclinical hypothyroidism:
Is characterized by elevated TSH in the presence of a normal free T4 levels. Subclinical
hypothyroidism is often asymptomatic with a variety of somatic & cognitive symptoms.
Pts with subclinical hypothyroidism and a TSH level of >10 microU/mL or elevated titers
of anti-thyroid antibodies have a high likelihood of progression to overt hypothyroidism
and are likely to benefit from levothyroxine treatment. Pts with a TSH level of 4.5-10
microU/mL and typical symptoms of hypothyroidism or diffuse goiter may also benefit
from treatment. In addition, women with ovulatory dysfunction/ infertility or who are
pregnant or planning to become pregnant shd be offered levothyroxine.
BOTTOM LINE: Subclinical hypothyroidism is characterized by a normal free T4 levels
with an elevated TSH. Indications for levothyroxine therapy in subclinical
hypothyroidism include hypothyroid symptoms , pregnancy, infertitlity
or ovulatory dysfunction, goiter, positive antithyroid antibody titers, and
TSH > 10 microU/mL.

114.T3 has a short half life and produces wide fluctuations in blood levels, so routine
use is not recommended.

115.Conversion of T4 to T3 is regulated by TSH. Pts with primary hypothyroidism can


have normal circulating T3 levels despite impaired thyroid function because of
increased peripheral conversion due to elevated TSH levels. Serum T3 measurement
is therefore of limited value in hypothyroidism.

116.Symptomatic tachycardia in pts with hyperthyroidism can be quickly controlled with


beta blockers (usually propranolol or atenolol). Treatment with PTU or methimazole
will certainly improve cardiac condition of thyrotoxic pt but only after several weeks.

117.Anion gap= Sodium chloride bicarbonate

118.Metformin can cause lactic acidosis , which is an anion gap metabolic acidosis.
Metformin also can cause diarrhea, that can lead to hypovolemia that is a further
precipitant or risk factor for the development of metformin related lactic acidosis.
Other important risk factors for development of Metformin related lactic acidosis
besides hypovolemia are severe liver disease, congestive heart failure, and renal
dysfunction.

119.Normal serum phosphorus=> 2.5 5 mg/dL

120.Normal serum PTH=> 10-65 pg/mL

121.Familial hypocalicuric hypercalcemia=> Urinary calcium is <100 mg/24 Hrs.


Primary hyperparathyroidism patients=> Urinary calcium >250 mg/24 Hrs.

122.Parathyroidectomy is indicated for ;


All symptomatic pts
OR
Pts with one of the following ;
=>Serum calcium > 1 mg/dL above the upper limit of normal
=>Young age < 50
=>Bone mineral density <T -2.5 at any site
=>Reduced renal function (estimated GFR < 60 mL/min)

123. Thyroid nodules shd be evaluated using TSH level and thyroid ultrasound. Pts with
normal or high TSH and a confirmed nodule >1 cm in diameter on U/S shd undergo
Fine needle aspiration biopsy with nodule cytology. Pts with low TSH shd have a
radionuclide thyroid scan.
Repeat ultrasound z recommended fr follow up of benign nodules aftr confirmation
with FNA. U/S can also be used to monitor nodules <1cm that are not suspicious for
malignancy.

124.MEN-2 Syndromes:
Autosomal dominant disorder caused by germline mutations involving the RET protooncogene located on chromosome-10.
MEN-2 can be divided into 3 classifications;
=>MEN-2A predisposes to Medullary thyroid cancer (MTC), pheochromocytomas
(PCC), & hyperparathyroidism (due to parathyroid hyperplasia, or multiple
adenomas)
=>MEN-2B is accompanied by MTC, PCC, marfanoid habitus, mucosal neuromas and
skeletal deformities (e.g kyphoscoliosis, lordosis).
=>Familial medullary thyroid cancer (FMTC) is a subset of MEN-2A associated with
MTC only w/o PCC or parathyroid disease.

125.All pts with medullary thyroid cancer require measurement of serum calcitonin and
carcinoembryonic antigen, neck U/S for evaluation of regional metastasis, genetic
testing for germline RET mutations, and evaluation of coexisting tumors (hyperparathyroidism, PCCs). PCC is present in approximately 40% of pts with MEN 2 and
undiagnosed PCC can cause life threatening hemodynamic complications during &
after thyroid surgery.Plasma free metanephrines are usually elevated in PCC and

confirmed with 24-Hr Urinary fractionated metanephrines, catecholamines and


abdominal imaging. Once PCC has been ruled out, pts with nonmetastatic MTC may
proceed to thyroidectomy for definitve management.

126.A serum Parathyroid hormone which is at the upper limits of normal may be
pathologic in pts with hypercalcemia since it shd typically be low secondary to
negative feedback. In pts with an elevated PTH and hypercalcemia , a urinary
calcium greater than 200 mg/day is consistent with primary hyperparathyroidism
and excretion of less than 100 mg/day is consistent with familial hypocalciuric
hypercalcemia.
Pts with familial hypocalciuric hypercalcemia have a mutation in a calcium sensing
receptor (CaSR) which is found in the parathyroid glands and kidneys. These pts are
generally asymptomatic from their mild hypercalcemia and no surgical treatment is
necessary.

127.Chronic diarrhea can be caused by endocrinologc conditns such as hyperthyroidism,


Addisons disease or diabetes mellitus type-2.When it presents in a young woman
with amenorrhea, weight loss w/o evidence of malabsorption, generalized muscular
weakness, tachycardia, or tremors, hyperthyroidism must be suspected. A careful
physical examination may reveal a goiter, skin softness, hand tremors and anxiety
manifestation.
The best screening test is TSH, which shd have suppressed levels.

128.Prolactinoma:
Prolactinomas r relatively common cause of amenorrhea, infertility , and galactorrhea
in women.Elevated prolactin level suppress gonadotropin releasing hormone , leutinizng

hormone and estradiol , leading to symptoms of hypogonadism including hot flashes,


vaginal dryness and potentially osteoporosis if left untreated. Male pts can develop
symptoms of hypogonadism and gynecomastia. Large adenomas may also cause
headaches and compression of the optic chiasm , leading to visual field defects
(bitemporal hemianopsia).
The diagnosis of prolactinoma is strongly suggested by a prolactin level of >200 ng/mL
(normal z 5-20 ng/mL). Mild elevations (20-200 ng/mL) may also be due to prolactinoma
but other causes such as medications (e.g certain antipsychotics) , nipple stimulation,
pregnancy, hypothyroidism & stress shd also be considered. If prolactinoma is suspected
,MRI of the pituitary can confirm the diagnosis. In most cases, hyperprolactinenmia will
respond to treatment with dopamine agonists (e.g cabergoline, bromocriptine), which
decreases both the size and prolactin secretion of the adenoma. Surgical intervention
can be considered in pts who fail to improve with dopamine agonist therapy, especially
if they have significant neurologic symptoms.

129.American Thyroid Association recommends all individuals over the age of 40 to be


screened for thyroid dysfunction. The American College Of Physicians recommends
screening women over the age of 50 years wth findings suggestive of thyroid disease

130.Asymptomatic subclinical hypothyroidism does not require treatment. Treatment


is warranted in the presence of ;
1.Antithyroid antbdoies (anti-TPO)
2.An abnormal lipid profile
3.Symptoms of hypothyroidism
4.Ovulatory and menstrual dysfunction.

131.Why dont u just treat asymptomatic individuals with levothyroxine ?


To avoid causing bone loss and atrial fibrillation

132.Failure to Thrive (FTT) :


FTT is not a diagnosis in itself ; rather, it is a term used to describe failure to gain weight
in children younger than two years old. Children caregorized as FTT weigh less than the
5th percentile for their age; more severe cases involve a slowing of linear growth & head
circumference as well. The 3 causes of FTT are inadequate calorie intake, inadequate
calorie absorption and increased calorie requirements. Newborn infants will require
110 Kcal/Kg/day , while children up to twelve months need 100 Kcal/Kg/day to grow at a
normal pace. Psychosocial factors r very commonly involved in cases of FTT; this is why
the clinician must explore whether there are stressors in home environment. Organic
causes of FTT, while less common,may include feedng problems,milk protein intolerance
,inborn errors of metabolism, infection, cystic fibrosis, gastroesophageal reflux, or renal
tubular acidosis. Dietary modification is the best initial approach in an otherwise well
appearing child.
BOTTOM LINE: FTT is not a diagnosis, but rather an indication of an underlying problem.
Psychosocial factors frequently play a causative role in FTT, thereby
meriting close examination. Organic causes of FTT are less common.
Dietary modification is the first line of treatment in an otherwise wellappearing child with FTT.

133.If the history and physical examination are suggestive of an organic cause for the
childs FTT, then laboratory evaluation is warranted. Appropriate tests include
urinalysis and culture, hematocrit, blood urea nitrogen, calcium, electrolyte levels,
HIV ELISA test and Mantoux tuberculin testing.

134.Delayed puberty:
A pt is diagnosed with delayed puberty if he does not have testicular enlargement by 14
years of age or if his testicles are 2.5 cm or less in diameter. Another criterion is a delay
in the development for 5 years or more from the onset of genitalia enlargement. The
most common cause of delayed puberty is constitutional delay. The initial evaluation
involves the use of an imaging test to determine the bone age. Bone age that is older or
equal to the chronological age warrants further testing to rule out chromosomal
abnormality and endocrine causes.

135.Osteoporosis:
Risk factors include advanced age, prior fragility fracture ( fractures due to minimal
trauma such as falls from standing height or less) and family history of hip fracture.
Other modifiable risk factors include medications (e.g glucocorticoids), secondary
causes (e.g celiac disease, Vitamind-D deficiency, IBD, Rheumatoid arthritis, hyperthyroidism, hyperparathyroidism, Cushing syndrome, Chronic liver or renal disease),
low body weight, current smoking and excessive alcohol intake (>3 drinks/ day). In
addition, regular weight bearing exercise combined with muscle strengthening can also
decrease fall and fracture risk by improving strength, agility and posture. Current
guidelines also recommend daily calcium and vitamin-D supplementation.
P.S-> HRT can effectively decrease a pts osteoporosis risk but is usually not a first line
treatment due to increased risk of breast cancer , endometrial cancer and CVS
events.

136.Glucagonoma:
New onset diabetes + Necrolytic migratory erythema ( erythematous itchy painful rash,
with central clearing and crusty borders) + abdominal pain, wt loss, cramping, diarrhea.

Can cause flushing of the face due to secretion of othr substances such as VIP, calcitonin
and GLP1. Most glucagonomas are malignant and have metastasis , mainly in the liver,
at the time of diagnosis. The diagnosis is suspected clinically and then confirmed by
measuring glucagon levels, which are generally very high in these pts. The primary
treatment is surgical.

137. Pellagra is due to niacin deficiency and is characterized by dementia, diarrhea,


dermatitis, stomatitis and cheilosis. The associated dermatitis or skin rash is
symmetrically distributed in sun-exposed areas and often forms vesicles & blisters.

138.The skin rash in systemic mastocytosis is urticarial; however, maculopapular ,


vesicular, or pustular rashes have been reported, particularly in children. The rash is
often generalized. Rubbing the edge of the rash produces wheals and erythema; this
is called Dariers sign. About 50% of these pts have also got hepatosplenomegaly.

139.Painless thyroiditis: (PT)


Painless thyroiditis ( silent thyroiditis, subacute lymphocytic thyroiditis) usually has
transient hyperthyroidism followed by hypothyroidism and then recovery. PT is a
variant of chronic autoimmune thyroiditis (Hashimoto thyroiditis) with many similarities
to post partum thyroiditis. It is more common in women and is frequently associated
with various forms of immunotherapy (e.g interferon therapy for Hepatitis-C,
interleukin-2) and lithium treatment.
PT is due to autoimmune destruction of thyroid nodules , which activates proteolysis of
thyroglobulin and leads to thyroid hormone and thyroglobulin release. The initial
hyperthyroid phase continues until the thyroglobulin stores are depleted. As the thyroid
follicles recover to synthesize thyroid hormone, there may be a hypothyroid phase and

then recovery in most pts. Hyperthyroid symptoms gradually develop over 1-2 months
without exophthalmos. There may be minimal goiter with decreased TSH and increased
T4 and thyroglobulin levels. The TSH may reverse and start increasing in the hypothyroid
phase. Over 50% of pts have elevated antithyroid peroxidase antibodies.
PT shd be differentiated frm other more common causes of hyperthyroidism, especially
Graves disease. Graves disease typically is associated with goiter, exophthalmos, and
markedly increased RAIU. PT has minimal goiter, no exophthalmos and markedly
decreased RAIU. PT is a self limited condition and most pts need no specific treatment.
Beta blockers (e.g propranolol) can be given during hyperthyroid phase to symptomatic
pts and those at increased risk for developing atrial fibrillation. Because PT has low RAIU
and is not due to excess thyroid hormones synthesis, antithyroid medications (e.g PTU)
and radioiodine therapy are ineffective. Pts shd be followed closely for onset of possible
hypothyroidism.
BOTTOM LINE: Painless thyroiditis (silent thyroiditis , subacute lymphocytic thyroiditis) z
a self limited condition due to autoimmune destructn of thyroid follicles
and release of thyroid hormones. Most pts have gradual onset of hyperthyroidism followed by a hypothyroid phase and eventual recovery of.
Pts usually do not need specific treatmnt. Beta blockers (e.g Propranolol)
can be given during the hyperthyroid phase to symptomatic pts & those
at increased risk for developing atrial fibrillation.

140.OCPs effect on levothyroxine dose:


The estrogen component of the combination pill increases TBG levels , resulting in an
increase in the total T4 levels and a decrease in free T4 levels. A person with a normal
thyroid gland can compensate for these changes by increasing thyroid hormone
production. A hypothyroid pt on thyroid replacement , however, cannot compensate

for these possible physiologic changes as pt is dependent on exogenous levothyroxine


therapy. The dose of levothyroxine may therefore need to be increased in hypothyroid
pts taking OCPs. After starting combination OCPs in pts with hypothyroidism, TSH levels
shd checked in 12 weeks and the dose of levothyroxine shd be adjusted accordingly.

141. Drugs that increase the TBG and decrease the free T4- may require higher dose of
thyroxine replacement;

Estrogen
Tamoxifen
Raloxifene
Methadone
Heroin

142.Drugs that decrease TBG and increase free T4-may require lower dose of thyroxine
replacement;

Androgens
Danazol
Anabolic steroids
Glucocorticoids

143.Pancreatogenic diabetes: (PD)


Occurs due to islet cell damage from chronic pancreatitis & resulting insulin deficiency.
PD, sometimes called type 3c diabetes, can be differentiated from type-2 diabetes by
measuring C-peptide and insulin levels. Pts with PD will have low C-peptide and insulin
levels compared to type-2 diabetes. In addition to impaired insulin secretion, PD is also
associated with insulin resistance.
Many pts with mild hyperglycemia associated with chronic pancreatitis can be treated
initially with metformin, which increases insulin sensitivity and also lowers the risk of
pancreatic cancer in chronic pancreatitis. However, due to the destruction of islet beta

cells, most pts with PD will require insulin therapy. Pts with PD have loss of both insulin
producing beta cells and glucagon-producing alpha cells and therefore at increased risk
of severe insulin induced hypoglycemia. (In type-1 diabetes, there is a loss of beta cells
but alpha cells are largely intact and can secrete glucagon in response to hypoglycemia).
*Insulin secretagogues (e.g glyburide) are ineffective in PD due to the loss of beta cells
and are associated with increased risk of pancreatic cancer in pts with PD.
*Sitagliptin and other DDP-IV inhibitors are associated with an increased risk of
pancreatitis and generally shd be avoided in pts with a history of pancreatitis.

144.Pioglitazone causes sodium and water retention and shd be avoided in pts with CHF

145.MEN-1:
MEN-1 syndrome is composed of hyperparathyroidisim, gastrinoma/ Zollinger-Ellison
syndrome (pancreatic tumor) and pituitary tumor (remember the 3Ps).
Hyperparathyroidism in MEN-1 is caused by hyperplasia of the parathyroid glands.
Removal of 3 glands or total parathyroidectomy with autotransplantation (keeping
part of parathyroid gland in forearm) is necessary. A normocalcemic state after the
parathyroid surgery causes marked improvement in the symptoms of Zollinger-Ellison
syndrome.

146.Acromegaly:
>Pts with untreated acromegaly have increased mortality mainly due to cardiovascular
disease.Approximately 50% of pts with acromegaly have HTN. A large proportion also
has LV dysfynction at the time of diagnosis. Other cardiovascular diseases in pts with
acromegaly are asymmetrical septal hypertrophy , conduction defects, coronary
atherosclerosis and myocardial fibrosis. Early cardiovascular disease in acromegaly is

reversible by treatment ( transphenodial surgery).


>Diabetes, respiratory problems and malignancies account for rest of the mortality and
morbidity rates.
>Pts with acromegaly have increased risk of colon cancer. Most authorities recommend
screening fr colon cancer every 3-5 years in all pts wth active acromegaly.The incidence
of colonic polyps z also increased in acromegaly.Despite thez facts, the risk of mortality
from colon cancer and colonic polyps is much lower compared to cardiovascular
mortality.

147.Primary hyperaldosteronism: (Conns Syndrome)


>HTN + Hypokalemia -> Primary Hyperaldosteronism.
>Hypokalemia-> Nephrogenic diabetes insipidus-> Polyuria and polydipsia.
>Plasma Aldosterone to renin activity (not renin levels) ratio z first test, with ratio >30,
with Aldosterone levels >15 ng/dL is diagnostic.

148. DIABETES:
>Diabetic pts shd receive annual ophthalmologic evaluations for diabetic retinopathy
as the problem is usually asymptomatic until the damage is severe. Strict glycemic
control can prevent the development & progression of microvascular complications
of diabetes mellitus including retinopathy.
>Statins have been proven to prevent cardiovascular events (e.g MI) and are
recommended for all diabetic pts aged 40-75.
>Mild non profilerative diabetic retinopathy does not require specific treatment in
most cases. Panretinal photocoagulation z used in treatment of severe profilerative
retinopathy and may be considered for pts wih mild to moderate retinopathy who
have proliferative retinopathy or impaired vision in the contralateral eye.

>ACEIs have been shown to decrease the progression of diabetic nephropathy in pts
with moderately increased albuminuria (30-300 mg/24 Hrs)
>Low dose aspirin is recommended to prevent cardiovascular events in diabetic pts
with a history of prior events (e.g stroke, MI).

149.Hypothyroidism and Surgery:


There z a risk that hypothyroidism may worsen perioperative outcomes,although urgent
or emergent surgeries can usually be done on pts with hypothyroidism as long as there r
no symptoms of myxedema coma or other findings to suggest severe hypothyroidism.
If a pt is in bad need of CABG, dont first start the pt on levothyroxine , firstly because it
can increase myocardial oxygen demand resulting in myocardial infarction, angina or
cardiac dysrhythmias resulting in prolonged wait for CABG and secondly because
hypothyroidism , in the absence of myxedema coma or other severe symptoms , usually
only mildly increases the perioperative risk.
*Levothyroxine generally needs to be started slowly and gradually in pts with cardiac dx
to avoid precipitating myocardial ischemia.

150.The most common causes of mental status changes in elederly pts are medications,
infections (e.g UTI, Pneumonia) and metabolic abnormalities. Thyroid dysfunction,
both hypothyroidism & thyrotoxicosis, also frequently causes mental status changes
in the elderly but it may present with nonspecific or atypical symptoms.

151.Elderly pts with thyrotoxicosis may not have classic manifestations.


Apathetic thyrotoxicosis is an atypical presentation of hyperthyroidism in older pts
that is characterized by lethargy, confusion and depression. It is often misdiagnosed
as dementia.

152. EEG can be useful in the diagnosis of nonconvulsive status epilepticus, esp in pts
showng subtle signs of convulsive disordr such as nystagmus, hippus (spontaneous
episodic pupillary contractions) or automatism (e.g facial twitching).

153.DKA:
>The initial management consists of IV fluids and IV insulin.
>Potassium is monitored closely and added to the IV fluids if the potassium z <5.2 mEq/L
to prevent hypokalemia due to IV insulin.Hold the insulin if the potassium has fallen to
<3.3 mEq/L
>IV insulin shd be continued until the anion gap has normalized.
>If the serum glucose falls to <200 mg/dL but the patient still has an elevated anion gap,
the rate of insulin infusion shd be halved and dextrose added to the IV fluids to prevent
hypoglycemia.
>Pts with DKA are switched to subcutaneous insulin once they are able to eat and the
glucose level has fallen to <200 mg/dL along with either normal anion gap <12 mEq/L
or bicarbonate > 15 mEq/L. SQ insulin is given as combination of long acting insulin
(basal insulin <e.g glargine/detemir>), rapid acting insulin (bolus insulin <e.g regular/
lispro/aspart>) with meals and correctional insulin for hyperglycemia on a sliding scale.
However, there z a slight delay in absorption of insulin when it is given subcutaneously.
It is recommended that the IV insulin infusion be continued for about 1-2 hours after
the SQ insulin is started to give the SQ insulin enough time to take effect and prevent
rebound ketoacidosis.
>Administration of bicarbonate is considered only in pts with severe acidosis (pH<7.0),
which can result in decreased cardiac contractility and systemic vasodilation.

154. Understand that pts with type-1 diabetes r predisposed to get other autoimmune
diseases such as Addisons disease, hypothyroidism, pernicious anemia, primary
hypogonadism, celiac disease etc to name a few.

155. Look for following to get to addisons disease;


=>Wt. loss
=>asthenia
=>Low blood pressure
=>Hypoglycemia
=>eosinophilia
=>Anion gap acidosis
=>Prerenal azotemia
=>borderline sodium levels
=>Hyperkalemia

156.Wt. loss and systemic illness can lead to functional hypothalamic amenorrhea

157.Hyperthyroidism in pregnancy:
In pregnant and non pregnant pts alike, Graves disease is the most common cause of
hyperthyroidism. The workup of hyperthyroidism in pregnant pts is different than it is
in the general population however, because radiodine cannot be used to obtain uptake
values.Measurement of Thyroid stimulating antibodies can instead be used to confirm a
diagnosis of Graves disease.
Thionamide medications are treatment of choice to control symptms of hyperthyroidism
in pregnant pts. Methimazole (MMI) z a potential teratogen & propylthiouracil (PTU) has

been associated with liver failure. Given these concerns, the recommended approach is
to treat with PTU during the first trimester when the susceptibility to teratogens is
highest, and then switch to MMI for the second and third trimesters to decrease the risk
of liver failure.

158.Adrenal incidentaloma:
All adrenal masses require workup for hormone secretion or malignancy. In any pt with
adrenal mass, essential lab studies include serum electrolytes, dexamethasone
suppression test, and 24-Hr urinary catecholamines, metanephrines, vanillyl mandellic
acid, and 17-ketosteroids measurement.
Surgical excision is recommended for all functional tumors, all malignant tumors (which
demonstrate a characteristic heterogenous appearance on imaging) and all tumors
greater than 4 cm. All other masses can be managd conservatively with serial abdominal
imaging and removed if they increase in size.

159.Hyperosmolar hyperglycemic state:


>As opposed to DKA, they have negative serum ketones, normal anion gap and normal
bicarbonate.
>Osmotic diuresis leads to dehydration and neurologic symptoms of confusion.
>Aggressive fluid resuscitation with isotonic saline is the first step in management as
these pts are generally volume depleted. IV insulin shd be started after the pt has been
partially resuscitated.
>Pts are frequently hyperkalemic due to relative insulin deficiency and hyperosmolarity
but actually these pts have low overall body potassium caused by increased urinary
excretion.
>Aggressive management of hyperkalemia shd be AVOIDED in HHS since these pts have

actually decreased total body potassium.


>Potassium supplementation is frequently necessary when the serum levels fall below
5.3 mEq/L

160. Stress can cause atrial fibrillation.

161.Peripartum Diabetes management:


The recommended blood glucose levels in peripartum period is 72-126 mg/dL.Levels
higher than this are associated with fetal hypoglycemia following delivery due to
glucose-induced insulin production in the fetus. However, insulin requirements typically
decrease durng active labor due to muscle contractions. Also, insulin resistanc decreases
rapidly following delivery of the placenta and pts with gestational diabetes mellitus may
not need any postpartum management.
In pts taking bedtime NPH, the best approach z to give full dose NPH on the night before
induction of labor.Blood glucose shd be monitored subsequently every 1-2 Hrs and short
acting insulin administered if the glucose is >126 mg/dL.
In pts treated with long acting insulin (glargine/detemir), most experts recommend
reducing the dose to 50-70% on the night before induction of labor.
In pts taking treated with multiple insulin injections, the morning dose of the NPH or
glargine/detemir shd decreased by atleast 50% before the induction of labor.
A light meal is typically allowed before the active phase of vaginal delivery; however, if
pts r not eating or drinking, IV infusion of dextrose-containing fluids may be warranted.

162.Post-thyroid cancer management :


After thyroid cancer is treated with thyroidectomy and radioiodine ablation, we give
suppressive doses of levothyroxine to firstly provide thyroid hormone replacement

and secondly to suppress TSH to prevent recurrent disease. However, suppressive doses
of levothyroxine are associated with bone loss and atrial fibrillation. Therefore degree of
TSH suppression will depend on the initial tumor stage and risk of recurrence.
If the risk of recurrence is low, then we keep TSH just below the normal range
(0.1-0.5 microU/mL) in the initial 6-12 months and then later on, keep in low
normal range.
In intermediate risk tumors, we target to keep the TSH persistently below
normal (0.1-0.5 microU/mL)
In pts with large, aggressive tumors or distant metastasis, the TSH is typically
kept below 0.1 microU/mL for several years after initial treatment.

163.Levothyroxine shd be taken on empty stomach, separately from other medications.


Taking levothyroxine with meals or other medications can impair its absorption and
make management erratic or unpredictable.

164.Pts with cushings syndrome due to ectopic ACTH overproduction have higher ACTH
than pts with cushings disease.
Inferior petrosal sinus sampling is used to differentiate the source of ACTH product
(Pituitary versus ectopic).

165.Cushings syndrome with an adrenal etiology is biochemically characterized by a


non-suppressible high dose dexamethasone suppression test and low plasma ACTH
levels.
The best next step wud be to do CT or MRI of the adrenals, although MRI z smtimes
better for further characterization of the adrenals.

166.Pseudohypoparathyroidism:
Occurs due to PTH resistance on its target tissues. Characterized by long standing
hypocalcemia & hyperphosphatemia, evidenced by bilateral cataracts & calcification
of basal ganglia (Fahrs syndrome).PTH levels are elevated.
Type-1A has features of Albright hereditary osteodystrophy in addition to
hypoparathyroidism. Albright hereditary osteodystrophy (AHO) is characterized
by short stature, round facies, short fourth and fifth metacarpals and a short
neck.
Type-1B does not have features of AHO.
Pseudopseudohypoparathyroidism is a variety of pseudohypoparathyroidism in
which pts do not have hypocalcemia and hyperphosphatemia becx the PTH
resistance is mild, but they do have features of AHO.

167.Acute hyperphosphatemia can cause a decrease in serum calcium levels. This is


classically seen in pts with seizures, tumor lysis, or acute renal failure.

168.Pseudohypoparathyrodism-> dec Ca, High Phosphorus, High PTH, Normal 25-OHD


Hypoparathyroidism---------> dec Ca, High Phosphorus, Low PTH, Normal 25-OHD
Vit-D deficiency---------------> dec Ca, dec Phosphorus, High PTH, decrzd 25-OHD

169. Anti-Diabetic side effects:


>Sulfonylureas (Glyburide, Glipizide, Glimepiride),
Meglitinides (Nateglinide,Repaglinide);
=>Weight gain
=>Hypoglycemia
>SGLT-2 Inhibitors (Canagliflozin, Dapagliflozin);
=>Urinary tract infections
=>Mycotic genital infections
=>Hypotension
>GLP-1 analogs (exenatide, liraglutinide)
=>Pancreatitis
>Biguanides (Metformin)
=>Lactic acidosis
>Thiazolidinediones (Pioglitazone, Rosiglitazone)
=>Fluid retention/Heart failure
=>Weight gain
>DPP-4 inhibitors (Sitagliptin, Saxagliptin)
=>Nasopharyngitis
>Alpha glucosidase inhibitors (Acarbose, Miglitol);
=>Diarrhea
=>Flatulence
*Nateglinide is a short acting insulin secretagogue that is typically dosed 3 times daily
with meals. Metabolism of Nateglinide in liver produces biologically active metabolites
that are excreted by kidneys. The risk of hypoglycemia with nateglinide increases in
renal failure due to accumulation of these active metabolites. The risk of hypoglycemia
also increases in pts who consume excessive alcohol.

*Newer agents such as GLP-1 analogs, DPP-4 inhibitors & SGLT-2 inhibitors do not cause
significant hypoglycemia when used as monontherapy.

170.Androgen insensitivity syndrome:


The combination of primary amenorrhea, bilateral inguinal masses and breast
development without pubic and axillary hair is strongly suggestive of androgen
insensitivity syndrome. They are 46 XY genotypes having normal concentration of
Testosterone, but due to a mutation in androgen receptor (AR gene), there is resistance
to testosterone and hence the pubic and axillary hair do not develop and testosterone
gets converted to estrogen in the peripheral tissues leading to breast development and
female phenotype. As the mullerian structures are dependent on Testosterone, so they
dont develop ( uterus, fallopian tubes ) and a blind vaginal pouch is present, leading to
primary amenorrhea.

46XY + Normal concentrations of testosterone + primary amenorrhea + bilateral inguinal


masses + blind vaginal pouch + Breasts development
I
Androgen insensitivity syndrome

171. Amenorrhea beyond 15.5 years is considered abnormal even if secondary sexual
characteristics are present.

172.A transverse vaginal septum presents with normal secondary sexual characteristics
and pelvic pain due to accumulation of blood at the level of obstruction.

173.Klinefilters syndrome;
=>Male phenotype
=>Small testes
=>Gynecomastia

174.Schmidts syndrome;
Also called polyglandular autoimmune failure type-II is characterized by combinatn
of Addisons disease, type-1 diabetes mellitus and autoimmune thyroid disease.
Other autoimmune disorders may also occur, such as pernicious anemia, premature
ovarian failure, vitiligo and celiac disease.

175.Pts with type-2 DM can have caloric deprivation during an acute illness and this will
result in starvation ketosis w/o a significantly increased anion gap.

176.Maturity onset diabetes mellitus of young (MODY) and type-1 DM are similar in that
these diseases occur in younger individuals ; however pts with MODY have a positive
family history (in most cases) and modest hyperglycemia and generally do not have
ketoacidosis.

177.Diagnose the diabetes with either of the following ;


->HbA1C >6.5%
->Random blood glucose >200 mg/dL with symptoms of hyperglycemia (polyuria,
polydipsia)
->Blood glucose >200 mg/dL 2 Hrs after 75 g glucose load (Oral glucose tolerance
test)
->Blood glucose >126 mg/dL after 8 Hrs of fasting

178. Impaired glucose tolerance / Prediabetic stage;


->HbA1c 5.7-6.4%
->Random blood glucose 140-199 mg/dL with symptoms of hyperglycemia
->Blood glucose 140-199 mg/dL after 2 Hrs of 75 g glucose load
->Blood glucose 100-125 mg/dL after 8 Hrs of fasting

179.Asymptomatic pts with an abnormal screening test for diabetes require repeat
measurement with the same test to confirm the diagnosis. Pts with symptomatic
hyperglycemia and abnormal screening tests can be diagnosed with diabetes
without repeat confirmatory testing.

180.Subclinical thyrotoxicosis:
Suppressed TSH levels along with normal thyroid hormone levels. The most common
causes of subclinical thyrotoxicosis are treatment with levothyroxine, nodular thyroid
disease, Graves disease & thyroiditis.Subclinical thyrotoxicosis inducd by leveothyroxine
is simply treated by reducing the dose. In some cases, the etiology cannot be determind
and TSH becomes normal if repeated in a few weeks.

181. Pts who have mildly suppressed TSH but normal T4 and T3, no symptoms, normal
heart rhythm and normal bone density are not intensively investigated because no
treatment is necessary and there is a high chance of normalization of TSH levels.
Repeating TSH after 6-8 weeks is generally performed.

182.70/30 insulin (70% NPH-30% rapid acting insulin); (versus Basal insulin)
=>Better glycemic control
=>Higher chance of hypoglycemia
=>Weight gain

183.Long acting basal insulins;


=>Lower chance of hypoglycemia compared to NPH (70/30)

184.Type-2 DM is characterized by both insulin resistance and progressive loss of


pancreatic beta cell function over time. As insulin levels decline with time, most pts
require advancement of of anti-diabetic medications and eventually insulin therapy
to maintain glycemic control.

185.Non-Alcoholic steatohepatitis: (NASH)


Look for obesity, HTN and dyslipidemia. There will be elevated liver enzymes.
Metformin is especially in the management of obese diabetic pts because its use is
associated with weight loss.It decreases fasting glucose by around 20%. Its helpful in
hepatic steatosis by improving the glycemic control and helping to stabilize the obesity.
Metformin along with Pioglitazone, decreases the triglycerides, increases the HDL and
improves NASH.
However, thiazolidinediones are not recommended because they can cause weight gain
and potentially hepatotoxic, both of which can potentially exacerbate NASH.
Insulin and sulfonylureas can cause weight gain.
Acarboses action is too weak to be used as monotherapy.

186.DKA:
>Defined as metabolic acidosis (pH <7.3 or serum bicarbonate <15 mEq/L) in the setting
of hyperglycemia (serum glucose >200 mg/dL).Clinically characterized by polyuria,
polydipsia, vomiting, abdominal pain, dehydration, kussmaul respirations (deep, rapid
breathing)
>Treatment begins with 10 mL/Kg bolus of isotonic fluids (e.g normal saline, ringers
lactate) given over an hour followed by initiation of an insulin drip. A small bolus given
over an hour prior to initiating insulin therapy has been shown to minimize the risk of
cerebral edema compared to starting an insulin drip immediately.Potassium containing
IV fluids shd be administered simultaneously with the insulin drip for pts with normal
or low potassium levels as insulin moves potassium intracellularly causing hypokalemia
>Ongoing management consists of blood work every 1-2 Hrs & close titration of IV fluids
to correct electrolyte derangements and acidosis. During this period, pts shd be closely
monitored fr signs of cerebral edema such as altered mental status, lethargy, headache
and vomiting. A head CT shd be ordered for pts in whom cerebral edema is suspected.
Once the metabolic acidosis has resolved and the anion gap has closed, the pt can be
transitioned to a subcutaneous insulin regimen.
>Avoid bicarbonate as it can cause hypokalemia, cerebral edema and metabolic alkalosis
>When to admit to Intensive care unit?
Mild cases of DKA with no vomiting can be managed by an experienced outpatient
team or on a general inpatient ward. However, pts with severe DKA (e.g pH <7.1,
bicarbonate <15 mEq/L, altered mental status) shd be admitted to an intensive
care unit for close monitoring as they are at greatest risk for complications (e.g
cerebral edema).

187.Premature menopause treatment:


Although HRT does increase the risk of MI, strokes, breast cancer, DVTs in older pts , but
these do NOT apply to younger pts with menopause (e.g premature ovarian failure).
Young pts with hypoestrogenic symptoms (e.g hot flashes) and low bone density can be
successfully treated with combination of estrogen and progesterone either continuously
or cyclically. Estrogen is the gold standard for hypoestrogenic symptoms. There will be
reduction in bone turnover and improvement in bone density.
*Estrogen is combined with progesterone to prevent endometrial cancer
*Bisphosphonates are used low bone density in older pts, not young pts.

188.Etidronate is first generation bisphosphonate that has to be used intermittently in


the treatment of osteoporosis. Its continuous use produces a mineralization defect.
Its not approved by FDA for treatment of osteoporosis.

189.Raloxifene z selectve estrogen receptor modifier (SERM) whch selectively stimulates


estrogen receptors on bone cells and has antagonistic properties on other estrogen
responsive tissues. Although raloxifene increases bone mineral density at vertebral
and non vertebral sites and reduces the risk of fractures, it is likely to worsen the
hypoestrogenic symptoms.

190.Thyroid lymphoma usually presents as rapid enlargement of thyroid gland in pts


with Hashimotos thyroiditis.(rubbery goiter + high titer Anti-TPO Abs + hypothyroid)
Pembertons sign is the presence of facial plethora or neck vein distension when the
arms are raised and confirms an enlarged thyroid gland as the cause of esophageal
obstructive symptoms.

191.Historical features suggestives CAH include menstrual irregularities, hirsutism,


or virilization, cushingoid habitus and acanthosis nigricans.

191.Inability to palpate inferior edge of thyroid indicates substernal extension.

192.Colloid goiter is a cause of thyroid enlargement in adolescent girls who have normal
thyroid function tests and negative antithyroid antibodies.

ALLERGY AND IMMUNOLOGY:


1.PRIMARY CNS LYMPHOMA:
Primary CNS lymphoma is a common malignancy in pts with advanced HIV infections
and is strongly related to EBV. It is usually associated with a significant degree of
immunosuppression. Most pts with PCNSL have a persistently depressed CD4 cell count
(<50/microL).
Institution of HAART therapy is associated with an improved prognosis, especially in pts
who demonstrate an improvement in the immune status (e.g increase in CD4 count and
decrease in viral load), becx the degree of immunosuppression seems to be the major
determinant of these patients surivival.

2.Recurrent bacterial infections in an adult pt may indicate a humoral immunity defect.


Quantitative measurement of serum immunoglobulin levels helps to establish the
diagnosis
*Selective deficiency IgG3 alone is more common in adult females and is associated
with recurrent sinopulmonary infections as well as gastrointestinal infections.
*Common variable immunodeficiency is associated with suppressed cell immunity
and increased risk of malignancy.

3.The allergens most frequently associated with asthma is house dust mites.

4.DTaP;
Anaphylaxis within seven days is an absolute contraindication
Encephalopathy within seven days of administration is an absolute
contraindication

Temporary contraindications include moderate or severe illness, with the vaccine


administered as soon as the illness resolves.
Mild acute illness with or without fever is not a contraindication to vaccine
administration.
If high fever (>104.8 F), shock, inconsolable crying for 3 or more hours, or seizure
occurs within 24 hours of receiving the vaccine, then subsequent doses should be
given with caution.

5.There are currently no contraindications to the administration of Hib, IPV and PCV
vaccines.

6.When possible, avoidance of the antigen triggers is typically the first step in the
management of allergic disorders.

7.Allergic rhinitis;
Avoiding antigen trigger is first step.
If allergen not identified, nasal corticosteroids is the first line therapy.

8.TRANSFUSION REACTIONS:
Anaphylaxis;
>occurs in secs-mins
>Occurs in Ig-A deficient pts.
>Red cells shd be washed to remove as much of the plasma as possible for pts
with IgA deficiency or prior allergic transfusion reaction.
Acute hemolytic reaction;
>Occurs in the first hour
>Presents with fever, chills, flank pain and hemoglobinuria. These can progress
to renal failure and DIC.
>The direct antiglobin test (Coombs test) is positive and plasma free Hb is 25
mg/Dl or more. Urinalysis also shows hemoglobinuria.
>Careful cross matching of the blood prevents acute hemolytic reactions.

Febrile non-hemolytic reaction;


>Occurs from 1-6 Hrs.
>When red cells and plasma are separated from whole blood, small amounts of
residual plasma and/or leucocyte debris may remain in the red cell concentrate.
During blood storage, these leucocytes release cytokines, which when transfusd
can cause fever, chills, and malaise without hemolysis.
>Management includes stopping the transfusion to exclude other serious rxtns,
administering antipyretics and using leukoreduced blood products for future
transfusions.
>Leukoreduction involves reducing the number of transfused leukocytes through
filtering or other methods such as saline washing, freezing and deglycerolizing or
buffy coat removal.
>Leukoreduction also reduces the risk of HLA alloimmunization and transmission
of CMV (which typically resides in leukocytes).
Transfusion related acute lung injury (TRALI);
>Occurs from 1-6 Hrs.
Delayed hemolytic reaction;
>Occurs within 2-10 days.

9.Whole blood is transfused rarely except in cases of acute hemorrhage e.g trauma

10. Warming blood is recommended only during the periods of massive transfusions.

11.ANAPHYLAXIS:
Skin and mucous membrane symptoms are common and include itching, hives,
and swelling of the lips and/or tongue.
10-20% of the pts dont have skin findings and their absence does not rule out
the diagnosis.
Respiratory distress is common and can be particularly severe in the setting of
underlying pulmonary problems such as emphysema.
GI symptoms such as nausea, vomiting, diarrhea and crampy abdominal pain may
also occur.
Hypotension may be the only manifestation of anaphylaxis at times

Presentation of anaphylaxis is variable and the absence of certain does not rule
out the diagnosis.
IM epinephrine injection into the thigh with the pt in supine position (or semi
recumbent if vomiting) with elevation of lower extremities is recommended.
If hypotension does not respond promptly and completely, large volume of IV
fluids resuscitation with a crystalloid such as normal saline is recommended.
IV epinephrine may be necessary if the pts blood pressure worsens despite the
IM epinephrine and fluid resuscitation.
Bottom line: Anaphylaxis presents with hives, wheezing, GI symptoms &/or hypotension
following exposure to a known or potential allergen. The first line treatmnt
is rapid administration of intramuscular epinephrine with the patient in a
supine or semi-recumbent position. Fluid resuscitation, bronchodilators,
antihistamines and glucocorticoids may also be required.

12.ALLERGIC BRONCHOPULMONARY ASPERGILLOSIS:


Is a hypersensitivity disorder that occurs in pts with asthma or cystic fibrosis. It is
associated with noninvasive colonization of the airways by Aspergillus species. The
pathophysiology of ABPA is related to an exaggerated IgE and IgG mediated immune
response to the Aspergillus fungus in the context of preexisting asthma.
Typical clinical features include recurrent asthma exacerbations, fever, lethargy, cough
with production of brown mucus plugs, occasional hemoptysis and fleeting infiltrates on
lung imaging. Other diagnostic criteria include immediate reaction to Aspergillus antigen
skin testing, elevated total and Aspergillus specific IgE, serum precipitins to Aspergillus
antigen, central bronchiectasis on high resolution CT scan and peripheral eosinophilia.
Once the diagnosis of ABPA is confirmed, glucocorticoids and itraconazole are used to
control inflammation and prevent irreversible damage.

13.CHURG STRAUSS SYNDROME:


Aka eosinophilic granulomatosis with polyangitis. It is an uncommon autoimmune
vasculitis. It is often associated with difficult to control asthma. Other clinical features
include allergic rhinitis with nasal polyps, chronic sinusitis, mononeuropathy multiplex
and skin manifestations (e.g granulomas, palpable purpura).

14.HPV VACCINATION:
Although the vaccination can be started at any time at age 9-26, the
recommended time for the first dose is age 11-12, ideally well before the onset of
sexual activity to maximize the protection.
The HPV vaccine series consist of 3 doses that are ideally administered within a 6
month period. When more than the recommended time has elapsed between
the doses, it is appropriate to resume the series where the patient left off; the
entire series need not be restarted. If the vaccine used initially is unknown, the
series can be resumed with any type.
History of prior sexual activity, previous HPV infection, anogenital disease and
abnormal Pap smear are NOT contraindications to vaccine administration.
The HPV vaccine should not be administered to pregnant women due to limited
safety data.
History of hypersensitivity reaction to yeast is an absolute contraindication to the
quadrivalent HPV vaccine as it is produced using Saccharomyces cerevisiae
(Bakers yeast).

Some forms of bivalent vaccine contain latex and should not administered to pts
with latex allergies.
HPV vaccine can be administered to immunocompromised and
immunosuppressed individuals.

15.VITAMIN B12 DEFICIENCY ANEMIA/PERNICIOUS ANEMIA:


Is characterized by the presence of two major type of antibodies
a.Anti-intrinsic factor antibodies.
b.Anit-parietal cell antibodies.
Anti-intrinsic factor antibodies testing has 50 to 84% sensitivity and almost 100%
specificity. It is the recommended initial test for pernicious anemia.
Pernicious anemia is associated with a type of gastritis called autoimmune metaplastic
atrophic gastritis (AMAG). AMAG is caused by autoimmune aggression against gastric
mucosa. An immune response is mainly detected against oxyntic cells & intrinsic factor.
The three main components of AMAG are;
Glandular atrophy, mainly affecting the body and fundus. In advanced stages, the
mucosa becomes thin and atrophic. Typically little changes are observed in the
gastric antrum.
Intestinal metaplasia, replacing the oxyntic glands. The mucosa becomes villiform
and resembles normal small intestine.
Inflammation.

16.DERMATOMYOSITIS:
Symmetric proximal myopathy associated with Gottrons papules or a heliotrope rash is
highly characteristic of Dermatomyositis. Look fr raised Creatinine phosphokinase levels.
Antinuclear antibodies (ANA) screening is initial test of choice as it is positive in ~80% of
the pts.
Evaluation also includes specific antibody testing including anti-Ro, anti-La, anti-Sm,

anti-RNP and anti-Jo 1 antibodies.


Chest X ray is performed to screen for interstitial lung disease if no pulmonary symptmz
are present.
In symptomatic pts, or those with abnormal Chest X ray findings, Chest CT and PFTs
are recommended.
Tissue biopsy z not required in pts with highly characteristic featurz of Dermatomyositis
in the absence of alternative diagnosis.
All pts with dermatomyositis should undergo cancer screening due to an increased risk
of malignancy. Symptoms of dermatomyositis may resolve if the cancer is treated
successfully.

17.Myopathy is unusual with oral prednisone <10 mg.

18.HIV;
Proximal muscle weakness is more prominent in the lower extremities and is
accompanied by myalgias and muscle tenderness.
Violaceous lesions of HIV associated psoriasis have more scaling and more
extensive involvement (nails, palms and soles).

19.TETANUS PROPHYLAXIS:
>What are the tetanus toxoid containing vaccines ?
Td, Tdap, TT, DT
>The wound is clean OR the wound is minor.-> No need of TIG at all.
Give them tetanus toxoid containing vaccine if the last tetanus vaccine was
more than 10 years ago
Give them tetanus toxoid containing vaccine if they were unimmunized
previously
Give them tetanus toxoid containing vaccine if they have an uncertain
immunization history.
Give them tetanus toxoid containing vaccine have an incomplete vaccination
status (<3 doses). Incomplete vaccination status is more commonly
encountered in IV drug abusers, immigrants, individuals from rural areas and
the elderly.
>The wound is dirty (e.g dirt, feces, saliva) OR the wound is severe (e.g punctures,
avulsions, crush injuries, burns, frostbite).
Give them tetanus toxoid containing vaccine only if they received their last
tetanus vaccine 5 or more years back.
Give them tetanus toxoid containing vaccine + TIG, If they are severely
immunocompromised.
Give them tetanus toxoid containing vaccine + TIG, if they have never been
immunized.
Give them tetanus toxoid containing vaccine + TIG, if they have an uncertain
immunization history.
Give them tetanus toxoid containing vaccine + TIG, if they have an incomplete
vaccination status i.e less than 3 doses. Incomplete vaccination status is more
commonly encountered in IV drug abusers, immigrants, individuals from rural
areas and the elderly. This is becx the early doses only prime the immune system
but do not confer immunity.

20.INFLUENZA VACCINATION:
Should be given to most people age 6 months or more.
People at risk of life threatening outcomes from influenza infection include
people with asthma, those with chronic illness and those <2 years age. Hence
specially recommended for those people.
Intramuscular vaccine is inactivated one and intranasal one is live attenuated.
Egg allergy recommendations;
>Those with no prior egg allergy of any kind can receive both intranasal and IM
influenza vaccines.
>Those with prior urticarial reactions to egg ingredients should receive
inactivated IM vaccine only and after that, 30 minutes of observation.
>Those with true anaphylaxis to vaccine should not receive any kind of influenza
vaccine and be considered for allergy consultation.
Asthma, pregnancy and immunocompromised state are contraindications to the
live attenuated intranasal vaccine.

21.If the hives are generalized after exposure to allergen, then concern for anaphylaxis
should raise.
If localized, then oral anthistamine should suffice.

22.Anaphylaxis should be suspected in pts with cutaneous, respiratory, GI and CVS


symptoms after exposure to a known or potential allergen. Its presentation is
variable and it is not necessary to have every sign and symptom for the diagnosis.
Epinephrine should be given intramuscularly if there is concern for anaphylaxis.

23.Bee allergy;
Self injectable epinephrine shd be carried with oneself all the time
Referral to an allergist for venom immunotherapy can reduce the been sting
anaphylaxis risk from 60% to <5%.

24. MMR VACCINATION:


Live attenuated vaccine
Generally safe in pregnancy but shd not be administered to pregnant women.
Contraindicated in immunocompromised pts (e.g congenital immunodeficiency,
severe HIV infection, hematologic or solid tumors, long term immunosuppressive
therapy).
If the pt is febrile at the time of appointment, vaccination shd be deferred until
the acute illness has resolved so that symptoms are not confused with vaccine
side effects and vice versa. However, a history of fever after vaccination is
common and is not a contraindication to future vaccination.
Precautions shd be observed in pts with history of thrombocytopenia and
receiving MMR becx of the rare but increased risk of immune thrombocytopenia.
The second dose of MMR shd not be given to the pts who developed
thrombocytopenia following the first dose if they have adequate antibody titers.
Recent administration of immunoglobulins (e.g treatment of Kawasaki disease)
can diminish the efficacy of MMR; therefore, vaccination should be delayed for
3-11 months depending on the strength of antibody in the preparation.

Asymptomatic HIV pts without severe immunosuppression shd be vaccinated


with MMR as they are at high risk of developing natural measles infection.
Pts with households who are pregnant, can safely receive MMR.
A personal or family history of epilepsy is a precaution to MMR vaccination due
to increased risk of febrile seizures after receiving the combined vaccination. In
these pts, MMR and varicella vaccines should be given separately.
A history of anaphylaxis after receiving gelatin (e.g marshmallows) is a
contraindication to MMR
A history of anaphylaxis to neomycin is a contraindication to receiving the MMR.

OPHTHALMOLOGY:
1.Anterior Uveitis (IRITIS):
Anterior uveitis (iritis) is characterized by pain, redness, variable visual loss, and a
constricted and irregular pupil. To properly distinguish iritis from other causes of a red
eye, it is important to visualize anterior segment of the eye with slit lamp examination.
If leucocytes are seen in anterior segment, which contains the aqueous humor found
between the cornea and the lens, then the diagnosis of iritis is confirmed. A hazy flare
, which is indicative of protein accumulation secondary to a damaged blood-aqueous
barrier, may also be seen. Treatment for iritis depends on the etiology of the condition
and typically includes antimicrobial therapy for viral or bacterial causes and topical
corticosteroids for noninfectious causes.

2. Infectious keratitis is characterized by severe photophobia and difficulty in keeping


the affected eye open. Penlight examination reveals a corneal opacity or infiltrate.

3.Central retinal artery occlusion: (CRAO)


Look for cardiovascular risk factors in question stem like DM, HTN, dyslipidemia, obesity.
Most common cause is carotid artery atherosclerosis as the central retinal artery is a
branch of ophthalmic artery, which arises from the internal carotid artery. Other less
frequent etiologies include cardiogenic embolism, clotting disorders, carotid artery
dissection, and vasculitis (e.g giant cell arteritis).
CRAO is characterized by acute-onset, painless, monocular vision loss. Pts may lose the
ability to make fine distinctions (e.g finger counting), but can frequently identify objects
(e.g hands). Typical examination findings include an afferent pupillary defect and
fundoscopy showing a pale, ischemic fundus with cherry red spot at the macula (due
to preserved circulation via posterior ciliary arteries).

Evaluation includes laboratory studies to exclude giant cell arteritis (ESR, CRP), carotid
artery imaging, and cardiac evaluation in pts with a suspected cardioembolic source.
CRAO can cause irreversible retinal damage within 90-100 minutes. As a result, pts
require urgent ophthalmology consultation and interventions to lower intraocular
pressure (e.g ocular massage, anterior chamber paracentesis, IV acetazolamide, or
mannitol). Long term management includes atherosclerotic risk factor modification
(lipid reducing and antiplatelet agents) for prevention of recurrent vascular events.
Prognosis is poor in pts with severe vision loss at the time of presentation.

4.Acute angle closure glaucoma would cause monocular vision loss, eye pain, headache,
nausea/vomiting, and conjunctival erythema. Pts may also report seeing light halos.
Examination shows dilated, poorly reactive pupil. Diagnosis is confirmed by raised
intra ocular pressure.

5.Optic neuritis typically causes subacute (hours to days) monocular vision loss associatd
with eye pain that worsens with eye movement. Pts can have an afferent pupillary
defect. Fundoscopy characteristically shows optic disc edema.

6.The presence of orbital fat tissue in the wound indicates a high probability of orbital
septum injury and a possible levator palpebrae injury.

7.The most common pathogen isolated from culture of corneal foreign bodies is
coagulase negative staphylococcus. Other common pathogens include streptococcus,
Haemophilus, and Pseudomonas. Therefore the treatment for all foreign-body
associated corneal abrasions should include empiric broad-spectrum antibiotic eye
drops or ointments (e.g erythromycin, sulfacetamide, ciprofloxacin, ofloxacin).

8.Pseudomonas is a common cause of bacterial infection in contact lens wearers.

9. Central retinal vein occlusion presents with painless, acute or subacute monocular
vision loss. CRVO is usually due to nonembolic causes. Examination can show tortuous
, and dilated veins, diffuse hemorrhages, disk swelling, and cotton wool spots.

10. Macular degeneration usually presents with progressive distortion and loss of vision,
primarily in the center of the visual field (central scotomas). The dry form can have
cellular debris (drusen) accumulate between the retina and the choroid, which
sometimes leads to the retinal detachment. The wet form is more severe with
blood vessels growing up from the choroid behind the retina, which can cause retinal
detachment.

11. Retinal detachment presents with vision loss (peripheral followed by central) and
photopsia with showers of floaters. It is usually associated with trauma, previous
eye surgery, aging and myopia.

12. Diabetic retinopathy:


Blurry vision can occur due to;
Macular edema with background diabetic retinopathy
Optic lens swelling
Macular edema occurs due to increased vascular permeability of retinal blood vessels.
Usually, pts with macular edema hve background diabetic retinopathy (microaneurysms,
hard exudates, soft exudates, dot or blot hemorrhages). Diaebtic retinopathy requires

some years to develop. Blindness can occur as a complication of proliferative diabetic


retinopathy, leading to vitreous bleeding, followed by retinal detachment. Blindness can
be prevented in majority of these pts with timely detection and treatment using laser
photocoagulation. Diabetes is the leading cause of legal blindness in the United States.

Poor glycemic control can also lead to swelling of optic lens secondary to osmotic
changes. Improving glycemic control can settle down this issue.

13. Common causes of decreased vision in elderly pts (especially above 75 years of age)
are cortical cataracts and associated macular degeneration and this is extremely
common. Elderly pts should be thoroughly evaluated for the severity of macular
degeneration, as they may not benefit from cataract surgery.

14. Bacterial conjunctivitis:


Eye redness and mucopurulent discharge are the cardinal findings. The condition is
usually unilateral although it can be bilateral. The thickness and color of the discharge
is helpful in distinguishing the condition from allergic and viral conjunctivitis, where
the discharge is typically watery. Also, it shd be differentiated from simple morning
crusting followed by watery discharge that can be seen in viral or allergic conjunctivitis.
Viral conjunctivitis, often caused by Adenovirus, frequently has other systemic
manifestations such as fever, pharyngitis and Upper respiratory symptoms. Bacterial
conjunctivitis is thought to be self-limited in most instances, although antibiotics are
typically prescribed to shorten the time of infection and reduce the person to person
contact.
When the infection is severe and/or untreated, there is an increased risk of keratitis.
Keratitis refers to inflammation of the cornea, where the conjunctivitis involves only

the thin conjunctival membrane overlying the cornea. Ths complication z more common
in pts who wear contact lenses. Pts with keratitis typically have a foreign body sensation
, which is different from the feeling of eye grittiness that pts with conjunctivitis often
describe. Other findings in keratitis include photophobia and a corneal opacity or
ulceration. Treatment of keratitis is also with antibiotics, but the pt shd have an urgent
ophthalmologic referral as well given the increased seriousness of the condition.
Culture is negative in 50% cases of bacterial conjunctivitis but the presence of purulent
eye discharge warrants antibiotic treatment. Most common microbiologic causes of
bacterial conjunctivitis include Staphylococcus aureus, Streptococcus pneumoniae,
Moraxella catarrhalis, and Hemophilus influenzae. The most commonly used antibiotics
include erythromycin ointment, sulfa drops and polymyxin/trimethoprim drops since
they cover all of these bacteria. The choice of agent shd be based on cost and local
resistance patterns.
Fluoroquinolones shd be reserved for contact lens wearers becx of their activity against
Pseudomonas, which is a common pathogen in these pts.
Primary care physicians shd generally not prescribe corticosteroids in these cases w/o
ophthalmologic referral.
Bacterial conjunctivitis is very contagious. Ideally pts shd stay away from work or school
while discharge is present, although this is not possible for most pts. A minimum of 24
hours of antibiotic therapy z generally requird by most institutions before the individual
returns to work or school.
Pts shd typically have marked improvement in symptoms by the first three days after
antibiotics are started. If a pt still has symptoms and drainage at 1 week after antibiotic
therapy, this shd be considered a lack of response and the patient shd be referred to an
ophthalmologist.

15. Endophthalmitis occurs post-cataract surgery or post-traumatic.

16.Lens dislocation in Marfan syndrome, Homocystinuria and Ehlers Danlos syndrome.

17. Endophthalmitis:
It is an infection of the vitreous fluid. The classic presentation is a 12-24 hour history of
mild eye pain or discomfort, accompanied by a decrease in visual acuity. Examination of
the affected eye may reveal swollen eyelids and edema or erythema of the conjunctiva.
Hypopyon, a layer of leucocytes in the anterior chamber is frequently present. In 80% of
the cases, the retinal vessels cannot be seen.
Endophthalmitis is an ophthalmic emergency. Immediate management must be done by
an ophthalmologist, who will confirm the diagnosis using aspiration and vitrectomy to
obtain vitreous or aqueous cultures (as needed) and administer antibiotics by the
intravitreal route.

18.Orbital cellulitis and Preseptal cellulitis:


Both preseptal cellulitis and orbital cellulitis have eyelid erythema, swelling and
tenderness, but orbital cellulitis also has proptosis, ophthalmoplegia and pain with
eye movements. Distinguishing between preseptal cellulitis and orbital cellulitis is
critical because preseptal cellulitis is typically a mild infection and orbital cellulitis can
be life-threatening. In both conditions, the most common offending organisms are
Staph. aureus, Strep. pneumoniae and other streptococci.
For orbital cellulitis, broad spectrum IV antibiotics (e.g Vancomycin plus ampicillinsulbactam) shd be started immediately becx the infection can spread quickly. Dangerous
complications include abscesses within the orbit or brain, blindness or sinus venous
thrombosis. If orbital abscesses are identified or the patient begins to deteriorate,

orbitotomy may be indicated. In contrast, preseptal cellulitis can be managed with


oral antibiotics as long as there are no toxic findings on examination and close follow up
can be ensured.
Bacterial sinusitis causes most of the orbital cellulitis due to proximity. Although most of
the cases have co-existing sinusitis caused by bacteria, fungal pathogens can also cause
life-threatening orbital cellulitis in pts with impaired host defenses (e.g poorly controlled
diabetes mellitus, HIV). Alarming symptoms such as facial numbness and septal necrosis
shd raise the suspicion for fungal infection. Therefore, sinus drainage and culture shd be
performed if the pt does not improve with antibiotics or if examination shows septal
necrosis and facial numbness.

19.Cavernous sinus thrombosis:


Headache is a common and early symptom and can become unbearable. Fundoscopy
typically reveals papilledema and dilated tortuous retinal veins in addition to the
symptoms seen in orbital cellulitis. Because cranial nerves 3,4,5 and 6 pass through the
cavernous sinus, which has anastomosis crossing midline, unilateral symptoms (e.g
ophthalmoplegia) can rapidly become bilateral.
Magnetic resonance venography is the imaging modality of choice to identify sinus
venous thrombosis.

EAR, NOSE AND THROAT:


1.Presbycusis:
Presbycusis is a gradual, symmetrical, sensorineural hearing loss that affects more than
50% of adults by age 75 and can significantly impair the quality of life. Pts typically
withdraw from their social lives & may stop leaving home to avoid conversations. Their
social interactions deteriorate and they often must listen to the radio or television at
high volumes. Physical examination may show no abnormalities except for the hearing
loss itself. Although audiometry is used to confirm the diagnosis, the whispered voice
test, a simple procedure that can be performed in the office without equipment can be
used. In this test, the examiner stands behind the pt & whispers letters/numbers while
occluding the non-tested ear. Repeating 3 out of 6 letters/numbers correctly is passing.
Most pts with presbycusis benefit from the use of hearing aids, which shd decrease the
social isolation.
BOTTOM LINE: Hearing loss due to presbycusis is one of the most frequent causes of
social withdrawal and isolation in elderly. It must be differentiated from
depression and dementia. Screening can be accomplished through simple
hearing tests performed in the office.

2.Thyroglossal duct cyst:


Presents as a midline neck mass that moves with the protrusion of the tongue. The
thyroid is formed as an outpouching from the base of the tongue and then descends to
the base of the anterior neck. The thyroglossal duct connects the tongue and the thyroid
gland & a cyst can develop from the epithelial remnants within the duct. Ectopic thyroid
tissue z present in large number of pts within the thyroglossal duct cysts but sometimes
this is the only functional tissue present. Therefore, imaging studies, like thyroid nuclear

scan, Ultrasound or CT is mandatory before subjecting the pt to surgery. A CT scan is


thought to be more useful becx it not only delineates the normal thyroid gland but also
clarifies the anatomy of thyroglossal duct cyst in relation to the surrounding structures.
Preoperative assessment of the thyroid gland is also required. A fine needle aspiration
biopsy is not generally required unless there is high suspicion of malignancy.
BOTTOM LINE: Ectopic thyroid tissues is present in a large number of pts within the
thyroglossal duct cysts, and sometimes this is the only functional tissue
present. Therefore, the imaging studies, like a thyroid nuclear scan, U/S
or CT is mandatory before subjecting the pt to surgery.

3.-> Chances of malignancy in thyroglossal duct cysts is around 2-3%


-> Thyroglossal duct cysts have high chances of getting infected due to connection
with oropharynx.

4.Mastoiditis:
Presents as pain and swelling behind the ear. It is the most common suppurative
complication of acute otitis media (AOM). Acute inflammatory response during acute
otitis media frequently results in a mucosal response of the mastoid air cells but this
response usually resolves quickly upon institution of antibiotic therapy. In rare cases,
when the response is severe or otitis media is not adequately treated, the process may
persist and result in accumulation of pus within the mastoid cavities. Acute mastoiditis
results in such cases. Sometimes, the pus can spread further and lead to dangerous
complications such as Bezold abscess (a neck abscess resulting from an erosion through
the medial aspect of the mastoid tip).

5.The decreaed ability to discriminate speech in pts wth presbycusis z especially obvious
in a noisy, distracting environment.

6.Pts with presbycusis hve difficulties understanding rapid speech and a complex or less
familiar vocabulary.

7.About 60% of head and neck cancers are locally advanced at the time of diagnosis and
are inoperable. Combined chemotherapy and radiotherapy promises superior results
than chemotherapy or radiotherapy alone.

8.Rhinitis:
Allergic rhinitis typically presents at an earlier age (<20) with predominant eye symptmz
(e.g watery eyes), sneezing, nasal congestion and watery rhinorrhea. Pts usually can
identify a trigger (e.g animals, pollen) with seasonal or year round symptoms. There is
also an increased association wth othr allergic conditions (e.g eczema, asthma). Physical
examination can also show normal or pale blue nasal mucosa with pallor & occasionally
shows polyps. Treatment involves allergen avoidance & topical intransal glucocorticoids
for symptomatic relief.
Non-allergic rhinitis (NAR) typically presents aftr age 20 with nasal blockage, rhinorrhea
and postnasal drip. Pts have limited eye symptoms and are usually unable to identify a
clear trigger. Although symptoms may worsen during seasonal changes, they can occur
thoughout the year. The nasal mucosa can appear normal or boggy and erythematous
on examination. NAR is less commonly associated with other allergic conditions.
Treatment includes topical intranasal glucocorticoids (e.g fluticasone) or intranasal
antihistamines (e.g azelastine). Pts with more severe symptoms require combination
therapy.

9. Epistaxis:
Anterior nasal mucosa is the most common source of epistaxis as it is where the
sphenopalatine, greater palatine, anterior ethmoid, and superior labial arteries
anastomose (kiesselbachs plexus). Anterior epistaxis that does not cease after
nostril pinching (which usually stops the bleeding) is next treated by the topical
vascoconstriction. Oxymetazoline is the preferred agent in children and should be
applied with a squirt bottle (nasal spray) or cotton pledget. Direct pressure should
then be applied to the nasal alae for an additional 5-10 minutes. Pts shd be advised
to use topical decongestants for <3 days to avoid rebound nasal congestion.
If vasoconstrictor doesnt work, chemical or electrical cauterization with silver nitrate
is the next step to do, with topical lidocaine applied for minimizing pain. (dont do
bilateral cauterization becx of risk of septal ulceration and perforation).
If above strategies fail, then anterior nasal packing with bacitracin-covered sponge or
tampon is the next step.
*Posterior nasal packing is done for posterior epistaxis in elderly with HTN or
atherosclerosis.

10.Otitis externa:
Otitis externa often arises in the setting of excess moisture leading to skin maceration
or other factors that disrupt the skin-cerumen barrier (e.g excessive cleaning or
scratching of the canal). It can also be associated with foreign bodies that occlude the
canal (e.g hearing aids, Headphones). The organisms most commonly responsible for
OE are those found in normal skin flora, including Staph. aureus and Pseudomonas
aeruginosa. The diagnosis is based on the history and physical examination findings.
Pain caused tragal pressure or traction on the auricle is a characteristic finding. Physical
examination might reveal erythema, edema and otic discharge.

The following treatment plan is recommended for most cases of OE;


Gently clean the canal if needed to remove cerumen and purulent debris using
a wire loop under direct visualization. This allows antbiotics to have maximal
impact.
Topical acidifiers (e.g acetic acid, Domeboro solution) can be used to treat the
infection in mild cases. Moderate to severe cases shd receive topical antibiotics
(e.g polymyxin B/neomycin, ciprofloxacin). Most cases are treated for 7-10 days.
Topical corticosteroids (e.g hydrocortisone, dexamethasone) may be added to
relieve itching and pain. More severe pain may require additional analgesics.
Pts shd be advised on preventive measures, including avoiding moisture in the
ears, drying ears thoroughly after swimming or bathing, and use of acidifiers at
the onset of recurrent symptoms.
BOTTOM LINE:otitis externa z associatd with swimming, excessive cleaning or scratching
of the ear, or use of occlusive devices. Most cases are managed with the
topical antibiotic solutions. Treatment may be facilitated by clearing the
canal of cerumen and purulent debris with a wire loop irrigation with
hydrogen peroxide is an acceptable alternate cleaning method if the
tympanic membrane is visualized and intact.

11.Studies have shown that the use of non-sedating decongestants (specifically


pseudoephedrine) before diving reduces the incidence of ear and sinus barotrauma
by 75%.
Scuba divers shd be advised to avoid flying in a plane within 24 Hrs of diving to avoid
pressure related related injuries..

12.A classic triad of periodic vertigo, unilateral hearing loss and tinnitus is characteristic
of Menieres disease.

13.Cleft lip:
Cleft lip with or without cleft palate is typically a multifactorial disorder. It has been
associated with the use of teratogenic agents during pregnancy e.g alcohol and the
modes of inheritance can be autosomal dominant, autosomal recessive and X-linked.
Reconstruction of the cleft lip is generally performed at approximately three months
of age, according to the rule of 10, 10 lbs of weight, 10 weeks of age and 10 g of
hemoglobin.
Autosomal dominant inheritance with variable expressivity in family is called
Van der Woude syndrome.

14. A 2 year old child has a 50+ word vocabulary, use of 2-3 word phrases and follow
two step commands.

15.Delayed speech development may indicate an underlying hearing impairment and


shd prompt referral of the pt for audiologic assessment. Other candidates for hearing
evaluation include children with a family history of hereditary childhood hearing loss,
history of meningitis, history of recurrent or persistent otitis media with effusion for
more than 3 months, documented intrauterine infections, craniofacial anomalies and
use of ototoxic medications such as aminoglycosides.
Hearing loss early in life, even mild or unilateral, may affect speech and language
development, social and emotional development and academic achievements.

16.Recurrent sinsusitis:
Exposure to cigarette smoke and air pollution causes recurrent or chronic sinusitis by
damaging the cilia responsible for moving the mucus through the sinuses, resulting in
buildup of of mucus, obstruction of sinuses and secondary bacterial overgrowth.

Other common causes of recurrent sinusitis in otherwise healthy people include


inadequately treated acute sinusitis, structural abnormalities of the nasal septum or
palate and allergic rhinitis.

17.Kartagener syndrome is an autosomal recessive disorder consisting of situs inversus,


chronic sinusitis and airway disease leading to bronchiectasis. It is a subgroup of a
congenital mucociliary disorder known as primary ciliary dyskinesia.

18.Malignant otitis externa:


It is an invasive infection of the external auditory canal and the bones forming the skull
base. The infection usually begins as external otitis progresses rapidly to involve the
adjacent bones at the base of the skull. Pts have marked pain (otalgia) and purulent
drainage or discharge from the ear (otorrhea). The finding of granulation tissue at the
floor of the bone-cartilage junction in the external auditory canal is pathognomonic of
this condition. The tympanic membrane is usually intact.
Malignant otitis externa is typically seen in the elderly pts with diabetes mellitus. Pts
with HIV disease and other immunocompromised states are also at higher risk of having
malignant otitis externa. A number of pts with malignant otitis externa usually have an
associated history of aural irrigation for removal of cerumen. Pseudomonas aeruginosa
is the usual causative organism in these pts. An untreated infection can progress rapidly
to involve skull base, temporomandibular joint and cranial nerves, causing osteomyelitis
and cranial nerve palsies.
Anti-pseudomonal antibiotic therapy is the treatment of choice. Fluoroquinolones
(ciprofloxacin), anti-pseudomonal penicillins (piperacillin, ticarcillin) with or without
aminoglycosides and third generation cephalosporins (ceftazidime) are all effective
in treatment of malignant otitis external otitis. All pts shd be treated with intravenous

antibiotics initially and then switched to oral antbiotics (depending upon the clinical
response) to complete 6-8 weeks of antibiotic therapy.

19.Acute otitis media is associated with inflamed erythematous, bulging and immobile
tympanic membrane due to the presence of fluid in the middle ear.

20.Ramsay hunt syndrome:


ear pain + vesicles in external auditory canal + ipsilateral facial paralysis

21.Diphenhydramine is highly sedating and has anti-cholinergic properties. Becx of the


effect of diphenhydramine on the level of alertness and coordination, use of the drug
must be discontinued before performing coordinated tasks such as driving, working
on machinery, flying planes, or diving.

22.

POISONING:
1.Carbon monoxide poisoning:
Suspect it in cases of house fire. Typical presentation is nausea, throbbing headache,
malaise, dizziness and altered mental status. Severe symptoms may result in seizures,
syncope and coma. Delayed neuropsychiatric syndrome develops in up to 40% of pts
with significant CO poisoning.
Several people simultaneously presenting with a headache is an important clue. It is
especially common in cold winters in cold climates. It might be because of improper
functioning heating system and improperly vented fuel-burning devices (e.g kerosene
heater).
CO-oximtery is used for the diagnosis of carbon monoxide poisoning. This method
distinguishes normal hemoglobin from carboxyhemoglobin.

2.Herbal medicines toxicity:


Licorice is used for stomach ulcers and bronchitis/viral infections. Chronic
ingestion of licorice can cause or aggravate hypertension in pts. Licorice inhibits
the enzyme 11-beta hydroxysteroid dehydrogenase, thereby preventing the
conversion of cortisol to cortisone. The available cortisol binds to mineralocorticoid receptors and causes hypertension, hypokalemia & metabolic alkalosis.
Gingko has been marketed as an antioxidant. It is used for the treatment of mild
memory loss, dementia, macular degeneration and peripheral vascular disease. It
doesnt have any significant adverse effects; however, it can interact with aspirin
or warfarin, leading to a potential risk of spontaneous bleeding.
Use of Kava Kava (piper methysticum) with alcohol, benzodiazepines or other
prescription sedatives can potentiate their effects and cause excessive
drowsiness or disorientation. It has been used for anxiety, insomnia and
menopause. its use may result in hepatotoxicity and liver failure several weeks
or more after intake.
Black cohosh has been used for the treatment of premenstrual syndrome and

menopausal symptoms. Its excessive use can cause hypotension. It shd be used
with caution in pts already on anti-hypertensive medications. It can also cause
hepatic injury & increased bleeding problems because of its anti-platelet effects.
Horse chestnut has been in pts with venous insufficiency or chronic venous stasis.
It can inhibit platelet aggregation and can cause bleeding in pts already on aspirin
, aspiring-containing products or warfarin.
Ginseng has been used for improved mental performance. Most common
adverse effects include headache, insomnia, and gastrointestinal symptoms.
Its use is associated with increased vaginal bleeding and hypoglycemia.
Saw palmetto is used for BPH. Its use has been associated with abdominal
discomfort and increased bleeding risk.
St. Johns wort has been used for depression and insomnia. It has drug interactnz
with antidepressants ( serotonin syndrome ), OCPs, anticoagulants (decreased
INR), and digoxin. It can also cause hypertensive crisis.
Echinacea has been used for the treatment and prevention of cold and flu. It has
been associated with anaphylaxis, especially in asthmatics.
Ephedra has been used for the treatment of flu and cold, weight loss, & improved
athletic performance. Its use hz been associated with hypertension, arrthythmias
, MI, sudden deaths, strokes and seizures.

3.Symptomatic Hypocalcemia:
In general, symptomatic hypocalcemia resulting from transfusion of citrated blood is
rare, because normal individuals rapidly metabolize citrate in the liver and kidney;
however, pts with renal failure, hepatic failure, hypothermia, shock and lactic acidosis
, who receive blood transfusions have a high risk of hypocalcemia. This is due to their
inability to metabolize citrate, which is concomitantly transfused with every blood
transfusion. The conversion of citrate to lactate is impaired, which leads to excess
amount of citrate in the blood. The excess citrate then binds calcium and this leads to
hypocalcemia. The measured serum calcium levels may remain normal despite this
occurrence, because the deficit of ionized calcium is not reflected in total calcium levels.
Prophylactic administration of atleast 10 cc of 10% of calcium gluconate is therefore

recommended for every 500 mL of packed red blood cells transfused.

4. Sodium bicarbonate is given when bicarbonate is extremely low (5 mEq/L or less).

5. Organophosphate poisoning:
S/S include lethargy, confusion, wheezing, miosis, bradycardia, excessive salivation,
drooling, sweating, lacrimation, diarrhea, muscle aches, fasciculations & skin flushing.
Sudden onset of symptoms and garlic-like odor from clothing are also characteristic.
RBC cholinesterase activity test can be used to confirm diagnosis of organophosphate
poisoning and assess the degree of toxicity.
It should be treated with two medications. Atropine which reverses the muscarinic
receptor effects and pralidoxime, which is a cholinesterase activator.

6.Scombroid poisoning:
Caused by the ingestion of improperly stored sea food. If seafood is stored at greater
than 15C, histidine can undergo decarboxylation and form histamine. Symptoms of
scombroid poisoning include flushing, a throbbing headache, palpitations, abdominal
cramps, diarrhea and oral burning. These typically begin 10-30 mins after ingesting the
fish and are self limited. Pts sometimes describe a bitter taste but this is not always
present. Physical findings may include skin erythema, wheezing, tachycardia and
hypotension.

7. Pufferfish poisoning is characterized by the prominence of neurological symptoms


(perioral tingling, incoordination, weakness etc).

8.Botulism:
Botulism is an acute, presynaptic, neuromuscular junction disorder caused by infection
with Clostridium botulinum, an anerobic bacteria found widely in improperly canned
food and preserved foods. The botulinum toxin released by this bacterium acts at the
peripheral nerve endings to inhibit the release of acetylcholine into the synaptic cleft.
This leads to the inability to achieve a threshold end-plate postsynaptic potential
(action potential). The ultimate result in neuromuscular junction failure and subsequent
weakness. Clinically, pts develop acute bulbar muscle weakness and descending limb
weakness. It also leads to autonomc dysfunctn (e.g blurry vision due to accommodation
failure and impaired pupillary responses). There are absent reflexes. Multiple cranial
nerve distributions may become weak. Sensory function is preserved.
EMG results may also suggest the diagnosis, which is confirmed by assay of the serum
for botulinum toxin.

9.In Myasthenia Gravis, autonomic dysfunction is rare and reflexes as well as sensory
functions are usually normal.

10.Infant botulism:
Infant botulism is most commonly caused by ingestion of microscopic environmental
dust containing soil spores of Clostridium botulinum. The incidence is highest in states
with high soil botulinum spore counts (California, Pennsylvania and Utah), especially in
areas where soil is disturbed (construction or farming sites).
Young infants age 2 weeks to 8 months are particularly vulnerable to life threatening
infection from spore ingestion due to less normal bowel flora compared to children and
adults. Clinical manifestations include constipation and cranial nerve palsies (e.g ptosis,

pupillary paralysis, weak suck) followed by progressive hypotonia & loss of deep tendon
reflexes (flaccid neuropathy).
Infants with botulism shd be admitted to the Intensive care unit for monitoring of
respiratory status, Nasogastric tube feedings, laxative medications and physical and
occupational therapies. Intravenous human-derived botulism immune globulin shd
be given as early as possible in the course of illness to reduce the severity and duration
of symptoms by neutralizing the circulatory toxins. Botulism immunoglobulin shd be
administered once the diagnosis is suspected and shd not be delayed while awaiting
confirmatory stool studies. Antibiotics (especially aminoglycosides) should not be
administered due to potential lysis of colonic Clostridium botulinum and increase in
toxin absorption. Equine derived botulinum antitoxin is avoided in infants and reserved
for food borne botulism only, because of risks of severe anaphylactic reactions in infants
& lifelong hypersensitivity to equine antigens.
Most children with infant botulism who are diagnosed and treated promptly are
hospitalized for 1-3 months and can expect a full recovery.

11.Foodborne botulism is caused by canned fruits, vegetables and fish may cause
outbreaks of foodborne botulism if tainted by preformed toxin. The treatment
is equine derived botulinum antitoxin.

12.Pooled human immunoglobulin is the preferred therapy for autoimmune conditions


such as Guillian barre syndrome.

13. Majority of burn center deaths are due to inhalation injuries. Supraglottic damage is
one of the most alarming types of inhalation injuries and stems from inhaling hot air,
steam or smoke. The ensuing inflammatory response results in edema of sorroundng

soft tissues and significant narrowing of the airway. Fiberoptic laryngoscopy or


bronchoscopy may be necessary to properly assess extent of airway involvement.

14.Lead toxicity:
Screening is commonly performed by capillary (fingerstick) testing but elevated
capillary results must be confirmed by venous lead levels due to risk of contamination
of capillary samples (e.g inadequate cleansing of childs finger). Children with mild
intoxication (<45 ug/dL) require follow-up testing but chelation therapy is not usually
administered due to the lack of evidence that it improves neurological outcomes. Public
health officials shd be notified and children shd be relocated to a lead-free environment.
Those with moderate intoxication (> 45 ug/dL) shd receive chelation therapy i.e DMSA
/Succimer which can be taken orally with few adverse effects. Chelation therapy
increases the urinary lead excretion. Children who cannot take orally shd be given
IV EDTA.
Severe intoxicatn (> 70 ug/dL) z medical emergency requiring immediate hospitalization
& administration of intramuscular Dimercaprol (British anti-Lewisite) followed by
intravenous calcium disodium EDTA.
After venous blood confirmation of lead level, abdominal X rays can be considered in
children with symptoms (e.g abdominal pain, vomiting, constipation) or a history of pica
(e.g appetite for ice, dirt) to evaluate for ingestion of radiopaque foreign bodies
containing lead flecks.
Long bones X rays show Lead lines, but usually not performed as that does not change
the management.
Erythrocyte protoporphyrin levels may be elevated but non-specific.
Levels as low as 10-20 ug/dL are associated with cognitive impairment and behavior
problems (e.g attention deficit disorder) that may not manifest until school initiation.

Cognitive deficits can persist throughout childhood and beyond adolescence.


Unfortunately, chelation therapy has not been shown to improve neuropsychiatric
outcomes in children with levels of 5-44 ug/dL. Therefore avoidance of ingestion in
paint in buildings built before 1978 & lead-containing products (e.g imported crayons,
toys, cosmetics) & laboratory screening are crucial to preventing further complications
of lead intoxication.
Constipation, abdominal pain and vomiting typically occur at levels >45 ug/dL.
Blood lead levels ~45 ug/dL are associated with decreased hemoglobin production, and
hemolytic anemia can occur at >70 ug/dL. Many children with lead intoxication may hve
iron deficiency anemia and shd have serum iron, ferritin levels, & iron-binding capacity
checked.
Acute encephalopathy typically occurs at levels >70 ug/dL.
Interstitial nephritis also occurs with lead poisoning.

13.Risk factors for lead intoxication;


-> Houses built before 1978
-> All international adoptees and immigrants shd be screened routinely

14. Oral D-Penicillamine is the primary chelating agent for copper removal in Wilson dx.
Adverse effects are leukopenia, thrombocytopenia, hepatotoxicity and angioedema.

15. Treatment of lead intoxication;


=> Mild -> 5-44 ug/dL -> No Medication
=> Moderate -> 45-69 ug/dL -> DMSA, Succimer
=> Severe -> >70 ug/dL -> Dimercaprol + EDTA.

16. Dry unknown chemicals shd always be brushed off the skin first. Once the remaining
visible powder is removed, then the area shd be irrigated with copious amounts of
low pressure water for 15-30 minutes.
Unknown liquid chemicals, in contrast, r simply immediately washed off with water.
The victims clothing shd be promptly removed and stored in a plastic bag, although
irrigation shd begin before the victim is undressed.

17. Ethylene glycol poisoning:


Its found in antifreeze solutions. It gets metabolized in the body by an enzyme, alcohol
dehydrogenase (ADH) , into a variety of toxic metabolites, including glycolic acid,
glyoxylic acid, and oxalic acid.
Ethylene glycol ingestion leads to a severe anion gap metabolic acidosis. This causes a
typical rapid and deep breathing pattern known as Kussmauls respiration. Some of the
other early signs include nausea, vomitng, slurred speech, ataxia, nystagmus, & lethargy.
Further toxicity may lead to tachypnea, agitation, confusion, flank pain, renal failure,
pulmonary edema, changes in mental status, and eventually progression to coma.
Fomepizole is a competitive inhibitor of ADH and is the antidote of choice in cases of
ethylene glycol intoxication ( also methanol intoxication ). If used early, it prevents the
formation of toxic metabolites, causes a dramatic improvement in academia & prevents
renal failure. It also prolongs the half life of ethanol ; therefore, simultaneous use with
ethanol is not recommended.

18. Anticholinergic poisoning occurs with TCAs, certain plants (Datura stramonium),
and mushrooms. Dry skin and mucosal surfaces, as well as dilated pupils, are seen.
Ventricular tachyarrhythmias can occur.

19. Second generation anti-histamines such as astemizole and terfenadine can cause
ventricular tachyarrhythmias.

20. Excessive salivation;


-> Organophosphates poisoning
-> Carbamate poisoning.

21.Hypothermia:
Mild hypothermia (core body temperature of 32-35 C) may cause only increased
shivering, ataxia, slurred speech, tachycardia, and tachypnea. Moderate hypothermia
(core temperature of 28-32 C) is characterized by marked CNS depression along with
hypoventilation, hypotension, bradycardia, hyporeflexia and cardiac conduction
abnormalities. Severe hypothermia (core temp <28 C) can cause marked hypotension,
areflexia, coma, malignant ventricular arrhythmias (ventricular fibrillation), & asystole.
Endotracheal intubation is necessary in the unconscious hypothermic pt for adequate
ventilation and airway protection. Endotracheal intubation shd protect against
aspiration, help in oxygenation, & allow for suction of cold-induced bronchial secretions.
Active rewarming measures (e.g heating pads, warm blankets) are also indicated.
Bradycardia would settle down with rewarming measures and does not need atropine
or cardiac pacing.
Hypothermic pts shd be handled gently as rough physical handling can precipitate
serious ventricular arrhythmias. Even the myocardial irritation that could occur
during placement of jugular or subclavian lines can cause cardiac arrhythmias. If
central line placement is required, femoral lines are preferred.
Hypothermia pts are usually hyperglycemia becx insulin action stops at <30 C, and
rewarming would settle down this issue.

Pulmonary edema is common in significant hypothermia and does not necessarily


indicate additional pathology.
Hypothermia is associated with several biochemical abnormalities such as anion gap
metabolic acidosis due to decreased tissue perfusion, respiratory acidosis due to
hypoventilation, azotemia due to decreased renal perfusion, hyperkalemia due to
cellular lysis, hyperglycemia due to decreased insulin effect, elevated lipase due to
cold inducd pancreatitis, elevated hematocrit due to hemoconcentration, coagulopathy
due to impaired coagulation pathways and leukopenia/thrombocytopenia due to splenic
sequestration. Most of these abnormalities resolve with appropriate rewarming and
hydration.
For mild rewarming, passive external warming (removing wet clothes, covering with
blankets) is generally adequate.
For moderate hypothermia, or mild hypothermia not responding to passive rewarming,
active external warming (heated pads and blankets, forced hot air, warm baths) is
required.
Active core rewarming (warm humidified oxygen, warm peritoneal lavage) is reserved
for pts with severe hypothermia not responding to external warming.
22.Lead intoxication;
Painted toys and decorations manufactured before 1976 or outside of the Unite
States may be overlooked sources of lead.
Lead poisoning is diagnosed when venous lead level is > 5 mcg/dL
Children with mild lead levels ( <45 mcg/dL ) shd have their venous lead levels
repeated within 1 month to ensure that the value is not rising.
Lead intoxication can cause language delays and hearing loss
Many pts present with non-specific symptoms such as anorexia, decreased activity
,irritability, vague abdominal pain and insomnia; therefore, careful history taking
(living conditions, potential environmental exposures and hand-mouth behavior)
and a high degree of suspicion are important for early diagnosis.
The initial laboratory investigation in pts with suspected lead poisoning should

include CBC, serum iron and ferritin levels and reticulocyte count. These tests will
help detect the presence of anemia and iron deficiency.

23. Lithium toxicity:


Therapeutic lithium levels are 0.8-1.2 mEq/L. Serum lithium levels >1.5 mEq/L confirm
toxicity and levels > 2.5 mEq/L require emergency management.
Risks for lithium toxicity include low GFR (elderly pts & renal failure), volume depletion,
and drug interactions. Thiazide diuretics, such as chlorthalidone, increase sodium
excretion in the distal tubule and cause slight volume depletion. The resulting increased
proximal tubular reabsorption of sodium also promotes lithium reabsorption, leading to
lithium toxicity. ACE inhibitors and NSAIDs (but not aspirin and acetaminophen) also
increase lithium levels. Pts who are co-administered these medications require close
monitoring of lithium levels and lithium dosage reduction if necessary.
S/S of toxicity include confusion, slurred speech, tremors and ataxia.
Lithium toxicity presenting with severe neurologic symptoms is an absolute indication
for urgent hemodialysis. Other indications for hemodialysis include a lithium levels of
>4 mEq/L or an increasing lithium levels despite the saline administration.
Hydration with isotonic saline increases renal filtration and lithium excretion by the
kidneys. All pts with lithium overdose, except those with specific contraindications (e.g
decompensated CHF) shd be hydrated with IV fluids. However, hydration alone at higher
lithium levels is not sufficient.

24.Giving honey to the infants can cause infant botulism. Constipation is typically the
first manifestation of the disease and is followed by lethargy, poor sucking and
weak crying. Gag reflex is frequently impaired. This may result in aspiration if
airways are not protected.

25.Heat stroke:
Heat stroke is a life threatening emergency that occurs when an individual is exposed to
sun for an extended period, but doesnt sweat enough to lower the body temperature.
Infants, the elderly and people, who work outdoors are especially prone to developing
heat stroke. The most common symptoms of heat stroke include hyperthermia,
tachycardia, loss of consciousness, seizure, fatigue, headache, dizziness, agitation, or
confusion and hot dry skin that is flushed but not sweaty. Once the heat stroke is
recognized, the patient shd be immediately removed from the hot environment and his
airway, breathing and circulation (ABCs) shd be stabilized. The initiation of rapid cooling
is crucial and the naked patient shd be sprayed with a tepid water mist or covered with
a wet sheet while large fans circulate air to maximize evaporative heat loss. Ideally, the
pts core temperature shd be dropped by 0.2 C/min.
Antipyretics are not indicated becx the cause of increased body temperature is decrzd
sweating, not increased hypothalamic set point.
Bottom line: Heat stoke shd be treated with augmentation of evaporative cooling and
the naked pt shd be sprayed with a tepid water mist or covered with a wet
sheet while large fans circulate air to maximize evaporative heat loss.Other
cooling methods such as ice packs, ice water lavage, or cold intravenous
fluids are helpful adjuncts but not the first line treatment.

MISCELLANEOUS:
1.The Swiss cheese model shows that a process has many layers of defense but all these
layers have holes. When all the holes line up, an error can occur.
Multiple efforts to improve patient safety have been developed to decrease the size
of the holes in the layers of defense. One such method is to increase the redundancy
and double checks, especially for high risk processes. For example, allergies can be
verified multiple times by multiple providers, flagged by the Electronic medical record
system (EMR) and documented in multiple locations in the medical record and on the
patient wristbands.

2.Automating key functions can help prevent errors from occurring due to variations in
patients & processes. For example, EMR systems r often set to automatically prompt
clinicians to verify allergies before ordering medications on newly admitted pts or to
signal possible drug-drug interactions between medical orders.

3.With increasing shift work in health care, patient handoffs have become increasingly
prevalent and a source of medical errors. Standardized handoffs that use checklists
& mnemonics have resulted in improved safety and decreased medical errors. These
tools take the burden off the clinicians to conduct handoffs purely by memory.

4.Safety generally improves when multiple members of a care team participate in


communication and decision making. For instance, procedural timeouts often require
the involvement of surgeons, anesthesiologists, nurses and technicians to all verify the
details of a procedure.

5.Meconeum ileus is usually the earliest manifestation of cystic fibrosis and is almost
pathognomonic for the disease. Ucomplicated meconium ileus is characterized by
distal intestinal obstruction, wherein the terminal ileum is dilated and filled with thick,
tar-like, inspissated meconium. Plain abdominal X rays would reveal dilated, gas-filled
loops of small bowel with absent air-fluid levels. Since CF is inherited in an autosomal
recessive pattern, a family history of recurrent respiratory infections (or some other
manifestations of CF) is an important clue to the diagnosis of this disease.

6.Family history of severe constipation z sometimes present in pts with Hirshsprungs dx

7.Duchene muscular dystrophy:


Is an X-linked recessive myopathy, that is the most common muscular dystrophy of
childhood that affects boys who have inherited a defective dystrophin gene on
X-chromosome p21. Weakness begins in the proximal lower-extremity muscles and
manifests as difficulty walking, running, jumping and climbing stairs. Boys may push
their arms on their thighs (Gower sign) to transition from sitting to standing. Bilateral
calf enlargement (pseudo hypertrophy) is characteristic.
The first step in evaluation is serum CPK which is markedly elevated by 10-20 times at
age 2 and then decreases with advancing age as the muscle is replaced by fat & fibrosis.
Genetic testing showing deletion of one or more exons of DMD gene confirms the
diagnosis.
Electromyography (EMG) shows myopathic (non-neuropathic) pattern.
Muscle biopsy shows significant fibrosis, fat, muscle degeneration and absent
immunochemistry staining of dystrophin.
Treatment consists of physical therapy and glucocorticoids.
Most the pts are wheel chair bound by adolescence. Death occurs at the age of 20-30

from respiratory or heart failure.

8. Dementia and Urinary incontinence:


Urinary incontinence has a strong epidemiologic association with dementia.
Dementia-associated urinary incontinence is a multifactorial disorder that may
not be etiologically related to the dementia itself. Careful assessment of an
individual patient and revealing predisposing urinary tract and non-urinary tract
conditions are important because many of these conditions may be treatable.
Until recently, it was a common belief that Alzheimers & multi-infarct dementia
damage cortical & subcortical inhibitory structurz, thereby causing an uninhibited
bladder & creating a pathophysiological basis for urinary incontinence; however
several studies have proven that even severely demented pts may stay continent.

9. SLE:
SLE is a significant risk factor for development of premature coronary atherosclerosis
and coronary artery disease. SLE is associated with accelerated atherosclerosis due to
a combination of traditional (e.g HTN, hyperlipidemia) and disease related risk factors
(e.g chronic inflammation, glucocorticoid use). Cardiovascular events are the leading
cause of mortality in pts with SLE.
Although pericarditis occurs with SLE, large pericardial effusions and/or tamponade
are rare.
Pts with SLE are at increased risk for non-Hodgkins lymphoma, especially diffuse large
B-cell lymphoma (DLBCL). Pts with DLBCL typically have rapidly enlarging symptomatic
mass (usually in the neck or abdomen) that may cause vascular or airway compromise
by compression.

10.Turner syndrome:
Is characterized by short stature and hypogonadism. All pts who are initially diagnosed
must be screened for the presence of other associated somatic abnormalities. The most
importnt of these are cardiac defects, which include coarctation of aorta, bicuspid aortic
valve, mitral valve prolapse, and hypoplastic heart. Since some of these defects cannot
be picked up by clinical examination, an echocardiogram is necessary. The other
associated defects are visual and hearing defects, kidney malformations (including horse
shoe shaped kidney) and an increased predisposition for autoimmune endocrinopathy
(especially primary hypothyroidism); therefore, in addition to an echocardiogram, all
pts initially diagnosed with Turner syndrome require visual and hearing assessment,
renal ultrasound, and TSH level measurement.
Streak gonads in Turners syndrome increase the pts chances for malignancy when
there is an associated mosaicism with a Y chromosome, so in cases of mosaicism with
Y chromosome, removal of such streak gonads is necessary but not otherwise.
Pts with Turners syndrome develop moderate to severe insulin resistance and diabetes
when they are older.
Pts with Turner syndrome have hypogonadism & eventually would require estrogen
replacement therapy; however, if estrogen therapy is given at an early age, there may
be premature fusion of epiphysis which will potentially decrease the pts final height.
Most physicians begin prescribing hormone replacement therapy when the pt reaches
14 years of age. Growth hormone is approved for use in pts with Turners syndrome to
improve their final height.

11.Palliative care:
Palliative care is an interdisciplinary intervention that seeks to reduce unnecessary
medical interventions and improve the quality of life for pts and families dealing with

serious or terminal illness. Palliative care can occur concurrently with life-prolonging
medical treatments and has been found to lead to better outcomes in mental health,
satisfaction and quality of life.
In contrast, Hospice is a model of palliative care offered to pts at the end of life when
life prolonging therapy is no longer indicated.

12.Restless legs syndrome:


More common in older individuals. Half of the pts with idiopathic RLS have a positive
family history because of autosomal dominant inheritance. Diagnosis is based on
clinical history. RLS z characterized by spontaneous, repeated leg movements to provide
relief in association with unpleasant sensations that occur while at rest. It is associated
with iron deficiency & iron supplementation improves symptoms in some pts with this
condition. Ferritin level z most accurate measurement of iron stores. Levels <75 ng/mL
are consistent with clinical symptoms. A normal complete blood count does not rule out
the condition as iron deficiency is frequently present in the absence of anemia. No other
lab tests are routinely indicated. Polysomnography may assist in ruling out other
contributing sleep disorders.
RLS can be classified as intermittent, daily & refractory. Intermittent or daily RLS of mild
severity is often treated with non-pharmacologic methods including iron replacement
therapy, mentally stimulating activities (e.g crossword puzzles) and the avoidance of
exacerbating factors (e.g nicotine, alcohol, caffeine). In moderate-to-severe daily RLS,
dopamine agonists such as pramipexole or ropinirole are considered first line therapy.
Second line choices include Benzodiazepines (e.g clonazepam) and gabapentin.
Benzodiazepines are typically used in pts requiring intermittent therapy only or as
treatment augmentation in refractory cases.
Levodopa has been shown to cause augmentation in RLS pts, which is the paradoxical

worsening (occurring earlier in the day, spreading to other body parts, shorter latency
to symptoms onset at rest) of RLS symptom severity. For this reason, Levodopa is
primarily recommended for intermittent RLS because as needed use may decrease the
risk of developing augmentation symptoms.

13.Sarcoidosis:
15-20% of pts with sarcoidosis can have ocular involvement and from these pts, 5% will
have it as the presenting symptom. Ocular involvement may manifest itself as keratoconjunctivitis, iridocyclitis, both anterior and posterior uveitis, papilledema, retinopathy,
or Heerfordts syndrome (a uveoparotid fever, involving both uvea and parotid glands).
Among the different ophthalmic presentations, anterior uveitis is the most common
form of disease and is characterized by acute onset of red eye, photophobia, blurring of
vision, and mild to moderate pain. Cataract formation, glaucoma, and blindness are
usually late complications of untreated disease.

14.Cryoglobulinemia:
Mixed cryoglobulinemia manifests as nonspecific systemic symptoms, arthralgias,
palpable purpura, lymphadenopathy, hepatosplenomegaly, and peripheral neuropathy.
Most forms of generalized vasculitis cryoglobulinemia commonly involves the kidneys.
At the time of diagnosis, renal disease z evident in 20% of the pts, but ths can eventually
in up to 60% of pts. Initial manifestations of renal disease usually include asymptomatic
hematuria, proteinuria and mild elevations of serum creatinine levels. Complement
levels are usually decreased (C3 and C4).
Chronic Hepatitis-C infection is the most common cause of acquired essential mixed
cryoglobulinemia. Once cryoglobulinemia is suspected, circulating cryoglobulins should
be measured to establish the diagnosis. All pts presenting with vasculitis and elevated

cryoglobulin levels shd be screened for Hepatitis-C.


Antiviral therapy is the mainstay of treatment in pts with mixed cryoglobulinemia
associated with Hepatitis-C infection. In pts without renal dysfunction, alpha-interferon
& ribavirin are employed, while in pts wth impaired renal function, only alpha-interferon
is typically used.

15.Following are the indications for parathyroidectomy in secondary or tertiary


hyperparathyroidism in chronic renal failure;

Calcium levels over 10.5 mg/dL, not responding to conservative treatment


Moderate to severe hyperphosphatemia, not responding to medical mngment
PTH levels of more than 1000 pg/mL
Intractable bone pain
Intractable pruritis
Episode of calciphylaxis
Soft tissue calcification

Pts who undergo subtotal parathyroidectomy have increased chances of relapse.


Coexisting bone disease significantly worsens in pts who have undergone a
parathyroidectomy; therefore it is very important to rule out a low bone turnover or
adynamic bone disease before parathyroidectomy is performed.

16. All pt handoffs shd contain pertinent information about the patient demographics
and clinical status, care plan, anticipated problems and pending actions or results
that require follow up. Listing anticipated problems (and possible course of action)
is especially important as it allows the receiving providers to take faster, more
effective action if any of these complications arise.

17.Drug-Induced Lupus:
Is caused by some specific drugs such as Hydralazine, Procainamide, Minocycline,
and anti-TNF-alpha drugs (Infliximab and etanercept). Clinical findings include
fever, malaise, myalgias, fatigue, arthralgias/arthritis, serositis (pleuritic and
pericarditis) , hepatomegaly/splenomegaly, and night sweats.
Diagnosis is made by typical symptoms, history of taking an offending medication,
and a positive test for anti-nuclear and anti-histone antibodies.
Treatment is generally symptomatic and discontinuation of the offending drug typically
leads to resolution of symptoms within weeks to months.

18.Intravenous nutrition and Hyperglycemia:


Nutritional complications can occur from intravenous nutrition in critically ill pts. The
most dangerous metabolic complication, hyperglycemia, is most commonly caused by
too rapid initiation of the infusion. This complication is best prevented by initiation of
the infusion at 40-60 ml per hour and slowly increasing the rate at 20 cc/hour every 24
hours, while monitoring the pts glucose levels. Pts with normal glucose tolerance may
manifest glycosuria for the first 48 hours of parenteral nutrition. Before initiating insulin,
one must verify that the glucose is high & that glycosuria is not secondary to a reduced
renal threshold for glucose.
The most common cause of sudden hyperglycemia is sepsis and hyperglycemia may
antedate other signs of sepsis within 24 hrs. The sudden appearance of hyperglycemia
shd initiate a thorough search for the source of infection. The infection may be due to
line sepsis, pneumonia, wound infection or another process. Blood cultures and a
thorough examination of the patient are required.
Hypophosphatemia occurs in 30% of pts receiving IV nutrition. The mechanism is
enhanced phosphate uptake into cells associated with enhanced glucose uptake.

The consequences of hypophosphatemia include respiratory muscle weakness,


hemolysis & impaired oxygen release from the hemoglobin. Phosphate supplementation
in the diet will prevent these complications.
Bowel rest due to prolonged IV nutrition can lead to bowel atrophy and this is risky becx
the atrophic mucosa leads to disruption of mucosal barrier & hence intestinal microbes
can make their way into circulation.

19. Carbon dioxide retention can occur when excessive glucose is given to the pts with
severe lung disease. Glucose metabolism produces large quantities of CO2 for each
liter of O2 consumed. When the ability to eliminate CO2 via alveolar ventilation is
impaired, this enhanced CO2 production leads to hypercapnia and impaired ability
to wean the pt from mechanical ventilator.

20. Amiodarone:
Prolonged administration causes adverse effects like photosensitivity, skin discoloration,
bone marrow suppression, thyroid dysfunction, abnormal liver function tests, and
pulmonary toxicity. Among all these, pulmonary toxicity is the most serious adverse
effect that is responsible for most of the deaths associated with amiodarone therapy.
Pulmonary toxicity with amiodarone can be seen in the form of chronic interstitial
pneumonitis, organizing pneumonia, acute respiratory distress syndrome, and rarely
with a solitary pulmonary mass. Chronic interstitial pneumonitis is the most common
presentation of amiodarone-induced pulmonary toxicity. It is characterized by the
presence of nonproductive cough, fever, pleuritic chest pain, weight loss, dyspnea on
exertion and a focal or diffuse interstitial opacity on the chest radiograph. It is usually
seen after months to years of amiodarone therapy, especially in pts who are on higher
maintenance doses (more than 400 mg/day). It is a cumulative dose effect, and the

serum amiodarone levels are usually within the normal range in these pts. The chest
radiograph may reveal the presence of diffuse or focal interstitial or alveolar opacities.
Discontinuation of amiodarone is the mainstay of treatment for amiodarone induced
pulmonary toxicity. Corticosteroids can be used in pts with severe or life threatening
pulmonary disease. The prognosis is usually good in most cases and majority of pts
either stabilize or improve after the complete withdrawal of the drug.

20.Delirium:
It is an acute confusional state that develops over hours to days and this acute onset
distinguishes it from dementia. It is extremely common in old nursing homes pts. It
manifests as a change in the level of awareness, easy distractibility, cognitive and
perceptual disturbances, including memory loss, disorientation and even agitation.
Delirium is a multifactorial disorder. The factors that increase the risk of delirium are
subdivided into those that increase susceptibility to delirium & those that precipitate
delirium. The most common factors that increase the pts susceptibility to delirium
include advanced age and an underlying disorder (i.e dementia, Parkinsons disease,
and history of prior stroke). The factors that precipitate delirium include multiple
medications use (polypharmacy), infections (i.e urinary tract infections & pneumonia),
fluid and electrolyte imbalances (dehydration, hyponatremia, hypernatremia),
malnutrition, immobility (including the use of restraints), use of bladder catheters and
a wide variety of other medical conditions. In fact, delirium may be the only presenting
complaint in an older demented pt suffering from an acute medical illness.
The first step in delirium is to check the pts vitals and do a pulse oximetry.
P.S : Use of psychotropics as a first line therapy for behavioral problems z not
recommended.
Focused history, physical examination and basic laboratory work-up shd be performed

early in the initial evaluation of delirium in any patient.


The initial step in counteracting a case of delirium shd be reassurance, trying to reorient
the pt to environment and constant supervision.
If the above measures fail, then use a low dose haloperidol. Start with low dose & then
titrate to higher doses as antipsychotics have many adverse effects.
Physical and chemical restraints shd be used as a last resort and be used judiciously in a
combative and disruptive patient.

21.Henoch-Schonlein purpura:
Is primarily a pediatric vasculitis that results from systemic IgA deposition. The
presentation varies in severity but most pts have a rash that begins as erythematous
macules that coalesce into palpable purpura. The purpura is non-blanching and nonpruritis & z distributed symmetrically across gravity dependant parts (e.g legs, buttocks).
Intermittent abdominal pain & nausea are usually due to small bowel intusscusception.
Edema and hemorrhage in the small bowel serve as the pathological lead points.
Involvement of the joints and kidneys is also common and usually manifest later. Some
boys have scrotal pain/swelling. An antecedent upper respiratory infection is present in
50% of pts.
HSP is diagnosed clinically, especially when the presentation is classic and involves lower
extremity purpura. Lab findings that support the diagnosis include normal platelet count
, and coagulation studies and elevated inflammatory markers (e.g WBC, ESR). Urinalysis
may show casts, hematuria, pyuria, and proteinuria. HSP resolves spontaneously in most
pts, some require NSAIDs or corticosteroids.

22.Multiple medications are a major cause of adverse drug reactions in the elderly.

23.The major treatable risk factors for ischemic stroke include Hypertension, Diabetes,
smoking and dyslipidemia. Hypertension is a risk factor for atherosclerotic disease &
z the most important modifiable risk factor for stroke. Current guideline recommend
a systolic blood pressure of <150 mmHg and diastolic goal of <90 mm Hg in pts >60
years old. Some studies have also shown a modest benefit with smoking cessation
and control of blood sugar and lipids.

24.Brain dead organ donors management:


Brain death leads to hypotension (loss of sympathetic tone), volume depletion (e.g
secondary to central diabetes insipidus) and systemic infections. Adrenal insufficiency
further contributes to hypotension.
Brain dead pts shd be managed in an ICU with continued hemodynamic support,
adequate volume resuscitation and vasopressors if needed. Hormone replacement
therapy (methylprednisone, thyroid hormone, and vasopressin) shd be administered.
Mechanical ventilation shd be continued to prevent hypoxia and hypercapnia.

25.Hemochromatosis:
It is an autosomal recessive disorder. Results in excessive accumulation of iron in the
parenchymal organs. It can ultimately manifest as liver disease, skin pigmentation,
Diabetes mellitus, arthropathy, impotence, or cardiac enlargement. Symptoms are
often present for ten years or more before the diagnosis is made. The classic triad of
cirrhosis, diabetes mellitus, and skin pigmentation (bronze diabetes) is a late finding
that occurs when the total body iron content is 20 grams or more.
Therapeutic phlebotomy is the preferred means of treating hemochromatosis.

26.Most pts with Turner syndrome are infertile due to atretic or insufficient ovaries. Pts
with Turner syndrome shd be informed that the chance of becoming pregnant w/o
donor oocytes is extremely low. Estrogen-progestin replacement therapy is used to
prevent osteoporosis but it has not been shown to improve fertility.

27.Although there is no uniform approach for a child with suspected language delay or
mental retardation, there are several points that shd not be missed. First of all, the
initial evaluation shd include a comprehensive history and physical examination;
these may provide importnt clues to the potential causes of observed developmental
delay (e.g microcephaly, dysmorphic features, high risk behavior for lead poisoning &
family history of neurological abnormalities).
Another essential part of the initial evaluation is sensory screening; this shd include
a complete ophthalmologic examination and audiometry, preferably using brainstem
auditory evoked response. It is estimated that 13-50% of pts with mental retardation
have visual abnormalities and approximately 18% have hearing impairment. Further
evaluation will depend on the initial findings in the history, physical examination and
sensory screening.

28.It is important to recognize Vit-B12 deficiency as a potential cause of delirium or


dementia in the elderly patient. Subtle laboratory findings, such as increased MCV
and reticulocytes, or pancytopenia, can help to identify the condition.

29.Aspiration pneumonia:
This condition is due to an infection of the lung parenchyma by microbes from the oral
cavity, commonly gram positive cocci, gram negative rods & anaerobes (polymicrobial).
Its more common in elderly demented and stroke pts. Aspiration pneumonia shd be

suspected in pts with recent history of foreign body aspiration. Pts typically present 1-5
days following an aspiration episode with respiratory difficulty, cough, sputum, fever
and chills. Clindamycin has excellent activity against gram positive cocci and anaerobes.
It is usually the first line drug as it is inexpensive, widely available and associated with a
low rate of methicillin-resistant staphylococcus aureus superinfection. A combination
of beta lactam and beta lactamase inhibitor would be another option.
It is important to differentiate aspiration pneumonitis from aspiration pneumonia.
Aspiration pneumonitis is a chemical injury with inflammation of lung parenchyma due
to the inhalation of foreign materials such as gastric contents (Mendelsons syndrome).
Pts usually have a history of depressed level of consciousness, a history of vomiting
or aspiration event. Respiratory distress occurs 2-5 hours after an aspiration event and
there are generally no features of infection. Treatment z supportive (without antibiotics)
Speech and swallowing evaluation, followed by diet modification is performed in high
risk pts (e.g dementia/stroke) to prevent future episodes of aspiration. It is unclear
whether percutaneous endoscopic gastrostomy tube placement prevents aspiration
episodes or improves long-term survival of pts with swallowing difficulties.

30.Acute kidney injury:


Acute tubular necrosis;
Acute tubular necrosis (ATN) is responsible for most cases of acute renal failure
in hospitalized pts. The etiopathology generally involves a perfusion deficit due
to hypovolemia, hypotension, shock, sepsis, or low cardiac output states. The
typical presentation is oliguria following a hypotensive episode, with elevated
BUN and Cr. (though the ratio is typically normal) and anion gap acidosis.
It does not respond to fluid challenge test. Examination of the urinary sediment
shows muddy brown casts. There is high fractional excretion of sodium.
Prerenal Azotemia;
Prerenal azotemia is associated with decreased fractional excretion of sodium
i.e <1%. However, once ATN sets is, the fractional excretion of sodium increases
i.e >2%. Response to fluid challenge is the gold standard in distinguishing
prerenal azotemia (which responds to fluid challenge with improved urine
output) from ATN. Examination of the urinary sediment in prerenal azotemia
shows bland casts.
Muddy brown casts and high fractional excretion of sodium and no response to
fluid challenge test are the key factors differentiating ATN from Prerenal
Azotemia.

31. oliguric/anuric pts are susceptible to fluid overload, so avoid giving excess of IV fluids

32.Indications for dialysis include uremia, fluid overload, intractable hyperkalemia,


and metabolic acidosis.

33.Pts with indwelling urinary catheters may grow Candida on urine culture but they
do not require treatment with antifungals unless there is evidence of a systemic
infection or tissue invasion.

34.Intravenous albumin can expand intravascular volume and is used for volume
expansion in cirrhotic pts with hepatorenal syndrome or spontaneous bacterial
peritonitis, as they are at risk for third spacing of fluids.

35. Autoimmune Hepatitis:


Autoimmune Hepatitis causes progressive parenchymal liver damage. Its course is
variable and severe cases can progress to cirrhosis and liver failure in 6 months. Most
cases occur in young to middle aged women. Autoimmune manifestations such as
arthritis, erythema nodosum, thyroiditis, pleurisy, pericarditis, anemia & sicca syndrome
are common. Common findings are elevated AST & ALT levels, a normal or near normal
alkaline phosphatase level and a normal bilirubin level. Autoantibodies are common and
the most typical are antinuclear antibodies (in a homogenous staining pattern, hence
the term lupoid hepatitis) and anti-smooth muscle antibodies (against actin).

36.ERCP is the gold standard for diagnoses of sclerosing cholangitis, which is seen in
association with ulcerative cholitis.

37.Herbal toxicity;
Gingko biloba leaf extract is an increasingly popular herbal supplement that many
pts use as a memory booster because of its suggested propensity for increasing
the cerebral blood flow. Some studies have suggested that gingko is atleast
somewhat effective in the treatment of intermittent claudication and Alzheimer
dx. Gingko has been associated with with a number of side effects, including
seizures, headaches, irritability, restlessness, diarrhea, nausea, and vomiting. It is
most notorious for increasing the risk of bleeding and its potentiation of the
effects of anticoagulation therapy through various mechanisms, including the
inhibition of platelet-activating factor. Several cases of serious intracerebral
bleeding associated with gingko use have been reported; in most cases, pts were
concurrently taking anticoagulant medications.

Aconite is an ingredient in Chinese Herbal medicines used to treat pain or heart


failure. It is known to cause serious and sometimes fatal arrhythmias.
Hepatotoxicity occurs with usage of unsaturated pyrrolizidine alkaloids. These
compounds are found in several herbal supplements, including comfrey, borage
leaf, and coltsfoot. Other herbal supplements known to cause liver toxicity
include ephedra, chaparral, germander, and a Chinese medicine called jin bu
huan.

38.Psychosis can occur with use of corticosteroids, appetite suppressants, quinacarine,


antidyskinetic agents (e.g levodopa, carbidopa, amantadine).

39.Polypharmacy is frequent in the elderly. Orthostatic hypotension is not uncommon


in this setting and is usually related to the use of diuretics, alpha blockers or nitrates.

40.Hepatorenal syndrome:
Hepatorenal syndrome is a common cause of acute renal failure in pts with cirrhosis
but shd be considered a diagnosis of exclusion. An ultrasound shd be performed to
exclude the presence of urinary obstruction. Spontaneous bacterial peritonitis (SBP)
is a frequent precipitant of hepatorenal syndrome and as such paracentesis is indicated
if ascites is present.
Pts with cirrhosis develop decreased peripheral vascular resistance due to splanchnic
vasodilation, which can cause the decreased renal perfusion of hepatorenal syndrome.
Similarly, volume depletion due to over-diuresis can cause renal dysfunction of a prerenal etiology. A fluid bolus is needed to confirm that the renal failure is not secondary
to volume depletion.
A combination of midodrine and octreotide along with albumin (for intravascular
volume expansion) is the treatment of choice after the diagnosis is confirmed.

41.Opiod intoxication:
Opioid toxicity presents with the clinical triad of respiratory depression, miosis and
depressed mental status. Decreased bowl sounds are also common. Morphine acts
directly on brainstem respiratory centers to produce respiratory depression that
results in decreased depth and frequency of breaths (leading to respiratory acidosis).
It also decreases both the hypercapnic and hypoxic respiratory drive from interaction
with central and peripheral chemoreceptors.
Morphine undergoes a 2-step metabolic process beginning with hepatic conjugation
with glucuronic acid to form the metabolites morphine-3-glucuronide & morphine-6glucuronide (which is more potent than morphine). The kidney then eliminates the
metabolites. In the setting of kidney injury, morphine-6-glucuronide accumulates and
potentiates the effects of morphine.
Bottom line: Opiod toxicity presents with the clinical triad of respiratory depression ,
miosis and depressed mental status. Decreased bowl sounds are also
common. Opioids shd be monitored and dosed appropriately as their
effects can be worsened in the setting of renal injury or liver disease.

42.Atrial fibrillation occurs after CABG in 15-40% of pts.It is usually self-limited in those
without a prior history of atrial fibrillation and reverts to sinus rhythm 6-8 weeks
after surgery.

43.Uremia involves a buildup of toxins that can cause nausea, vomiting, neuropathy,
lethargy, seizures and coma.

44. Obesity:
Pts with BMI > 35 kg/m2 are at high risk for complications from obesity such as
osteoarthritis, obstructive sleep apnea, back pains, hypertension, and dyslipidemia.
The cornerstone of therapy is dietary caloric restriction, exercise, and behavioral
modifications of dietary habits.
Pts who fail to lose weight with above measures can be treated with orlistat.
Pts who who do not lose weight with either of these modalities shd be considered for
Bariatric Surgery. Current guidelines recommend bariatric surgery for pts with BMI
greater than 40 kg/m2 who have a low risk for surgery and failed previous weight loss
treatments and those with BMI greater than 35 kg/m2 who have the obesity related
comorbidities.

GENERAL PRINCIPLES:
1.Digoxin toxicity:
Digoxin toxicity is characterized by nausea, vomiting, anorexia, fatigue, confusion,
visual disturbances, and cardiac abnormalities.
Verapamil can potentiate digoxin toxicity by increasing serum digoxin levels through
inhibition of renal tubular secretion of digoxin.
Other medications that can potentiate the digoxin toxicity are quinidine, amiodarone
and spironolactone.
Captopril can cause slight elevation of digoxin levels (not other ACEIs).

2.Chest X ray abnormalities may suggest a variety of specific causes in pts with
hemoptysis, including malignancy, focal infection (e.g pulmonary tuberculosis), or
heart disease (e.g mitral stenosis).

3.COPD:
Acute bronchitis can cause an exacerbation of underlying COPD. Airway inflammation in
acute bronchitis can cause erosion of superficial vessels, leading to hemoptysis, which is
usually scant or small in volume. Plain chest radiograph remains initial test of choice
as it may identify the site & cause of bleeding (e.g cavitary lesion, lung mass, or stigmata
of mitral stenosis) in over one-third of pts. In combination with clinical picture, a chest
radiograph may also help direct further management (bronchoscopy versus CT scan).
Generally, antibiotics are not required for the treatment of acute bronchitis as it is of
viral etiology, but per the guidelines they shd be given in COPD exacerbation when any
two of the following features r present; increased sputum purulence, increased sputum
volume or increased dyspnea. Antibiotics are also recommended for those requiring

mechanical ventilation (invasive or non-invasive).

4. Information obtained about a pts condition, including that obtained by history taking
and chart review, should not be discussed with family and friends unless authorized
by the patient.

5.Marfan syndrome:
Is an autosomal dominant disorder characterized by mutations in the extracellular
matrix protein fibrillin-1. Diagnosis is often made based on clinical features.
The main cause of morbidity and mortality in pts with Marfan syndrome is aortic root
disease manifesting as aneurysmal dilatation, aortic regurgitation, or dissection. All
pts with Marfan syndrome shd have a routine screening for aortic root disease with a
transthoracic echocardiogram or chest CT scan. Pts found to have aortic root disease
and/or a family history of aortic dissection or sudden death shd be counselled against
any strenuous physical activity. They can participate in low to moderate intensity
recreational noncompetitive sports (e.g bowling, golf, doubles tennis). They shd also
have a repeat echocardiogram on a regular basis to check for progressive aortic disease.

6.Abdominal gun shot wounds:


Pts shd be first undressed completely to evaluate for abdominal wounds, entry and
exit points and wounds in other parts of the body.
Unstable pts with signs of peritonitis (abdominal distension, tenderness) or organ
evisceration shd undergo immediate laparotomy.
Stable pts may be evaluated first with non-operative methods, like plain films or
contrast CT scans of the abdomen, followed by surgical evaluation.

7.When combining sildenafil with an alpha blocker, it is important to give the drugs
separately with atleast 4 hours intervals to reduce the risk of hypotension.

8.Pts with PPD >5 mm r considered positive if they had close contact with an individual
recently diagnosed with active tuberculosis.

9.INH:
The most common side effect of INH is hepatotoxicity, which can occur within
the first 2 months of therapy and range from mild hepatotoxicity to hepatitis.
Risk factors include daily alcohol intake, HIV, underlying liver disease (e.g chronic
viral hepatitis), age >50 and active intravenous drug use. Mild INH hepatotoxicity
presents with increased transaminases (usually <100 IU/L). However, pts are
usually asymptomatic. They typically have self-limited transaminitis and can
continue the INH with close monitoring. All pts shd have baseline measurement
of aminotransferases before starting INH and monthly monitoring during the
therapy. Pts with normal baseline aminotransferases shd discontinue the drug if
they have aminotransferases >5 times the upper limit of normal (ULN) or
develop symptoms with aminotransferases >3 times the ULN.
Can lead to neurotoxicity. Susceptible individuals (e.g alcoholic pts, malnourished
, children, diabetic pts, HIV pts, those with chronic kidney disease) can develop
ataxia, peripheral neuropathy and paresthesias. Giving Vit-B6 along can prevent
the neuropathy.
Drug induced lupus can occur and present with joint pain, malaise, and low grade
fever. However, this rarely occurs.

10.Ethambutol-> Ocular toxicity.

11.Pyrazinamide->Hyperuricemia.

12.Menopause:
Menopause is an important risk factor for osteoporosis. Pts shd be encouraged on good
nutrition & weight bearing and anti-gravity exercises (walking, hiking, jogging). Dietary
sources of calcium include milk, yogurt and dark green leafy vegetables.
If the dietary sources are unable to meet the requirements, then Vitamin-D and calcium
supplements should be taken. But dietary sources shd be encouragd as the supplements
are associated with potential adverse effects such as hypercalcemia, hypercalciuria and
nephrolithiasis (kidney stones).

13.Bone density screening shd begin at age 65 in women without risk factors. Postmenopausal women age <65 may be screened if they have risk factors such as body
weight <58 Kg, chronic steroid use, smoking, malabsorptive disorders (e.g pancreatic
insufficiency in cystic fibrosis), and parental history of hip fracture or personal hx of
low impact fracture. Bone anti-resorptive agents (e.g bisphosphonates, raloxifene)
are approved for osteoporosis prevention in high risk women.

14.If the father is affected by an X-linked recessive disorder and the mother is not a
carrier, their children will not be affected by the disease, although all their female
children will be carriers. (do the crossing and u will know).
Hemophilia is an X-linked recessive disease

15.Heart failure:
A combination of hydralazine plus nitrate (Isosorbide dintrate) therapy hz been shown
to provide additional symptomatic and mortality benefit in African American pts with
persistent New York Heart association class III or IV symptoms due to left ventricular
systolic dysfunction i.e left ventricular ejection fraction <40%, not responding to optimal

medical therapy (beta blockers, ACEIs, Aldosterone antagonist and diuretic).


It is also recommended in pts who cannot tolerate ACEIs or ARBs due to hypotension,
renal failure or hyperkalemia.

16.Patient Handoffs:
Ineffective patient handoffs can cause significant delays in care, workflow redundancies
and adverse patient outcomes due to errors or oversight. Handoffs frequently occur in
less-than-ideal environments, with time constraints & interruptions from staff members.
However, most errors result from a failure to communicate in a systematic manner that
incorporates the big picture and assists with anticipatory decision making. Using an
organized template or a checklist has been shown to improve sign out efficacy and
accuracy. Sign-outs shd be conducted verbally, face-to-face, with an updated, succinct
written component.
Bottom line: Undesired medical outcomes (e.g errors, inefficiencies, delays) often result
from inadequate patient handoffs between medical providers. Use of
sign-out procedures that incorporate systematic checklists and templates
can reduce the incidence of poor outcomes.

17. Turner syndrome:


Turner syndrome is characterized by 45 XO. The clinical presentation z short webbed
neck, dorsal feet and hands edema. Cardiac murmur may be present. There is no
barry body on the buccal smear.
No increased recurrence risk is present after having an infant with Turner syndrome.
The risk of 45,X does not increase with increased maternal age ( unlike Down synd.
and Klinefilter synd. <47 XXY> ).

18.Maintenance of an adequate airway & hemodynamic stability is the most important


step in management of a trauma patient. Intubation is recommended for a GCS score
8 or less.

19.Animal bite:
All animal bites, regardless of the site should be thoroughly cleaned and irrigated with
normal saline and all devitalized tissues shd be debrided. A plain radiograph should be
obtained if a patient is suspected of having a foreign body or has a bite occurring close
to a bone. Most open lacerations can be closed primarily within a few hours of injury.
This is especially true for injuries of the face where infections are less common due to a
good vascular supply;however, bites involving the hands should not be sutured or closed
primarily due to a high risk of subsequent wound infection. These shd be left open to
drain and examined frequently for signs of infection. Other situations where primary
closure is not recommended include puncture wounds, cat and human bites (high risk of
infection) and pts presenting late after the bite.

Bottom line: Dog bite injuries of the hand and puncture wounds anywhere on the body
should not be closed primarily due to high risk of development of wound
infection.

20.Metoclopramide induced dystonia:


Diphenhydramine IV is the drug of choice for acute dystonia by metoclopramide, becx of
its anti-cholinergic properties. It provides relief within 3-5 minutes.
If that does not work, go for Benztropine IV.

21.Herbal medicines:
None of the following herbal preparations have been shown to be significantly
efficacious. However, following are the conditions for which they are commonly used
in USA.

Saw palmetto is commonly used in the treatment of BPH.


Garlic is often used in the treatment of hypercholesterolemia
St Johns wort is frequently used in the treatment of depression
Glucosamine and chondroitin are used for osteoarthritis

Saw palmetto is generally well tolerated but shd be used cautiously in pts undergoing
surgery due to a possible increased risk of perioperative bleeding (likely due to platelet
dysfunction). Other popular herbal remedies associated with increased bleeding risk
include ginkgo biloba, ginseng, black cohosh, and garlic.

22.Fish oil supplementation can be effective in the treatment of refractory


hypertriglyceridemia.

23. Adverse effects of herbal remedies;


Echinacea-> anaphylaxis
Ephedra-> Myocardial infarction
Aristolochic acid->Nephrotoxicity especially due to concurrent use with appetite
suppressants such as fenfluramine, diethylpropion.
St johns wort-> Short term side effects include GI distress, dizziness, fatigue,
photosensitivity and dry mouth. Long term adverse effects include increase in
anorgasmia, urinary frequency and swelling.

24.Thiazolidinediones:
PPAR-gamma receptor agonists (pioglitazone, rosiglitazone) act on the PPAR-gamma
receptors in nephrons to stimulate the sodium and fluid resorption, resulting in fluid

retention. This is the action similar to aldosterone causing sodium retention. Most of
such pts who experience fluid retention and edema with thiazolidinediones would have
an underlying heart failure. Such pts can be treated with aldosterone antagonists such
as spironolactone in order to remove the excess fluid.

25.Dental caries:
Risk factors for dental caries include frequent exposure to sugary snacks or beverages,
nighttime bottle feeding, especially with juice (baby bottle tooth decay), low
socioeconomic status and caregivers with cavities. Poor dietary habits are the most
detrimental and should be discouraged. Dental screening examinations by primary care
physician shd begin as soon as the first teeth erupt (usually around 5-8 months), with
immediate referral to dentists for obvious decay, abscess, or risk factors identified
during screening.
For children aged <3, a fluoride toothpaste shd be used twice a day as a very thin
layer (smear) covering less than half of a toothbrush. Fluoride toothpaste helps
prevent caries. However, caries in infants are more likely to result from cariogenic
dietary habits than lack of fluoride use.

Bottom line: Education on oral hygiene and healthy dietary behaviors shd be provided
at every well child visit during infancy. Bedtime bottle use and frequent
exposure to sugary snacks are the primary cariogenic risk factors. As soon
as a childs first teeth erupt, fluoride toothpaste shd be used judiciously
& dental plaques, white spots & cavities shd be assessed on examination.

26.Klinefilters syndrome is the strongest known risk factor for male breast cancer. It
carries a 50-fold increase in the risk of breast cancer compared to men with a normal

genotype.

27.Neurofibromatosis-1:
The diagnostic criteria for NF-1 are two or more of the following: a first degree relative
with NF-1, the development of more than six CALS of 5 mm in greatest diameter (if
children) or 15 mm (if adult) or the presence of more than two neurofibromas, lisch
nodules, optic glioma, bone dysplasia, or axillary freckling.
NF-1 must be suspected in all first degree relatives of a patient with the disease. This
condition is especially important in the pediatric population, becx the children affected
with this disease can develop optic gliomas, other CNS tumors, developmental problems
, or bone dysplasia.
When the pt is diagnosed with NF-1, an immediate ophthalmologic referral is needed
due to 15% risk of developing optic gliomas. If there are any ophthalmologic or
neurologic abnormalities, other tests such as MRI, head CT scan, EMG or neurosurgery
evaluation may be needed.

28.Hospitalized patients:
Adverse events occur in approximately 5-15% of hospitalized pts, although these
numbers appear to be decreasing and vary significantly depending on how an adverse
event is defined. These events can be grouped into 4 broad categories: operative and
post-procedure, adverse drug and general care (e.g falls, pressure ulcers) events and
hospital acquired infections.
The most common adverse events in hospitalized pts, such as wound infections,
bleeding, or deep venous thrombosis, are related to surgery. In pts not undergoing
surgery, the largest group is adverse drug events (ADEs), which are commonly related
to use of hypoglycemic agents, anticoagulants, antibiotics, or analgesics. ADEs are

estimated to occur in slightly over 5-7% of all hospitalized pts and account for almost
one-fifth of all adverse events in some studies.

27.Alcohol intake:
Moderate alcohol intake (<2 drinks/day for women, <3 drinks/day for men) has been
associated with greater insulin sensitivity, which may lower the incidence of diabetes
and improve overall glycemic control in pts with established type diabetes. Moderate
alcohol consumption may also result in lower doses of antidiabetic medication needed
to achieve treatment targets. Other effects of moderate alcohol intake include lower
risk of atherosclerotic heart disease (in both diabetic and nondiabetic pts) and total
mortality.
Intermittent heavy drinking-defined as >14 drinks/week for men (or 4 drinks at a time)
and >7 drinks/week for women (or 3 drinks at a time)- and binge drinking- defined as
>4 drinks at a time for women and >5 drinks at a time for men- do not have the same
benefits as steady, moderate consumption. Heavy alcohol intake can cause unstable
glucose levels and is associated with potentially severe hypoglycemia in pts treated with
sulfonylureas (e.g glyburide). Other harmful effects include atrial fibrillation, dilated
cardiomyopathy, increased risk of cardiac events, increased overall mortality, and
elevated blood pressure with refractory hypertension.

28.

29.Complementary and alternative medicine (CAM):


CAM is a term that includes folk medicine, body manipulation, massage and herbal and
traditional ethnic practices.
If CAM is inherently harmful and would displace effective conventional care, then the
therapy should be discouraged.
CAM can be used if they are not inherently harmful & do not interfere with conventional
care.
A court order can be indicated if CAM therapy is life threatening.
*Motherwort is a CAM that is not harmful.

30.Age is the single most important risk factor for osteoporosis and osteoporotic bone
fracture.
Other contributing factors such as family history, smoking and alcohol consumption
are also important.
The medications which predispose a pt to have osteoporosis include glucocorticoids,
anti-androgens and some anti-convulsants.
Caucasians are more susceptible to osteoporosis than African americans.

31.Tachycardia is the earliest sign of hypovolemia.

32.ACEIs and Angioedema:


Angioedema presents with facial swelling and prominent lip and tongue swelling. It is an
uncommon but well recognized side effect of ACEIs. Some pts may develop abdominal
pain due to intestinal swelling. ACE catalyzes the degradation of bradykinin, inhibition
of ACE leads to elevated levels of bradykinin that can cause angioedema as well as a
dry cough. Angioedema associated with ACEIs is usually self-limited, lasts 24-48 Hrs, and

unlike allergic angioedema, is not associated with itching or urticaria. Half of all cases of
ACEIs related angioedema occur in the first few weeks of therapy, although it can occur
even in pts who have been on medication for several years. ACEIs related angioedema
affecting the airway can be life-threatening. Pts who develop angioedema should not
receive ACEIs again.
ARBs are generally used as a replacement for ACEIs in this situation as they do not
affect the bradykinin system and otherwise have the same general benefits. Some pts
may have a recurrence of angioedema during the first few months after discontinuing
the ACEIs and still need monitoring on the ARB. It is not clear whether this is a direct
effect of ARBs, residual effects of ACEIs or underlying tendency of the pt to develop
angioedema.

33.Many herbal medications are not strictly regulated and some may contain unknown
toxic ingredients and cause dangerous drug interactions.

34.Physicians must be cautious when asked about new therapies that have not been
adequately tested. They should defer recommending alternate therapies until
obtaining adequate information frm well designed research studies that substantiate
the efficacy, safety and quality of these products.

35.Mitochondrial disease:
Only mother to offspring transmission is possible. If daughters get the disease, they will
transmit it to all their children but males who get the disease wont transmit it.
So, Only FEMALES TRANSMIT, males DONT.
Leber hereditary optic neuropathy is one example where there is bilateral optic atrophy
at age 15-30 rzlting in blurred vision. Other examples of mitochondrial diseases include

mitochondrial encephalopathies and myopathies (e.g MELAS).

36.Clinical microsystems:
Clinical microsystems are frontline health care settings where pts, families and health
care teams interface. These microsystems may also include support staff, equipment
, information technology, billing and processes specific to that setting. They are the
building blocks of the larger health care system. Quality improvement measures
targeting microsystem components can enhance delivery of high quality care and
improve overall patient outcomes in the system as a whole.
An example is bedside scoring system to be used all clinical staff members to ensure
that all staff members are aware of the most unstable pts, allowing them to take
proactive measures that may prevent an emergency response code.

37.Of all the antibiotics, beta lactams are the most commonly associated with adverse
CNS events. Specifically, penicillins, cephalosporins, monobactams, carbapenems
(including imipenem) and fluoroquinolones are the antibiotics most likely to trigger
seizures.
Imipenem is associated with increased risk of seizure.

GASTROINTESTINAL
1.Managing pts with chronic medical problems related to non adherence to recommend
lifestyle changes requires a patient-centered approach tht is empathic,non judgmentl
and collaborative.Never blame a patient and encourage their participation to improve
rapport and adherence to treatment.

2.The most common cause of Lower GI bleed in elderly pts is angiodysplasia ( vascular
ectasia) or diverticulosis. Remember the association of angiodysplasia with aortic
stenosis and ESRD.
The cause of angiodysplasia is unknown but may be related to degenerative changes
associated with aging and to intramuscular hypertrophy that obstructs submucosal
veins.

3.In elderly population who present with lower GI bleed, even if hemorrhoids are prznt,
colon cancer must be ruled out.

4.Cancers of both left and right side can cause a trace amount of bleed and often prznt
with anemia.

5.Diverticulosis bleed is generally painless and can be massive.Diagnose with clonoscpy


and add fiber to the diet for recurrent cases.

6.Diverticulitis bleed is usually mild.

7.Diarrhea in children:
>In a child with diarrhea and no clinical signs of dehydration,preferred intake includes
fluids low in sugar and the resumption of a normal,age-appropriate diet with plenty of
complex carbohydrates & minimal sugar and fat.Foods containing excessive sugar can
increase the osmolarity of the stool in the intestinal lumen and fatty foods are known
to delay gastric emptying.Both can worsen the diarrhea.
>The previously used BRAT diet ( Banana,rice,apple sauce, tea/toast) is no longer
recommended for refeeding children with diarrhea becx its quite low in calories and
protein.Moreover,the bananas and apple sauce in the BRAT diet add excessive sugar.
>Dont use loperamine as its linked with paralytic ileus,toxic megacolon,CNS depression
,coma and death in children.
>Clear liquids like apple juice and lemon-like soda contain excessive sugar.Better choice
would be Gatorade.
>Admission to the hospital for IV hydration is only necessary when there are signs of
dehydration.

8.Drug induced pancreatitis is mild and responsible for 5% of cases:


>Pts on diuretics=>furosemide,thiazides
>Pts with IBD=>Sulfasalazine,5-ASA
>Pts with immunosuppression=>Azathoprine,L-asparaginase
>Pts with hx of seizures or bipolar disorder=>Valproic acid
>AIDS pt=>Didanosine,Pentamidine.
>Pts on antibiotics=>Metronidazole,tetracyclines.

9.Dumping syndrome:
Dumping syndrome is a common complication of gastrectomy.Liquid and food passage
through the stomach into the jejunum is faster.This leads to the abdominal pain,diarhea
,nausea and vomiting as well as some neurovegetative symptoms such as dizziness,
generalized sweating, and dyspnea.The treatment is aimed at decreasing the speed of
the passage of fluids and food into the small gut.A high protein and low-carbohydrate
diet is advised, as well as smaller but more frequent meals throughout the day.
P.S: =>Dont give metoclopramide as it stimulates the contraction of the Lower
esophageal sphincter and increased the gastric emptying and would worsen the
symptoms.
=>A low fiber diet may improve the diarrhea but will not control the other features
of dumping syndrome.

10.Dysphagia:
>Oropharyngeal dysphagia is characterized by difficulty initiating a swallow,often
often accompanied by coughing,drooling, or aspiration.
>Esophageal dysphagia is characterized by delayed sensations of food sticking in the
upper or lower chest.
>Dysphagia initially involving both solids and liquids would suggest a neuromuscular
disorder
>Dysphagia that occurs initially for solids and later includes liquids is characteristic of
mechanical obstruction.
Structural lesions that lead to dysphagia in the pharynx and upper esophagus
may be visualized with nasopharyngeal laryngoscopy.This pattern of dysphagia
can also be evaluated using a standard barium esophagogram,which will often
identify upper esophageal disorders(e.g zenkers diverticulum).An esophagoram

is also indicated when there is suspicion for achalasia or a long-segment stricture


(seen in pts with prior radiation exposure or caustic ingestion).If no diagnosis is
made on laryngoscopy or barium esophagogram, an Esophagogastroduodenscopy
(EGD) may be needed to visualize the esophageal mucosa fully.

11.Esophageal adenocarcinoma is found primarily in the lower esophagus and is


associated with chronic gastroesophageal reflux and Barrets esophagus.Squamous
cell carcinoma is often seen in pts who use tobacco and alcohol and is usually
located in the upper esophagus.

12.Tobacco and alcohol causes squamous cell cancers of head and neck.

13.Erosion of the artery is the most common source of the diverticular bleeding.The
erosion is caused by a fecalith in the diverticular sac.Vasa recta in the diverticulum
are separated from the lumen only by mucosa and are exposed to luminal trauma.
This results in weakness of artery,which predisposes it to rupture by fecaliths.

14.Hemorrhoids-> Venous bleeding.

15.Boerhaves syndrome:
Esophageal perforation, aka Boerhaaves syndrome, usually presents as acute chest pain
following episodes of repeated vomiting.Other associated symptoms are dyspnea,
epigastric pain or shoulder pain.Most esophageal tears occur in the distal third of
esophagus and this leads to pleural effusion.In 75% of cases, the pleural effusion will
develop six hours after perforation.The effusion is located on the left side in 66% of the
cases. and can be accompanied by pneumomediastinum,pneumothorax or both.

It takes a mean of more than 4 hrs for fever to develop due to mediastinits.Urgent
management is needed since mediastinits carries a mortality rate of more than 40% if
not properly diagnosed within the first 24 Hrs.
The best diagnostic test for esophageal perforation is an esophagogram with water
soluble contrast.This test provides a definite diagnosis in 90% of cases.If the test is -ve,
but the clinical suspicion is high,barium contrast can be used.Water soluble contrast is
preferred to barium becx the latter can produce further mediastinal irritation and injury.
CT scan of the chest is helpful but it may not detect small tears or ruptures.
Upper GI endoscopy has no role and shd not be used.

16.Binge drinking-> vomiting

17. a.Pancreatitis=>decreased bowl sounds


b.Boerhaave syndrome=>Normal bowl sounds

18.Gastrostomy tube placement is a safe and effective means of providing nutrition to


pts with oropharyngeal dysphagia after acute stroke.It is often temporary as most pts
will recover significant function within a year of cerebral infarction.
P.S. =>NG tube feeding is associated with tube dislodgement,pt discomfort &
pulmonary complications, so its not recommended.
=>Total parenteral nutrition is associated with central line placement risk &
increased risk of infection.So,its not a good long term solution.

19.Upper GI bleed:
>Hematemesis
>Melena can occur
>BUN:Cr >20:1 is suggestive
>Hemodynamic compromise and orthostasis are common.
>NG tube with aspiration of bile and blood is highly suggestive.
>After resuscitation, the next step is to perform EGD.

20.Lower GI bleed: (LGIB)


>Bright red blood per rectum (BRBPR) is nearly always due to Lower GI bleed.
>NG tube with aspiration of bile and NO blood increases the likelihood of LGIB.
>After hemodynamic stabilization,Colonoscopy is the next best step in management.
Colonoscopy has diagnostic and potentially therapeutic abilities in the lower GI bleed.
>If no bleeding source is found,EGD shd be considered.
>Angiography can be considered in pts with BRBPR in whom EGD did not reveal a source
of bleeding.
>If angiography is unsuccessful, other techinques ( e.g tagged RBC scan,enteroscopy,
surgery) can be considered.

21.Pts who hve undergone cholecystectomy do not need to change their dietary habits.
Half of them may have transient and mild episodes of diarrhea,flatulence or bloating
that will be self-limited and do not necessitate any interventions.

22.Familial adenomatous polyposis:


Transmitted in an autosomal dominant fashon, so pts typically have a hx of first degree
relative with early onset colon cancer.However upto 1/3 of pts with FAP have no such

family hx due to a new mutation.Polyposis typically begins in the 2 nd or 3rd decade of


life and pts usually have over 100 adenomatous polyps on colonoscopy.Symptoms
preceding the colonoscopy are generally vague and non-specific.Genetic testing for
mutations in the APC gene is available to confirm the diagnosis.The vast majority of
these pts will progress to develop colorectal carcinoma if colectomy is not performed.
Pts with FAP are also at risk of developing extracolonic neoplasms.Both gastric and
duodenal adenomas/carcinomas are more prevalent among pts with FAP ,so screening
upper GI endoscopy shd be performed after a diagnosis of FAP is made.
The vast majority of pts with FAP will go on to develop colon cancer by the age of 45.
Prophylactic colectomy is therefore strongly recommended.

23.Life time risks of colon cancer:


>General population=> 10%
>Peutz Jeghers syndrome=>40%
>Lynch syndrome(Hereditary non polyposis colorectal cancer)=>70-75%
>FAP=> >95%

24.Infant bowl habits:


The normal frequency of passage of stools in the infant is around 6-8 times daily.This
approximates to one stool passage per episode of breast feeding.On the fourth week of
life , the pattern changes.The stool frequency decreases to one or two episodes daily or
even less, such as three episodes per week.
This has to be differentiated from true constipation in order to avoid exageratd concern.
Hirshsprungs is a cause of infant constipation that can also prznt insidiously,thus being
diagnosed later in life.Hirshsrpung disease also has failure to thrive.Untreatable and
prolonged constipation in a child may prompt the clinician to consider this diagnosis.

25.Gall stones:
Biliary colic is due to hormonal or neural stimuli that contract the gall bladder and force
the gallstones or sludge into the cystic duct.The increased pressure within the gall
bladder causes pain, which usually decreases as the gall bladder relaxes and the stone
moves back from the cystic duct.
Pts typically develop a constant epigastric or right upper quadrant dullness & discomfrt.
Pain can radiate to the right shoulder.Associated symptoms include nausea,vomiting &
diaphoresis.The entire episode usually lasts for <6 Hrs.Although fatty meals can trigger
the pain, many pts can develop nocturnal symptoms or pain independent of meals.Pts
usually improve spontaneously but can have multiple recurrent episodes before seeking
medical attention.Abdominal examination is usually benign w/o peritoneal signs or
guarding.
Transabdominal U/S is initially preferred for diagnosing gall stones.Gallstones would
appear as echogenic foci with shadow and sludge would appear as echogenic foci w/o
shadow.Pts with negative U/S and suspected biliary colic usually have a repeat U/S in a
few weeks.If the repeat U/S is negative, they likely need further testing ( e.g endoscopic
U/S ) to evaluate for missed gall stones or microlithiasis.
Rx:
=>Asymptomatic pts with confirmed gallstones do not require any specific treatmnt as
the progression to symptomatic disease is low.However,prophylactic cholecystectomy
is recommended for those with increased risk for gall bladder cancer ( e.g porcelain
gall bladder ) or recurrent gallstones ( e.g hemolytic anemia ).
=>Typical biliary colic symptoms and confirmed gallstones pts are given acute pain
management (e.g NSAIDS) and prophylactic elective cholecystectomy (usually
laproscopic ) to prevent recurrence.Pts who are poor surgical candidates or refuse
surgery can receive a trial of ursodeoxy cholic acid (UDCA).

=>Confirmed gallstones and atypical symptoms are given UDCA.Pts who respond to
UDCA undergo cholecystectomy.Non responders are evaluated for other causes of
abdominal pain.
=>Typical biliary colic symptoms w/o gallstones on imaging undergo Cholecystokinin
stimulated cholescintigraphy that evaluates for gall bladder ejection fraction, which
is low in pts with functional gall bladder disease.Pts with a positive test ( low gall
bladder ejection fraction) undergo cholecystectomy.Those with a negative test are
treated empirically for dyspepsia or irritable bowl syndrome.

26.Pancreatitis due to gall stones typically dilates the common bile duct.

27.Lithotripsy uses shock waves to fragment and remove difficult biliary and pancreatic
duct stones.However,its not as effective for gallstones within the gall bladder.

28.Normal diameter of common bile duct=> <0.6 cm

29.Choledocholithiasis:
>Presents with typical biliary symptoms (e.g RUQ pain,nausea,vomiting).Pts with
choledocholithiasis typically have longer duration of pain and can have elevated
serum liver function studies ( elevated bilirubin, alkaline phosphatase, & GGT out of
proportion to elevated transaminases )
>Pts with uncomplicated choledocholithiasis are usually afebrile,symptomatic and have
normal CBC, and pancreatic enzymes ( amylase,lipase ).
>Complicated choledocholithiasis with complete obstruction of CBD can cause acute
cholangitis (e.g fever,hypotension,mental status changes,leukocytosis).The stone can
also obstruct the pancreatic duct to cause acute biliary pancreatitis with elevated

serum amylase and lipase >3 times normal.


>Although lower sensitivity than cholelithiasis, but transabdominal U/S shd be done for
choledocholithiasis as well.
>Pts with stone visualized in CBD shd have ERCP done for removal of sotnes followed by
elective cholecystectomy.
>Pts in whom stone is not visualized but there is dilation of CBD on U/S shd also have
ERCP done to confirm the diagnosis and possibly provide therapy for a confirmed CBD
stone before undergoing elective cholecystectomy.

30.Perforated Peptic ulcer:


>Perforated peptic ulcer is the most common cause of peritonitis.Look for a Hx of Peptic
ulcer disease in the question stem.Peritonitis shd be suspected in pts experiencing
sudden onset abdominal pain with significant tenderness and guarding. Pts with
peritonitis tend to lie flat and motionless to limit peritoneum irritation.An upright
chest X ray showing pneumoperitoneum can identify the perforation.Its important to
keept the pt in an upright position so that air within the peritoneal cavity can be
visualized beneath the diaphragm.Pneumoperitoneum identification in a supine pt can
be much more difficult.
>With early diagnosis and treatment,the prognosis is excellent and most pts recover
fully.However, if left untreated the pt can have a rapid clinical deterioration leading to
death within 12-24 Hrs.Emergency surgery ( open or laproscopic ) is indicated in all pts
with perforated peptic ulcer.
>In preparation for expedited surgical intervention, pts with suspected or confirmed
peptic ulcer perforation shd receive Intravenous fluid resuscitation,broad spectrum
antibiotic therapy with good coverage for Gram -ve organisms and intravenous proton
pump inhibitor therapy.

31.Abdominal U/S=> gall bladder dx, ruptured AAA.

32.Bowel infarction=>raised Lactic acid.

33.Postoperative adhesions are the most common cause of small intestinal obstruction
pts with a history of abdominal surgery.

34.Ogilive syndrome (aka acute pseudocolonic obstruction) is characterized by the


dilation of the cecum and right colon in the absence of a mechanical obstruction to
the flow of intestinal contents.It tends to involve the right side of colon.

35.Blood transfusion:
>The threshold for blood transfusion in most stable pts with upper GI bleed is
Hb<7 g/dL as this is associated with fewer complications,fewer rebleeding episodes
and reduced mortality.(restrictive red blood cell transfusion ).
>Pts at risk for morbidity in the setting of severe anemia (e.g Unstable coronary disease)
can benefit from liberal transfusion ( transfusion for Hb <9 g/dL ).
(Note: Not just the fact that the pt has comorbidities like CHD, and he is a candidate
for transfusion at <9 g/dL, but he also needs to be severely symptomatic for him
to qualify for transfusion at <9 g/dL e.g ECG changes start appearing etc )
>Those with massive bleeding and hemodynamic instability might require blood transfsn
regardless of hemoglobin levels as the levels do not accurately reflect the amount of
blood loss.

36.He has life threatening bleeding and his platelets are <50,000 /mm3 ?
Transfuse him platelets.

37.Lactose intolerance:
Occurs when when there is insufficient amounts of lactase enzyme in the brush border
of the duodenum , thereby resulting in the inability to break down ingested lactose into
glucose and galactose.Levels of this enzyme decline naturally with aging and as much as
75-90% of the Asian,African and South American populations are lactose intolerant.
Symptoms can include nausea,abdominal pain and bloating,flatulence, and diarrhea,
which arise when the free lactose is fermented by colonic bacteria to form short chain
fatty acids and hydrogen gas.
The preferred means of diagnosis is the lactose breath hydrogen test , which requires
the patient to drink a lactose-containing beverage and then breathe into a bag at set
intervals over two to three hours.The exhaled air is tested with gas chromatography for
the presence of hydrogen.Typically, and increase in breath hydrogen concentration
>20 ppm is suggestive of lactose intolerance.In preparation for this test, pts shd fast for
eight hours, consuming no food or water.

38.Delayed Gastric Emptying:


>Delayed/impaired gastric emptying manifests as nausea,bloating and postprandial
fullness.A succussion splash is heard with stethoscope placed over left upper quadrant
and is suggestive of retained gastric material.
>Delayed gastric emptying can be due to mechanical obstruction ( intrinsic or extrinsic )
or delayed motility.
>The first step in evaluating impaired gastric emptying is to exclude a mechanical
obstruction.Upper GI endoscopy can diagnose an intrinsic obstruction (e.g peptic ulcer
stricture,gastric malignancy,bezoar/foreign body,gastric volvulus) & may show signs of
external compression as well. ( a radio contrast study e.g barium swallow can
also be considered but is less specific ).

>If endoscopy is non diagnostic but external malignant compression is still a concern,
abdominal imaging with CT scan or MRI can be performed.
>Once mechanical obstruction is ruled out,confirmatory tests for gastric motility
disorders can be performed.Gastric motility disorder can occur due to medication
side effects such as anticholinergic medication, autonomic neuropathies e.g diabetes,
,nerve injuries e.g post surgical or can be idiopathic.Scintigraphic gastric emptying
study can be performed for impaired motility.Gastroduodenal manometry can be
considered in pts who have evidence of delayed gastric emptying on a scintigraphic
study but lack a clear underlying disorder such as diabetes.

39.Diabetic Gastroperesis:
Dietary modification is generally the first step in management of pts with Diabetic
gastroparesis.Changing to smaller,more frequent meals may help to improve the
symptoms.Foods that are high in fat or fiber can slow gastric emptying and shd be
avoided.Feeding tubes or parenteral nutrition may be required in severe cases.Fluid
and electrolyte supplementation is often necessary due to repeated episodes of
vomiting.Improved blood sugar control may also help to improve gastric motility over
time.
Promotility medications may be helpful for some pts in whom dietary changes are not
effective or in those with acute worsening of gastroparesis symptoms.Commonly used
medications include erythromycin and metoclopramide.

40.Diverticulitis:
Abdominal CT scan is the imaging of choice for the diagnosis of diverticulitis.Possible
findings include colonic wall thickening, and stranding of mesenteric fat. CT is also ideal
for the visualization of diverticulae themselves as well as complications of diverticulitis

such as perforation,fistulas, and abscesses.This information will determine the need for
surgery or percutaneous CT-guided drainage.Pts with mild disease can be treated as
outpatients with a combination of ciprofloxacin and metronidazole.
Colonoscopy is contraindicated in acute diverticulitis.

41.Lactose intolerance:
Yogurt is a good alternative source of calcium for pts with lactose intolerance, as
studies have shown that the fermented milk and live cultures in yogurt contain betagalactosidase , which is well tolerated in these pts.

42.Milk and ice cream have high cocentrations of lactose.Complete restriction of these
and other lactose-containing products is necessary to confirm the diagnosis and rid
the pt of his symptoms.Once the pt is symptom free , the pt may be instructed to
gradually add lactose-containing products to his diet as long as he is able to tolerate
it.Ice cream has high concentrations of sugar and fat and can be used initially in small
quantities after a lactose-free interval.

43.Acute Mesenteric ischemia:


Acute onset of severe abdominal pain wherein the physical examination findings do not
match the severity of pain,along with metabolic acidosis, is genrally mesenteric ischemia
unless proven otherwise.This condition is frequently overlooked initially.All radiologic
studies may be initially normal.Laboratory data may show a mild to moderate metabolic
acidosis.Leukocytosis and elevated Hb and Hct % in the CBC may occur secondary to
hemoconcentration.The ischemia may be due to an occlusion secondary to thrombosis,
embolism and/or vasospasm.The most commonon cause is Superior mesenteric artery
(SMA) occlusion secondary to an embolism.The SMA is the most common site involved

becox it runs off at a minimal angle from the aorta and has a wide diameter.The origin
of the embolus is commonly from the left atrium, left ventricle or cardiac valves.Risk
factors are cardiac arrhythmias,recent MI and abdominal malignancy.The patient may
later develop abdominal distention and signs of acute peritonitis.If not treated promptly
,consequences may be very serious.Untreated mesenteric ischemia may cause bowl
infarction, sepsis and death.Mortality in this case may be as high as 60%.
Serum amylase,lactic acid dehydrogenase ( LDH ), creatine phosphokinase, or alkaline
phosphatase can be elevated in mesenteric ischemia.

44.Primary biliary cirrhosis: (PBC)


>Chronic , progressive autoimmune liver disease common in middle aged women of 3065 years of age
>Pathophysiology involves anti-mitochondrial antibodies formation that leads to
leads to cholestasis and destruction of small and mid-sized bile ducts.There is
progressive fibrosis and end stage liver disease can supervene 5 to 10 years after the
diagnosis.
>Signs & symptoms include raised alkaline phosphatase due to cholestasis,unexplained
pruritis ( can manifest as skin excoriation secondary to scratching ), progrssive jaundice
,steatorrhea,fatigue (an early and sometimes debilitating symptom),hyperlipidemia
with formation of xanthomas, bone disease (osteoporosis and/or osteomalacia),
hyperpigmentation of exposed areas, and autoimmune manifestations such as type-1
diabetes,keratoconjunctiivtis sicca,CREST syndrome,rheumatoid arthritis, & antithyroid
antibodies.
>Antimitochondiral antibodies have high sensitivity (>90%) and 98% specificity for PBC.
Diagnostic confirmation requires liver biopsy.This procedure can also give information
about disease stage and prognosis.

>Ursodeoxycholic acid can slow the progression of PBC,improve overall survival and
maybe transplantation-free survival.The only curative treatment is liver transplantatn.
Recurrence is uncommon after transplantation and when the disease does recur,it
tends to have slower progression, even though most pts remain positive for antimitochondrial antibody.A significant percentage of transplant recepients develops
histologic features of PBC.
>Steroids and immunosuppressive drugs are NOT useful, despite the disease apparently
autoimmune nature.
>Osteomalacia and osteoporosis are important complications of long-standing
cholestatic disease and of PBC in particular.Screening with bone densitometry,calcium
supplementation and eventual treatment with Vitamind-D and/or bisphosphonates are
essential in these pts follo-up.
>Paradoxically the incidence of atherosclerosis is low in pts with PBC, despite the high
incidence of hyperlipidemia.
>Cholestasis leads to malabsorption of fat soluble vitamins A,D,E and K.PBC pts should
receive regular supplementation of these vitamins.

45.Bile-salt induced diarrhea:


Postcholecystectomy diarrhea is a form of bile salt-induced diarrhea that occurs in
5-10% of pts following cholecystectomy.Bile salts are conjugated bile acids.Primary bile
acids produced in liver cells are secreted into the intestinal lumen, where they are
converted into secondary bile acids by bacteria.These secondary bile acids can cause
colonic stimulation if present in excess amounts.Gall bladder surgery alters gut dynamcs
and leads to increased bile acid flux to the colon, resulting in an increased proportion of
secondary bile acids and causing diarrhea.Bile-salt induced diarrhea is also seen wth ileal
resection or short bowl syndrome.Cholestyramine is a bile-salt binding resin that

sequesters bile salts and is often effective against this form of diarrhea.

46.The antibiotic rifaximin is used to treat small intestine bacterial overgrowth.This is a


malabsorption syndrome due to anatomic ( e.g surgically created blind loops ) or
motility (e.g diabetes, systemic sclerosis) disorders that typically present with
abdominal bloating,flatulence, and diarrhea.It has also been used to avoid
recurrences of hepatic encephalopathy and treat clostridiym difficile associated
diarrhea.

47.The somatostatin analogue Octreotide is useful in some forms of AIDS related


diarrhea and in diarrhea due to neuroendocrine tumors (e.g VIPomas)

48.Infantile Functional Gastroesophageal reflux (GER):


>Spitting is normal occurrence in infants up to 24 months old.It typically requires no
intervention if the child is otherwise healthy and developing appropriately ( the happy
spitter ).
>In children with mild GER symptoms,the mainstay of conservative treatment is
thickening of formula with cereal, which usually results in decreased emesis, decreased
cry and better weight gain.
>Prone positioning is another concervative therapy but avoid it as much as possible due
to risk of SIDS.
>Prescription medications ( Ant acids ) and Surgery is reserved for more severe cases.

49.Cisapride enhances myenteric plexus acetylcholine release, and is highly effective in


increasing GI motility. However becx it can cause cardiac arrhythmias in some
individuals, its availability is heavily restricted in USA.

50.Infantile hypertrophic pyloric stenosis: (IHPS)


>Presents in a male infant aged 3-6 weeks who develops postprandial projectile vomitng
There is usually no blood or bile in the vomitus and the child is immediately hungry aftr
vomiting.
>Physical examination reveals a palpable olive-shaped mass in the right upper quadrant
of the abdomen and perilstaltic waves are sometimes seen travelling from left to right
in the upper abdomen immediately before vomiting.
>Historically lab evaluation will show hypokalemia,and a hypochloremic,metabolic
alkalosis secondary to the loss of gastric hydrochloric acid , although electrolyte
imbalances are seen less often now that the diagnosis is made earlier.
>The imaging modality used most commonly to diagnose IHPS is ultrasound though an
upper GI contrast study is preferred in some medical centers.
>Electrolyte drangements and dehydration must be corrected before proceeding with
surgical correction of infantile hypertrophic pyloric stenosis.Studies have shown that
children who undergo surgery without first correcting the electrolyte imbalances are
are at increased risk for postoperative apnea.Once the child is well hydrated and
electrolyte imbalances corrected, Immediate pyloromyotomy shd be performed.
>Studies have shown an association between the development of infantile hypertrophic
pyloric stenosis and the usage of oral erythromycin which is typically given as PEP for
pertussis.In addition there is some indication that the usage of macrolides in breast
feeding women is linked to the development of infantile hypertrophic pyloric stenosis
, esp in infant girls.

51.Abdominal radiograph is helpful in diagnosing conditions such as dudodenal atresia,


or malrotation.

52.Colon cancer:
>Early development of colon cancer in a first generation relative is a very important risk
factor for evaluating a pt for colon cancer prevention.The risk is further increased if
more than one first generation relatives are affected or if colon cancer develops at an
age younger than 55.
>A diet rich in fiber,folate and calcium has a protective effect against colon cancer.

53.Partial small bowel obstruction:


>Present with s/s of obstruction but clue to the diagnosis of partial rather than
complete obstruction is on abdominal radiograph which shows some air distal to
the obstruction while proximal gut shows multiple air fluid levels consistent with
obstruction.
>Pts with partial small bowl obstruction shd be admitted and managed initially with
conservative therapy ( e.g Intravenous hydration, nasogastric suctioning and correction
of electrolyte abnormalities ) and observed for improvement.If pt fails to improve in
the next 12-24 Hrs, early surgical intervention is recommended.

54. A.DES=>High amplitude perilstatic contractions.Normal LES relaxation response.


B.Achalasia=>Absence of perilstalsis.High LES pressure.
C.Scleroderma=>Loss of perilstalsis in lower 2/3rd of esophagus.Low tone of LES.

55.Diffuse Esophageal Spasm:


Manifests with chest pain and dysphagia intermittently in young females.Association
with emotional factors and functional GI disorders.Manometry demonstrates high
amplitude perilstaltic contractions.In contrast to achalasia, the lower esophageal
sphincter has a normal relaxation response.Manometric findings may be intermittent,

thus making the diagnosis difficult.The esophagogram is frequently normal, although


the classic corkscrew esophagus is seen occasionally.Treatment is with anti spasmodics,
dietary modification and psychiatric counselling.Surgery is very rarely required for this
disorder.

56.Achalasia:
The LES does not relax.Histopathology reveals hypertrophied , inner circular muscle with
the absence or degradation of ganglia in Auerbachs plexus.Manometry will show the
absence of perilstalsis.The cause is not known but a similar condition in South America is
caused by the parasite, Trypanasoma cruzi.An esophagogram typically reveals a dilated
esophagus with a birds beak narrowing of the distal esophagus.Therapy is balloon dilatn
of the narrowed esophagus or surgery.

57.Scleroderma:
Is a collagen vascular disorder which can present with loss of distal perilstalsis of the
esophagus.There is complete atrophy of the esophageal smooth muscle and fibrosis.
The LES becomes incompetent ( low tone ) with time, leading to reflux esophagitis and
a stricture.The condition is progressive and difficult to treat.

58.Zenkers diverticulum:
Is a disorder of the proximal esophagus generally seen in females.The diverticulum may
vary in size and is generally asymptomatic in presentation.The occasional patient may
present with complaints of food sticking in the throat,halitosis and regurgitation.There
is no pain associated with the diverticulum.Treatment is surgery.

59.Wireless video endoscopy through the video capsule is an effective tool to diagnose
small bowel disease becox visualization of the majority of small bowel mucosa is not
possible with push endoscopy.Though the resolution is higher than the conventional
endoscopy but it provides limited view of the stomach and esophagus in which case
conventional endoscopy is preferred.The capsule video endoscopy is rarely associatd
wth any complications, though retention of capsule might occur in <1 % of the cases.

60.Failure to pass meconium:


>Full term infants normally pass meconium within 48 Hrs of birth.Failure to pass
meconium is usually pathologic and can lead to abdominal distension, poor feeding &
bilious emesis.Initial evaluation consists of Abdominal X rays to rule out free intraperitoneal air and to evaluate bowel gas pattern.The presence of multiple dilated bowl
loops and the absence of rectal air are concerning for distal bowel obstruction.Contrast
enema is performed to delineate the level of obstruction in the distal intestinal tract.
>Hirshsprung disease;
The classic finding is a transition zone between a normal or narrow caliber rectosigmoid (aganglionic segment) and a normally innervated but markedly dilated
descending colon.The absence of ganglion cells on rectal suction biopsy definitively
confirms the diagnosis.Treatment consists of surgical resection of the aganglionic
segment followed by anastomosis of normal bowel to the anus.Upto 25% of pts with
Hirshsprung disease have another congenital anomaly ( e.g renal ) and approximately
10% of pts with Hirshsprung disease have a chromosomal defect(e.g Down syndrme)
>Cystic fibrosis;
The tenacious secretions in CF can cause meconium ileus as the inspissated stool in
CF is difficult to propel, resulting in impaction in the ileum and a narrow,underused
microcolon.Sweat chloride test is the gold standard test for cystic fibrosis.

61.Pyloric stenosis->non bilious emesis w/o abdominal distension.

62.Anorectal manometry-> can be used in older infants or children as a screening test fr


hirshsprung dx as failure of internal sphincter relaxation
suggests HD.However its less accurate than rectal biopsy.

63.Upper GI series is the best test to diagnose malrotation with midgut volvulus in
children which can cause bilious emesis.In addition, abdominal X ray may show an NG
tube in a displaced duodenum.

64.Colonscopy is the initial test of choice in pts with iron deficiency anemia, and positive
fecal occult blood test.In pts with no obvious pathologic findings on colonoscopy,
upper GI endoscopy shd be performed.

65.Pancreatitis:
>Characterized by an acute onset of upper abdominal pain radiating to the back with
associated nausea and vomiting.
>No need to do imaging.Confirm the diagnosis by biochemical markers, Amylase and
Lipase.Amylase is non-specific and can be elevated in other conditions such as acute
parotitis,intestinal diseases,fallopian tube disease,cholecystitis,kidney disease.Lipase
is more sensitive and specific and is the diagnostic test of choice.
>In addition Liver function tests are also obtained as an elevated ALT and AST may point
to gallstones as the etiology.
>Pts can have significant amount of third space loss of fluids causing hypotension,acute
kidney injury and even worsening of the pancreatitis.Therefore all pts shd have early
and aggressive fluid resuscitation with close monitoring of urine output to ensure

adequate tissue perfusion.After the pt is started on IV opioids and IV fluids,further


diagnostic testing she be performed.
>Plain radiographs though not done but the characteristic findings are absence of bowel
gas in colon distal to splenic flexure and ileus.
>CT scan is not done unless the biochemical markers fail to show the diagnosis or if the
pt fails to improve or if the diagnosis of pancreatitis is in question due to atypical
features
>Pts with hx of biliary colic,biliary disease or elevations of Liver function tests should
undergo further evaluation for a potential gall stone source of their pancreatitis.For
that purpose, an Abdominal Ultrasound shd be done that can visualize the gallstones
as the etiology.
>All pts with pancreatitis shd have their triglycerides and serum calcium checked as thez
etiologies may be missed otherwise.
>IV antibiotics are only needed in pts with clinical or tissue evidence of infection of
necrotic pancreatic tissue or evidence of extrapancreatic infection.Antibiotics such as
Imipenem, metronidazole or fluoroquinolones are preferred for empiric and targeted
coverage.
>Surgical debridement is indicated only in pts with extensive tissue necrosis or localized
abscess formation.
>A pt who is hemodynamically stable but deteriorates several days later shd raise the
concern of a possible complication such as pancreatic necrosis, an acute peripancreatic
fluid collection or infection.The first step in identifying the underlying etiology for the
clinical change is a CT scan of the abdomen with contrast to help guide further
intervention.
>For those who cannot tolerate oral feeding,enteral nutrition using a jejunal feeding
tube placed beyond the ligament of treitz is recommended.

66.Pt has pancreatitis and they are not alcoholic ?


Do abdominal ultrasound for checking gallstones.

67.Which polyps are neoplastic ?


>Serrated
>Adenomatous ( tubular, tubulovillous,villous )
>Hamartomatous

68.Which polyps are non-neoplastic ?


>Hyperplastic
>Mucosal
>Submucosa
>Inflammatory

69.We have several small i.e < 1 cm rectal hyperplastic polyps ?


Ans:The colonoscopy wud remain at 10 years interval.The interval is decreased only if
there are neoplastic polyps found.

70.We have small i.e < 1 cm tubular adenomas, 1-2 in number ?


Ans: colonoscopy shd be done at 5 years intervals.

71.We have large i.e > 1 cm tubular adenomas ?


OR
We have 3-10 tubular adenomas ?
OR
We have adenomas with high grade dysplasia/villous features ?
Ans: The colonoscopy shd be done every 3 years

72. We have more than 10 adenomas ?


Ans: The colonscopy shd be at <3 years intervals.Such pts shd be evaluated for possible
underlying familial syndromes.

73.There was a large i.e >2 cm sessile polyp removed by piecemeal excision ?
Ans: Colonoscopy shd be done every 2-6 months

74.A polyp with adenocarcinoma that has minimal invasion and has >2 mm margin ?
Ans: Colonoscopy shd be done every 2-3 months.

75.Celiac disease:
>In a young patient with multinutrient malabsorption, the most likely explanation is
celiac disease.
>Usually associated with other autoimmune conditions like vitiligo, type-1 diabetes,
hypothyroidism.
>Although celiac disease is classically associated with prominent GI symptoms such as
diarrhea, a substantial subset of pts develops malabsorption symptoms such as iron
deficiency anemia and Vit-D deficiency even in the absence of diarrhea.Vit-D defency
wud lead to hypocalcemia and hence secondary hyperparathyroidism.

>Weight loss can occur in celiac disease.


>Screening for celiac disease is performed by checking the anti-endomysial antibody
and anti-tissue transglutaminase antibody levels.Simultaneously checking both
antibody levels increases the sensitivity for diagnosis.
>Gold standard for diagnosis is small intestinal biopsy.

76.Radiation proctitis:
Acute radiation proctitis may present with diarrhea,mucus discharge and tenesmus
(ineffectual/painful straining on defecation) during or within 6 weeks of pelvic radiation.
Chronic radiation proctitis is characterized by similar symptoms occurring >9 weeks to
years after radiation therapy and is more commonly associated with strictures,fistula
formation and rectal bleeding.
The diagnosis is typically made after excluding other causes of colitis ( e.g infection,
inflammatory bowel disease,ischemia,malignancy ) as symptoms can be nonspecific.
Colonoscopy may show continuous lesions with pallor,friability,telangiectasias, and
mucosal hemorrhage.Acute radiation proctitis is typically treated with supportive
measures ( e.g fluids,antidiarrheals ) ; chronic cases may require sucralfate or
glucocorticoid enemas.
BOTTOM LINE:
Radiation proctitis may present with bloody diarrhea,mucus discharge, &
tenesmus during or months after the completion of pelvic radiation thrpy.
The diagnosis is typically made after excluding other causes of colitis (e.g
infection,inflammatory bowel disease,ischemia,malignancy).

77.Pseudomembranes->raised yellow/off-white plaques scattered over the colonic


mucosa.

78.Endoscopic ultrasound (EUS) with aspiration is the best test to evaluate a pancreatic
cyst to differentiate malignancy from non-malignant causes.If the lesion is beign,
then serial CT scans in 3-6 months.If the lesion is malignant, then do surgery.Distal
pancreatectomy is dne fr lesions in tail and body whereas pancreaticoduodenectomy
is done for lesions in head of pancreas.

79.EUS is most effective for biopsying lymph nodes and lesions in the pancreas,liver,bile
duct,adrenal gland,peritoneal fluid and pleural fluid.

79.Hemolytic uremic syndrome is caused by shiga toxin producing E.Coli ( E.Coli


O157:H7)

80.Variceal bleed:
Pts who are admitted to the hospital becx of recent variceal bleeding are at increased
risk of complications during their hospitalization.The principal complications in these
pts that lead to increased mortality are: infections, hepatic encephalopathy and renal
failure.The most common complication is the development of an infection ,whch usually
occurs as a urinary tract infection,spontaneous bacterial peritonitis,respiratory infection
,aspiration pneumonia, or primary bacteremia.A large number of trials have evaluated
the efficacy of prophylactic antibiotics in cirrhotic pts that were hospitalized for variceal
bleeding.All these trials have suggested a decreased incidence of infectious complicatns
with the use of prophylactic antibiotics,The optimal choice of antibiotics & the duration
of therapy remain unclear.The currently preferred regimn is the use of fluoroquinoloes
(ofloxacin,norfloxacin,ciprofloxacin) agent for for 7-10 days.
The development of renal failure in these pts can have multiple etiologies such as acute
tubular necrosis (ischemic or toxic ) or precipitation of hepatorenal syndrome.

81.The primary treatment for all types of hernias is surgical repair.

82.A femoral hernia is much more predisposed to strangulation than an inguinal hernia.

83.Chronic Pancreatitis:
Abdominal pain accompanied by fat malabsorption shd raise the suspicion of chronic
pancreatitis , particularly in pts with a high history of alcohol intake. The pain tends to
be epigastric with radiation to the back.The pain may be either continuous or episodic
and may be worsened shortly after eating.A sudden change in the character of pain can
be caused by an acute flare sumperimposed on chronic pancreatitis.Glucose intolerance
is also frequently seen in pts with chronic pancreatitis.
CT abdomen is commonly used to assess for pancreatic calcifications, pancreatic
enlargement, ductal dilation and pseudocysts in order to confirm the diagnosis.
Cessation of alcohol intake and dietary modifications consisting of smaller meals that
are low in fat are first line treatment of chronic pancreatitis.
Panncreatic enzyme replacements and possibly opiate medications are the next
treatments if conservative measures are unsuccessful.
A variety of minimally invasive and surgical treatments are possible in pts with chronic
pancreatitis refractory to medical interventions.
(Celiac nerve block,endoscopic decompression of pancreatic duct and extracorporeal
shock wave lithotripsy to disrupt pancreatic duct calcifications are all minimally invasive
tools that have been used.Possible surgical options include decompression of the
pancreatic duct by anastomosing it to the jejunum , partial or complete pancreatic
resection and denervation of the afferent nerve fibers leaving the pancreas).

84.Lactose intolerance-> Flatulence and watery diarrhea.

85.Spontaneous bacterial peritonitis:


SBP is diagnosed when there are more than 250 neutrophils/mm3 in the peritoneal
fluid.Empiric antibiotic therapy must be started immediately while waiting for the
culture results, especially if there are other indicators of infection.Cultures are not
always positive in this condition.

86.An episode of SBP can precipitate hepatic encephalopathy. The presence of tremors
is suggestive of stage 2.Management involves administration of lactulose.Mannitol is
used with lactulose only in the more advanced stages of hepatic encephalopathy.

87.MELD scoring: (Model for end-stage liver disease )


>Its an objective measurement that uses serum bilirubin, INR and serum Creatinine
levels to determine 90-day mortality in pts with advanced liver disease.
MELD score < 15 => 95% survival rate
MELD score is 30 => 65% survival rate
MELD score is 40 =>10-15% survival rate
>For TIPS ( transjugular intrahepatic portosystemic shunt );
a.MELD score is <14 => Go for TIPS, it would have best outcome in such pts.
b.MELD score is >24 =>Avoid TIPS, dont do it in such pts.
>For liver transplant;
Higher MELD score => Better candidates for liver transplant.

88.Platelet levels are often low in pts with cirrhosis and portal Hypertension.

89.Long term PPI use:


>Increases the risk of osteoporosis and hip fracture.PPIs possibly decrease calcium
absorption,inhibit osteoclastic acitivity and eventually reduce bone mineral density.
Hip fracture risk increases and is higher in pts with other risk factors for hip fracture.
Some studies have shown an increased risk of spine and arm fractures as well in pts
with long term PPI use.
>Increased risk of enteric infections ( e.g clostridium difficile )
>Hypomagnesemia
>Interstitial nephritis
>Decreased absorption of Vit B12 and iron
>Decreased GI toxicity of Aspirin
>Decreased GI bleeding risk associated with anticoagulants (e.g warfarin)
>Chronic atrophic gastritis
>Decreased gastric acid secretion which aloows for increased bacterial colonization in
the upper GI tract and possible increased risk for pneumonia.

90.Pts with familial adenomatous polyps are especially at high risk and shd have annual
colonoscopy.

91.ALS is a neurodegenerative disease that progresses to severe muscular weakness,


dysarthria,dysphagia, and dyspnea.The dysphagia that develops is irreversible and
must be treated through percutaneous gastrostomy (PEG) tube placement.Although
placement of a PEG tube will not decrease the risk of aspiration when compared with
an NG tube, it is more comfortable for the patient.Other advantages of the use of a
PEG tube compared to the NGT are that it does not lead to the development of
sinusitis and it does not affect the patients breathing or speech.Temporary measures

(total parenteral nutrition and peripheral parenteral nutrition) are only used in the
emergency setting but shd not be part of the standard long-term management.

92.Button Battery ingestion in children:


>Button battery ingestion can lead not only to a mechanical damage of the GI tract due
to pressure effect but also to tissue damage due to the release of alkaline solutions,
heavy metals and local electrical discharge.Severe corrosive injury and liquefaction
necrosis may result.
>Button batteries lodged in the esophagus shd be removed immediately under direct
endoscopic visualization.
>Batteries that have made it to the stomach usually pass without complications and
outpatient management is indicated.

93.Pts with chronic hepatitis-C having HCV RNA positive, consistently elevated ALT and
atleast moderate inflammation on liver biopsy shd be treated with interferon &
ribavirin.
Pts with normal ALT level and/or mild inflammation without fibrosis on liver biopsy
can be observed closely.

94.Hepatitis-A vaccine shd be given to all unimmunized pts with underlying chronic liver
disease.

95.Cholangitis:
>Presence of fever, RUQ pain and Jaundice is known as Charcots triad and is suggestive
of ascending cholangitis.
>Addition of confusn&hypotension is known as Raynolds pentad and is associated with

suppurative cholangitis which has poorer prognosis.


>Look for ductal dilation of CBD on abdominal U/S, raised bilirubin and raised Alkaline
phosphatase and leukocytosis
>Associated nausea and vomiting can occur.
>Post-cholecystectomy biliary leak and biliary occlusion can occur predisposing to
cholangitis.
>Antibiotic therapy and fluid resuscitation shd be started.Many pts will be adequately
treated with conservative antibiotic management.However,pts with persistent
abdominal pain , hypotension despite aggressive fluid resuscitation, fever greater than
39 C or mental confusion shd be treated with biliary drainage.Biliary drainage can
usually be performed non surgically or percutaneous means.

96.Mallory Weiss tear:


The classic presentation of hematemesis preceded by a bout of recting/vomiting only
occurs in 30% of pts.Endoscopy is the gold standard in establishing the diagnosis.This
procedure typically reveals a single longitudinal tear at the gastro-esophageal junction.
In pts with Mallory-Weiss tear who are not actively bleeding,Observation and supportive
care are typically necessary.
Hiatal hernia is the most well known anatomical predisposing factor for Mallory Weiss
syndrome.According to different sources, its present in 40-100% of pts with this
syndrome.During retching or vomiting,the transmural pressure gradient is greater withn
the hernia than the rest of the stomach,thereby making this location the most likely to
sustain a tear.
Other precipitating factors include retching,vomiting,straining,hiccupping,coughing,
primal scream therapy,blunt abdominal trauma,cardiopulmonary resuscitation, and
diagnostic or therapeutic manipulation (e.g endoscopy).

97.Enteral feeding:
>Enteral feeding through a gastrostomy tube is preferred to parenteral feedings for pts
who have a functioning GI system.The standard composition of 30 Kcal/Kg/day and
1 g/kg/day of protein is satisfactory for most pts with adequate baseline nutrition.
>A lower calorie enteral feed could be considered for pts with severe malnutrition in
order to prevent refeeding syndrome.
>A low calorie with elevated protein could be considered for pts with malnutrition.

98.Chronic Mesenteric ischemia:


Found in pts with multiple atherosclerotic risk factors.Shd be considered in pts with
crampy epigastric pain that worsens with meals.The disease is sometimes referred to
as intestinal angina since it presents as abdominal pain when there is increased oxygen
demand in the bowel after eating , much as the pts with cardiac ischemia experience
chest pain with increased myocardial oxygen demands.The pain may becomes so severe
that pt avoids food and cosnsequently lose weight.Due to the large number of collateral
vessels supplying the bowel, multiple high grade stenosis are typically needed in the
mesenteric vasculature before the pts become symptomatic.
CT angiogram,MR angiogram and duplex ultrasound are all non invasive means of
making the diagnosis.Angiography remains the gold standard for making this diagnosis
and intervention with angioplasty and/or stenting can be done in the same setting.
Non invasive tests are helpful in localizing the obstruction and to identify the best route
it can be reached with angiography.

99.Lactose intolerance is frequently associated with borborygmi and abnormal bulky


stools.

100.Any sharp object in the esophagus (e.g fish bone, chicken bone,toothpicks, needles
or pins ) , an urgent flexible endoscopy shd be performed to remove the foreign
body and prevent the esophageal perforation.

101.Management of Hepatic encephalopathy includes the rapid identification & reversal


of the precipitating causes. The common causes of HE include:
>GI bleeding
>Hypokalemia
>Hypovolemia
>Hypoxia
>Sedatives or traquilizers
>Hypoglycemia
>Metabolic alkalosis
>Infection ( including spontaneous bacterial peritonitis )

102.Immediate potassium replacement must be done when HE is associated with


moderate or severe hypokalemia.Hypokalmia itself can HE; this situation is usually
associated with loop diuretics ( e.g furosemide ) use. Other situations in which
potassium must be replaced immediately are: ventilator failure and cardiac
arrhythmias ( Hypokalemia-related ECG changes ).

103.Bacterial enteritis in children:


>Salmonella is the most common cause of dysentery in United States.
>Shigella, E.Coli ( enterohemorrhagic or enteroinvasive strains ) , Yersinia and
Campylobacter are other common causes.
>These bacteria produce inflammation ( fever ) and enterocyte necrosis ( sloughing &

bloody diarrhea ) by directly invading enterocytes or by injecting toxin into cells (i.e
Shiga toxin).
>Identification and correction of hypovolemia are the most important aspects in pt
management.Rehydration can be accomplished with oral rehydration therapy or with
parenteral fluids in severe cases.
>Antibiotic therapy has not been shown to improve outcomes for bacterial enteritis and
may prolong symptoms or carriage of the organism ( i.e Salmonella ). In addition,
antibiotic therapy may predispose infants with E.Coli O157:H7 infection to Hemolytic
uremic syndrome development.Therefore antibiotic therapy is reserved for pts with
invasive dysentery (e.g sepsis ) or cholera ( which presents with voluminous nonbloody
stools).Antibiotic therapy is an option for pts with severe enteritis or those at highr risk
for invasive disease, such as young infants (<3 months) or immunocompromised hosts
BOTTOM LINE:
Bacterial enteritis is characterized by fever and bloody diarrhea.Therpy z
focused on rehydration.Antibiotics are only indicated for pts with severe
enteritis or invasive disease and should not be administered until Enterohemorrhagic E.Coli has been ruled out due to the risk of HUS.

104.Intussusception:
>Most cases occur between the age of 6 months-3 years.
>The classic clinical triad is intermittent,severe,crampy abdominal pain, a palpable
sausage-shaped mass on the right side of the abdomen and currant jellystools.Other
manifestations include drawing the legs up to the abdomen during episodes of pain &
vomiting.
>When the presentation is unclear, U/S is the imaging of choice which shows target sign
>When the diagnosis is obvious, then U/S is not required.Such pts can proceed directly

to treatment with diagnostic and therapeutic air(pneumatic) or water soluble


(hydrostatic contrast) enema.
>Resulting venous and lymphatic congestion in intussuception can cause significant
intestinal edema and may ultimately result is bowel ischemia and perforation.
>Intestinal peforation is also the primary risk of non-operative reduction of
intussusception with therapeutic enema,although it is rare overall (<1% incidence).
Children who have had symptoms for several days, those with evidence of small bowel
obstruction and those aged <6 months are at greatest risk for perforation.
>An air enema is generally preferred for reduction becx it tends to be fast, therby
minimizing radiation exposure.Additionally, perforations from air enemas tend to be
smaller than those occurring with water-soluble (hydrostatic) or barium enemas.
>Barium enemas are no longer recommended due to the risk of peritonitis with
perforation.

105.Diagnostic testing and treatment can proceed after informed consent has been
obtained from >1 parent who has legal custody of the child.

106.Dyspepsia:
>Defined as > 1 of the following symptoms: post-prandial fullness , epigastric pain or
burning and early satiety
>Pts with predominant heartburn,regurgitation and cough likely have GERD and can be
treated empirically with a proton pump inhibitor therapy.
>Pts taking NSAIDs or COX-2 inhibitors shd discontinue the medication and also be
treated empirically with an 8 week therapy of PPIs.
>Pts age > 55 shd undergo endoscopy.
>Pts with alarm features ( e.g unintentional weight loss, persistent vomiting or

hematemesis, unexplained iron deficiency anemia ) shd undergo endoscopy.


>In all other cases, consider and evaluate for H.pylori testing ( e.g stool antigen, urea
breath test ) and treat if positive.

107.Breat Milk Jaundice:


Is a common cause of unconjugated hyperbilirubinemia in exclusively breast-fed infants
that develops during first 3-5 days of life and peaks by the age of 2 weeks.The etiology is
thought to be due to high beta-glucoronidase activity in breast milk that deconjugtes
intestinal bilirubin and allows for increased intestinal absorption and enterohepatic
circulation of bilirubin.Neonates are otherwise well-appearing,asymptomatic and breast
feeding successfully.
Treatment of Breast milk jaundice includes frequent follow up and monitoring of the
infants hyperbilirubinemia.Exclusive breastfeeding shd be continued and encouraged
as the jaundice would resolve spontaneously by the age of 3 months.Formula is not
required.Cessation of breastfeeding can accelerate the decline in serum bilirubin levels
but has no clinical benefits as kernicterus from breast milk jaundice is very rare.Rather,
discontinuing breastfeeding prevents the infant from receiving the benefits of breast
milk,including maternal anitbodies , mother-infant bonding, and immunity.Interrupting
breastfeeding with bottles,esp during the first month, can decrease the mothers milk
supply and the newborns ability to latch and suck at the breast.

108.Breastfeeding failure jaundice:


Occurs during the first week of life and is caused by lactation failure form inadequate
milk supply, poor latch or infrequent feeding.This decrease in oral intake results in
decreased bilirubin elimination and increased intestinal absorption of bilirubin ,
thereby increasing serum bilirubin.The infant is usually dehydrated.

109.Biliary atresia=> Presents as jaundice in the first 2 months of life due to conjugated
hyperbilirubinemia ( direct bilirubin > 20% of total bilirubin ) from
progressive obstruction of the extrahepatic biliary tree.Ultrasound
shd be performed to evaluate for biliary atresia.

110.conjugated hyperbilirubinemia is always pathologic.

111.Toxic megacolon:
>Hx of severe colitis , particularly secondary to inflammatory bowel disease.
>Toxic signs/appearance.
>Distended typmanitic abdomen due to bowel dilatation.
>Multiple bloody bowel movements
>Fever and tachycardia
>Findings of peritonitis may be absent , so lack of rebound tenderness and guarding
does not exclude the diagnosis.
>Abdominal X ray shd be performed to evaluate for colonic dilation.Transverse colon
usually shows the most prominent dilation among the colonic segments in cases of
toxic megacolon, generally measuring atleast 6 cm in diameter.Multiple air fluid levels
are commonly seen along with loss of the normal haustral position and possibly
mucosal ulcerations.Pneumoperitoneum may be present if the toxic megacolon has
progressed to perforation.
>CT scan is best utilized for detection of complications.
>The first line treatment of toxic megacolon is medical management to lessen the
degree of colitis , with glucocorticoids used for pts with underlying inflammatory bowel
dx and appropriate antibiotics used for pts with infectious colitis ( steroids are then

contraindicated in infectious colitis ). Do NG suctioning and do fluid resuscitation and


correction of electrolytes and admit the pt to ICU for close monitoring.
>Avoid the medications that decrease the perilstalsis (e.g anticholinergics and opioids).
>Avoid 5-Amino salicylic acid ( 5-ASA ) during the acute attacks as it can precipitate the
further attacks.
>Surgical treatment is warranted in cases of non-responsive cases to avoid
perforation.Surgery is also done if perforation has already occurred.

112.Variceal bleeding:
>The ABC mnemonic (Airway, Breathing, Circulation) designates the order of
management in any patient who is unconscious , unresponsive or pts with altered
sensorium.Endotracheal intubation is indicated in unresponsive pts with a large
amount of hematemesis due to aspiration risk,followed by assessment of breathing
& circulation.Upper endoscopy shd be performed early on since it may be both
diagnostic and therapeutic.
>Cirrhotic pts who have upper GI bleed frequently have a coexisting bacterial infectn
or develop one after admission , so prophylactic antibiotics preferably prior to
endoscopy is recommended for these pts.
>In acute variceal bleeding,an endoscopic intrvention is the most effective treatmnt.
In cases of rebleeding,repeat Upper GI endoscopy with additional treatment using
either sclerotherapy or banding shd be performed.If this second round of endoscpc
treatment fails, then portal shunting with either a surgical shunt or TIPS may be
required.
>IV beta blockers are only given prophylactically and not during the acute episode to
avoid systemic hypotension
>TIPS is usually preferable to Surgical shunting becx of increased mortality wth surgry

in pts with chronic liver dysfunction and other comorbid conditions.


>Prophylactic non-selective beta blockers such as propranolol and nadolol can be givn
prophylactically to prevent the rebleeding episodes.If we give it before there is any
bleeding episode, then it would be primary prevention and if we give it after a bleed
episode to prevent further episode then it would be secondary prevention.The
combination of beta blockers with repeat endoscopic surveillance and band ligation
as necessary, results in an even lower mortality than giving beta blockers alone. Oral
nitrate medications may also be used as part of the medication regiment for variceal
bleeding prevention.

113.Spontaneous bacterial peritonitis:


Spontaneous bacterial peritonitis can present with non specific symptoms of fever,
abdominal pain or discomfort or altered mental status in pts with ascites. One shd
have a low threshold for obtaining a paracentesis and excluding the diagnosis of
SBP in cirrhotic pts with ascites.
PMN count > 250 cells/mm3 with positive gram stain or culture would confirm the
diagnosis.
Most cases are caused by translocation of enteric bacteria such as E.Coli into the
ascites fluid.However, streptococcal and rarely staphylococcal bacteria can also
cause the infection and and thus a broad spectrum empirc thrpy with an antibiotic
such as third generation cephalosporin ( cefotaxime ) is often inititated.

114.Splenic vein thrombosis:


The presence of isolated gastric varices in a pt with a history of chronic, recurrent
pancreatitis is suggestive of splenic vein thrombosis, which is one of the less frequent
complications of chronic pancreatitis.The splenic vein runs along the posterior surface of

the pancreas and can get directly inflamed and thrombosed due to recurrent pancreatic
inflammation.Apart from gastric varices, pts with chronic splenic vein thrombosis may
develop non cirrhotic portal hypertension , ascites and massive splenomegaly with
associated features of hypersplenism ( anemia, thrombocytopenia and leukopenia ).

115.Small Intestinal Bacterial Overgrowth:


Small intestinal bacterial overgrowth ( SIBO ) is due to an increased number of native &
non-native intestinal bacteria that alter the normal flora & cause excessive fermentation
,inflammation and malabsorption.The proximal small intestine normaly contain relativly
minimal bacterial colonization due to gastric acidity and perilstalsis.Other protective
mechanisms against SIBO include bacterial degradation by proteolytic enzymes, trappng
bacteria by the intestinal mucus layer and an intact ileocecal valve preventing retrogrde
bacterial movement from the colon.
Causes of SIBO include anatomical changes (e.g surgical anastomosis with blind loops,
strictures,fistulas), motility disorders (e.g diabetes, systemic sclerosis,radiation enteritis)
and others (e.g acid suppression, immunodeficiency states, chronic pancreatitis,cirhosis)
Pts typically develop bloating, increased flatulence, abdominal discomfort and diarrhea.
Lab studies are usually normal in most pts but some may have nutritional defeciencies
(e.g calcium, Vit B12 and fat soluble vitamins).Sevre cases may devlop significnt diarrhea
,malabsorption and weight loss.
The gold standard for diagnosis is endoscopy with jejunal aspirate shwing >10^5 org/mL
(normal is <10^4 org/mL).Other tests such as hydrogen breath test using lactulose ,
showing an early peak in hydrogen levels due to rapid lactulose metabolism in the small
bowel ( normally broken down in the colon ). Treatment includes dietary changes (high
fat,low carbohydrates) , avoiding drugs with anti-motility effects , trial of promotility
drugs and antibiotics ( e.g rifaximin , amoxicillin-clavulanate ).

116.Vit B12 defeciency may occur in Crohn disease due to terminal ileal involvement.

117.Acute mesenteric ischemia:


is seen in pts who have evidence of atherosclerotic dx and is usually manifested by
acute abdominal pain followed by bloody diarrhea.The most vulnerable areas are
watershed areas , which include splenic flexure and rectosigmoid junction.Pts usually
have an elevated white cell count.X rays and sigmoidoscopies usually show mucosal
edema and mucosal ulcerations.

118.Acute pancreatitis usually does not usually cause bloody diarrhea.

119.Colonoscopy:
>General population:
=>Start screening at age 50
=>Do colonoscopy every 10 years, OR flexible sigmoidoscopy every 5 years OR fecal
occult blood testing (FOBT) or fecal immunochemical test (FIT) every 1 year.
=>Some experts recommend dual contrast barium enema or CT colongraphy every
5 years.
>Pts who have ONE first degree relative diagnosed with colon cancer or adenomatous
polyps at age >60 are at slightly increased risk and would follow the same guidelines
as for general population.
>Who are at high risk ?
=>Those with one first degree relative diagnosed with colon cancer or advanced
adenomatous polyps (size >1cm, villous features, or high grade dysplasia)
before age 60.
=>Those with >2 first degree relatives diagnosed with colorectal cancer or advanced

adenomatous polyps at any age.


>What to do with high risk group ?
Start their colonscopies either 10 years before the age at which the relative was
diagnosed OR start it at age 40 ( whichever comes first )
MNEMONIC=> <60 , 2 relatives

120.A sticking sensation in the throat (dysphagia) accompanied by heartburn is


characteristic for scleroderma.

121.Alcoholic liver disease:


>A disproportionate elevation of AST levels is the most typical biochemical pattern in pts
with alcoholic liver disease.
>AST/ALT ratio > 2 is quite specific. The relatively lower elevations of ALT is attributed to
hepatic deficiency of pyridoxal-6-phosphate in alcoholic liver disease which is a co-factr
for the enzymatic activity of ALT.
>AST level is usually <300 and almost always <500 U/L.
>GGT is elevated.=> Sensitivity is ~70%
>Macrocytosis and thrombocytopenia are seen in moderate to severe alcoholic liver dx
and not in mild dx.

122.Barrets esophagus:
>Actually a metaplasia of normal stratified squamous epithelium in distal esophagus to
columnar epithelium
>Seen on the endoscopy as reddish and velvety like ( salmon-colored )
>Can precede the development of adenocarcinoma
>If you see barrets esophagus on endoscopy, take it biopsy specimen.
=> No dysplasia on pathology specimen;
Endoscopic surveillance every 3-5 years.
=>Low grade dysplasia on pathology specimen;
Surveillance every 6-12 months
OR
endoscopic eradication (i.e ablation/mucosal resection)
=>High grade dysplasia on pathology specimen;
Endoscopic eradication ( ablation/mucosal resection)

123.Slow transit chronic constipation:


>The first thing to do is to exclude secondary causes (e.g hypothyroidism) and check for
medications causing it (e.g Tricyclic anti-depressants, certain anti-hypertensives).Also
consider doing colonoscopy to exclude malignancy and other structural diseases.
I No?
>Counsel the pt i.e pt education and ask them to increase the dietary fiber and water.
I No?
>Consider starting bulk forming laxatives (e.g psyllium,methylcellulose,polycarbophil)
I No?
>Consider either of the following;
=>Saline laxatives ;

=> they can cause hypermagnesemia in CKD,

Milk of magnesia,magnesium citrate

=>Stimulant laxatives;
Bisacodyl, Senna , Castor oil

so avoided in CKD pts.

=>they can cause hypokalemia in CKD with


long term use, so avoid its long term use
in such pts.

=>Osmotic laxatives;

=>they can cause hypokalemia in CKD with

Polyethylene glycol,

long term use, so avoid its long term use

Lactulose,sorbitol

in such pts.

I No?
>Consider either of the following;
=>Suppositories (e.glycerin)
=>Chloride channel activator (e.g lubiprostone)
I No?
>Consider surgery ( colectomy/hemicolectomy).

124.Medications causing constipation:


>Anticholinergic drugs;
=>Antihistamines
=>Tricyclic antidepressants
=>Antipsychotics
=>Muscle relaxants
>Non-dihydropyridine calcium channel blockers;
Verapamil.

125.Docusate sodium is a surfactant stool softener that increases stool hydration and is
most effective when there is need to avoid straining (e.g postoperative pts)

126.Pts who wear dentures all the time are prone to develop oral lesions.The most
appropriate initial step for such pts is to simple advise them to remove their
dentures for a couple of weeks.

127.Stress ulcers:
>When do you need stress ulcer prophylaxis ?
Recommended for Intensive care unit (ICU) pts with coagulopathy,Hx of GI bleed in
the last year or has been on mechanical ventilation for >48 Hrs.
OR
2 of the following ; Sepsis, ICU admission > 1 week, occult GI bleed > 6 days, and
glucocorticoid therapy.
>What is the recommended regime of Stress ulcer prophylaxis;
PPIs will do the job. H2 blockers can be considered as reasonable alternatives. but if
needed both can be given intravenous. Remember that they would increase the pH

of Gastrointestinal tract as well that increases the risk of bacterial growth in stomach
and hence increases the risk of aspiration pneumonia.
>What is the mechanism of stress ulcers ?
Critically ill pts may have uremic toxins and reflux of bile salts into the stomach.Both
can disrupt the protective glycoprotein layer, predisposing to ulcer formation.Head
trauma causes increased gastrin secretion , leading to to parietal cell stimulation &
acid secretion.Most of these ulcerations occur within the first 72 Hrs and tend to be
located in the duodenum.

128.Fecal incontinence in elderly:


Fecal impaction is the most common cause of fecal incontinence in elderly pts.Fecal
impaction is generally managed by using enemas followed by suppositories to ensure
complete emptying of the bowels.Once complete emptying has been achieved,the pt
is instructed to increase his fiber and fluid intake.

129.Acute Mesenteric ischemia:


Acute mesenteric ischemia is typically due to sudden occlusive or nonocclusive
obstruction of intestinal blood flow ( arterial or venous ). Occlusive ischemia is most
commonly due to embolic or thrombotic involvement of the superior mesenteric system
(artery or vein),segmental intestinal strangulation or volvulus.Emboli typically are due to
a dislodged cardiac thrombus (e.g underlying atrial fibrillation) that occludz the superior
mesenteric artery.Superior mesenteric venous occlusion is typically due to conditions
such as hypercoagulable states , portal HTN and abdominal infections. Non occlusive
ischemia is usually due to hypoperfusion (e.g low cardiac output) leading to splanchnic
hypoperfusion and vasoconstriction.
Pts usually develop sudden onset of periumbilical abdominal pain with nausea & vomtng

However, the initial abdominal examination is usually normal w/o peritoneal signs (e.g
rebound tenderness,guarding).Pain is often out of proportion to the exam findings.
Progression of small bowel ischemia to infarction leads to a grossly distended abdomen,
abdsent bowel sounds and peritoneal signs.As a result, diagnosis requires a high index
of clinical suspicion, especially in elderly pts with risk factors (e.g atrial fibrillation,
peripheral vascular disease).Marked leukocytosis, hemoconcentration and elevated
lactate resulting in metabolic acidosis are common. CT of the abdomen shows signs of
small bowel ischemia (e,g focal or segmental bowel wall thickening, intestinal
pneumatosis with portal vein gas, dilated bowel, mesenteric stranding ) and is helpful
in excluding other causes of abdominal pain. However, the diagnosis is best established
with CT angiogram. Treatment involves fluid resuscitation , correction of metabolic
acidosis , broad spectrum antibiotics and NG suctioning for decompression. Surgical
consultation is required.

130.Acute colonic ischemia is usually due to transient reduction in blood flow to the
colon due to hypovolemic states or transient ischemia to the bowel.It affects primarily
the water shed areas (e.g splenic flexure,rectosigmoid). Pts usually have a more
lateralized abdominal pain followed by bloody diarrhea.Pain is mild to moderate and
not severe as colonic ischemia is usually not due to obstructed blood flow to the colon.

131.Acute intermittent porphyria usually presents with recurrent episodes of abdominal


pain worsened by factors including medications (e.g phenytoin) , menstruation ,systemic
illness, and surgery. other findings can include peripheral neuropathy, tachycardia and
hyponatremia.Porphyria commonly occurs earlier in life, usually after puberty.

132.Familial adenomatous polyposis:


FAP is an autosomal dominant syndrome caused by inherited mutation in Adematous
polyposis coli (APC) gene.The disease is characterized by the growth of more than 100
colonic adenomatous polyps.Some pts with FAP develop other benign extraintestinal
manifestations (e.g soft tissue tumors of the skin, desmoid tumors, osteomas, hepatoblastomas). The polyps appear at a mean age of 15 years and will inevitably progress to
colorectal cancer by age of 50 unless a prophylactic colectomy or proctocolectomy is
performed.

133.Celiac dx, Intestinal lymphoma:


It is important to remember that the pts with celiac sprue are at an increased risk of
developing intestinal T-cell lymphomas.The jejunum is most commonly affected and
the tumors are usually nodular or ulcerative.Some pts may present with perforative
peritonitis.The most common presenting symptoms are abdominal pain,weight loss
and diarrhea.Malabsorption results in anemia and poor nutritional status.The diagnosis
of intestinal lymphoma must be suspected in any pt with celiac dx who presents with
GI symptoms despite adherence to a gluten-free diet.Treatment is with surgery and
chemotherapy but the relapse rates are high and the prognosis is such cases is poor.

134.Inguinal Hernias:
>Direct inguinal hernia occurs due to the muscular weakness of abdominal muscles.It
is usually seen in the elderly.
>Indirect inguinal hernia occurs due to the failure of obliteration of processus vaginalis.
More common in the pediatric age group.The hernia usually bulges into the scrotum.
These shd be surgically repaired as early as possible.These will not resolve with age.The
risk of potential complications , including incarceration, is particularly high if it remains

unrepaired during the first month of life.

135.Prognosis indicator of pancreatitis:


The APACHE-II score is the system of choice to predict the severity of acute pancreatitis
and guide management decisions given its good predictive values and ability to be
calculated both at admission and on a daily basis. It has replaced the RANSON criteria in
predicting the prognosis and triaging the pancreatitis pts becx RANSON score has 11
variables out of which 6 wont appear until 48 Hrs after admission.
APACHE-II score >8 means CT scan shd be done at 72 Hrs to evaluate the degree of
pancreatic necrosis.

136.GERD:
>Mild symptoms can be managed with simple lifestyle and dietary modifications ,
antacids, and non prescription H2 blockers.
>A trial of proton pump inhibitors shd be given for atleast 8 weeks to those who fail the
conservative management.
>Endoscopy shd be done early if the pt complains of dysphagia,odynophagia,significant
wt loss and GI bleeding.

137.Appendicitis in pregnancy:
>McBurneys sign ( tenderness at McBurneys point,the site two-thirds b/w umbilicus
and right anterior superior iliac spine ) , Rosvings sign ( transmission of pain from the
left to the right lower quadrant ) and ilipsoas sign ( RLQ pain on passive extension of
the right hip ) are all positive.
There is a left shift in WBC counts in 95% of cases i.e increased bands would be seen.
>One third of the women in the first trimester having acute appendicitis may experience

abortion.
>Around 14% of those in the second trimester can have premature delivery
>During the third trimester, the main complication is appendix perforation with
peritonitis and subsequent pylephlebitis ( infectious thrombosis of portal veins ).

138.Aspiration:
>Some of the conditions that can predispose a pt to aspiration are cerebrovascular
accidents, seizures , alcoholism, general anesthesia, drug overdose, protracted vomtng,
multiple sclerosis, Pakinson disease , endotracheal intubation and upper GI endoscopy.
>The upright supine position is thought to be the most protective and is used most
frequently to prevent the risk of aspiration in pts with impaired consciousness or
dysphagia due to neurological disorders.
>Recumbent position predisposes pts to a high risk of aspiration and she be avoided , if
possible.

139.Dyspepsia:
Pts who come from high prevalence areas ( Asia, Eastern Europe, Mexico, Latin & South
America ) shd have H.Pylori testing and treatment with appropriate antibiotics if +ve.

140.Rectal prolapse:
>Due to a mucosal or full-thickness layer of rectal tissue sliding through the anal orifice.
>Risk factors for rectal prolapse include prior pelvic surgery, pelvic floor dysfunction or
anatomincal defects, conditions causing increased intra-abdominal pressure (e.g
chronic straining with constipation , benign prostatic hypertrophy ) or women > 40
with prior vaginal deliveries.
>Pts usually develop difficulty with defecation, diarrhea/fecal incontinence, discomfort

(not significant pain) in the anal area , rectal bleeding and a protruding rectal mass.
Symptoms may occur intermittently.Significant rectal pain suggests a different
undelying disorder.
>Diagnosis is made clinically by physical examination showing a mass extending through
the anus with concentric rings of rectal mucosa.
>Medical management is usually sufficient for pts w/o a full thickness rectal prolapse.
Uncomplicated prolapsed rectal mucosa can be reduced by gentle digital pressure.
Additonal therapies include adequate fluid and fiber intake, pelvic floor exercises and
possible biofeedback for fecal incontinence.
>Surgery ( intra-abdominal or perineal approach ) is preffered for pts who hve complete
rectal prolapse or prolapse with fecal incontinence and/or constipation.
>Untreated rectal prolapse can lead to strangulation and gangrene of the rectal mucosa.
The prognosis is generally good with prompt and appropriate care.

141.Gallstones:
>Asymptomatic pts require no further intervention.
>The most effective therapy for symptomatic gall stones is laproscopic cholecystectomy.
>Pts with mild symptoms and small cholesterol stones may respond to ursodeoxycholic
acid.The drug slowly dissolves gallstones over one to three years but provides
symptomatic relief approximately three months after the start of therapy.It is effective
in 50% of pts who must have a functional gall bladder.However it causes diarrhea in
some of the patients.
>Extracorporeal lithotripsy may be used for those with bigger cholesterol pts.
>Hig risk pts especially those with non-cholesterol stones may be treated with
endoscopic electrohydraulic therapy.

142.Acute Cholecystitis:
>Usually due to gall stones obstructing the cystic duct.
>10% of cases are due to acalculous cholecystitis ( no gallstones ) which usually occurs
in elderly and critically ill pts and is associated with higher morbidity and mortality.
>Acute cholecystitis typically presents with fever, leukocytosis and steady upper
upper abdominal or right upper quadrant pain that can radiate to the back or right
shoulder.Pts can be ill appearing with significant right upper quadrant tenderness,
voluntary or involuntary guarding and positive Murphys sign ( palpation of gall bladder
fossa with deep inspiration causing increased pain and catching of the breath).
>Abdominal ultrasound is initially preferred to document gall stones, gall bladder wall
thickening or edema and a sonographic Murphys sign ( Murphys sign with
radiographic palpation of the gall bladder ).
>HIDA scan is usually recommended for pts with unclear ultrasound findings i.e above
findings described are not seen fully/paritally.A radioactive tracer is injected IV and
taken up by hepatocytes which release the tracer into the bile.A HIDA scan is considerd
positive If the gall baldder does not visualize , usually from cystic duct obstruction due
to a gallstone or acute cholecystitis causing gall bladder edema.

143.H.Pylori:
>70% of duodenal ulcers and 50% of gastic ulcers.
>Rx;
=>Normal pts;
PPI-Amoxicillin-Clarithromycin-> 2 weeks.
=>Penicillin allergic pts;
PPI-Metronidazole-Clarithromycin->2 weeks.
*Keep your eyes open for any drug allergies or contraindications.

=>Persistent infection after two weeks of triple therapy; ( indicated by persistent


+ve urea breath test/stool antigen test after 4-6 weeks of treatment completion);
PPI-Metronidazole-Tetracycline-bismuth->2 weeks of therapy with meals.
145.Pts with gastroparesis in diabetes often have labile diabetic control and frequent
hypoglycemia as its difficult to time their insulin dose to correspond with the
delayed intestinal absorption of glucose.

144.The typical symptoms of diabetic gastroparesis include early satiety & postprandial
fullness.Its often accompanied by other autonomic symptoms and labile glucose
control.Mechanical obstruction shd be excluded.A nuclear gastric emptying study is
the procedure of choice to confirm the diagnosis.

145.Physiologic Jaundice in neonates:


>Unconjugated.
>Elevated unconjugated hyperbilirubinemia in neonates can be caused by increased
bilirubin production due to a relatively high hematocrit and short RBC lifespan ,
decreased clearance secondary to reduced levels of UGT enzyme and increased
enterohepatic circulation.

146.Neonatal hyperbilirubinemia:
>Although unconjugated bilirubin would increase with neonate age , but bilirubin
levels greater than 18 mg/dL is always considered high regardless of age.First
line therapy for hyperbilirubinemia is phototherapy which helps to convert
bilirubin into a water-soluble form that can be excreted more easily.
>Exchange transfusion is considered when phototherapy is ineffective or bilirubin
is already at toxic levels e.g infants at bilirubin >25 mg/dL are at risk of neurologic

dysfunction.
>IVIGs are used when jaundice in neonates is caused by hemolysis and jaundice due
to hemolysis usually presents in firs 24 Hrs of life.
>Phenobarbital can increase the rate of bilirubin excretion but not typically used due
to side effects.

147.Chronic diarrhea:
>First thing to do is to take a comprehensive history.
>Second thing to do is to do microscopic examination of the stool for leucocytes,ova,
parasites, occult blood and special staining for fat,pH and electrolytes for calculating
the osmotic gap.

148.Celiac dx:
>Diagnosed with small intestinal biosy of distal dueodenum and the characteristic
histologic findings are loss of the normal villus architecture , mucosal flattening (villus
blunting) and lymphocytic infiltration.
>Management;
=>Avoid gluten containing products ( i.e wheat, rye and barley )
=>Soya bean, rice and potatoes are safe
=>If pts develop dietary defeciencies of iron, folate, calcium and other vitamins, then
replenish that.
>Most pts will show clinical improvement ~2-3 weeks afte the initiation of gluten-free
diet, however, normalization of the villus architecture can take a few months.The
diagnosis of celiac disease is in question in pts who fail to show clinical or histologic
improvement after being on a gluten-free diet for 4-6 months.
>Oral steroids not routinely used but in cases refractory to gluten free diet.

149.Whipple disease treated with antibiotics

150.Malnourishment treatment:
>The intial treatment of pts with severe malnutrition shd address temperature control
(warming), possible infection, dehydration and malnutrition ( feeding ).
>Dehydration shd be treated with oral rehydration in malnourished pts whenever
possible.Intravenous hydration shd be used only if there is severe dehydration resultng
in shock becx it can result in heart failure and edema in malnourished pts due to alterd
cardiac function and low albumin levels.
If pts have oral ulcer, vomiting or unconscious, then insertion of Dobhoff tube for
rehydration and feeding may be needed.Placement of the enteric tube beyond the
pylorus reduces the risk of aspiration.
>Systemic infection is commonly present in these pts so obtaining blood cultures and
starting empiric antibiotics would also be reasonable.
>Feeding shd be started cautiously to prevent refeeding syndrome.Feeding solutions
shd not initially contain excessive calories and protein becx the pts GI reserves (i.e
Gastric acid production,Intestinal motility and pancreatic anzymes) have decreased in
response to malnutrition.
>Early iron supplementation is not recommended becx it can lead to GI ulceration.Iron
supplementation shd begin later in the rehabilitation phase.

160.The MELD scoring system is preferred to the Child-Turcotte-Pugh scoring system


for determining prognosis in pts with chronic liver disease.The components of the
MELD score include bilirubin , INR and Creatinine.

161.Meckels diverticulum:
>Young children usually present with painless bleed due to irritation from ectopic
gastric tissue while adults present with obstruction.
>Classically presents as painless hematochezia in a child <2
>Technetium-99 nuclear scan ( Meckels scan ) is highly specific.
>Can serve as a lead point for intussusception.
>Symptomatic Meckels diverticulum is usually treated with surgical resection.
>Intestinal lymphagiectasia or other vascular malformations can also cause painless
bleed but Meckel scan is negative.

162.Upper GI endoscopy is the preferred method to evaluate upper GI bleeding becx it


also has therapeutic applications.The most common reason for bleeding in a pt with
a Hx of previous gastric lesion is the development of a new ulcerated lesion or the
recurrence of the previous one.These lesions are readily diagnosed and managed
with endoscopy.

163.LIGHTS CRITERIA:
A.Exudative effusion;
>Pleural fluid protein/Serum protein >0.5
>Pleural fluid LDH/Serum LDH >0.6
>Pleural fluid LDH >2/3rd upper limit of normal serum LDH.
=>Autoimmune disease
=>Esophageal rupture
=>Infection(parapneumonic , TB , fungal, empyema )
=>Malignancy

=>Post-CABG

=>Pancreatitis

=>PE

B.Transudative effusion;
>Pleural fluid protein/serum protein <0.5
>Plerual fluid LDH/serum LDH <0.6
>Pleural fluid LDH <2/3rd of upper limit of normal serum LDH.
=>Hypoalbuminemia (cirrhosis, nephrotic syndrome)
=>Congestive heart failure
=>Constrictive pericarditis

164.Hepatic hydrothorax:
>Usually presents as a right sided transudative pleural effusion in pts with cirrhosis and
ascites.
>The primary treatment is sodium restriction and diuretics.
>TIPS is second line therapy.
>Pts shd be considered for liver transplantation.

165.Management of adenocarcinoma arising in a polyp is dependant upon its pathologic


features.Most cases of adenocarcinoma require surgical resection.However, if the
invasive adenocarcinoma is in the head of the polyp, the margins are uninvolved
(>2mm), the lesion is well-differentiated, and there is no lymphovascular invasion,
the pt can usually undergo nonoperative management.Pts shd be advised of Rx
recommendations after careful consultation on a case-by-case basis.In pts who are
managed nonoperatively, follow up colonoscopy shd be performed in 2-3 months
to exclude residual or recurrent disease.This short interval follow-up can also assess
for a synchronous lesion that may have been missed.The endoscopist can tattoo the
area of the polyp at the time of initial biopsy so that the area can be found easily in
the future.Subsequent follow-ups shd be obtained 1,4 and 9 years following the

initial polyp resection.

166.Confirmation of H.Pylori eradication is recommended for pts with ulcers or ongoing


dyspepsia.Either Urea breath test or fecal antigen testing can be used after 4 weeks
to confirm H.Pylori eradication.H.Pylori serology shd not be usd since it may remain
positive for a year or more after eradication.
*Fecal antigen test is more readily available but may be less accurate than the
urea breath test.
*Intake of antibiotics or bismuth can result in false negative results for both the
urea breath test and fecal antigen test.
*Eradication shd also be confirmed in pts with persistent dyspepsia, MALT
lymphoma or who have had resection of early gastric cancer.

167.Duodenal ulcers symptoms occur more frequently at night.

168.Gastric ulcers are more commonly assocated with cancer, so surveillance


endoscopy generally indicated.

169.Constipation in children:
>Although it can occur with some serious underlying conditions ( Hirshsprung dx, CF,
hypothyroidism), but most of the children are normal.
>Occurs due to resistant toilet training,increased intake of dairy products and diet low in
fiber.
>First step is dietary modification i.e inc water, inc fiber and inc and inc vegetables.
>If dietary modification fails then laxatives shd be used until the constipation is relieved.
>For moderate constipation with no encopresis, use magnesium hydroxide/milk of

magnesia, a saline laxative , that causes the osmotic fluid retention in the gut lumen.It
can be titrated to produce soft,but non-liquid stool and is safe for use in young
children.
>Treatment with enemas and suppositories are reserved for severe constipation and
fecal impaction.

170.Castor oil is a lubricant that coats the bowel and stool with oil to allow for easier
passage through the colon and rectum, however, lubricants also inhibit the
absorption of fat-soluble vitamins in the GIT and have been linked with oil-related
aspiration pneumonia in individuals prone to GE reflux or vomitng.Castor oil is knwn
to have harsh stimulant effect, therefore its use is not recommended in children.

171.All pts with adenomatous polyps on screening sigmoidoscopy shd have a complete
colonoscopy for detection of synchronous adenomas.

172.Isolated elevation of anti-HBc:


>There are only 3 situations in which isolated anti-HBc may be seen;
=>During the window period of acute Hepatitis-B virus infection,when HBsAg has falln
but anti-HBs has not risen yet.Pts may be symptomatic with a history of recent
exposure but about 70% have subclinical hepatitis.IgM anti-HBc is typically positive
and liver enzymes are elevated.
=>Years after recovery from acute HBV infection once anti-HBs has waned off.Liver
enzymes are normal and IgM anti-HBc is usually negative.
=>After many years of chronic HBV infection when HBsAg has fallen to an undetectble
level.Pts may have evidence of chronic liver disease and HBV DNA is detectable.
IgM anti-HBc is typically negative.

>Steps to follow afte you see an isolated elevated Anti-HBc;


Repeat the HBV serology ( e.g anti-HBc, HBsAg, anti-HBs, anti-HBe ) to make sure its
not a false-positive result.
I
If results are unchanged, measure IgM anti-HBc ( checks window period ) and
liver function tests ( elevated LFTs means active infection ).
I
If liver enzymes are abnormal or if there are signs of chronic liver disease , then
HBV DNA shd be checked to evaluate for chronic HBV infection.

173.GERD:
>The chest pain of GERD is described as an uncomfortable squeezing or burning
sensation in the retrosternal chest that radiates towards the back, neck , jaw or
arms.The pain may resolve spontaneously of after consumption of antacids and
usually occurs postprandially , can awaken pts from sleep and worsens with
emotional stress.
>Hypersalivation is very unusual to occur and causes the pts to foam at the mouth
from secreting as much as 10 mL of saliva per minute.
>Peptic stricture is a well known complication of GERD that results from the healing
process of ulcerative esophagitis.It causes obstructive dysphagia.Dysphagia usually
starts with solids, followed by liquids.Pts typically report difficulty following solid
food , prolonged and careful chewing and swallowing small portions.

174.Achalasia typically causes dysmotility type dysphagia that is characterized by a


difficulty in swallowing both solids and liquids.

175.Pancreatitis:
While CT is not generally used to confirm a diagnosis of acute pancreatitis, it may be
helpful in determining its severity or assessing for complications. Pts with pancreatic
necrosis over 30% may benefit from prophylactic antibiotic therapy with imipenem or
meropenem.

176.Dyspepsia:
>Dyspepsia is defined as epigastric fullness or pain w/o significant heartburn.Pts with
heartburn shd be classified as having GERD and treated accordingly.
>A test and treat strategy is recommended for pts <55 years old from an endemic area
where prevalence of H.pylori infection is >10% ( Asia, Eastern Europe, Mexico, Latin &
South America ).
>Current guidelines do not recommend dietary modifications for dyspepsia as they are
less effective.

177.Porcelain gall bladder:


Is a condition characterized by calcium salt deposits in the wall of a chronically inflammd
gall bladder. The calcifications can be thin or fairly visible or may be amorphous , patchy
and thick.The gall bladder is generally large , but its size can vary considerably. Most
porcelain gall bladders are associated with gall stones.A plain radiograph generally
detects these but CT scan has higher specificity ; therefore CT scan is performed to
confirm the diagnosis.
Due to their high risk of gall bladder carcinoma, all pts with porcelain gall bladder shd
have elective cholecystectomy.

178.A GI source of bleeding must be sought whenever there is fecal occult blood.It is
essential to rule out colorectal cancer.Colonoscopy is indicated in pts with positive
fecal occult blood tests.Other colorectal cancer screening methods are doublecontrast barium enema and sigmoidoscopy.Stool tests for oncogene-DNA (e.g
damaged APC or p53 genes) are being studied.

179.Acute cholangitis is characterized by Charcots triad; fever, jaundice and RUQ pain.
Pts shd be admitted to the hospital.Immediate management includes blood cultures
followed by antibiotics(ampicillin+ gentamicin OR monotherapy with imipenem or
levofloxacin) , hydration and close monitoring of vital signs.If the patient clinically
improves , an elective ERCP can be scheduled. If conservative therapy fails,an urgent
bilary decompression (emergent ERCP for biliary drainage) is required. A pt in such a
conditn can be critical.Hypotension and confusion may develop due to sepsis.When
these two signs occur with Charcots triad , the pt is said to have Raynolds pentad,
which has a mortality of 50%.

Hematology & oncology


1. Normocytic anemia-> do a retic count to know the major
pathophysiologic mechanism
A.hypo proliferative states: decreased retic count
1.renal dx
2.hypothyroidism
3.aplastic anemia
B.increased retic count: hemolytic anemias

2. Brain metastasis
A.Multiple metastasis:
a.Whole brain radiation therapy is mainstay treatment.
b.Corticosteroids are also recommended
B.Solitary metastasis:
Stereotactic surgery might be indicated
3.Iron deficiency anemia:
A.Breast milk provides enough iron for an infant in first 6 months of
Age .After 6 months of age, supplemental foods(e.g pureed meats)
Are necessary to maintain adequate iron stores and prevent iron
Deficiency Anemia.Foods rich in vit-C should be given to infants
Starting at age 6 months.Formula or cereals given to infants should
Always be fortified with iron.Breast milk predominant diet after
Age 6 months will result in Iron deficiency.
B.Iron deficiency anemia in infants presents as lethargy, irritability,
Pallor,cardiomegaly,systolic murmur, and tachypnea if its severe.
C.Classic labs in iron deficiency anemia are HB<11 g/DL, decreased
MCV and decreased total red cell count.

D.Most children wit iron deficiency anemia are asymptomatic and are
Identified through Universal screening at age 9 to 12 months.
E.Toddlers consumption of greater than 24 ounces of milk daily or
Less than 3 servings per day of iron rich foods (e.g meat,fortified
Cereals) would result in iron deficiency anemia.
F.Premature babies tend to have iron deficiency anemia.
G.Consumption of cow milk,soy milk or goat milk before 1 year of age
Would result in iron deficiency anemia
H.Do give empirical trial of oral iron in iron deficiency anemia.
4.MCV/RBC=Mentzer index:
A. >13= Iron deficiency anemia
B. <13= Thalasemia
5. Chronic GI bleeding in cow milk fed infants.
6.Iron deficiency anemia pts have height and wt retardation on percentile
chart
7.Ferrous sulfate is the most effective treatment for iron deficiency anemia
And is given with orange juices in between meals as vit-C enhance iron
Absorption.
8.Pts with iron deficiency anemia have decreased reticulocytes ,as the bone
marrow is unable to produce the requisite number of RBCs w/o adequate
iron substrate.Within 1 to 2 weeks of initiating supplementation , the
reticulocytes count increases quickly. However, in pts with moderate to

severe iron deficiency, it usually takes 3 to 4 weeks for the HB and HCT to
increase and a few months to return to regular levels. Iron supplementation
shd be maintained for 2 to 3 months after HB Normalization to Replenish
iron stores.
9.Iron deficiency anemia-> anisocytosis ( different sized RBCs) -> inc RDW
10.LHRH agonists (leuprolide) + Anti-Androgen ( Flutamide) :
a.Best initial therapy for Metastatic Prostate cancer.
b.Give Flutamide one week before Leuprolide to avoid initial
Testosterone surge.
11.Pts with advanced symptomatic hormone resistant prostate cancer can
Be treated with palliative chemotherapy
12.Diethylstilboesterol -> estrogen Analog >blocks HPA axis
13.DES banned in USA due to effects of MI,Stroke,PE
14.:
A.Sickle cell trait: HB A : HB S = 60: 40
B.Sickle cell beta (0) thalasemia: No HB A at all
C.Sickle cell beta(+) thalassemia:
a.Type I : 3-5 HB A
b.Type II: 8-14 HB A
c.Type III: 18-25 HB A

15.Sideroblastic anemia> Hepatosplenomegaly


16.ESR is elevated in anemia of chronic disease
17.No anemia and microcytosis in sickle cell trait
18.children with hypochromic microcytic anemia are usually treated
with iron because iron deficiency anemia is the most common cause of
this type of anemia.More sophisticated studies are indicated only if
the response to iron is inadequate or if the medical history is
complicated.
19.Three dietary factors were found to be associated with anemia in
children:
A.low intake of meats,vegetables, fruits and grains
B.High intake of milk
C.Daily intake of fatty snacks ,sweets or soft drinks.
20.Risk factors for colon cancer:
a. Family history
b. Familial adenomatous polyposis
c. Inflammatory bowel disease
d. African american race
e. Alcohol intake
f.Alcohol intake
g. Cigarette smoking
h.Obesity

21.Protection against colon cancer:


a.Hormone replacement therapy
b.NSAIDS use
c.High fiber diet rich in fruits and vegetables
d.regular exercise
22.Several studies have suggested that even moderate alcohol intake ( 2-3
drinks per day) is associated with increased colon cancer risk , though the
highest risk was seen in heavy drinkers (equal/greater than 4 drinks a day).
Alcohol likely interferes with folate absorption and heavy drinkers may have
decreased folate intake also
23.Cigarette smoking is also associated with increased colon cancer risk but
the risk is primarily seen in current long term smokers (eg equal/greater
than 30 years )
24.Digital systolic pressure in response to cooling is occasionally used to
confirm the diagnosis of Raynaud phenomenon.
25.Dactylitis ( painful swelling of hands and feet ) might be the first
manifestation of sickle cell disease.Do HB electrophoresis to confirm the
diagnosis.
In older children and adults these episodes primarily occur in joints,back and
chest.
26.Worse prognosis in breast cancer:
a.Her 2/ Neu over expression
b.Tumor aneuploidy
c.High S fraction

d.Accumulation of p 53 protein
e.increased cathepsin D ( lysosomal proteolytic enzyme )
27.Over expression of HER 2 / Neu ( an oncogene that encodes a
transmembrane receptor belonging to the epidermal growth factor receptor
family ) is traditionally associated with a worse prognosis in breast cancer.
There is some clinical evidence that Her 2 / Neu over expression may be
associated with relative resistance to alkylating agents and probably
endocrine therapy as well.
28. Trastuzumab / herceptin is a a monoclonal antibody directed against
Her2/Neu.these patients usually demonstrate good therapeutic response to
these antibodies and improved sensitivity to chemotherapy drugs.
29.Thalasemia major:
a.severe microcytic / hypo chromic anemia
b.extra medullary hematopoiesis resulting in hepatomegaly and
ssplenomegaly.
c.Hyper transfusion regimen has a great role in treated pts ( I.e the child
may survive several years after diagnosis).Hyper transfusion regimen can
suppress the effects of chronic severe anemia and extra medullary
hematopoiesis but results in significant iron overload and resultant organ
damage.
d.Interestingly iron metabolism is significantly affected even w/o
transfusion therapy in these pts.Iron overload is caused by erythroid
hyperplasia,RBC destruction and ineffective hematopoiesis.
30.Severe anemia > high output congestive heart failure

31.Agnogenic myeloid metaplasia ;


A. <60 => Allogenic hematopoietic stem cell transplantation
B. >60 => palliative therapy
32.Pts with untreatable diseases (e.g myeloid metaplasia) ,a poor quality of
life and those who have already tried different therapies in the past may
sometimes refuse to accept any kind of treatment. The principle of
autonomy includes a pt's right to refuse treatment in such situations. The
physician must respect his pt's decision and not look for extra ordinary
therapeutic measures ,unless the pt's evaluation of his / her case is not
realistic. Psychiatrist can be consulted if there is any suspicion of psychosis
or depression.
33.Major problem that leads to difficulties finding cross matched blood in
pts with a hx of multiple transfusions is allo antibodies ( e.g in pts with sickle
cell DX or myelodysplasia).The most commonly implicated RBC antigens in
that case are E,L and K. Moreover these pts tend to develop multiple allo
antibodies that make finding compatible blood even more difficult.
34.HLA allo sensitization increases the risk of graft rejection in pts awaiting
organ or bone marrow transplantation and platelet refractoriness in those
requiring subsequent platelet transfusion support.Remember RBCs do not
express HLA antigens.
34.Acute severe anemia in sickle cell disease can occur and presents as
weakness, pallor and lethargy.Three major causes:
A.Increased reticulocytes:
a.hyperhemolytic crisis: Associated with G6PD deficiency.
b.Splenic sequestration crisis: Associated splenomegaly

B.Decreased reticulocytes:
Aplastic crisis: Transient failure of erythropoesis with abrupt reduction
In blood HB and number of erythroid cell precursors in
Bone marrow. There is virtual absence of reticulocytes.
35Life threatening hemorrhage in pts on warfarin therapy shd be treated
with fresh frozen plasma infusion.
36.Almost all pts with serum ferritin of less than 15 ng/ ml are iron
defecient.However the sensitivity using this cutoff is only modest at best
and it cannot be used to exclude the diagnosis of iron deficiency anemia
since the majority of pts with a level of 15-30 ng/mL are also iron ddeficient.
37.GIT is the major source of blood loss leading to iron deficiency
anemia.Therefore a fecal occult blood test is a reasonable initial choice in
iron deficiency anemia in males. A series of three fecal occult blood tests
helps to maximize the sensitivity of the test,although endoscopy would
likely be needed even if negative. Colonoscopy is typically performed before
EGD in these pts unless there is a particular reason to suspect an upper GI
source of bleed such as NSAIDS use.
38.Pts who develop DVT as a result of reversible or time limited risk factors
(e.g surgery, pregnancy, OCPs use,trauma ) warfarin anticoagulation shd be
considered for a minimum of 3 months.Treatment for however longer than
6 months is not necessary.
39.In pts with idiopathic DVTs ,at least 6 months of Warfarin therapy is also
recommended, with reevaluation at the end of treatment cycle for long

term anticoagulation. This decision is individualized however and should be


based on the risk to benefit to ratio for anticoagulation.
40.Mechanical valves are more durable and last longer than Bio prosthetic
valves.However mechanical valves confer a significant long term risk for
thromboembolism and require life long anticoagulation (warfarin) and anti
platelet therapy (Aspirin).
41. Warfarin is typically prescribed for 3 months following a bio prosthetic
mitral valve replacement therapy. Aspirin is continued indefinitely.
42.
A.Low risk mom
a. Pre conception: Warfarin.
b. 1st trimester : LMWH
c. 2nd trimester: LMWH
d.3rd trimester: LMWH
e.Just few weeks before delivery: UFH
B.High risk mom
a.1st trimester: LMWH
b.2nd trimester: Warfarin
c.3rd trimester: Warfarin
d.Just few weeks before delivery: UFH
43.LMWH: Easier to administer and achieves therapeutic levels than UFH
44.UFH: Fast acting and fastest reversal ( Protamine)

45. Warfarin can be used safely in breast feeding moms.


46.LMWH is cleared renaly while UFH is cleared hepatically.With the
exception of pts with severe renal insufficiency , prolonged LMWH use ( e.g
during first trimester ) is preferred over UFH as UFH prolonged use is
associated with increased bone density loss , bleeding and
thrombocytopenia.
47. UFH is the anticoagulant of choice preceding delivery due to itd rapid
reversibility.
48.No anti coagulation in
a.Labor
b.Prior to epidural anesthesia due to risk of epidural hematoma
c.During delivery due to risk of significant intra partum hemorrhage
49.Warfarin causes fetal nasal and limb hypoplasia at 6 to 12 weeks due to
effect on cartilage and bone respectively.
50.Warfarin can cause fetal bleeding at anytime during pregnancy.
51.Aspirin is generally safe in pregnancy but shd be discontinued
temporarily at the time of delivery due to increased bleeding risk.

10

52.SAAG:
A. >/= 1.1 > portal HTN
a. CHF
b.Budd chiairi syndrome
c.Constrictive pericarditis
d.Cirrhosis
e.Alcoholic Hepatitis
f. SBP
B.<1.1
a.Infections ( except SBP )
b.Cancer
c.Nephritic syndrome
d.Peritoneal tuberculosis
e.Pancreatitis
f.Serositis
53.Most common cause of as cites in USA is cirrhosis
54.
a.Anti platelet antibodies > HIT
b.Antibodies against phospholipids > Anti phoshpholipid syndrome
c.Massive release of tissue factor > DIC

11

55. HIT
A.Type-I HIT:
Occurs due to a non immune direct effect of heparin on platelet
activation and usually presents within the first 2days of Heparin
exposure.The platelet count then normalizes with continued heparin
therapy and there are no clinical consequence.
B.Type-II HIT:
Its a more serious immune mediated disorder due to antibodies to
platelet factor 4 complexed with heparin. This leads to platelet
aggregation ,thrombocytopenia and thrombosis ( both arterial and
venuous ).Platelet count usually drops greater/equal than 50 percent from
baseline with a nadir of 30,000-60,000 / uL. Type 2 HIT usually presents after
5 days of heparin exposure and may cause life threatening
consequences(stroke, limb ischemia ).
56.Thrombocytopenia and prolonged PTT in anti phospholipid syndrome.
57.All forms of heparin (including low molecular weight such as enoxaparin)
must be discontinued in pts with HIT while awaiting diagnostic confirmation.
Pts with HIT remain at high risk of thrombosis even after discontinuation of
heparin.Therefore ,an alternate, rapidly acting non heparin anticoagulant
such as direct thrombin inhibitor ( e.g Argatroban,Bivalirudin) or
fondaparinux( synthetic polysaccharide) must be started immediately.
Clinical observation is recommended only in high risk bleeding pts.
58.Diagnostic confirmation tests for HIT:
a.ELISA for Platelet factor 4 antibodies
b.Serotonin release assay
c.Heparin induced platelet aggregation assay.

12

59.The bet way to prevent HIT is to use low molecular wt heparin instead of
unfractionated heparin whenever possible. However once pts are diagnosed
with HIT , they must avoid all forms of heparin ( including low molecular
weight heparin such as enoxaparin ) for life.This includes Heparin flushes for
arterial lines and heparin coated catheters.Information on the Heparin
allergy must be conveyed in the pt's medical record.
60.
a.Trimethoprim
b.Methotrexate

> Folic acid deficiency anemia

c.Phenytoin
61.Methotrexate is a dihydrofolate reductase inhibitor.Folinic acid
( leucovorin ) and not folic acid, is therefore the drug of choice for rescuing
RBCs from the folic acid deficiency induced by methotrexate use , by
bypassing the block on dihydrofolate reducatase inhibitor (DHFR).
62.Metoclopramide is a central and peripheral D-2 receptor blocker.It is
used to treat (and prevent ) chemotherapy induced emesis;however its use
has been associated with extrapyramidal symptoms such as akinesia,
akathesia , dystonia , and parkinsonian symptoms. Metoclopramide has now
been largely replaced by 5HT3 receptor antagonists (e.g Ondansetron) and
aprepitant.
63.Anemia in ESRD:
A.Men : < 12g/dL
B.Women : <11 g/dL

13

64.Initial evaluation tests for anemia in ESRD:


a.B12/Folate
b.FOBT
c.Iron studies
d.Retic count
65.Anemia evaluation results in ESRD:
a.Iron deficiency: Iron supplementation
b.Other deficiencies: Treat accordingly
c.None: EPO Deficiency actually > ESA rx
66.Causes of Anemia in ESRD:
a.Iron deficiency ( frequent blood testing,GI blood loss,dialysis it self )
b.severe Hyper parathyroidism (induces EPO resistance)
c.Folate deficiency
d.systemic inflammation
e.Aluminum toxicity
67.Pts with ESRD can have functional iron iron deficiency ( normal iron
stores with inability to mobilize the stores in response to
erythropoeitin),defined as transferrin saturation of <20 percent with ferritin
of 100-800 ng/ml or higher.
68.Absolute iron deficiency is defined as transferrin Saturation of <20
percent or ferritin <100 ng/ml.However, the underlying inflammation
associated with ESRD and dialysis can significantly increase serum ferritin
and make it a less accurate measure of iron deficiency.Other markers of iron

14

stores (e.g serum iron,transferrin saturation,total iron binding capacity) can


help clarify iron status.
69.Because erythropoeitin stimulating agents (ESAs) increase Iron
demand ,these pts shd have iron studies prior to starting ESAs.Testing shd
continue periodically ,especially in those without appropriate response to
ESAs.ESAs such as erythropoeitin are recommended for ESRD pts with
HB<10 g/DL.The goal is to increase HB by 1.5 to 2 g/Dl over 4 to 6 weeks to
target HB to 10-11.5 g/DL.
70.Iron supplementation is recommended for ESRD pts with transferrin
saturation less than/equal to Greater than 30 percent and ferritin less than /
equal to 500 ng/ml.These pts can have higher iron requirements than they
can absorb from the GIT and may not respond to oral iron.As a result,
Intravenous iron is suggested for pts on hemodialysis or peritoneal dialysis.
( oral iron is preferred for non dialysis pts due to cost and convenience ) .
71.Pts with ferritin >500 ng/ml (generally reflecing good iron stores ) usually
respond to increased ESA dose but may require intravenous iron if there is
no response.
72.Transfusion associated circulatory overload causes
dyspnea,hypoxemia,HTN and tachycardia due to rapid transfusion of blood
products in individuals with depressed cadiac funtion .Pts develop clinical
hypervolemia with increased central venous pressures ,pulmonary
edema,and elevated N-terminal pro hormone BNP levels.Treatment includes
oxygen, non invasive positive pressure ventilation and diuretics.
73.Pts who are transfused with bacteria contaminated blood have fever and
chills followed by fulminant septic shock , and DIC.

15

74.Most bacteria with exception of some gram negative species ( e.g


Pseudomonas) do not grow at the cold temperatures at which blood is
stored.
75.Blood transfusion may be continued if its confirmed that the pt had only
an uriticarial reaction to the transfusion (Hives).
76.Pre medication with anti histamines and acetaminophen does not
prevent blood transfusion reactions
77.Acute hemolytic transfusion reaction is a medical emergency caused by
ABO incompatibility (possibly due to clerical error) or presence of other
antibodies.Pts who require chronic transfusions (e.g African American pts
with sickle cell anemia) may form multiple antibodies to common Rh,kell
and other blood group antigens
78.Acute hemolytic reaction occurs within an hour of tranfusion.Pts with
acute hemolytic transfusion reactions develop fever,chills , flank pains
hemoglobinuria and discomfort at the infusion site.This may advance to
renal failure ( due to immune complex deposition) and DIC.Diagnosis is
made by direct Coomb'test and pink plasma(plasma free HB>25
g/DL),hemoglobinuria, and repeat type and crossmatch showing a mid
match.
Rx: Managed by immediately stopping the transfusion and aggressively
hydrating the pts with normal saline ( Not ringer or dextrose) to treat
hypotension and prevent renal failure.
79.IgA deficient pts develop antibodies against IgA.Residual plasma in red
cell concentrates contain proteins including IgA.These antibodies, in

16

recipient, can react with IgA containing donor products and induce an
anaphylactic reaction.To reduce this risk ,red cells shd be washed to remove
as much of plasma as possible for pts with IgA deficiency or prior allergic
transfusion reaction.
80.Febrile non hemolytic transfusin reaction Occurs within 1-6Hrs of
transfusion.when red cells and plasma are separated from whole blood
small amounts of residual plasma and/or leucoycte debris may remain in red
cell concentrate.During blood storage,these leucocytes release
cytokines ,which when transfused can cause transient fever,chills and
malaise without hemolysis.
Management includes stopping the transfusion to exclude other serious
reactions ,admisintering anti pyretics and using the leucoreduced blood
products for future transfusion reactions. Leucorediction involves reducing
the number of transfused leukocytes through filtering or other methods
such as saline washing, feezing,deglycerolizing,or buffy coat removal. It also
reduces HLA alloimunization and CMV transmission.
81.TRALI:
Occurs within 6 Hrs of transfusion. Caused by donor anti leukocyte
antibodies.There is respiratory distress and signs of non- cardiogenic
pulmonary edema.
82.Delayed Hemolytic reaction:
Occurs within 2-10 days of transfusion. Caused by anamnestic antibody
response. There is mild fever and hemolytic anemia.There is positive direct
coomb's test and positive new antibody screen.

17

83.
a.secs-mins > Anaphylaxis
b.0-1 Hrs > Acute hemolytic reaction
c.1-6 Hrs > Febrile non hemolytic reaction
d.2-10 days > Delayed Hemolytic reaction.
84.Molar pregnancy (partial and complete hydatidiform mole) is most
common type of gestational trophoblastic disease and is usually benign.
85.Nearly 20 % of pts who have had a molar pregnancy develop persistent
gestational trophoblastic disease and are at risk for malignant
transformation.
86.Ultrasound is useful for initial diagnosis and management of GTD.and for
reassessing recurrent GTD.Its not needed following evacuation.
87.All pts shd be followed with serial Beta HCG values until there are 3
consecutive normal values to confirm resolution. A plateau or rise in HCG
suggests persistent disease which requires further intervention.
88.Initial evaluation of molar pregnancy is with three things
A.Ultrasound
B.Baseline HCG
C.thyroid function
|
Suction evacuation
|

18

6-12 months:
A . effective contraception
B.weekly HCG levels
|
A.HCG declines to normal>3 consecutive HCGs>Stop
B.plateau /rise in HCG>consider for chemotherapy,+/-surgery,+/-radio
88.Sickle cell disease:
a.Autosomal recessive
b.abnormally folded Hb chains
c.increased LDL
d.more common in African American and hsipanics
e.Vaso occlusive pain crisis more precipitated by
hypoxia,dehydration,infection,fever,cold temperatures,
menstruation.Presents as severe pain in chest,abdomen,back and thighs.
A.1st line RX: Analgesia
>Outpatient: NSAIDS and Acetaminophen
>Inpatient: IV opiates (e.g MORPHINE).NSAIDS are adjunct.
B.2nd line RX: Hydration
>1/4 to Normal saline only because sickle cell pts have decrease
Ability to excrete sodium,so dont give them full strength normal
Saline.
f.penicillin until age 5 years age to sickle cell pts.

19

89.Acute chest syndrome:


Most common cause of death and 2nd most common cause of
hospitalization In pts with sickle cell dx!.In children infection,asthma
exacerbation,and pulmonary infarction are most common causes .In
adults ,bone marrow or fat emboli are typical etiologies. Nearly half of all
cases occur in pts who have already been hospitalized for other reasons.
Diagnosis requires presence of a new pulmonary infiltrate on CXR and >/=
one of the following clinical findings, including fever >38.5 C( 101.3 F) ,
hypoxaemia,chest pain , or respiratory distress.
B/c infection is most common cause in children, empiric antibiotics shd be
started promptly. The recommended agents are a 3rd generation
cephalosporin (e.g ceftrixone or cefotaxime) to cover strep.pneumonia plus
a macrolide ( e.g azithromycin) to cover mycoplasma pneumonia.
Pain control is also critical to prevent splinting and subsequent
hypoventilation and atelectasis.
IV fluids shd be administered to prevent dehydration and further sickling.
Bronchodilators (e.g albuterol) shd be used in Acute chest syndrome pts if
they are wheezing even if they dont have prior asthma.
Pulmonary embolism is a complication of acute chest syndrome more
common in adults.
90. Tension pneumothorax
a.dyspnea , hypotension and absent breath sounds on affected side
b.tracheal deviation away from affected side.
c.Needle thoracostomy above the 2nd or 3rd rib in mid clavicular line is
indicated for decompression
91.Neonatal polycythemia is diagnosed when the peripheral venous
hematocrit is higher an 65 percent.High hematocrit levels that are obtained

20

from capillary samples ( usually from heels) are only screening values that
need to be confirmed through venous blood sampling because peripheral
venous hematocrit is usually 5-15% lower than hematocrit taken from a
capillary sample.
Hematocrit reaches its maximum value when the infant is approximately
two hours of age.For this reason it is important to recheck the hct 12-24 Hrs
after delivery if the hct is borderline high.The sample must be venous(not
capillary sample).
Capillary sample>venous sample at delivery time> confirmation with venous
Sample 12-24 Hrs after
Delivery
92.Neonatal polycythemia presents with irritability, drowsiness,poor
feeding,abdominal distension, and hypotonia.peripheral cyanosis may
occur.infants usually appear plethoric and lethargic.Hypoglycemia, jaundice
and apnea are common presentations of neonatal polycythemia.These signs
and symptoms are due to blood hyper viscosity ,which decreases blood flow
to tissues including brain and gut.
Rx:
> IV hydration
> partial exchange transfusion
93.Accurate histological identification, and staging of the tumor ,along with
preoperative physiologic assessment of the lung function.It is an important
initial step in the optimal management of pts with lung cancer.
A.Staging: Imaging such as chest CT scans/PET scans/ radionuclide bone
Scans for metastases
B.Histologic: CT Guided biopsy.
C.Physiologic:

21

a.FEV1( low FEV 1 pts do not tolerate the surgery well)


b.Quantitative ventilation-perfusion scans to determine the Pulmonary
Reserves of that lung( if there is high reserve then resection becomes
Questionnable).
94.Lung Cancer:
A.Stage I and II: Surgery
B.Stage III and IV: Surgery + Chemo and radio
95.Multiple myeloma:
A.Age >65
B.Back or bone pain
C.Pathologic fractures
D.Hypercalcemia
E.Renal failure
F.Anemia(usually normocytic)
G.Elevated ESR( sometimes not always)
H.Hyperprotenemia
I.Diagnosis:
> Do serum and urinary electrophoresis to check for monoclonal protein
( standard dipsticks would miss the monoclonal protein )
> Do bone marrow biopsy and greater than 10% plasma cells is
confirmation.but patients with patchy bone marrow involvement can have
normal bone marrow biopsies.
J.A complete X ray skeletal series ( e.g skull,long bones,spine) shd be
performed once the pt has been diagnosed with multiple

22

myeloma.Conventional skeletal surveys typically reveal punched out lytic


lesions ,diffuse osteopenia,or fractures in nearly 80 percent of pts.
Other imaging modalities (e.g CT,MRI,PET scans ) are usually reserved for pts
with bone pains and negative initial x-ray skeletal surveys.
K.Studies requiring iodinated contrast agents shd be avoided in myeloma
pts with renal insufficiency.
L.MRI is more useful in assessing prognosis in early /asymptomatic
(smoldering) myeloma, identifying focal and diffuse bone marrow
involvements amenable to biopsy and monitoring disease progression.
M.Nuclear bone scans/ technetium 99m bone scans will show nothing
becx there is no blastic activity.
96.Complications of multiple myeloma
A.Hyper viscosity syndrome:
Typically due to increased serum protein content with large molecular
size, abnormal polymerization and abnormal immunoglobulin shapes.The
impaired microcirculation in the brain can cause Neurologic
symptoms(headache,dizziness,vertigo,nystagmus,hearing loss,and in severe
cases somnolence, confusion, seizures),blurry vision,heart failure,mucosal
hemorrhage ( e.g nasal or oral bleeding) due to impaired platelet fucntion.
Rx: plasmapheresis
B.Hypercalcemia:
Asymptomatic or presents with anorexia,nausea,polyuria,constipation,
weakness and confusion.
Rx:
>hydration
>mild=>Dexamethason
>moderate to severe=> Bisphosphonates

23

C.Renal failure:
Can occur acutely or gradually due to light Chain cast nephropathy or
deposition disease.Pts can develop dyspnea, nausea,peripheral
edema,mental status changes,and eventual normocytic anemia due to
kidney damage.
Rx=> plasmapheresis or dialysis
D.Infections:
Pneumonia and urinary tract infections.there is highest risk in first 3-4
months of therapy .Vaccines and prophylactic antibiotics during therapy
help to prevent infections.
E.Thrombosis:
Increased risk of both arterial ( e.g stroke, MI, TIA) and venous thrombi
F.Lytic lesions:
Usually presents with bone pains or pathologic fractures
Rx: Bisposphonates help prevention.
97.Children aged <3 are at highest risk of acute splenic sequestration
crisis .Accumulation of RBCs in the spleen causes splenomegaly and acute
exacerbation of anemia.Look for reticulocytosis and thrombocytopenia in
question stem.Parents shd be taught how to palpate the abdomen of child
for splenomegaly and to seek immediate medical care if the spleen feels
enlarged.
98.Recurrent episodes of splenic sequestration compromise the splenic
function ; ongoing infarction causes atrophy of spleen (auto
splenectomy).Therefore all pts are at high risk of infection from
encapsulated bacteria.Vaccination against
pneumococcus,meningococcus,H.influenzae is extremely important.

24

99.Vaccination and penicillin prophylaxis are important measures to prevent


pneumococcal infection in the setting of splenic dysfunction.
100.Young adults with sickle cell anemia are at risk of myocardial oxygen
ischemia and infarction due to oxygen demand exceeding the oxygen
carrying capacity of abnormal red blood cells.
101.Strokes are one the primary causes of death in sickle cell DX pte .The
incidence approximately 10% by age 20.
102.Ocular melanoma:
Is a primary malignant tumor arising from the melanocytes within the
Uvea (iris, ciliary body,choroid) , usually from a choroid pigmented nevus.Its
often remains asymptomatic for a long time and is frequently diagnosed
during an examination done for other reasons.When symptoms occur ,it
usually presents as blurry vision or progressive and painless visual field
abnormalities.Some pts may report floaters or pain (occasionally) if the
tumor impinges on nerves.Ultrasound of the eye is the most sensitive
imaging modality to diagnose the lesion while MRI is used to document any
extra scleral extensions for staging and treatment decisions.
Asymptomatic pts with small pigmented lesions ( diameter <10 mm,
thickness <3 mm) often experience an extended period of slow growth ,
during which they can be safely observed w/o treatment.A common
protocol is to repeat examination in 3 months and every 6 months
thereafter.
103.Treatment modalities of Ocular melanoma:
A.Radiotherapy:
Large ( Diameter > /= 10mm or thickness >/= 3mm ) or symptomatic
choroidal melanomas ( e.g eye pain,visual disturbances ) require definitive

25

treatment with radio therapy ( brachytherapy or external beam


radiotherapy ) .
B.Enucleation:
Preferred for large tumors or tumors with extra scleral extension or
severe associated pain.
C.Photocoagulation:
Photo coagulation of ocular melanomas and its sorrounding small
vessels blood supply has been advocated for small lesions and marginal
recurrence following radiotherapy.
D.Chemotherapy:
Not proven effective.
104.Inferior vena cave filters:
A.Are an option for patients with acute DVT who have contraindications
to anticoagulation ( e.g recent surgery,hemorrhagic stroke,bleeding
diathesis,active bleeding)
B.Acute complications of IVC filter placement include guide wire
entrapment within the filter and post procedural complications ( e.g acute
insertion site thrombosis) hematomas, arteriovenous fistula).
C.Main long term complications are recurrent DVTs and IVC thrombosis.
D.IVC filters do not appear to affect overall mortality significantly.
105.Retroperitoneal bleeding is most commonly due to trauma to the lower
back,vascular procedures near the retro peritoneal space( e.g cardiac
catheterization, aortic catheter) ,anticoagulation or hemorrhage from a
malignancy in the retroperitoneal organs ( e.g pancreas)

26

106.Risk factors for post operative DVT :


A.Old age
B.Previous DVT
C.Medications
D.Hyperoagulable states
E.Obesity
F.Hx of malignancy
107.Anticoagulation is the primary preferred prophylaxis for Deep
venous thrombosis in pts at high risk for post operative DVT.Low
molecular weight heparin is superior to both unfractionated heparin
and warfarin in preventing DVT in pts undergoing total hip or knee
replacement or repair surgery.
Current studies are evaluating direct thrombin inhibitors such as
rivaroxaban and dabigatran which are promising for future use.
108.Mechanical methods can be used to prevent post operative DVT in
pts with contraindiactions to anticoagulation.
>Pneumatic compression devices are preferred over compression
Stockings for serving that purpose.
109.CML is caused by a translocation of chromosomes 9 and 22 which
produces the Philadelphia chromosome.The presence of bcr/abl fusion
protein is diagnostic of CML and results in unregulated tyrosine kinase
activity.Tyrosine kinase inhibitors such as imatinib are the initial
treatment choice in almost all instances.
BMT can be offered to pts who are young with stable disease and who
have a suitable donor.
110.Look for splenomegaly, markedly elevated WBC count,blasts ,
basophilia,bands and metamyelocytes,anemia for raising suspicion
against CML.

111.WBC on the order of 4,00,000/mm3 -> hyperviscosity syndrome->


leukapheresis recommended.
112.normal calcitonin levels=> <4 ng/ml (imp in medullary thyroid CA)
113.Elevated calcitonin levels in pts with medullary thyroid cancer
following total thyroidectomy indicates metastatic disease.
CT scan of neck and chest with or w/o high resolution ultrasound
Of neck is recommended as next step for metastatic disease.
I No ?
Abdominal CT scan and bone scan may be required.
I No ?
I111 octreotide or PET scan may be helpful.
114.Medullary thyroid cancers do not take up iodine , therefore totally
body iodine scan is not useful.
115.normal iron binding capacity = 250 370 ug/dL.
116.supression of erythropeoisis in anemia of chronic disease by
inflammatory cytokines of inflammatory disorder.Some pts might have
low levels of erythropoietin as well.
117.Rx of Anemia of chronic dx in Rheumatoid arthritis:
a.Infliximab ( anti- TNF alpha ) will suppress the inflammation.
b.erythropoietin/darbepoetin shd be given if it EPO is low.pts with
appropriately elevated EPO levels ( > 500 mU/mL ) do not respond.
c.Red Cell transfusions In severe symptomatic anemia pts not
responding to initial measures.

118.ACD:
low serum iron
high ferritin
Low TIBC
Low transferrin saturation.
119.ACD:
> MCV is low-normal to mildly decreased.
> usually a mild anemia but ~20% of pts have Hb<8 g/dL.In all such
Cases clinicians shd rule out additional causes of anemia such as
Iron deficiency,myelodysplasia,thalassemia.
120. Plasmapheresis:
> Myasthenia gravis
> GBS
> TTP
121.Splenectomy:
> hypersplenism
> auto immune hemolytic anemia
> hereditary spherocytosis
122.Along with ACD,Coexisting iron deficiency anemia can also occur in
Rheumatoid arthritis which can result from GI bleed secondary to drug
therapy or decreased absorption of dietary iron.If oral iron
supplementation is not adequately absorbed , as may occur in pts with
rheumatoid arthritis , parenteral iron stores may be needed.
123.Management of ACD ( anemia of chronic dx) in pts with chronic
inflammatory conditions such as rheumatoid arthritis involves treating
the underlying condition and ruling out concurrent causes of anemia.
124.pancoast tumor is a subset of non small cell lung cancer.

125. Shoulder pain is the most common initial presentation of Pancoast


tumor.
126.Pancoast tumor:
> Shoulder pain
> Horner syndrome (ptosi,miosis,anhidrosis and enophthalmos)
> Weakness and atrophy of ( C8-T2 involvement )
Intrinsic hand muscles and
Assymetric upper extremity
Deep tendon reflexes, called
Brachial plexopathy.
127.There can be pain and paresthesias of 4th and 5th digits , and medial
arm and forearm due to C8-T2 involvement in Pancoast syndrome.
128. Spinal cord compression occurs in upto 25% of pts with Pancoast
tumor and dictates a worse prognosis. Urgent interventions are
required if it happens (corticosteroid therapy, surgery and radiation ).
129. There can be supraclavicular lymph node enlargement in Pancoast
tumor.
130. Asymmetric lower extremity deep tendon reflexes , particularly in
the setting of low back pain suggests that Pancoast tumor has spread to
spinal cord.
131.Metastatic tumors to brain ( secondary ) are more common brain
tumors than the tumors that orginate inside the brain ( primary ).
132.Secondary brain tumors in order of frequency:
Lung>Breast>unknown primary>melanoma>colon cancer.

133.small cell lung CA-> hemtogenous spread-> multiple brain lesions->


do contrast enhanced MRI and you woud see multiple,well
circumscribed lesions,surrounded by large amount of vasogenic edema
compared to the size of lesion.
134.Organomegaly ( hepato splenomegaly ) in CLL.
135.CLL
A.characterized by significant lymphocytosis
B.Staging systems(Rai,Binet) guide treatment and prognosis by taking
Into account :
a.Physical examination findings: Organomegaly,lymphadenopathy.
b.CBC: Anemia, thrombocytopenia.
The presence of any of these findings , including thrombocytopenia,
Indicates a worse prognosis.
C.Confirmed by combination of peripheral smear( smudge cells ) and
Flow cytometry.
D. lymph node and bone marrow biopsy are not generally needed
E. Median survival of 10 years.
F. Infection and secondary malignancy(richter transformation)
are principal causes of death.
G. Monocytic B-cell disease.
H. CLL is often asymptomatic and found on routine blood work up
I. Auto immune hemolytic anemia in CLL.
136.Worse prognosis in CLL:
> lymphadenopathy
> Organomegaly
> Anemia
> Thrombocytopenia

137.Superior vena cava syndrome: ( SVCS)


A. Atleast 80 percent of the cases caused by bronchogenic carcinoma.
B. The classic presentation begins with dyspnea , persistent cough,
Facial fullness, neck pain and progresses into hoarseness,
Dysphagia,chest pain and syncope.Pertinent physical findings are
Edema and erythema of neck ( which may sometimes compromise
The face) and dilated veins of arms and neck.Advanced disease is
Manifested by cyanosis, collateral veins in thorax, ocular proptosis
And lingual edema.
C.The best diagnostic test for SVCS is a contrast CT scan of chest and
Neck , which will reveal an obstruction of the superior vena cava
Due to the pulmonary mass,the metastatic nodes or an intravenos
Thrombus.CT scan is very useful because it can reveal the extent of
Obstruction and provide a histopathologic diagnosis(via a
Percutaneous biopsy) , which will determine the particular
Therapeutic regimen required to manage the underlyng mlignancy.
D.MRI is a more expensive procedure and does not offer any visual
Advantage over CT scan.Its only used when a pt is allergic to the
Contrast dye used in CT scan.or when venous access cannot be
Obtained for contrast-enhanced studies.
E.SVCS shd be suspected in any pt who has a high risk of malignancy
( e.g family history or smokers ) and presents with dyspnea,
Orthopnea, neck pain and swelling, and has cervical and upper
Extremeties venous dilation.
138.Physiologic jaundice presents 24 hours after birth.
139.Neonatal jaunice appearing in the first 24 hours of birth is always
pathologic.Immune or non-immune hemolysis is frequently present in
such pts.

140.G-6-PD deficiency anemia:


> X-linked.
> Should be suspected in male infants of African , Mediterranean or
Asian descent.
> No triggering even is usually present.
> Infants developing severe jaundice usually have Gilberts syndrome
as well.
141.Osteolytic metastasis:
a.multiple myeloma
b.non small cell CA
c.Non-Hodgkins lymphoma
I
Poorly visible on radioisotope bone scans( technetium 99 MDP ) as the
labelled technetium is preferentially taken up in areas of new bone
formation
I
Plain X rays and PET scans are usually more effective.
142.Osteoblastic metastasis:
a.Prostate CA
b.Small cell CA
c.Hodgkins lymphoma
I
Radionuclide bone scans shd be done for bony metastasis detection.
143.Mixed Osteolytic/Osteoblastic lesions:
Breast CA
I
Radionuclide Bones scans.
I No?
PET scans to be done then.

144. Bone metastasis.


>Consider CT or MRI to assess cortical integrity & risk of fracture.
>Consider MRI if signs of Neurological involvement.
145.Bone pains of prostate CA:
> mostly at night
> Dull , boring , aching
> Targets axial skeleton mostly.
> Often unremarkable findings on physical ex. and Plain X rays.
I
Do radionuclide bone scan.
146.In the absence of local pain, Bone scans are not usually necessary
for routine staging of low grade prostate CA. However, it shd be
considered in pts with possible high grade disease ( e.g PSA > 20
ng/Ml,Gleason score > 8, extra prostatic extension ).
147.soft tissue dx in chest, abdomen and pelvis-> CT scan to be done.
148. Shd I start androgen deprivation therapy for prostate CA?
Do DEXA scans for detecting osteopororsis/osteopenia due to risk
of further bone loss with therapy.
149.Diabetics-> frozen shoulder-> physical therapy needed.
150.Gabapentin:
a.Post herpetic neuralgia.
b.Diabetic neuropathy.
151.hemolytic disease of the newborn is possible only when the mother
is Rh-negative and father is Rh-positive.

152.RhoGAM shd be given within 72 hours of delivery when its


required.
153.Factor V leiden is the most common cause of inherited or
hereditary thrombophilia , accounting for approximately 40-50 % of
inherited thrombophilias.
154.Polycythemia:
> Men= >18.5 g/dL.
> Women= >16.5 g/dL.
I
Hb shd be repeated to confirm polycythemia and to avoid spurious
results.
155.Serum Erythropeoitin level is always the best initial in polycythemia
> low EPO= Polycythemia vera, a chronic myeloprofilerative disorder
> High EPO= Secondary polycythemia due to:
a.chronic hypoxia.
b.Neoplasms( commonly renal cell carcinoma)
156.Hb-> repeat Hb-> EPO levels.
157.If EPO is elevated, its secondary polycythemia.Chronic hypoxia the
MCC of secondary polycythemia.ABGs shd be done to exclude hypoxia.
Carboxyhemoglobin levels shd be obtained to exclude Carbon monoxid
poisoning in certain pts , including those who smoke heavily. Sleep
apnea / Obesity hypoventilation syndrome shd be considered in pts
with a relatively normal oxygen saturation on physical examination, as
hypoxia may only be intermittently present at night , so do
Nocturnal oximetry.
158.Warfarin is usually started in HIT after treatment with a nonheparin anticoagulant ( direct thrombin inhibitors or fondaprinux )

and platelet recovery to >/= 1,50,000 / uL.


159.The primary goal of treatment in pts with metastatic hormone
refractory prostate cancer is palliation.Specific aims in management are
adequate pain relief,improvement of the functional status,and
prevention of vertebral compression fractures and spinal cord
compression.
160.Focal external beam radiation therapy shd be used for pain
alleviation in pts with single or few focal bone metastatic lesions due to
hormone-refractory prostate cancer in which the pain is not adequately
controlled with narcotic analgesics.
161.When do you use Corticosteroids or do surgical consultation for
metastatic prostate cancer ?
When there is spinal cord compression
162. Metastatic -> Main complaint -> focal external beam radiation
prostate CA
is pain
therapy.
163.Infectious mononucleosis:
>Tonsillar pharyngitis
>fever
>fatigue
>symmetric
-> peaks in first week-> subsides in 2-3 weeks.
posterior cervical
lymphadenopathy
164.Persistent and localized abornmal lymph nodes shd be biopsied.
Abnormal and persistent lymphadenopathy is usually the first
clinical manifestation of an underlying lymphoma.

165.Pts with localized lymphadenopathy should be observed for three


to four weeks.A biopsy shd be performed if the abnormal nodes fail to
resolve after four weeks.
166.Hard cervical lymph nodes, esp in older pts and in smokers , are
suggestive of malignancy.These pts shd be referred to an
otolaryngologist for fibreoptic examination of the oropharynx to look
for a primary malignancy.
167.ITP in children:
a.characterized by sudden onset of bleeding , manifested as
petechiae,purpua, epistaxis,gingival bleeding.More severe bleed is
rare.
b.Treat with steroids and/or IVIGs when:
> symptomatic OR/and
> platelets <30,000 / uL.
c.Do splenectomy in children with chronic ITP who persistently
have hemorrhagic symptoms.
168.Radiation-induced cardiotoxicity:
Radiation therapy causes fibrosis in the interstitium of the
myocardium , along with progressive fibrosis of the pericardial
layers, cells in the conduction system, and/or leaflets of the valves.
It also causes injury to the intimal layers, with arterial narrowing
typically involving the ostial parts of coronary vessels.These effects
lead to:
A.Myocardial ischemia and / or infarction( due to coronaries ostial
narrowing probably I think)
B.Restrictive cardiomyopathy with diastolic dysfunction.
C.Constrictive pericarditis.
D.Valvular abnormalities ( mitral or aortic stenosis/ regurgitation )
E.Conduction defects ( sick sinus syndrome or variable degrees of
heart block ).

169.Anthracycline toxicity-> Dilated cardiomyopathy->Reduction of EF


170.Secondary malignancy ( e.g breast, lung or GIT cancer;acute
leukemia,non-hodgkin lymphoma ) is the leading cause of death
in the survivors of hodgkins lymphoma.
180.When pts do not respond to therapy for osteoporosis (e.g
bisphosphonates ) and/or have rapidly progressive osteoporosis ,
secondary causes shd be actively investigated. Multiple myeloma
shd be considered in such pts especially those who progress very
rapidly with multiple fractures because Mutliple myeoloma can
present as rapidly progressive generalized bone loss due to
secretion of osteoclast activating factor ( OAF ).
181.Calcitonin and Bisphosphonates mechanism of action:
Osteoclast inhibition.
182.Injectable PTH fragment ( 1-34 amino acids, teriparatide ) is
indicated in pts with a very high risk for fractures and in those who
fail other modes of osteoporosis therapy.It is very useful improving
bone mineral density.The bone density gain is much more than
that seen with bisphosphonates , calcitonin or estrogen.
Teriparatide also reduces the vertebral as well as non-vertebral
fractures rate in pts with primary osteoporosis.
183.Very obese and high BMI pts might require more than normal
amount of Wafarin compared to others.
184.Goal of INR with warfarin should be 2 to 3.
185.Under anti-coagulation can lead to worsening of thrombotic
disease and such pts shd be started on Intravenous heparin
( or subcutaneous low molecular weight heparin ) until a

therapeutic INR is achieved with warfarin. Such cases should not


be considered anti-coagulation failures that warrant placement of
IVC filter.
186.ITP:
A.A presumptive diagnosis of ITP can be made when the history,
physical examination, CBC , differential count and peripheral
blood smear show isolated thrombocytopenia without any
obvious causes.
B.Due to Platelet destruction by antiplatelet autoantibodies which
are most often directed against membrane proteins ( GPIIb/IIIa ).
C.Circulating platelets are rapidly removed by the auto antibodies ,
limiting the role of platelet transfusions.Platelet transfusions are
limited to life threatening emergencies(e.g intracranial
intracranial hemorrhage).
D.There might be antecedent viral infection.
E.The asymptomatic petechiae and ecchymosis are most common
presentation.
F.Mucocutaneous bleed ( e.g epistaxis, hematuria , GI bleed ) can
occur.
G.Lab findings show you isolated thrombocytopenia of
<1,00,000/uL and nothing else.
H. Peripheral smear would show you megakaryocytes and no other
abnormalities.
187. Treatment of ITP:
A.Children:
a. He only has skin manifestations?
Observe him and dont treat him
b. He has bleeding?
Give him either IVIG or Glucocorticoids.
B.Adults.
a. He is not bleeding PLUS his platelets are >30,000/uL ?

Do NOT treat him.


b. His platelets are less than 30,000 / uL ?
Give him either IVIGs or Glucocorticoids.
c. He is bleeding ?
Give him either IVIGs or Glucocorticoids.
C.Treatment options for ITP are:
>Corticosteroids are first line.
>IVIGs can be considered for pts who have failed or have
contraindications to steroid therapy or require a more rapid
increase in platelet counts.
>Anti-Rh(D) is an alternate treatment option in rhesus+ve
non-splenectomized pts.
>Rituximab can be considered for pts who have failed initial
treatment.
>Splenectomy is reserved for refractory cases.
188.Thrombotic thrombocytopenic purpura is characterized by
thrombocytopenia and microangiopathic hemolytic anemia, which
manifests as schistocytes on peripheral smear and elevated LDH.
Other features may include acute renal failure, fever and neurologc
abnormalities.
189.TTP-> Plasma exchange.
190.Renal dysfunction
I
Functional platelet dysfunction
I
> Normal platelet count.
> Increased Bleeding time
I
Treatment options:

a.Desmopressin-> 1st line and least toxic treatment.It acts by


releasing the factor VIII-von Willebrand factor multimers from
endothelium.
b.Estrogen
c.Cryoprecipitate-> associated with infectious complications.
d.Dialysis.
Note: Treat this condition only when the patient is having
active bleeding or he is about to undergo surgery.
191.Radionuclide ventriculography/MUGA scan (multigated acquisition)
has high accuracy and reproducibility for measuring the ejection
fractions.Its most common use is in the initial evaluation and
subsequent follow-up of patients receiving cardiotoxic chemo
such as Anthracycline agents ( doxorubicin and daunorubicin).
These chemo agents would be contraindicated if the EF is less than
30 percent and modified dosing would be required for pts with
baseline EF less than 50%.A decrease of EF by ten or more percent
points may warrant discontinuation of therapy.The risk of Cardiotoxicity with anthracycline therapy is related to the cumulative
dose and is increased in pts with pre-existing cardiac disease with
a low ejection fraction(EF).MUGA scan is before initially at baseline
before starting chemo and then before each subsequent dose.
192.Sickle cell anemia is definitively diagnosed by Hb electrophoresis.
193.Hb electrophoresis:
> Sickle cell disease
> thalassemia
194.Sickle cell dx rarely presents before 6 months of age when fetal Hb
is present.It manifests when fetal Hb declines and functional
asplenia typically develops.

195.Benign pulmonary nodule:


>Popcorn calcification characteristically wth pulmonary hamartoma
>Concentric calcification
>Laminated calcification
>Central calcification
>Diffuse homogenous calcification
>Smooth borders
196.Suspicion for malignancy
>Eccentric calcification ( areas of asymmetric calcification )
>Reticular calcification
>Punctate calcification
>Spiculated margins
197.All pregnant women shd be screened for anemia at the first
visit.
198.Thalasemia:
A.Decreased MCV ( microcytic )
B.Decreased MCH ( hypochromic )
C.Mildly elevated ferritin due to increased RBC turnover.
D.The RBC count may be normal or increased.
E.RDW is normal because all RBCs are uniformly small.
F.Thalasemia is more common in those of Medeterrinean,Middle
Eastern, Southeast Asian, African and Indian ethnicity.
G.Hb electrophoresis shd be ordered to distinguish alpha
thalassemia from Beta thalassemia minor.Electrophoresis is normal
in pts with alpha thalassemia: Beta thalassemia casuses abnormally
elevated Hemoglobin A-2 levels.
H.Partners of pregnant women with thalassemia minor should
undergo Hemoglobin analysis to assess fetal risk of inheriting
thalassemia.

199.Maternal conditions requiring partner testing include clinically


clinically significant maternal hemoglobinopathies ( e.g SS,SC,Hb-S/
B-thalasemia ) , as well as carrier states ( alpha and B-thalasemia ).
200.Diabetic ketoacidosis can provoke hemolysis in G6PD deficiency.
201.Physiologic anemia of pregnancy is characterized by mild
normocytic , normochromic anemia. This results from a greater
expansion of plasma volume relative to the increase in RBC mass
(i.e physiologic or dilutional anemia ).The nadir typically occurs
during the late second to early third trimester of pregnancy.
202.Interestingly , only 30 percent cases of Sickle cell anemia are
diagnosed at the age of 1 year.
203.The child has hx of anemia and has an episode of Salmonella
osteomyelitis ?
This is sickle cell anemia -> go for Hb electrophoresis.
204.Salmonella osteomyeilitis usually affects long bones and multiple
foci are often present.

Infectious diseases

1.Cat-Scratch disease.
A.affects young immunocompetent.
B.Caused by Bartonella henslae
C.Suppuration of lymph nodes is the most common
complication occurring in 10% of pts.
D.Other complications are Neuroretinitis, encephalopathy,
fever of unkown origin and hepatosplenomegaly.

2.Percutaneous or mucous membranes exposure to HIV+ve pt.:


First step=> thoroughly wash the skin.
Second step=> Start PEP within 2 hours.
A. The source pt is either asymptomatic or low viral load ?
Two drug regimen of two nucleoside reverse transcriptase
inhibitors shd be given for four weeks.
B.The source pt is symptomatic or high viral load ?
Three drug regimen for four weeks is given. Two nucleoside
reverse transcriptase inhibitors PLUS a protease inhibitor.

3.Neonatal Hepatitis:
A.IVIG and Hepatitis vaccine shd be administered within 12
Hrs of birth in
neonates born to HBV+
B.Vaccine first dose at birth, second dose at one month age
and third dose at 6 months of age.
C.Serology controls for HBV shd be monitored either 3-4
months after the third dose OR between 9-15 months of age.
D.With prophylaxis, the risk of chronic Hepatits B in infants
born to mothers with Hepatitis B decreases from 30% to 2%.
E.Infants born to mothers with Hepatitis B infection has 30%
risk of acquiring chronic Hepatitis B and 25% risk of developng
Hepatocellular carcinoma.

4.Cellulitis:
A.Infection of deep dermis and superficial fat in contrast to
the erysipelas which involves the superficial dermis and
lymphatics.
B.Local anesthetics such as lidocaine are not effective for
reducing pain because the acidic environment created by the

infection neutralizes the basic local anesthetics hence


neutralizing the medication.
C.Cellulitis is characterized by irregular and elevated borders as
well as increased local temperature, tenderness and erythema
D.Diabetes mellitus predisposes to cellulitis.
E.If the pts infection progresses to abscess formation, then the
local anesthetic shd be used to anesthetize the epidermis in
order to drain the abscess.

5.Women who have sex with women have a cervical cancer risk
similar to that of heterosexual women.Screening for cervical CA
in this pt population shd be performed according to current
guidelines.
>Screening starts at age of 21.
>Incase of immunocompromised (HIV, SLE, organ transplant
recepients on immunosuppressant therapy) , start screening at the
onset of sexual intercourse and do it every 6 monthly x 2 times, then
annually.
>Women aged 21-29 can be screened with Pap smear every 3 years.
>Those aged 30-65 can be screened with either pap smear every 3
years or Pap smear with HPV testing every 5 years.

> equal/greater than 65, stop the screening if negative prior screens
and not high risk of cervical cancer.
>Dont screen those who have done hysterectomy with cervix
removed PLUS there is no hx of high grade pre cancerous
lesion,cervical cancer, or exposure to diethyl stilboesterol.
>Women aged lesser than 30 should not be screened with HPV
testing as infection in this population is transient and typically does
not lead to cervical cancer.HPV testing in younger women also leads
to unnecessary colposcopies.

6.Most pts with active TB have abnormal chest X rays with


findings such as upper lobe infiltrates,cavities,hilar
adenopathy or pleural effusions.

7.Latent TB:
>An induration of >/= 10 mm on a tuberculin skin testing
in a recent immigrant ( within the past 5 years ) from a
country with high incidence of TB ( countries in Asia, Latin
America,Eastern Europe and Africa ) is considered +ve.
>Isoniazid given daily for 6-12 months reduces the risk of
active TB.
>Asymptomatic pts without radiographic evidence of active TB

who have been treated previously for active TB or LTBI


(latent tuberculosis infection) need no further TB treatment.
There is no need for a repeat Tuberculin skin test as it will
likely remain positive.
>Pts who have completed treatment for latent tuberculosis
infection should not undergo repeat tuberculin skin testing as
it will likely remain positive.

8.Interferon gamma release assays measure Interferon gamma


release by T-cells stimulated by specific M tuberculosis
antigens.They are an alternate means of diagnosing LTBI and
cannot distinguish LTBI from active TB.Unlike Tuberculin skin
tests, they do not require repeat visits and are not affected by
vaccination with Bacille Calmette-Gerin.

9.HIV:
>HIV+ve child can attend the public school in normal manner
( i.e comingling with other students in the class room and on
the the playground ) and may participate in all sports.
> Any disclosure of HIV status by the family is voluntary.

>In rare instances, exclusions can be instated if an HIV+ve


child has aggressive tendencies (i.e biting).Such issues shd be
resolved before the child is enrolled in school.
>HIV is usually transmitted by blood, semen, vaginal fluids
and breast milk.
>Standard precautions including the use of gloves when in
contact with blood or mucous membranes , are sufficient to
preclude the transmission of HIV.

10.Undercooked meat consumption of infected animals


( including lamb,beef or game) or contact with cat feces
during pregnancy may be associated with congenital
toxoplasmosis, which can manifest as microcephaly as a late
manifestation.Other TORCH infections can also result in
microcephaly.

11.Typical triad of Congenital toxoplamosis:


>Hydrocephalus
>Intracranial calcifications
>Chorioretinitis

Other manifestations are mental retardation, seizures,


deafness

12.Maternal smoking during pregnancy is associated with low


birth weight infants.

13. Excessive maternal caffeine consumption is associated with


increased risk of spontaneous abortion and still birth.

14.Testing a pt for HIV following an exposure is highly


recommended to avoid unnecessary HIV prophylaxis.When
a pt refuses testing,testing reults are pending or HIV status
the pt is unknown , following occupational exposure the
first step is to administer HIV post exposure prophylaxis to
the health care worker immediately as efficacy decreases
over the course of hours.

15.Chlamydial conjunctivitis:
>Chlamydia is the most common cause of infectious neonatal
conjunctivitis.

>Cannot be prevented with erythromycin ointment


>Maternal testing and treatment is recommended for
prevention and that shd be done at first prenatal visit.
>caused by direct maternal vaginal contact during delivery.
>The conjunctivitis occurs at 5-14 days.
>The pneumonia occurs at 4-12 weeks:
Presents as nasal congestion,staccato cough, rales on
examination and hyperinflation on chest x ray.
Differentiate from Respiratory syncytial virus bronchiolitis
which would have wheezing instead of rales and there
would be fever present which is not there in chlamydia.
>Rx of neonatal chlamydial conjuncitivits is 14 days course
of Oral erythromycin,regardless of the fact that it increases
the risk of hypertrophic pyloric stenosis becx other therapies
like azithromycin have not been well studied and topical
therapies dont work.

16.
A.Airborne transmission:
>TB

>Varicella
>Measles
B.Breast milk transmission:
>HIV
>Human T-cell lymphocytic virus transmission
C.Direct physical contact:
>Staph aureus
>Clostridium difficile
>Scabies
D.Droplet transmission
>Influenza
>Perussis
>Neisseria mengitidis.
E.Intrauterine transmissions
>HIV
>Syphilis
>CMV

17.Dengue:
>Spread by Aedes mosquito

>Prevalent in tropical and subtropical regions( South and


South east Asia,Pacific islands , Carribean, Americas )
>Can present as acute febrile illness with headache,retroorbital pain,joint/muscle pain,macular rash,cervical
lymphadenopathy,pharyngeal erythema.
>Other findings can include hemorrhagc tendencies(petechiae
with tourniquet application) or spontaneous hemorrhage(e.g
purpura, echymoses),thrombocytopenia,leukopenia.and
elevated transaminases.
>Dengue hemorrhagic fever (DHF) is the most serious dengue
viral infection and is due to increased capillary permeability
leading to hemoconcentration,pleural effusion,and ascites.
Circulatroy failure can develop with significant plasma
leakage and is sometimes referred to as Dengue shock
syndrome.Pts typically have marked thrombocytopenia
(<1,00,000/mm3) and prolonged fever and respiratory.
Respiratory failure can occur.
>The rash of Degue fever is described as islands of white in
sea of red.

18.Cerebral edema can occur in Faciparum malaria which


usually presents with fever, anemia and splenomegaly.

19.Leptospirosis:
>There can be occupational exposure in farmers and
veterinarians.Can be from fresh water swimming or river
rafting.Household exposure can occure from pet dogs or
rodent exposure.
>There is abrupt onset of systemic symptoms e.g fever
,headache and myalgias. Conjunctival suffusion can occur.
There will be GI symptoms i.e Nausea, vomiting and diarrhea
Hepatosplenomegaly and / or lymphadenopathy can occur.
Muscle tenderness and myalgias can occur.Jaundice is seen
is most severe cases ( Weils syndrome ).

20.Rheumatic fever:
>Syndenham chorea OR carditis alone is sufficient to diagnose
rheumatic fever.
>Syndenham chorea is characterized by emotional lability
and decline in school performance.There will be distal hand

movements that will progress to facial grimacing and feet


jerking.Movements are typically irregular and rapid.Pts also
have decreased strength throughout and the relaxation
phase of the patellar reflex is usually delayed.In addition,
pronator drift is present.
>Sydemham chorea develops 1-8 months after the initial
streptococcal infection; carditis and arthritis develop within
3 weeks.
>Pts with Sydenham chorea shd receive long-acting IM
penicillin until adulthood for secondary prevention, even if
there is no active pharyngitis.The goal of antibiotic therapy is
to eradicate group-A streptococcus to prevent recurrent
acute rheumatic fever and worsening rheumatic heart dx.
>Treatment of Sydenham chorea in itself is supportive and
with complete resolution occurring within months in most
pts.Corticosteroids can reduce the duration of symptoms
but are typically reserved for severe cases.
>NSAIDS such as ibuprofen can be used for arthritis in ARF.
Aspirin shd be avoided due to the risk of Reye syndrome.

21.Haloperidol is a powerful dopamine antagonist that can be


used to control chorea in pts with severe symptoms that
may result in self injury.

22.Methylphenidate -> for ADHD pts.

23.Terbenazine-> Dopamine antagonist-> for Hutington dx.

24.Chlamydia:
>Genitourinary infection caused by Chlamydia is extremely
common and results in insidious and often chronic
unrecognized dx.Its sexually transmitted.
>70 percent of women are asymptomatic.
>Major cause of infertility,PID and ectopic pregnancy.
>Risk factors include younger age, multiple sexual partners,
inconsistent use of barrier contraceptives and history of
prior STDs.
>CDC recommends testing all pregnant women at first
prenatal visit.
>Women under age 25 and those at increased risk for

chlamydial infection shd have repeat testing in 3rd trimester.


>Chlamydia endometritis during pregnancy can lead to chorioamnionitis and premature delivery of the fetus.

25.Chlamydia treatment protocols:


>Options for treatment;
A.Macrolides.
a.Azithromycicn ( 1gm PO as single dose )
b.Eryrthromycin base ( 500 mg PO QID for seven days )
c.Erthromycin ethylscuccinate( 800 mg PO for seven days)

B.Tetracylines.
Doxycline (100mg PO BID for seven days )
C.Fluoroquinolones.
Ofloxacin ( 300 mg PO BID for seven days )
Levofloxacin ( 500 mg QD for seven days )
D.Amoxicillin ( 500 mg PO TID for seven days)
>Most commonly used regimens are Azithromycin and
Doxycycline.

>Tetracylines,fluoroquinolones and Eryrthomycine estolate


are contraindicated in pregnancy.
>Recommended treatment regimens in pregnancy are
Erythromycin base and Amoxicillin.Azithromycin is an
alternative though it has not been well tested in pregnancy.
>Sexual partners are presumptively treatment to prevent
recurrence and reinfection.

26.Toxic shock syndrome:


>Almost all cases are caused by staph aureus.
>Toxic shock syndrome toxin-1 acts is an exotoxin released
by staph aureus that causes widespread activation of T-cells
acting as a superantigen i.e can activate T-cells directly w/o
needing to processed by antigen recognition cells.
>Presents as a prodrome of fever, chills, myalgias that evolves
into a multisystem syndrome ( e.g hypotension and shock
with dermatologic , gastro intestinal , muscular , renal &/or
neurologic findings ).

27. massive bacterial lysis and circulating endotoxin->Septic shk

28.Bacterial overgrowth and dissemination->bacterial


endocarditis.

29.Management of toxic shock syndrome:


>Remove any associated foreign bodies ( e.g nasal packing )
>Extensive fluid replacement that can reach to 20 L per day
>Clindamycin therapy ( theoretically prevents toxin synthesis )
with or without anti-staphylococcal therapy ( vancomycin or
oxacillin or nafcillin if susceptible ) to eradicate the organism
and prevent TSS recurrence.
>Corticosteroids do not affect mortality and are not routinely
recommended.
>Hypotension persists ? -> vasopressors
>Cardiogenic shock ? -> Dobutamine.
>Polyclonal human immunoglobulin seems to be more effect
in streptococcal rather than staphylococcal TSS.

30.Chagas disease:
>Caused by trypanasoma cruzi, a protozoan.
>Most common in Mexico, Central and South America.
>Has 3 phases :
fever&myalgias->serologic/parasitic evidence only->Final
phase of Chagas cardiomyopathy and GI dx.
>The GI dx is characterized by progressive dilation of
esophagus and colon.
>Causes dilated cardiomyopathy.
>Chagas cardiac pts can remain asymptomatic or develop
heart failure.Findings of right sided heart failure ( e.g Jugular
venous distension,ascites,edema ) are more pronounced
than those of left heart failure (e.g dyspnea, crackles ).These
pts frequently develop arrhythmias due to fibrosis of
conduction system, including sinus node dysfunction
complete heart block and V-tach.Echo shows
varying degrees of biventricular dysfunction along with
left ventricular apical aneurysm which is considered
pathognomonic of Chagas cardiomyopathy.The mural
thrombosis can lead to embolic complications.

32.Moderate to heavy alcohol consumption can lead to a dilated cardiomyopathy with


global hypokinesis and dilation of left ventricular cavity.

33.Lyme disease aka borreliosis caused by spirochete Borrelia burgdorferi.

34.Giant cell myocarditis is a rare form of idiopathic myocarditis thought to be mediated


by an auto-immune process.Depending upon the severity, the echo can show left
ventricular dilation with segmental and/or global left ventricular systolic dysfunction.

35.Infection with HIV has been associated with development of dilated cardiomyopathy.

36.Presence of left ventricular apical aneurysm in absence of coronary artery disease is


considered pathognomonic for Chagas cardiomyopathy.

37.Cryptococcal meningitis:
>Subacute course
>More severe symptoms seen in HIV+ve pts.
>CSF analysis:
a.Markedly elevated opening pressure , often > 250-300 mm H20 on spinal tap.
b.Mononuclear ( lymphocytic ) predominance with a leucocyte count that is
characteristically low ( < 50/mm3) compared to other forms of meningitis.
c.Elevated protein
d.Low glucose.
e.Positive India Ink Preparation. OR cryptoccal antigen test.

>Eye examination shows papilledema due to elevated CSF CNS pressure caused by
fungus.The papilledema would cause blurry vision.The elevated pressure also causes
vomiting.
>Initial treatment consists of Amphotericin B PLUS Flucytosine
>Due to outflow obstruction of CSF by yeast and its capsular polysaccharide,the
cryptococcal meningitis causes increased intracranial pressure and is manifested as
severe headache, altered sensorium and blurred vision.Pts may require repeated
lumbar punctures , although some require lumbar drains or ventriculostomies.
>In HIV+ve pts with cryptococcal meningitis, typical induction therapy consists of
Amphotericin B and flucytosine and usually lasts 10-14 days.If there is clinical
improvement during this period, Amphotericin B and flucystosine can be discontind
and fluconazole can be used as consolidation and maintanence therapy to prevent
recurrence.
>In the setting of cryptococcal meningitis , the inititation of HAART therapy can lead to
immune reconstitution inflammatory syndrome ( IRIS ), a paradoxical worsening of
infection ( due to strengthening of the immune system ) that can result in increased
mortality.Based on trial data showing mortality benefits, HAART is typically started
4-10 weeks after the initiation of anti-fungal therapy in pts with cryptococcal
meningitis.

38.Ampicillin + Gentamicin:
Used in neonatal sepsis less than one month age, including Neonatal meningitis.
Most common agents are GBS and E.Coli.

39.Ceftriaxone PLUS vancomycin ( to cover penicillin resistant strains of Strep


pneumoniae ) used for empirical therapy in adults with meningitis. Ampicillin added
for pts > 50 years age to cover the Listeria monocytogenes.

40.Dexamethasone is used in Pneumococcal meningitis.

42.Pyrimethamine, sulfadiazine and leucovorin -> toxoplasmic encephalitis.

43.Caspofungin-> treats infection of candida species ( candidemia ) and aspergillus.

44.PID:
>Caused by Neisseria gonorrhea and chlamydia trachomatis, so the antibiotics must
cover these two organisms.
>There is abdominal pain, tenderness on palpation of adnexae and lateralization of
cervix.
>chills , bandemia , nausea and vomiting are signs of severe infection.
>Treatment:
a.Mild infection: IM ceftriaxone + Oral Azithromycin/Oral Doxycycline.
b.Severe infection: IV cefoxitin/ IV ceftriaxone + IV doxycycline.

45.Otitis media:
>A normal appearing tympanic membrane with decreased mobility on pneumatic
otoscopy is suggestive of an effusion in the middle ear.An effusion commonly persists
upto 3 months after an acute episode has been treated.

>Therapy is not usually required , unless there are persistent symptoms of infection,
the effusion is bilateral or more than three months have elapsed since the acute
episode.
>Switching to a second line antibiotic shd be considered if there is inadequate
improvement of symptoms or appearance of the tympanic membrane or when there
persistence of a purulent ear discharge.According to the CDC, three drugs can be used
as alternatives.These are amoxicillin-clavulanate, cefuroxime axetil and IM ceftriaxone
>Tympanocentesis or myringotomy with culture are indicated in children in whom the
clinical response to a second line agent has been unsatisfactory
>Myringotomy and insetion of tympanostomy tubes shd be considered for pts with
otitis media and effusion after antibiotic therapy and an ample period of watchful
waiting.

46.Cystic fibrosis:
>Are at risk of respiratory failure from recurrent pneumonias.
>Staph aureus is the most common cause in children and Pseudmononas aeruginosa
is the most common cause in adults.
>Treatment protocol for pneumonia:
Resistant strains should be considered in CF, esp in pts with a hx of recurrent
hospitalizations.Add coverage for resistant strains of both staph aureus and
pseudomonas because many children are infected with both.
A.Staph aureus: Vancomycin
B.Pseudomonas:
Intravenous regimen

a.Tobramycin PLUS
b.Anti pseudomonal semisynthetic penicillin ( ticarcillin-clavulanate,
piperacillin-tazobactam) OR a 3rd or 4th generation cephalosporin
(e.g ceftazidime, cefepime ) OR a carbapenem ( e.g meropenem,
imipenem/cilastatin ).
C. A combination of tobramycin, ticarcillin-clavulanate and vancomycin is a preferred
empiric regimen in pts experiencing severe pulmonary exacerbations.

47.Ampicillin-sulbactam provides good coverage for anaerobes in aspiration pneumonia

48.HIV testing:
>All individuals aged 15-65
>In each pregnancy, regardless of normal results in previous pregnancy.
>Annual or more frequent testing in high risk pts ( men who have sex with men,
IV drug users, sex workers, sexual partners of HIV+ve pts, those with a hx of another
STD and those who engage in unprotected sexual intercourse )
>Method of screening is P24 antigen and HIV antibody 4th generation assay.Those with
+ve results shd undergo HIV1/HIV2 antibody differentiation assay , as this influences
the treatment regimen.
>Plasma HIV RNA testing is recommended for pts with negative serologic tests and
high clinical suspicion of acute HIV.

49.HCV screening is recommended in pts with elevated ALT,HIV positive status and HCV
risk factors ( e.g Hx of IV drug use,hemodialysis, born in US between 1945-1965,

received clotting factors before 1987, blood transfusion before 1992 ). Sexual contacts
of HCV+ve pts shd also be screened.

50.Anti-pseudomonal agents used in the treatment of CF pneumonia:


> those listed above
> others:
a.certain fluoroquinolones.
b.Aztreonam
c.Colistin.
d.Amikacin

51.Ampicillin-sulbactam->Rx of a host of infections , esp intra-abdominal ones.

52.Oseltamivir-> neuraminidase inhibitor-> shd be started within 48 hrs

53.Common causes of pneumonia:


Strep pneumoniae + Mycoplasma pneumonia

54.Pseudomonas-> greenish sputum.

55.Acute rhinosinusitis:
>lasts less than 4 weeks
>usually viral etiology , that is complicated by secondary bacterial infection esp in
people with nasal obstruction,allergic rhinitis,intranasal cocaine use,recent tooth

infection.
>most common bacteria implicated are strep.pneumoniae, hemophilus influenza &
Moraxella catarhalis.
>signs of bacterial rhinosinusitis are fever,sinus pressure,headache,facial pain,maxillary
tooth pain,nasal obstruction and purulent nasal discharge.
>Acute rhinosinusitis is usually viral if symptoms last <10 days and does not require
treatment with antibiotics.Bacterial superinfection can occur and antibiotics indicated
without the need for further testing or imaging in pts who have symtoms >/= 10 days.
>CT scan is indicated in pts who fail a course of antibiotic therapy for ARS to help
visualize air fluid levels and diagnose blockage of the osteomeatal complex.Its also
indicated in pts who initially present with ARS and have decreased vision, diplopia,
periorbital edema,severe headache or altered mental status.
>Plain X rays not routinely indicated becx they are less sensistive than CT scan in
delineating the bony and soft tissue anatomy and detecting mucosal thickening of the
sinuses.
>Referral for surgery is only indicated for ARS if the pt has vision changes (e.g diplopia
and blurry vision ) or extra nasal complications ( e.g epidural abscess,meningtitis,brain
abscess and perioribital/orbital edema ).Surgery is also indicated in pts who fail
antibiotic therapy and develop chronic rhinosinsusitis.
>First line Rx: Amoxicillin-clavulanate.

56.Septic pulmonary embolism:


Is a well known complication of IV drug abuse.This may result from Septic
thrombophlebitis ( question stem may describe a painful subcutaneous mass in the

right cubital area ). and / or triscuspid endocarditis.Early tricuspid endocarditis is


difficult to diagnose becx heart murmurs are absent but blood cultures are positive.

57.Neonatal sepsis:
>Occurs at less than 28 days
>Most common causes are E.coli and GBS.
>Presents as fever,lassitude,poor appetite and indirect hyperbilirubinemia(jaundice).
>CBC may show leukocytosis.
>All febrile neonates should undergo following:
a.CBC
b.Blood culture
c.Urinalysis
d.Urine culture
e.CSF cell count
f.CSF culture
>RX:
Ampicillin + Gentamicin/Cefotaxime.
.Cefoxtaxime is preferred over Gentamicin in areas with high rates of gentamicin
resistance.
.Ceftriaxone and sulfonamides shd be avoided due to high risk of hyperbilirubinemia.

58.TMP-SMX shd be avoided in pts aged <6 weeks due to increased risk of
methemoglobinemia.

59.Pediatric pneumonia is characterized by fever, tachypnea, cough and adventitious


lung sounds.Severe cases may be accompanied by hypoxia and respiratory distress
(e.g grunting, nasal flaring, intercostal retractions ), esp in pts with hx of underlying
respiratory dx ( e.g asthma, chronic lung dx in premature infants ) or immunodefecny.
When the diagnosis is equivocal on examination , a 2 view (postero anterior and
lateral ) chest x ray showing infiltrates supports the diagnosis.

60.Community acquired pneumonia in toddlers with focal findings(lobar) is most


commonly caused by Strep. pneumoniae. The treatment of choice is high dosre oral
amoxicillin, as low-dose amoxicillin is less efficacious.

61.Azithromycin is the treatment of choice for atypical pneumonia caused by


Mycoplasma pneumonia in older children who are well appearing with bilateral
lung findings.

62.Ciprofloxacin is not recommended for pneumonia treatment due to poor lung


penetration.

63.Doxycycline contraindicated in children aged <8 years.

64.Recurrent Clostridium difficile infection: (CDI)


>recurrent in 25% of cases
>recurs in 1-3 weeks usually ( but can be 2-3 months )
>due to persistent spores of Clostridium difficile or impaired host immune response

>reinfection with same or different strain of Clostridium difficile.


>RX:
A.Initial Rx:Oral metronidazole
B.1st relapse:
a. Oral metronidazole for mild illness
b. Oral Vancomycin for severe illness( fever,WBC > 15,000/mm3 or Cr. >1.5x )
C.2nd relapse:
Oral Vancomycin 6-week pulse taper regimen.
D.Subsequent relapses:
a.Oral rifaximin followed by Oral Fidaxomicin.
b.Consider fecal microbiota transplant.
>Recurrent CDI shd be confirmed with stool studies for C.difficile ( assay for C.difficile
toxin testing or PCR for toxigenic strains of C.difficile ).
>CT scan of abdomen can help diagnose severe CDI complicated by toxic megacolon or
ileus.
>Colonoscopy can help diagnose pts with high clinical suspicion for CDI and negative
stool studies.

65.IV pentamidine adverse effects:


>Hyerkalemia
>Hypokalemia
>Hypocalcemia
>Hypoglycemia ->seizures -> finger stick blood glucose test
>Hyperglycemia

66.HBsAb titer of >10 mIU/mL is considered protective after HBV vaccination and not
lesser than that.

67.Post exposure prophylaxis consisting of Hepatitis B immunoglobulin and vaccination


shd be given to people who have had percutaneous or mucous membrane expsosure
to HBV and either have not been previously vaccinated to HBV or who did not mount
a satisfactory antibody response to the vaccine.

68.Pegylated interferon and Lamivudine -> chronic hepatitis B.

69.Why more UTIs in females ?


>Shorter urethra
>proximity of urethra to vagina and rectum
>decreases urethral resistance during pregnancy and menopause.
>Sexual intercourse would increase the frequency

70.Recurrent UTIs vs Relapsing UTIs:


>Recurrent ;
a.Different strains infecting subsequently
b.Same strain infecting >2 weeks after completion of antibiotic treatment
>Relapsing;
Same strain infecting within 2 weeks of antibiotic treatment completion.

71.When is prophylaxis indicated for recurrent UTIs ?


> 2 infections in a period of 6 months
>prophylaxis is used for a period of 6-12 months.
>urine culture shd be done before and after antibiotic treatment in pts with recurrent
UTIs.One shd ensure that the urine culture is negative before starting prophylactic
antibiotics to prevent recurrent UTIs.The choice of antibiotics shd be based on the
susceptibility of the strains that caused the previous UTIs.

72.How to prevent recurrent UTI secondary to sexual intercourse?


>Single dose of antibiotic immediately after coitus.
>Voiding after intercourse
>Drinking water after intercourse to augment diuresis.

73.HIV can be associated with defeciencies of all cell lines.

74.HIV associated thrombocytopenia : ( HIV-TP )


>Due to immune dysfunction and viral destruction of megakaryocytes.
>Rarely associated with bleeding ( <5% of pts have platelets <50,000.mm3)
>Associated with splenomegaly.
>can occur at any stage of the disease.
>Antiretroviral therapy is the mainstay treatment.

75.Anti-retroviral therapy effects: (ART)


A.Virologic failure is defined as failure to achieve a viral load <200 copies/mL within

24 weeks ( 6 months ) of anti-retroviral therapy (ART) and may be due to drug


resistance or non-compliance.
B.Following ART inititation, Viral load shd be measure approximately every 3-6 months
C.Pts who have a better baseline immune function (i.e higher CD4 cell counts and
lower viral load ) appear to have the best response , often achieving extremely low
viral load nadirs rapidly after ART initiation.
D.In general , Viral load in treatment nave pts initiated on ART is expected to derease
to the following:
a.<5,000 copies by 4 weeks
b.<500 copies by 8-16 weeks
c.<50 copies by 16-24 weeks

76.Urethritis in men:
>dysuria and urethral discharge
>sexual transmission
>Gonorrhea is the most common cause
>Chlamydia is the most common cause of non-gonococcal urethritis.
>Urtheral swab results:
a. Presence of neutrophils->its bacterial cause.
b.Gram negative diplococci-> gonococcal-> give ceftriaxone + Azithro ( 30%
of pts with gonorrhea have co-existing chlamydia, so give Azithro to cover
Chlamydia as well.
c.Only neutrophils->its non-gonococcal->chlamydia is MCC->give Azithromycin
OR Doxycycline.

Note: Azithromycin covers many of the organisms that cause


non-gonococcal urethritis including chlamydia.
>Nucleic acid amplification testing for gonorrhea and chlamydia can be done as an
alternative to urethral swab.
>PCR has 88% sensitivity and 99% specificity in diagnosing gonococcal and chlamydial
infections.
>In cases of non-gonococcal urethritis unresponsive to Azithromycin, noncompliance and repeated sexual exposures shd always be considered.
>Microbiologic causes of nongonococcal urethritis other than chlamydia include
Trichomonas vaginalis, Mycoplasma genitalium and Ureaplasma urealyticum.
Azithromycin provides adequate coverage for Mycoplasma genitalium and
Ureaplasma urealyticum, so in such cases Trichomonas vaginalis shd be strongly
considered.Metronidazole would be the treatment of choice in such situations.
>Purulent discharge in cases of gonorrhea.
>Watery urethral discharge in cases of Chlamydia and trichomonas.

77.Ofloxacin->epidydimits due to enteric organisms.

78.There is little to no risk to the fetus if the mother contracts lyme disesease during
pregnancy if it is appropriately treated.Lyme disease can be spread by a tick vector ,
there is no possibility of spread between humans.

79.While doxycycline is the usual first line agent for treating lyme diseasae,
in pregnancy we use Amoxicillin or Cefuroxime instead becx doxycycline is
contraindicated.

80.Repeated hospital admissions due to polymicrobial bacteremia in a healthcare


professional should raise your suspicion for a factitious disorder.

81.Bacterial endocarditis can cause persistent bacteremia but is typically associated


with similar organisms.It is rarely polymicrobial.

82.HIV transmission:
>Almost all cases of occupational transmission of HIV have been due to transmission
via exposure to blood and certain body fluids.
>the body fluids wherein standard precautions have been recommended include
semen,vaginal secretions and any other body fluid containing visible blood.
>Other standard precautions, according to CDC, also apply to Cerebrospinal,peritoneal
,pleural,pericardial,synovial fluid or any other tissue, even though the epidemiologic
date regarding the risk of HIV transmission from these fluids is insufficient.
>Standard precautions do not apply to urine,sweat,tears,sputum,vomitus and nasal
secretions or feces, as long as there is no gross visible blood.

83.Cryptococcosis:
>Occurs in immunocompromised and HIV+ve pts with low CD4 counts
>Most common manifestation of disseminated cryptococcosis is the infection of CNS

causing meningitis or encephalitis.


>Systemic spread of the infection can involve any organ, with skin being the most
common extra neural site of infection.
>Cutaneous cryptococcosis can be the first presenting sign of developing systemic
disease in high risk individuals.The lesions are usually multiple,discrete, flesh to
red-colored papules of varying sizes with slight central umbilication, often
resembling the skin lesions of molluscum contagiosum.
>The lesions are usually seen over the face and/or trunk but can be seen on any part
of the body.The significance of the lesions is for the early diagnosis of disseminated
cryptococcosis.Once a diagnosis of cutaneous crytpococcosis is made,it is mandatry
to look for systemic involvement with chest radiographs , blood and CSF cultures,
India ink stain for CSF as well as crytpococcal antigens in serum and CSF.
>Histopathologic examination of biopsy specimen from the skin lesions is necessary
to confirm the diagnosis of cutaneous cryptococcosis.Histopathologic examination
usually reveals a granulomatous inflammatory reaction with multinucleated giant
cells , histiocytes , lymphocytes , neutrophils and plasma cells with numerous yeastlike organisms present both in free spaces and within the hsitocytes and giant cells.
Special stains like periodic acid Schiff stain and Gomoris methenamine silver nitrate
are commonly employed to identify the organism in biopsy specimen.Microscopic
examination of skin scrapings is not a reliabel way to confirm the diagnosis.It does
not provide enough tissue samples required for a histopathologic examination.
>Antigen testing and India ink preparation is useful for identifying the organism in CSF
specimens.These are not useful in pts with cutaneous cryptococcosis.

84.Pyoderma is a generalized term that refers to any inflammatory pustular skin lesions
with eventual necrosis and ulceration.

85.The most common form of cutaneous tuberculosis, lupus vulgaris, is a chronic and
progressive form of tuberculosis.The lesions are usually solitary and involve the skin
of face and neck.The typical lesions are small marginated,reddish brown papules wth
a gelatinous consistency ( apple-jelly nodules ) that slowly progress by peripheral
extension and have central atrophy.

86.Cat and Dog bites should be treated prophylactically with amoxicillin/clavulanate


for a period of 5 days presuming the wound remains uninfected.

87.If the animal who attacked is already vaccinated with rabies, then we do not give
rabies vaccine to the person.We just put the animal under observation.

88.Tetanus toxoid must be given to individuals with dirty wounds who have received a
booster >5 years ago and to individuals with clean wounds who have received a
booster >10 years ago.A second dose of tetanus toxoid must be given in one
month and a third dose in twelve months.
Tetanus immune globulin shd be given to any individual with a dirty wound and an
unclear or insufficient immunization history.

89.NOROVIRUS:
>Norovirus is the most common cause of gastroenteritis, including

epidemic gastroenteritis , in adults and children. Vomiting is more prominent.


>Spread via fecal-oral contamination and the virus can be transmitted from food
and water,fomites,or airborne droplets from the vomitus.
>Outbreaks are common and can occur in restaurants,cruise ships and
institutionalized settings such as nursing homes and health care facilities.
>Can cause the following:
a.Asymptomatic infection
b.Fever with watery diarrhea-non inflammatory small bowel process
c.Severe illness-fever,vomiting,headache and other systemic symptoms
>Symptoms usually start suddenly and last 48-72 Hrs.
>Diagnosis is generally clinical and most pts improve with supportive care.If needed,
diagnosis can be confirmed by polymerase chain reaction or nucleic acid based
testing.
>stool cultures are negative.

90.Campylobacter infection most commonly presents with acute onset of cramping


abdominal pain and inflammatory diarrhea (e.g mucus,blood).Diagnosis is by stool
culture.

91.Clostridium perfringens is typically associated with traumatic gas gangrene but is also
an important cause of watery diarrhea.The spores of C.perfringens germinate in
warm food;once ingested , the bacteria produce a toxin in the GIT that causes
disease.

92.Enterotoxigenic E.Coli is the most common causative agent of travellers diarrhea but
is much less frequently a cause of diarrhea in outbreaks.It classically presents with
malaise, anorexia and abdominal cramps followed by the acute onset of watery
diarrhea.

93.Listeria monocytogenes is a rare cause of epidemic gastroenteritis (usually associated


with deli meats and soft cheeses ). In addition to watery diarrhea ,fever and nausea/
vomiting , Listeria monocytogenes can also present with non-GI symptoms such as
myalgias,arthralgias and headache.Diagnosis can be confirmed from special stool
culture media.

94.Vomiting predominant:
>Staph aureus
>Bacillus cereus
>Noroviruses (e.g Norwalk )

95.Watery diarrhea predominant:


>Clostridium perfringens
>Enterotoxic E.Coli
>Enteric viruses
>Cryptosporidum
>Cyclospora
>Intestinal tapeworms

96.Inflammatory diarrhea predominant:


>Salmonella ( both typhi and non typhi )
>Campylobacter
>Shiga toxin producing E.coli
>Shigella
>Enterobacter
>Vibrip parahemolyticus
>Yersinia

97.Non GI symptoms:
>Descending paralysis->Botulism
>Paresthesias->Ciguateria toxin
>Flushing,urticaria->Scromboid
>Meningitis->Listeria
>Cellulitis,sepsis->Vibrio vulnificus
>Jaundice->hepatitis A
>Fever,arthralgias-> Brucellosis

98.HIV:
>Obtain a detailed sexual history in all young pts who present with weight loss and
non specific complaints.
>The index of suspicion of HIV shd be high in any young pt who presents with nonspecific or vague symptoms and a significant , unintentional weight loss in the past
few weeks.

>New onset seborrheic dermatitis (erythematous and pruritic rash on the face and
axilla ) is commonly seen in HIV infected pts and can be one of the presenting
complaints of the disease.

99.Rubella: ( German measles )


>Low grade fever ( very high grade fever in measles in contrast )
>Tender lymphadenopathy involving the posterior cervical and occipital noes.
>Pink maculopapular rash that begins on the face and and spreads caudally.The rash is
similar in appearance to Measles but pts are usually much less sick at presentation.
Spares the palms and soles.
>Some pts may have forscheimer spots ( patchy erythema of soft palate )
>Non specific symptoms ( e.g coryza,conjunctivitis,malaise,anorexia ) may also be
noted.
>Potential complication include an acute arthritis that typically resolves within several
weeks, thrombocytopenia and encephalitis.

100.Rubeola aka Measles is a more severe illness characterized by three Cs ( cough,


coryza,conjunctivitis).Fever and photophobia are also common.The blue white
Kopliks spots found on the buccal mucosa precedes the appearance of the maculo
popular rash , which starts on the face and spreads caudally to the trunk and
extremeties.

101.Roseola is a mild illness characterized by a high fever that rapidly resolves.The fever
is followed by the eruption of a characteristic rosy nonpruritic rash originating on

the trunk and spreading to the extremeties.

102.Rocky mountain spotted fever is a rickettsial dx transmitted by a tick bite.It is


characterized by fever,myalgias,headache and a petechial rash.Clasically,the rash
first involves the distal extremities (especially the palms and soles ) and subsquntly
spreads to involve the trunk.

103.Lyme disease is characterized into three stages:


A.Localized dx;
Symptoms include erythema migrans,lymphadenopathy,fever,malaise,myalgias,
or arthralgias.
B.Disseminated dx;
Symptoms include secondary annular lesions,meningitis,carditis,neuritis,AV nodal
block or migratory musculoskeletal pain.
C.Persistent dx;
Symptoms include intermittent or chronic arthritis,encephalopathy,or
polyneuropathy or acrodermatitis.

104.Streptococcal pharyngitis:
>caused by Group-A streptococcus.
>Typical s/s are fever,sore throat,tonsillar exudates and tender anterior cervical
lymphadenopathy.Palatal petechiae can be present.Abdominal pain/vomiting can
occur.
>There is absence of cough and viral symptoms.

>First test is Rapid streptococcal antigen test (RSAT);


+ve->treat right away.
-ve->confirm with throat culture
Note: RSAT is specific but not sensitive, so the negative test does not exclude the dx
and shd be confirmed with throat culture and a positive test means disease is
present,just treat it(hence specific).
>Penicillin and Amoxicillin are the first line treatments.They reduce symptom
duration and severity, decrease the spread to close contacts and prevent acute
rheumatic fever.
>Complications are Peritonsillar abscess,cervical lymphadenitis,rheumatic fever
and post-streptococcal glomerulonephritis.

105.Viral pharyngitis typically presents with upper respiratory symptoms (e.g cough,
rhinorrhea ).
Rx:
a.NSAIDS
b.Hydration.

106.Asymptomatic bacteriuria :
> Greater than 10^5 colony forming units in a pt without s/s of UTI.
>Common in elderly pts and typically resolves over time w/o the need for oral
antibiotic therapy.

>Screening and treatment is recommended in;


a.Pregnant women
b.Pts undergoing urologic procedures in which mucosal bleeding is possible
c.Pts undergoing hip arthroplasty in which long-term catheter in anticipated.
>Antibiotic therapy is not recommended in;
a.Non pregnant premenopausal women.
b.Elderly
c.Diabetics
d.Pts with spinal cord injuries.
e.Pts with chronic indwelling urinary catheters.

107.When a health care worker (HCW) is exposed to a pt with a contagious form of


tuberculosis ( ie. laryngeal,bronchial or pulmonary) , the CDC recommends
immediate PPD testing to determine the baseline immunologic status; three
months after, PPD testing will be given to check for any changes due to the recent
TB exposure.

108.Tuberculosis in pregnancy:
>Pregnant women stricken with tuberculosis not likely to be drug-resistant shd be
treated with Isoniazid (INH) , rifampin and ethambutol for a period of nine months.
>Since there is a paucity of data regarding the teratogenicity of pyrazinamide,it shd
not be used in pregnant women unless there is strong evidence that multi-drug
resistant tuberculosis is present i.e MDR-TB ( e.g previous treatment of TB or
exposure to a known case of MDR-TB).

109.Otitis media:
>The most common complication after an episode of acute otitis media is another
episode of otitis media.Children who have had more than 2 episodes are especially
at risk.
>There can be other serious complications as well such as bacterial meningitis,
mastoiditis ( mostly inflammatory but infectious is on the rise now becox of practice
of avoiding antibiotic therapy ) , lateral sinus thrombosis, epidural or brain abscesses
and even cavernous sinus and carotid artery thrombosis; however,the advent of
antibiotic therapy has made these complications rare.
>Contrary to the normal belief, pneumonia is not a complication of acute otitis media.
Both conditions can coexist but one is not a risk factor for other.

110.Tuberculosis:
>Tuberculin skin testing shd be initially performed in all persons exposed to pts with
tuberculosis.
>Once infected, a persons body mounts a cell mediated immune response which is
detected by a tuberculin skin test.This is why the first step in evaluating an
asymptomatic person with recent significant exposure to tuberculosis pt is to obtain
a tuberculin skin test.If the initial test is negative, a second test shd be performed
10 weeks after the last known exposure.
>INH Rx for ;
a.Positive PPD pts.

b.Tuberculin skin test conversion (atleast 10 mm increase in skin reaction within


a two year period).
111.Pseudomonas aeruginosa bacteremia:
>Usually causes infections in immunocompromised
>In immunocompetent,it causes infection in pts with central venous or urinary
catheterization or following infections of traumatic or surgical wounds.
>It can cause perivascular invasion of media and adventitia of arteries and veins and
lead to ecthyema gangrenosum that is characterized by nodular patches marked by
hemorrhage,ulceration and necrosis.The lesion is described as black necrotic ulcer
surrounded by an erythematous rim.Although ecthyma gangrenosum is not
pathognomonic of infection with P.aeuginosa,its presence increases the likelihood
of Pseudomonas as a causative agent.
>Mortality is higher in pts with Pseudomonas bacteremia and concurrent pulmonary
infection.
>Rx:
a.Treat with two drug regimen rather than monotherapy to avoid antibiotic restnc
b.Popular two drug regimens include an aminoglycoside ( e.g tobramycin or
amikacin ) and an extended spectrum anti-pseudomonal penicillin(eg piperacilin)
or an an anti-pseudomonal cephalosporin (e.g cefepime or ceftazidime)
c. Neutropenics=>treat for 14 days until neutrophils count returns to normal
d.Catheter infection=>stop after 7-10 days once catheter has been removed.
e.The antibiotics shd be given IV.
f.Surgical debridement is not recommended for ecthyema gangrenosum.

112.Cellulits->caused by strep pyogenes or staph aureus.

113.Impetigo:
>bullous=> caused by staph aureus
>non-bullous=>caused by staph aureus,strep pyogenes or combination of two.

114.Pyoderma gangrenosum:
>Classical=>Found on legs and is characterized by deep ulcers with violaceous border
>Atypical=>Found on hands, arms and face, and are superficial ulcerations that have
violaceous borders.

115. HCV:
>It is important to maintain a high degree of suspicion in pts with risk factors.
>Acute HCV infection is frequently asymptomatic but can present with malaise,nausea
,RUQ pain and abnormal liver function test results.
>Lab studies usually show aminotransferases that can be 10-20 times the upper limit.
>Most pts develop anti-HCV antibodies 2-6 months after exposure while HCV RNA
can generally be detected within days to 8 weeks following viral exposure, so
diagnosis is usually confirmed with PCR for HCV RNA.
>Resolved infection=>Normal aminotransferases,-ve HCV RNA,+ve Anti-HCV antibody.

116.Giardiasis:
>Transmission;
a.Person to person transmission thru fecal-oral or anal intercourse.

b.Food borne thru contaminated water or food.


c.outbreaks in institutional setting with poor hand hygiene or fecal incontinence
(e.g children in day care facilities ).
>Asymptomatic pts shed cysts for upto 6 months.
>Symptomatic pts;
a.Can develop acute diarrhea,steatorrhea,flatulence,and weight loss
b.Chronic giardiasis can develop despite the treatment with symptoms including
steatorrhea,profound weight loss and fatigue.
>Treat the symptomatic pts with metronidazole/tinidazole/nitazoxanide and report
the outbreaks to the health department.
>Do not treat the asymptomatic carriers.Exceptions include food service workers,
children in day care facilities that have additional new cases despite infection contrl
measures or family members of symptomatic pts who were likely infected from
fecal-oral transmission. Isolating the asymptomatic carriers is not recommended as
it does not affect the disease transmission.
>Preventive measures include infection control ( e.g contact isolation for pts in
diapers,hand washing,diaper disposal ) and water treatment ( e.g boiling,iodine for
hikers). Symptomatic children shd also be kept from attending day care facilities.

117.Lyme disease:
>Vermont is notorious for lyme disease
>Symptoms of early localized lyme disease can resemble a viral syndrome and include
headache,fatigue,malaise,myalgias and fever.Erythema migrans presents as an
expanding painless lesion with central clearing.

>The diagnosis of early localized lyme disease can be made on clinical grounds given a
suggestive history and erythema migrans rash, as most pts will not have had serologc
conversion by this stage in the disease course.
>Serologic testing can be considered for confirmation of diagnosis in unclear cases.
The recommended strategy for serologic testing includes an initial ELISA , with a
Western blot for confirmation if ELISA is +ve or equivocal.
>Rx:
a.Oral doxycycline , amoxicillin or cefuroxime can be used to treat early localized dx
b.Intravenous ceftriaxone is required if there is carditis or neurologic symptoms
other than a cranial nerve palsy.

118.Osteomyelitis:
>Probe-to-bone test is considered highly suggestive,if not diagnostic.
>Once osteomyelitis suspected,though it may take 2 weeks to show up on
imaging,MRI is the fist line test.MRI would show osseous lucency and periosteal rxn.
>If pt is unable to have MRI due to some reasons,CT scan and three phone bone scans
can be considered.CT scan would show osseous erosion,periosteal reaction,sinus
sinus tracts and osseous sequestra.Three phase bone scans can show a false positive
in diabetics due to fractures or charcot arthropathy which would also demonstrate
increased radiotracer uptake.
>Bone biopsy is frequently used if there are positive findings of osteomyelitis on
imaging in order to identify the infectious agent and guide therapy.
>Intravenous antibiotics are used for a period of 6 weeks due to limited ability of
antibiotics to penetrate bone.

119.HBV vaccination:
>For new borns and school age children.
>All HIV+ve pts shd be vaccinated against Hepatitis-B
>Members of various high risk groups ( people with multiple sexual partners,
homosexual or bisexual males, IV drug abusers,healthcare workers,pts requiring
hemodialysis or repeat blood transfusions and household contacts of Hepatitis B
carriers)

120.HIV:
>Pts with CD4<200 => prophylaxis with TMP-SMX shd be started.
>Pts with CD4<50=> prophylaxis with clarithromycin/azithromycin shd be started.
>Ganciclovir prophylaxis for pts with CD4<50 is effective prophylaxis , however its use
its use is not widely recommended for several reasons,including the finding that
those pts taking ganciclovir have no increased survival advantage.
>HAART is not only credited with curtailing the development of AIDS and opportunistic
infections but it has also been shown to dramatically reduce the mortality in this
population as well.

121.Cats bite:
>Cats bite gets infected most of the time i.e in 80% cases.Furthermore , the lesion is
usually deeper than a dog bite, becx cats teeth are smaller and sharper.
>Although Pasteurella multocida is the major causative organism transmitted by the

cats bite, the resulting infection is often polymicrobial.

>Rx:
a.Amoxicillin/clavulanic acid is the most recommended prophylactic oral therapy in
both children and adults.It covers both aerobic and anaerobic organisms.
Administered for 3-5 days
b.Doxycycline can be used as an alternative in penicillin allergic pts.( adults, not
children ).
c.Alternate antibiotic regimen includes an agent that covers pasteurella(e.gTMP-SMX)
plus an agent with anaerobic coverage (e.g clindamycin).
c.More ill pts can be treated with an intravenous ampicillin-sulbactam combination.
d.Although the wound shd be cleaned with povidine iodine or benzalkonium
chloride and copiously irrigated with saline,they shd not be closed.
Closed wound significantly increases the infection risk.Bite wounds <24 hours old
on the face w/o clinical signs of infection can be closed.Antibiotic prophylaxis shd
be given for all wounds closed primarily.

122.The need for rabies prophylaxis is based on the type of animal bite and whether the
animal is available for observation.For cat or dog bites,if the animal is healthy and
available for a 10-day observation period,rabies prophylaxis is not required.
Rabies vaccine and rabies immune globulin shd be administered in pts with high risk
wounds from or exposure to bats,raccoons and skunks.

123.Rabies prophylaxis
A.High risk animals=>raccoon,skunk,fox,coyote,bat
>Start PEP if unable to test animal.
>If able to test animal,Start PEP if rabies test is +ve.
B.Low risk wild animal=>squirrel,chipmunk,mouse/rat,rabbit
>No PEP.
C.Dog,cat or ferret.
>Available for quarantine ?
a.yes->observe for 10 days and no PEP if animal is healthy
b.No->Test animal if possible.Start PEP and discontinue if rabies test is -ve.
D.Live stock or unknown wild animal ?
Contact public health dept.

124.Necrotizing fasciitis:
>Fulminant infection of subcutaneous tissue that spreads rapidly along the fascial
planes and leads to extensive tissue necrosis and shock.
>Look for erythema and swelling of skin.
>Look for fever and hypotension.
>Pain out of proportion to examination findings and tenderness.
>There is often antecedent hx of minor trauma.
>Type-I:
Usually seen in pts with underlying diabetes and peripheral vascular disease.It is

generally a polymicrobial infection;some commonly isolated organisms include


Staph aureus,Bacteroides fragilis,E.Coli,group A strep and prevotella species.
Crepitus is more common if anaerobic organisms such as clostridium perfringens or
B.fragilis are involved.
>Type-II:
Most common form.Usually occurs in individuals with no concurrent illness.Many
patients report a history of laceration,blunt trauma or a surgical procedure as a
predisposing factor.It is typically caused by group A streptococcus (strep pyogenes).
>Necrotizing fasciitis most commonly involves the extremeties and perineal region.
>Rx:
a.Urgent,aggressive Surgical exploration and debridement of the involved tissue.
These measures also help to ascertain the diagnosis by providing tissue samples
for diagnostic purposes.
b.Hemodynamic support shd be provided
c.Until the etiology is unknown, broad spectrum antibiotic therapy shd be started.
1.Piperacillin/tazobactam or a carbapenem (e.g iminpenem,meropenem) will
cover group-A strep and anaerobes.
2.Vancomycin will cover staph auereu including MRSA.
3.Clindamycin is added to inhibit toxin formation by streptococci/Staphylococci
P.S: A plain radiograph and CT scan may be useful in detecting the presence of gas in
the subcutaneous tissue.However therapy shd not be delayed to obtain imaging
or lab studies.

125.C.perfringens->Necrotizing fasciitis->crepitus on examination

126.Pseudomonas->Necrotizing fasciitis in immunocompromised.

127.S.epidermidis->Central line-associated bloodstream infections.

128.Malaria:
>Malaria shd be considered as a differential diagnosis for fever in travelers returning
from sub-saharan Africa , including those who did not complete an appropriate
prophylaxis course.
>Look for fever,headache,chills,myalgias.
>GI complaints are always present ( abdominal discomfort,nausea,vomiting)
>Look for splenomegaly
>Anemia might be there secondary to hemolysis
>Look for thrombocytopenia.
>Look for diaphoresis.
>Peripheral blood smear is useful in establishing the diagnosis.
>Prophylaxis
A.Areas with chloroquine resistant P.falciparum ( Sub Saharan Africa,Southern and
south east Asia )
a.Atovaquone-proguanil=> .expensive
.GI disturbances ( e.g abdominal pain)
.Increased liver function tests

b.Doxycycline=> .Inexpensive

.Photosensitivity
.GI disturbances
.Teratogenic

c.Mefloquine=> .Neuopsychiatric problems


.Agent of choice in pregnancy
.Weekly dosing

B.Areas with chloroquine susceptible P.falciparum:


a.Chloroquine
=>
b.Hydroxychlorquine

.Need to be started 1-2 weeks in advance


.Potential exacerbation of some skin conditions
.Weekly dosing

c.All other drugs listed above can also be used.

C.Areas w/o P.falciparum ( parts of south ameirca,Mexico,Korean peninsula )


Primaquine=> .Potential teratogenicity
.Hemolysis in G-6-PD deficiency
.Weekly dosing

129.Differential diagnosis of fever in a returning traveler:


A. <10 days => .Typhoid fever
.Dengue fever
.Chikungunya

.Influenza
.Legionellosis
B.1-3 weeks => .Malaria
.Typhoid fever
.Leptospirosis
.Schistosomiasis
.Rickettsial dx
C.>3 weeks => .TB
.Leishmaniasis
.Enteric parasitic infections.

130.Early Neurosyphilis:
>Occurs most commonly during the secondary stage of syphilis ( secondary syphilis
manifests as generalized maculopapular rash and lymphadenopathy )
>Manifests as;
a.Symptomatic meningitis: presents as headache,confusion,photophobia,Nausea/
Vomiting,stiff neck.Pts are usually not as sick as those of
bacterial meningitis and usually have a prodrome of
several days preceding presentation of meningtic
symptoms.A CSF-VDRL test is almost universally reactive
b.Ocualar syphilis: Can present as impaired vision,eye irritation,posterior Uveitis
(floaters),retinitis ,or optic disc neuritis.
c.Otosyphilis:Can present as tinnitus and hearing loss
d.Meningovascular: Infectious CNS arteritis causing ischemia and/or infarction.

e.Cranial neuropathies,esp of the optic , facial or auditory nerves can occur.

131.Early neurosyphilis affects the CSF,meninges, and vasculature.While late forms


more commonly cause disease of the spinal cord,and brain parenchyma.

132.Late Neurosyphilis ( teritay syphilis )


>General paresis=> progressive dementia
>Tabes dorsalis=> .Posterior column/dorsal root dx
.Pupillary defects,diffuse neurologic signs
.May have normal CSF

133.Pregnant women shd not receive live virus vaccines , but their household members
shd get all routine immunizations on schedule.Delaying vaccines is not recomended
as vaccines make it less likely that children will contact the disease and expose
pregnant women to wild-type infection.

134.Penicillin desensitization is considered to be the treatment of choice for pregnant


penicillin-allergic pts with syphilis.

135.Acute retroviral syndrome:


>Occurs 2-3 weeks after exposure to the virus and precedes seroconversion by 10-21
days.
>Fever
>Fatigue

>Arthralgias
>Myalgias
>Maculopapular rash on face,trunk and extremeties
>Oral ulcers,thrush
>Pharyngitis
>Lymphadenopathy
>Night sweats
>headache
>GI symptoms e.g nausea,vomiting,diarrhea
>Thrombocytopenia and leukopenia may occur
>HIV ELISA testing is often negative initially and can be supplemented by testing for
HIV RNA or p24 antigen if acute retroviral syndrome is suspected.

136.Classic triad of infectious mononucleosis:


>Moderate to high grade fever
>Tonsillar pharyngitis
>Lymphadenopathy

137.Viral hepatitis is often subclinical in its initial presentation, but has been known to
cause nausea,fatigue,low grade fever,right upper quadrant pain,myalgias,
arthralgias, and urticaria. Jaundice typically appears later.Anemia & leukocytosis
may also occur.

138.Central line associated blood stream infection ( CLABSI ) :

>Caused by coagulase -ve staph, staph aureus, candida spp. and aerobic gram -ve
bacilli
>Risk factors for infection include location ( lower > upper extremity,wrist > arm,
femoral>internal jugular ), duration > 1 week and site care ( e.g daily chlorhexidine
bathing reduces risk in ICU pts ).
>A comprehensive approach to CLABSI prevention include the following:
a.Checklist prior to catheter insertion.
b.Proper hand hygiene prior to handling catheter.
c.Avoiding femoral vein for central venous access.
d.Using all inclusive catheter kit.
e.Using maximal sterile barrier precautions ( e.g mask,sterile gown,drape and gloves )
f.Using chlorhexidine bases antiseptic for skin preparation
g.Ensuring proper catheter site care with catheter removal when no longer needed.
The most effective prevention is full body drape during catheter insertion to avoid
contamination as skin organisms most commonly cause CLASBI.
>Topical antibiotics applied to central catheters do not consistently reduce the infectn
rates but may actually increase the antibiotic resistance and candida colonization.
>Exchanging catheters through a guidewire shd be avoided as it increases the risk of
bloodstream infection.
>The femoral vein is the most common site of infection followed by internal jugular
vein and then the subclavian vein.However, the risk of pneumothorax is higher at the
subclavian site compared to the internal jugular.

139. Topical antibiotics application can reduce the infection rates and bacteremia in

hemodialysis catheters.

140.Ultrasound guidance for placing central lines can decrease the risk of mechanical
complications and number of placement attempts.

141.Subphrenic abscess:
>Abdominal ultrasound is the best diagnostic test for a suspected subphrenic abscess
or other abdominal abscess.
>Subphrenic abscess;
a.typically develops 14-21 days after abdominal surgery.
b.Swinging fevers
c.Leucocytosis with bandemia
d.Dry cough
e.Shoulder tip pain
f.Tenderness over 8-11th ribs.

142.HIV vaccines:
>Most of the vaccines recommended for general population are also given to HIV pts
>Live vaccines are generally contraindicated.
>MMR,varicella zoster and live attenuated influenza are contraindicated if the CD4
count is lesser than 200 or if the pt has AIDS defining illness,otherwise we can give
it to the pts.
>As with all adults, HIV pts shd receive Tdap followed by vaccination for Td every 10 yrs
>All HIV pts with negative immunity for HAV and HBV shd receive Hepatitis A and

Hepatitis B vaccines.
>Pts with HIV shd receive PCV 13 followed by PPSV 23 8 weeks later and then every
5 years.
>Annual influenza vaccine is also recommended in the fall for all adult patients.

143.The meningococcal vaccine is recommended for pts at high risk for invasive
meningococcal dx,including college students,military recruits, and those with
asplenia,complement deficiency, or who travel to regions where meningococcal
dx is endemic.

144.Tuberculosis diagnosis:
>Active TB is suspected based on the epidemiologic (e.g travel to endemic area,
previous TB) and clinical (e.g cough >2-3 weeks,fever,night sweats,weight loss).
>Pts with suspected active TB require chest imaging to document findings consistent
with pulmonary TB.
>Pts with abnormal imaging ( e.g infiltrate, cavitary lesion ) or clinically suspected TB
shd with normal imaging undergo sputum microscopy which is the easiest and
quickest method for confirming suspected active pulmonary TB.
>If a pt cannot produce sputum then sputum induction is done through aerosolized
inhaled hypertonic saline (3-15%) in negative pressure isolation rooms.
>Atleast 3 sputum samples at intervals of 8-24 Hrs are collected with atleast one early
morning specimen.
>Sputum smears are generated from the specimens and undergo acid fast staining
>Sputum cultures are ordered at the same time,which is the gold standard test

(81% sensitive and 98% specific) and they also help in determining the drug sensitvty
>Bronchoscopy with alveolar lavage is usually reserved for confirming the diagnosis
in pts unable to produce adequate expectorated or induced sputum samples,who
have negative sputum studies with high suspicion for active TB or who have possible
alternate diagnosis.
>Children cannot produce sputum and hence gastric aspirates are cultured for
Mycobacterium tuberculosis.
>PPD and IGRA cannot differentiate between latent and active TB. In those who have
been BCG vaccinated, IGRA can be utilized.

145.The decision to hospitalize a pt with suspected TB depends primarily on the clinical


circumstances and severity of infection.Infected pts can be managed as outpatients
if they are a low public health risk , can remain at home , avoid visitors and stay
away from family members.Hospitalization is recommended for pts who are a
public health risk with limited access to health care or resources ( e.g homeless pts )

146.Sputum microscopy for acid fast bacilli can serve as the surrogate marker for
infectivity for tuberculosis.Pts with 3 negative smears are considered noninfectious.
The positive predictive value of 3 sputum samples for acid fast bacilli ( AFB ) testing
can range as high as 90% for diagnosing tuberculosis. but AFB testing has low
sensitivity for ruling out TB.
meaning: . AFB smear testing is +ve-> disease is there.
. AFB smear is -ve-> becox of low sensitivity, we have to move forward
in high suspicion cases and diagnose with culture.

But YES before further results are available we can


say pt is -ve for now.but like the sensitivity is low,
so we have to move forward and do further
confirmation.

147.Chlamydia pneumonia:
>Staccato cough ( A staccato cough is essentially a cough that comes as a series of outbursts with
time for at least one breath in between each one. People cough once, then pause,
then cough again, often over and over again )

>Between 2-19 weeks age


>Auscultatory and radiologic findins out of proportion to the healthy appearance of
the child.
>Chest X ray shows hyperinflation, peribronchial thickening and bilateral,symmetrical
interstitial infiltrates.
>WBC count is usually normal but eosinophil count is elevated.
>Diagnosis can be confirmed by tissue culture isolation of the organism from the
nasopharyngeal specimens,direct fluorescence antibody tests, enzyme linked
immunoassays or polymerase chain reaction.

148.Maternal fever during last trimester is common in listeria pneumonia.

149.Progressive multifocal encephalopathy ( PML ) :


>Multiple focal neurologic deficits in an immunocompromised pt ( i.e pt wid AIDS )
suggest a diagnosis of PML and this diagnosis is best confirmed with MRI.
>Classic MRI findings in PML consist of multiple demyeilinating, non enhancing

lesions with no mass effects.


>Causes by JC virus, a human polyoma virus, which most commonly affects cortical
white matter but the brainstem and cerebellum may also be involved.
>Mode of transmission is unknown.
>Most common presenting symptoms are hemiparesis and disturbances in speech,
vision and gait. Cranial nerve deficits may occasionally develop.
>No specific treatment, just start HAART therapy if pt is AIDS pt.

150.HIV encephalopathy usually presents with dementia as the predominant symptom.


MRI findings might be similar to PML but these usually tend to be symmetrical.

151.PPD:
>PPD testing shd be promptly performed on individuals exposed to active infectious
tuberculosis.If the results are negative,PPD test shd be repeated in three months.
>In people with positive PPD results, a chest x ray shd be taken to check for signs of
infection; if positive,anti-tuberculous treatment shd be instituted.Public health
authorities shd be notified if active infectious tuberculosis is revealed.

152.Right sided murmurs accentuate with inspiration

153.A vast majority of pts with right sided infective endocarditis are IV drug users.

154.Infection of unimmunized pregnant woman during the 1st trimester , as congenital


rubella syndrome, can be devastating to the fetus.

155. Measles rash => Darker ( brick red )


Rubella rash=> Lighter.

156.Measles=> high grade temperature, darker rash, no lymphadenopathy.


Rubella=> lighter rash,low grade temperature,lymphadenopathy.

157.Rickettsial dx:
fever, headache and rash.

158.Varicella ( chickenpox ) is characterized by a low grade fever, malaise and a rash


that appears in crops that progresses through several stages,including papules,
vesicles,pustules,and crusting.The rash can be distinguished by the variety of lesions
at different stages.

159.HIV in pregnancy:
>HAART is a combination of 2 nucleoside reverse transcriptase inhibitors (NRTIs)
plus one protease inhibitor OR one non-nucleoside reverse transcriptase
inhibitor ( NNRTI )
>Triple antiretroviral therapy reduces the prinatal HIV transmission from ~25% to ~2%.
>Zidovudine/lamivudine is the first line dual NRTI therapy due to its extensive safety
profile and long history of pregnancy use.
>Efavirenz is the preferred NNRTI after the eighth week of gestation.Before 8 weeks
gestation ( primary period of organogenesis ) , women who are not on efavirenz shd
not begin this medication due to the risk of neural tube defects,facial clefts and

anopthalmia.However no changes shd be made to an effective efavirenz containing


regimen as modifications can compromise virological control and increase the risk of
perinatal transmission.The risk of perinatal HIV with anti retroviral switching
outweighs the risk of possible teratogenicity,especially because many pregnancies
are diagnosed after this period.
>Some treatment nave pts choose to start HAART after the first trimester but
initiation may be less effective in reducing perinatal HIV transmission.
>Pts shd be counselled that poor compliance is a significant risk factor for drug
resistance and viral failure ( HIV RNA >/= 50 copies at 24 weeks after HAART
initiation or any sustained HIV RNA >/= 50 copies/ml after initial suppression ).
>Although Nevirapine is an alternative NNRTI used during pregnancy,it is associated
life threatening hypersensitivity reaction in pts with CD4>250/ul.
>Pts with HIV viral load >1000 copies /ml shd undergo cesarean delivery to reduce
the vertical transmission to ~2%.Those who are compliant with HAART and have an
undetectable viral load can deliver vaginally with an equivalent transmission risk of
~2%.
BOTTOM LINE:
A pregnant women who is HIV+ve, and on highly active retroviral therapy
(HAART), shd continue her treatment regimen to maintain viral suppression
and reduce the risk of perinatal transmission.Discontinuing or modifying an
effective regimen can lead to viral failure and drug resistance.Pts on HAART
with an undetectable viral load can have a vaginal delivery.

160.HIV and Breast feeding :


A.In United States and other industrialized nations , where women have access to
clean water and commercially prepared formula, are advised to feed their infants
with formula instead of breast milk if they are HIV+ve.
B.In resource poor countries , formula feed is associated with increased infant
morbidity and mortality from diarrhea,pneumonia and other infectious diseases.
Breastfeeding is recommended in these countries due to overall decreased rates
of morbidity and mortality resulting from protective maternal antibodies and
avoidance of pathogens in dirty water.In addition, most antiretroviral drugs are
passed through breast milk in significant quantities and decrease the risk of
postnatal HIV transmission by passively providing the medication to the infant.
BOTTOM LINE:
Industrialized nations=> Forumula feed
Resource poor nations=>Breast milk

161.In all countries,the infant shd receive zidovudine prophylaxis for >/=6 weeks, while
the mother continues the 3-drug active antiretroviral therapy for viral suppression
indefinitely.

162.Acute otitis medie:


>limited mobility of tympanic membrane=>Effusion in middle ear
>Typmanic membrane inflammation=> Bulging of tympanic membrane
>Limited mobility + Bulging => Acute Otitis media.
>Other non specific signs of inflammation are tympanic membrane erythema,fever
and Otalgia.

>Most common organisms involved;


=>Strep pneumoniae
=>Non typeable Hemophilus influenza
=>Moraxella catarrhalis
>Recurrent cases of otitis media is not due to different organisms but its due to
resistance of these same organisms to the antibiotics because of tradition of
excessive prescription of antibiotics becx of overdiagnosis of acute otitis media.
>Concurrent purulent conjunctivitis + Otitis media => non typeable H.Influenzae.
>Uncomplicated acute otitis media shd be treated empirically with amoxicillin.
Recurrent AOM within the same month raise the concerns for beta-lactamase
resistance and warrants treatment with amoxicillin-clavulanic acid.Ototopical
medications are unnecessary, even if there is tympanic membrane perforation.
>Tympanocentesis with Gram stain,culture and antibiotic susceptibility testing
shd be considered in children with multiple treatment failures.However,this
procedure is invasive and unnecessary for most cases of AOM that resolves with
1 or 2 empiric antibiotic treatment courses.

163.H.influenzae type b -> Meningitis and bacteremia.We vaccinate against type b.

164.Pseudomonas aeruginosa is the most common pathogen responsible for acute


otitis externa (swimmers ear) which presents as inflammation of the external canal
and a normal tympanic membrane.

165.Ototopical antbitiotics are indicated for the treatment of Otitis externa.

166.When to hospitalize a hepatitis patient ?


>Significant fever
>Leucocytosis
>Hemodynamic instability
>biliary obstruction
>Impaired hepatic synthetic function ( e.g prolonged coagulation markers )
>fulminant hepatic failure ( e.g encephalopathy, significantly elevated bilirubin ).
>Older pts with comorbid conditions who have poor oral intake or are without
social support.
>Toxic hepatits pts ( e.g acetaminophen poisoning) who carry a significant risk of
acute deterioration , even in pts who initially appear stable.

167.Stable and otherwise healthy adults who present with jaundice can be evaluated
on an outpatient basis.

168.Patterns of different forms of hepatitis:


>Acute viral hepatitis has acute onset of symptoms w/o fulminant hepatic injury.
>Alcoholic hepatitis has moderately elevated transminases ( <300 IU/L ), AST > ALT
>Toxic and ischemic hepatitis associated with known injury ( e.g suicide attempt
and sepsis ) and causes fulminant hepatic failure with severe clinincal instability.
>Autoimmune hepatitis presents as a chronic rather than acute hepatitis.
>Liver injury due to herpes virus family viruses ( HSV,VZV,EBV,CMV ) is usually found
incidentally and presents as an acute mild hepatitis.

169.Budd chiari syndrome:


>Young and middle aged women
>Severe abdominal pain
>Ascites on Ultrasound (>90%cases)
>Thrombolytic therapy shd be given.

170.Liver biopsy is indicated for pts with unclear etiology of hepatic injury after initial
evaluations or for staging of chronic liver dx prior to treatment.

171.Hepatitis-B:
>Acute infection is characterized by positive HBsAg, HBeAg (an indicator of high
infectivity) and IgM anti-HBc, elevated ALT and detectable HBV DNA.
>Chronic Hepatitis B infection includes HBsAg with anti-HBe and IgG anti-HBc.
>Nearly 70% of acute Hepatitis B pts are asymptomatic.The remaining 30% develop
symptoms, including anorexia,nausea,jaundice,and RUQ discomfort.
>Antiviral therapy for acute Hepatitis-B is recommended for pts with concurrent
Hepatitis-C, immunosuppression or severe disease/fulminant hepatic failure.
>Most pts with acute Hepatits B can be managed with supportive measures and close
follow up.
>Liver transplantation is the only hope for pts with fulminant hepatic failure or chronic
end stage liver disease.
>Hepatits-B immunoglobulin + Hepatitis B vaccine is for PEP in healthcare workers
exposed to blood or other bodily fluids contaminated with Hepatitis-B

172.Interferon alfa-2b + ribavirin=> chronic hepatitis C.

173.Progression rate of Acute Hepatitis B to Chronic Hepatitis B:


>Perinatally acquired infection=> Progression rate is 90%.
>Pts aged 1-5 yrs age=> Progression rate is 20-50%.
>Adults=> </=5%.

174.Hepatitis C has an approximately 50% rate of progression to Chronic disease.

175.In both Hepatitis B and C, the severity of acute symptoms is inversely proportional
to the risk of chronic hepatitis.Pts with more severe illness are less likely to develop
chronic hepatitis.

176.Epiglottitis:
>Caused by H.influenzae type b, so vaccination reduces its risk.
>Inflammatory edema of the epiglottis occurs and leads to respiratory arrest.
>Muffled voice, toxic appearance of child.
>Pt tends to lean forward.
>Frequently diagnosed by clinical presentation.However, if a lateral neck radiograph is
obtained it wud reveal a swollen epiglottis ( Thumbprint sign ).
>Its a life threatening infection and treatment shd be focused on relieving any airway
obstruction and treating the infection.Threshold for performing endotracheal
intubation shd be very low.
>Laryngoscopy is contraindicated.

>Corticosteroids and racemic epinephrine have not been shown to be helpful.

177.Shistosomiasis:
>Microheamturia
>Anemia
>Immigration from endemic area ( e.g Africa )
>The most common method of diagnosing Shistosomiasis is demonstration of parasite
eggs in the stool or urine through stool/urine microscopy respectively.Its preferrabe
to collect urine samples for examination between 10 am and 2 pm, when egg
excretion is maximal.

178.Chemoprophylaxis for meningococcal meningitis:


>Close contacts would need it. >8 hours of exposure at <3 feet is a close contact;
Household members,roommates or intimate contacts,child care workers,
airline travelers seated next to affected pt during an extended flight and
those having had direct exposure to oral secretions (e.g kissing )
>Direct exposure to respiratory secretions within 7 days of onset of pts symptoms
>Health care workers do not need prophylactic antibiotics unless the workers are
directly exposed to respiratory secretions ( e.g mouth-to-mouth resuscitation,
endotracheal intubation ).
>Recommended regimens include:
=>Rifampin ( 600 mg orally twice daily for 2 days )
=>Ciprofloxacin ( 500 mg single oral dose )
=>Ceftriaxone (250 mg IM single dose )-> Can be used in pregnancy.

>Rifampin is preferred over Ciprofloxacin and ceftriaxone unless the pt cannot take
Rifampin ( e.g women taking OCPs becox Rifampin is enzyme inducer ).
>Chemoprophylaxis shd be ideally administered within 24 Hrs.Prophylaxis given
>14 days after exposure has limited efficacy and is not recommended.

179.Rabies prophylaxis:
A.Pre-exposure prophylaxis;
Persons whose work/travel involves likely exposure to rabies shd receive a
pre-exposure prophylaxis.
B.Post-exposure prophylaxis;
a.Those who have been previously vaccinated for rabies shd receive only active
immunization ( rabies vaccine ) and NOT passive immunization ( rabies IG )
as immune globulin blunts the immune response to the vaccine.Further, immune
globulin shd be avoided in previously vaccinated individuals as some amount of
antibody is still circulating.
b.Those w/o a hx of immunization require BOTH immune globulin and a full
vaccination series.

180.Trichomonas vaginalis:
>Affected pts have green frothy discharge,dysuria,dyspareunia,urinary frequency,
and/or vaginal pruritis.
>Vaginal and cervical petechiae ( Strawberry cervix ) are sometimes seen on speculum
exam.
>Trichomonas is a pear-shaped motile organism that can be seen on Pap smear and

wet mount.
>Affected women may or may not have symptoms.Males are almost always
asymptomatic.Regardless of symptomatic/asymptomatic, both male and female
partners shd be treated with oral metronidazole.

181.Prior BCG vaccination shd rarely cause more than 15 mm induration with PPD skin
testing.The effect of BCG vaccination on induration decreases significantly 15 years
after the vaccine is received.

182.Positive PPD-> next step is Chest X ray.

183.INH for 9 months is the recommended Rx for pts with latent TB.
Rifampin for 4 months is an alternative.

184.Tuberculous meningitits:
>All pts with tuberculous meningitis ( based on intitial hx and CSF examination ) shd be
immediately started on empiric antituberculous therapy , pending the results of
confirmatory tests.
>Drug-sensitive Tuberculosis;
12 months therapy.3 drug regimen of Rifampin,INH and Pyrazinamide (bactericidal
agents) for first 2 months with Rifampin and INH for remaining time. All 3 drugs have
good CSF penetration.
>Drug-resistant Tuberculosis;
18-24 months therapy.

>Adjunctive corticosteroids treatment in certain situations can significantly reduce the


neurological complications and mortality rates in pts with tuberculous meningitis;
however this does not affect the duration of antituberculous therapy.
>Intrathecal therapy for tuberculous meningitis is not recommended since all agents
have good CSF penetration.

185.Infants and children with tuberculous meningitis,military TB and tuberculous


osteomyeilitis shd receive 12 months of anti-tuberculous therapy.

186.Tuberculosis is transmitted by aerosols e.g coughing,sneezing,singing and even


speaking.Hence pts shd be kept in isolation unless they are confirmed to be
non-infectious.

187.Pts are considered non-infectious if they are clinically improving on effective


anti-tuberculous therapy and have three negative results on sputum smears on
different occasions.

188.Remember that cough and Chest x ray findings can persist even after the pts are
no longer infectious.So they are not reliable markers to be considered for noninfectiousness.

189.HIV lipodystrophy may present as lipoatrophy,fat accumulation or both in different


areas.A pattern with increased fat tissue deposition on the back of the neck and
abdomen along with thin extremeties and face is suggestive of HIV lipodystrophy.

Insulin resistance and dyslipidemia are closely interrelated with HIV lipodystrophy.
Antiretroviral therapy may also play a role in the development of HIV
lipodystrophy.
Insulin resistance is possibly because of altered secretion of hormones adiponectin
and leptin by abnormal adipocytes.The cause and effect relationship of insulin
resistance and HIV lipodystrophy is somewhat unclear.The diabetes medication
metformin and thiazolidenediones such rosiglitazone and pioglitazone may have
efficacy in treating HIV lipodystrophy , suggesting a link between the two conditions

190.Total cholesterol=> 150-24 mg/dL.

191.
A.HIV dyslipidemia=> Elevated triglycerides level, with low total cholesterol,low LDL
and low HDL.
B.Antiretroviral therapy=>Elevated triglycerides level, high total cholesterol, high LDL
and low HDL.

192.HIV infected pts have dyslipidemia, particularly hypertriglyceridemia, which can be


exacerbated by antiretroviral therapy.
Triglycerides < 500 mg/dL => Statins are the treatment.
Triglycerides >500 mg/dL => Fibrates are the treatment.

193.Nicotinic acid=> flushing, pruritis, hepatotoxicity.

194.Centor criteria for streptococcal pharyngitis:


>Tonsillar exudates
>Tender anterior cervical lymphadenopathy
>Fever
>Absence of cough
Strep antigen testing is indicated if a pt meets atleast two of these criteria, whereas no
diagnostic testing is indicated if a pt meets one or no criteria because in that case its
most likely a viral pharyngitis which is treated symptomatically.

195.In United States,PPD testing is presently being performed in all healthcare workers
prior to their employment.and if the PPD is negative,screening shd be repeated
annually to detect any tuberculin skin test conversions.

196.The interpretation of PPD is based on induration and NOT erythema.

197.Pts with prior Hx of BCG vaccination and positive PPD test present a clinical
challenge.In all such pts, the results of tuberculin skin testing shd be compared to
the baseline on the prior test.An increase in the skin test reactivity of greater than
10 mm ( in pts lesser than 35 years old ) or greater than 15 mm ( in pts older than
35 years age ) is considered a positive test for new infection.When baseline skin test
values are not available,the test results shd be interpreted as if the BCG vaccination
was never done.
BOTTOM LINE:
A prior hx of BCG vaccination shd not be presumed to be the cause

of a positive tuberculin skin test,especially in pts from countries


with a high prevalence of tuberculosis.

198.Isoniazid causes peripheral neuropathy by interference with the metabolism of


Vit-B6 (pyridoxine).

199.Health care workers with latent tuberculosis infection (LTBI) who refuse or do not
accept treatment for LTBI shd not be excluded from the workplace.They should be
counselled about the risk of developing active tuberculosis and instructed to report
for evaluation if they develop s/s of tuberculosis.They shd continue to have regular
follow up visits for symptom screening and offered treatment for LTBI at every
follow up visit.

200.The presence of diarrhea and eosinophilia can be seen in certain conditions (such as
intestinal parasitosis) due to Helminths , eosinophilic gastroenteritis or Addison dx.
Treatment of parasitic infections with roundworms( Ascaris Lumbricoides),
hookworms(Ancylostoma duodenale), whipworms(Trichuris trichiuria), or pinworms
can be achieved with albendazole or mebendazole.
Intestinal parasitosis is not common in United States but its highly prevalent in othr
regions of the world such as Central America.

201.Mycotic or infected arterial aneurysms can develop due to metastatic infection frm
infective endocarditis with septic embolization and localized vessel wall
destruction in the cerebral (or systemic) circulation.Intracerebral mycotic

aneurysms can presnt as an expanding mass with focal neurologic findings or may
not be apparent until aneurysm ruptures with stroke or subarachnoid hemorrhage (
headache and neck stiffness).The diagnosis of mycotic aneurysm can usually be
confirmed with CT angiography.Management includes broad-spectrum anitbiotics
( tailored to blood culture results ) and surgical intervention (open or endovascular)

202.Pts with basilar artery occlusion present with motor weakness,ataxia or incordinatin
,altered level of consiouness,facial weakness,dysphagia, and/or dysarthria and
unilateral/bilateral gaze paralysis.

203.Clostridium difficile infection(CDI):


>Risk factors include recent hospitalization,advanced age and recent antibiotic use
(most commonly fluoroquinolones,penicillins,cephalosporins and clindamycin)
>Mild colitis typically presents as watery diarrhea (sometimes upto 10-15 times a day),
low grade fever,abdominal pain and leukocytosis. Rx of Mild to Moderate CDI is
Oral Metronidazole.
>Pts with severe disease may not have profuse diarrhea but often have signs of
systemic toxicity such as high fever,WBC count > 15,000/uL and serum Cr. >1.5x
Rx of severe CDI is Oral Vancomycin.
>Severe colitis with ileus usually requires the addition of IV Metronidazole.
>Pts with severe ileus,or toxic megacolon, WBC >20,000/uL and/or Lactate
>/=2.2mg/dL usually require surgery.
>Intracolonic vancomycin is suggested for pts with significant ileus but must be given
carefully due to increased perforation risk.

>IV vancomycin is not excreted into the colon and is NOT effective for CDI.

204.Catheter infections:
> Catheter removal is recommended in catheter related infections where there is
hemodynamic instability , severe sepsis with organ hypoperfusion, endocarditis,
suppurative thrombophlebitis, or persistent bacteremia after 72 hours of
appropriate antibiotic therapy.
>Majority of catheter-associated infections are due to coagulase negative staph
and in the majority of cases are methicillin resistant.Thus, empiric vancomycin is
appropriate.Pts with neutropenia or sepsis shd receive gram negative coverage
as well.
>Vertebral osteomyelitis or diskitis ( infection of intervertebral disk space ), is a well
recognized complication of catheter related systemic infections.It usually presents
with insidious onset of back pain, local tenderness to spinal percussion, reduced
back mobility and spasm of nearby muscles.
>MRI is highly sensitive in detecting vertebral osteomyelitis and/or diskitis.Typical
MRI findings in vertebral osteomyeilits include decreased signal intensity in the disk
and adjacent vertebral bodies and involved paraspinal muscles , loss of endplate
definition, contrast enhancement of the disk,adjacent vertebral bodies and involved
paraspinal and paravertebral soft tissues.Finally, a CT guided needle biopsy is
generally necessary to confirm the clinical and/or radiographic suspicion of the
presence of vertebral osteomyelitis or diskitis.
>Appropriate therapy for vertebral osteomyelitis is to continue vancomycin alone for
atleast six weeks.Prolonged therapy (12 weeks) is usually required for pts with

extensive bone destruction,adjacent soft tissue or paravertebral infection.

205.Metastatic infections and complications are common in pts with Staph aureus
infections.Examples include septic thrombophlebitis,infective endocarditis,
osteomyeilitis, and rarely, retinitis.The diagnosis is suspected by persistent
bacteremia or unchanged clinical status despite adequate treatment.

206.Candida Endopthalmitis:
>Occurs in 10-28% of pts with candidiasis through hematogenous spread and caused
by candida albicans.Its a marker of disseminated candidiasis and shd not be taken
lightly.
>Risk factors include Central venous catheters,total parenteral nutrition,broad
spectrum antibiotic therapy,prior abdominal surgery,neutropenia,corticosteroid
therapy and injection drug abuse.
>The condition is frequently characterized by ocular pain,photophobia,scotomas and
fever and decreased visual acuity.
>Fundoscopic examination reveals off-white glistening lesions that extend from
chorioretinal surface into vitreous and cause a vitreous haze.
>Candida endophthalmitis with vitreal involvement shd be treated vitrectomy and
systemic antifungal therapy ( Amphotericin-B and/or fluconazole ).
P.S: Systemic amphotericin-B attains sufficient concentration within the choroid
and retina but not within the vitreous body.Therefore the pts with vitreal
involvement will require vitrectomy for positive outcomes.

207.Roth spot-> retinal hemorrhage with a pale center.

208.Mycobacterium avium complex is characterized by fever,night sweats,fatigue,


shortness of breath,abdominal pain,diarrhea,weight loss and lymphadenopathy.

209.Vancomycin + Gentamicin => Empiric treatment of Infective endocarditis.

210.Influenza:
>Fever,headache,myalgias,dry cough.
>Positive throat swab culture confirms the diagnosis.
>Its impossible for pts to contract influenza from injected influenza vaccine.However,
such vaccinated pts can still contract influenza from other infected people becx the
vaccine is not 100% effective.This is especially true in the first two weeks.
>Lesser than 48 Hrs-> Oseltamivir
>More than 48 Hrs->Symptomatic with acetaminophen.
>More than 48 Hrs, Oseltamivir is only considered when pt is;
=>greater than 65 yrs age
=>pregnant
=>high risk medical condition such as pulmonary and cardiac dx.
=>Hospitalized pts.
>Amantadine and rimantadine are no longer recommended becx high levels of
resistance is generated against these medications.

211.Osteomyelitis:
>MRI is the most accurate test in diagnosing osteomyelitis in the vertebrae and
diabetic foot.
>The gold standard for diagnosing osteomyelitis is Bone biopsy.
>Majority of pts require long term antibiotics and resection of affected bone.

212.Diabetic foot Osteomyelitis:


>Majority of foot infections in diabetics are caused by a mix of aerobic and
anaerobic organisms (80%).
>Most commonly involved organisms are S.aureus,Group B strep,Proteus,
Pseudomonas,E.Coli,Candida,Bacteroides,Peptococcus and Clostridium.
>Organisms isolated from culture of specimens from deep curettage correlates
closely with cultures obtained from surgical resection of the deep tissue.
>Antimicrobial therapy for diabetic foot infection shd be individualized.Most
superficial skin infections require outpatient therapy against S.aureus,whil extensive
,deep infections require parenteral antibiotic therapy with a broad range for
microbes , including anaerobes.Major surgical debridement may be necessary.

213.Suspect infectious mononucleosis in any adolesecent who presents with fever,


pharyngitis,tonsillitis,lymphadenopathy,splenomegaly and rash.It can cause mild
kind of hepatitis as well.The recognition of atypical lymphocytes in the peripheral
smear can be a clue to its identification but this finding can be seen in other cnditns
as well such as Toxoplasmosis, CMV infection and lymphocytic leukemia.

214.Rhinocerebral mucormycosis:
Pts with DKA are exposed due to acidic environment and high free iron.The pathogn
can affect the nasal mucosa and invade the surrounding structures rapidly.The
common clinical features are fever,facial swelling,maxillary pain and tenderness,
nasal discharge,ophthalmoplegia, and headache.The diagnosis is suggested by the
characteristic clinical picture and confirmed by performing a biopsy of the infected
tissues.Treatement consists of debridement of the necrotic tissues&amphotericin B.
Despite aggressive treatment,the mortality remains high.

215.Terbinafine is used for the onychomycosis of the fignernails.

216.Serum sickness like syndrome may develop in the prodromal phase of Hepatitis-B
infection.This manifests as fever,rash and arthralgias and usually resolves with the
onset of jaundice.The serum sickness like syndrome is attributed to the circulating
immune complexes ( type-III reaction ).Other extrahepatic manifestations of
Hepatitis-B infection that can be explained by circulating immune complexes are
polyarteritis nodosa and glomerulonephritis ( usually membrane nephropathy,less
often membranoproliferative glomerulonephritis ).

217.Latent tuberculosis treatment regimens:


>INH therapy for 6 months

Although both 6 mo and 9 mo regimes available,the

>INH therapy for 9 months

=> 9 mo regimen provides optimal benefit.6 mo regimn

>INH therapy for 12 months

only for those who have adherence issues.12 mo


regmn is not significantly more benfecial than 9 mo

>Weekly INH + Rifapentine for 3 months


>INH + rifampin for 4 months
>Rifampin for 4 months.

218.Local candidiasis presents as a white lesion on buccal mucosa that can easily
scrapped off with.The diagnosis is confirmed by scrapping the lesions with a tongue
depressor and performing a Gram stain or KOH preparation on the scrappings.
Although local candidiasis is common in certain patient groups (e.g young infants,
pts on chemotherapy or corticosteroids ).Its unusual for a young and otherwise
healthy adult.In such a setting,an underlying immunodeficiency shd be suspected.

219.Sporotrichosis:
Its a subacute infection caused by a dimorphic fungus , Sporothrix schenckii.The
infection usually arises when the soil or other organic matter containing the fungus
is inoculated in the skin or subcutaneous tissue.It is usually seen in persons involved
in outdoor activities or occupations.The infection initially manifests as a papular
lesion over the site of inoculation.The lesion usually ulcerates over time with nonpurulent discharge from the lesion.Similar lesions are seen along the lymphatic
channels proximal to the original lesion.Diagnosis is by clinical presentation and
culture of the tissue or aspirated material from the lesion.Itraconazole for 3-6 mo
is the treatment of choice for lymphocutaneous sporotrichosis.

220.Blastomycosis:
The infection can range from asymptomatic infection to acute pneumonia and

extrapulmonary spread to the skin, bones and genitourinary system.

221.Atinomycosis:
Its a chronic dx caused by a gram+ve organism from the actinomycosis species.Its
not found in the environment and humans are only reservoir for the species, causing
cervicofacial disease.It occurs by direct invasion of the organism and some of the
predisposing conditions include gingivitis,dental caries,extraction and oromaxillofacl
trauma.It typically presents as chronic,slow growing,non tender mass,which slowly
evolves into multiple abscesses,fistualae and draining sinus tracts with thick yellow
or serous discharge.

222.Coccidioidomycosis:
Its caused by the inhalation of the airborne dimorphic fungus,Coccidioides imitis.
Chest pain,fever,cough and rarely,hemoptysis are usually presented with a primary
infection.Erythema nodosum is a delayed hypersensitivity reaction , which manifests
as a tender nodule over the pretibial area.It can sometimes be the initial feature of
primary coccidioidal infection.

223.Disseminated gonococcal infection:


>Gonococcal arthritis is the most common cause of acute non-traumatic mono and
oligoarthritis in young healthy adults.
>Shd be strongly suspected in unprotected sexual relationships.
>Tenosynovitis is a unique finding in pts with disseminated gonococcal infection.Its
very unusual for other forms of infection.

>Another characteristic finding is pustular or vesiculo-pustular skin rash that is often


transient and disappears spontaneously in several days.
>The typical synovial fluid leucocyte count is around 50,000 cells/mm3;however cell
counts below 10,000 cells/mm3 can be present in occasional cases.
>Confirmation of the suspected diagnosis is usually done by culturing the joint fluid,
and the mucosal surfaces,including the urethral,cervical,rectal and oral mucosa.Mre
than 80% of pts with disseminated gonococcal infection have positive cultures from
atleast one of the mucosal sites.

224.Spondyloarthritis=> Sausage digits.

225.Rheumatoid arthritis=> Subcutaneous nodules.

226.Infectious mononucleosis:
>Pts shd be warned that the profound fatigue takes months to resolve.
>Diagnostic test of choice is the Monospot test that screens for heterophile antibodz
that agglutinates the horse RBCs.
>Atypical lymphocytes are commonly seen on peripheral smears but are not sensitive
or specific for EBV IM.
>In pts with suspected IM and negative Monospot test , evaluation for EBV-specific
antibodies shd be done next.The presence of IgM and IgG viral capsid antigen (VCA)
antibodies suggests an acute EBV infection and the diagnosis is confirmed if there r
no IgG EBV nuclear antigen (EBNA) antibodies present. ( The IgG EBNA antibodies are
not seen in acute infection and only appears 6-12 weeks after the onset of symptmz)

>EBV infectious mononucleosis is typically a self limited illness treated with supportive
care and NSAIDS.Antimicrobial therapy is not indicated.
>While Corticosteroids are not indicated in the classic self-limited cases of infectious
mononucleosis , they may be of help if severe complications develop,e.g;
=>Airway obstruction ( Shortness of breath while recumbent is classic )
=>Overwhelming infection
=>Aplastic anemia
=>Thrombocytopenia.

227.Although VZV is a live attenuated vaccine , it can be safely administered to


immunocompetent pts with household contacts who are immunocompromised.
Although transmission of VZV by a vaccine related rash is rare,vaccinated individulz
shd be monitored for a rash and isolated if one develops becx its potentially
contagious to the immunocompromised individuals.

228.Pretransplant recepients are recommended to receive 2 doses of the VZV vaccine


atleast 4 weeks prior to the transplantation.

229.Contraindications of VZV vaccine:


>Anaphylaxis to neomycin
>Anaphylaxis to gelatin
>Pregnancy
>Immunocompromised states

230.Salmonella gastroenteritis:
>Treated symptomatically with fluids and electrolytes in immunocompetent individulz
aged 12 months or older.
>Antibiotics are only indicated in pts lesser than 12 months age, esp neonates,
immunocompromised , pts atleast 50 yrs old with known atherosclerotic disease, as
they are more prone to developing bacteremia.Effective antibiotics include
Ciprofloxacin,TMP-SMX and Ceftriaxone.

231.Skin abscesses are most commonly due to Staph aureus. All abscesses require
incision and drainage.
Give antibiotics in;
=>Systemic signs of infection
=>Immunocompromised
=>Large abscesses ( >5 cm )
=>Multiple abscesses
=>cellulitis
=>Age <6 months
Clindamycin and TMP-SMX are appropriate options for empiric S.aureus therapy.

232.If a pregnant woman has S/S of cystitis, following antibiotics can be used for a
period of 3-7 days;
a.Nitrofurantoin
b.Amoxicillin
c.Amoxicillin-clavulanic acid

d.Cephalexin

Following antibiotics are contraindicated for UTI in pregnancy;


a.Tetracyclines (dental staining and reduced bone growth in fetus)
b.Fluoroquinolones (tendon rupture in young children)
c.TMP-SMX ( 1st trimester->birth defects, last trimester-> hyperbilirubinemia fetus)

233.Clindamycin is excellent choice for eradicating gram positive,anaerobic organisms.

234.Pyelonephritis in pregnancy:
>Pregnant women are susceptible to adverse outcomes (e.g septic shock, preterm
birth, low birth weight ).
>Pyelonephritis in pregnant women is traditionally treated with hospitalization and
intravenous antibiotics such as ceftriaxone OR ampicillin and gentamicin until she is
afebrile for 24-48 Hrs and her symptoms are improving.The pt shd be then dischargd
on oral antibiotics , with the entire treatment course lasting for 10-14 days.
>Some providers will consider the outpatient treatment if the symptoms are mild and
the pt can keep oral medications down ( no vomiting ). Whether the treatment is
provided on an outpatient or inpatient basis, treatment duration shd be 10-14 days.
>If there is no response to appropriate antibiotic therapy within 48 Hrs , renal U/S to
assess for perinephric abscess or renal calculi shd be performed.
>Nitrofurantoin does not achieve adequate levels in renal tissue and shd not be used
to treat pyelonephritis.
>There is no evidence supporting antibiotic prophylaxis to prevent future episodes of
pyelonephritis in pregnancy.

>Pyelonephritis antibiotics in pregnancy;


=> Mild/Moderate
a.Third generation cephalosporins
b.Aztreonam
c.Ampicillin and gentamicin
=>Severe ( Urinary retention,immunocompromised,urosepsis)
a.Piperacillin/Tazobactam
b.Ticarcillin/Clavulanate
c.Carbapenems

235.Fever of unknown origin (FUO) is defined as a fever greater than 38.3 C on multiple
occasions over a period of three weeks in the setting of negative basic evaluation.
In 50% cases,the underlying cause is never identified. Of the cases with an
idenftifiable cause, connective tissue disorders, infections and neoplasms,in that
order, are most common.

236.Immune reconstitution inflammatory syndrome ( IRIS ):


Is a paradoxical worsening of preexisting ( and sometimes subclinical ) infections in
HIV+ve individuals.The syndrome usually occurs days to weeks after the initiation of
treatment with HAART and is thought to arise secondary to the rapid improvement
in immune function allowed for by HAART.The inflammatory reactions that develop
at the site of preexisting infection are responsible for the worsening of the pts
clinical presntatn. Ocassionally this can cause severe long term sequalae, esp if
neurologic tissues are involved.

IRIS is typically self limited and is best managed with continued HAART & antibiotic
treatment of the underlying pathogen.
Discontinuation of the HAART may prove necessary if the manifestations of IRIS are
life threatening or organ threatening.Its also acceptable to temporarily delay the
initiation of HAART for one to two months in those pts with opportunistic infections
that are being properly treated.In most instances,however,its preferable to continu
Rx with both HAART and antibioitcs.
Corticosteroids shd not be used to replace esp provided the pt is immunsupressed.
Once the diagnosis of IRIS is made,all invasive efforts to locate an occult infection
shd be halted.

237.Meningococcal meningitis:
>Low risk contacts do not require antibiotic prophylaxis.These include casual contacts
(such as co-workers,class mates or teachers ) and health care providers who were
briefly exposed to the pt but had no direct with secretions.
>Individuals who have direct contact with a high risk contact but not the patient are
also considered low risk ( e.g babysitter of a pts younger sibling )
>Child care workers require prophylaxis due to the increased risk of exposure to the
saliva of a pre-school aged child.

238.Intravenous antibiotics indicated for pyelonephritis when :


> Vomiting

> Positive blood culture

> Those who fail oral anitbiotics

> Infants < 2mo age who hve high of urosepsis

> hemodynamically unstable

239.Empiric antibiotics shd be started in Pyelonephritis before the culture and sensitivty
results are available.

240.Renal U/S is performed in Pyelonephritis in children aged < 2 yrs age after the
resolution of acute symptoms to look for the underlying renal anomalies.

241.VCUG is performed in neonates with a first febrile UTI or in children aged <2 with
recurrent infection or abnormal renal U/S.

242.Chronic functional constipation can prevent full bladder emptying.The resulting


urinary stasis is an important risk factor for UTI in toddlers.Evaluation and treatmnt
of constipation is necessary to prevent dysfunctional voiding and recurrent UTIs.

243.Females are advised to wipe from front to back to prevent transferring potentially
pathogenic colonic bacteria near the urethra.

244.Enuresis is a common manifestation of constipation due to reduced bladder capacty

245.Streptococcal pharyngitis:
>Group A streptococcus ( strep. pyogenes )
>Oral Penicillin for 10 days is the optimal treatment.Benefits of therapy are;
a.Decreased symptom severity and duration
b.Prevention of spread to close contacts
c.Prvention f supuratve cmplicatns (eg peritonsillar abscess,cervical lymphadenitis)

d.Prevention of acute rheumatic fever.


>A 5-day course of Azithromycin is acceptable therapy for penicillin allergic pts.
>Penicillin does not prevent the post-strpetococcal glomerulonephritis

246.D/Ds of oral lesions:


>Herpangina=>Vesicles on posterior oropharynx from coxsackie virus.
>Herpes simplex virus/Cold sore=>Cluster of vesicles on vermillion border.
>Aphthous stomatitis/Canker Sore=>Shallow gray ulcers on anterior oral mucosa.

247.Hand-Foot-Mouth Disease: (HFMD)


The exanthem is typically not itchy but can be macular,maculopapular or vesicular &
can appear on the palms,soles,genitalia, and/or buttocks.The enanthem consists of
vesicles on the posterior oropharynx.This is also known as herpangina and the lesions
on the hard palate can be painful and lead to diminished oral intake.The distribution
of the lesions varies among patients; some have isolated lesions on the mouth or skin
and others have a combination.
HFMD is primarily caused by infection with group A coxsackie virus during the sumer.
The infection is transmitted by direct contact with contaminatd body fluids (eg diaper
changes at daycare facilities).Hand washing is extremely important as the virus can be
contagious for several weeks after rash resolution due to viral shedding in the stool.
Treatment is supportive and directed at maintaining adequate hydration and relieving
pain.

248.Herpetic whitlow ( vesicles on hands and fingers ) are unilateral in contrast to

bilateral lesions of HFMD.

249.Papular urticarial consists of raised pruritic lesions surrounding the area of an insect
bite ( e.g fleas, bedbugs, mosquitos ).

250. Pneumocystis pneumonia:


>Seen in pts with defects in cell mediated immune response esp in those with HIV.
>Seen with CD4 < 200 cells/mm3
>fever,non productive cough,progressive dyspnea,tachypnea.
>CXR shows diffuse and bilateral ground glass or alveolar infiltrates.
>Its generally recommended to establish a definite diagnosis of PCP before instituting
specific therapy.
>Induced sputum ----NO ? ---> Fiberoptic bronchoscopy with bronchoalveolar lavage
for direct identification of organisms in bronchoalveolar
tree or respiratory secretions.
>Those with significant respiratory distress and desaturation are at high risk of hypoxic
respiratory failure and shd be admitted to the hospital for close observation,oygen
therapy and IV antibiotics.
>IV Trimethoprim-sulfamethoxazole shd be used initially for all pts with moderate to
severe PCP pneumonia and shd be converted to oral therapy when the pt has signs of
clinical recovery.
>IV pentamidine is the drug of choice for moderate to severe pneumocystis pneumnia
in pts who are intolerant of TMP-SMX.( Aerosolized pentamidine is associated with a
higher relapse rate and is not recommended for treatment ).

>Corticosteroids use decreases the mortality rate and rate of respiratory failure.
When to use steroids ?
a.PaO2 of 70 or less on room air.
b.A-a gradient > 35 mmHg

251.Community acquired pneumonia:


>Hosptialize the pt if he is over 65 yrs age, has comorbidities , miltilobar involvement,
severe hypoxia, and low blood pressure ( systolic BP < 90 mmHg ).
>The coverage of typical (e.g S.Pneumoniae) and atypical agents (e.g legionella) shd be
provided in hospitalized pts with community acquired pneumonia.A comibation of
IV ceftriaxone with a new generation macrolide such as Azithromycin or
Clarirthromycin is typically used in seriously ill pts ( pts who are in ICU ).

252.Clostridium difficile colitis:


>Clostridium difficile colitis is usually diagnosed by stool studies for C.difficile toxin
such as Polymerase chain reaction (PCR) or Enzyme immunoassay (EIA) for C.difficile
toxins A and B. PCR is usually preferred due to higher sensitivity than EIA. Pts with
negative laboratory testing and high clinical suspicion for C.difficile colitis usually
undergo limited colonscopy or sigmoidoscopy to document pseudomembranous
colitis and confirm the diagnosis.
>Endoscopy can also help diagnose pts who do not improve with appropriate
antibiotic therapy for C.difficile,require more urgent diagnosis before lab tests are
available ( fulminant colitis ) or have atypical cases with ileus and minimal diarrhea.
>Clostridium difficile colitis is usually due to altered intestinal flora by antibiotic

therapy and thus growth of Clostridium difficile occurs which causes intestinal
colonization with its cytotoxins.
>Fluroquinolones,enhanced-spectrum penicillins,cephalosporins and clindamycin
are frequently associated with the development of Clostridium difficile infection.
>Amoinoglycosides,TMP-SMX and tetracyclines are less frequently associated with
C.difficile infection.
>Macrolides are intermediate and can cause infection in some pts.
>In general, broad spectrum and multi-drug antibiotic therapy rasies the risk for
C.difficile disease more than narrow spectrum and single drug therapy.

253.Rocky mountain spotted fever:


>Is a tick borne illness caused by intracellular gram-ve organism Rickettsia rickettsii.
>Seen in spring and early summer when outdoor activities are at their peak
>Early symptoms are non specific=>fever,headache,myalgias and lethargy.
>Rash is seen on 3rd-5th day of illness.Its a petechial rash which usually begins on the
ankles and wrists and spreads to the palms,soles and to the central body.In severe
fulminant cases,the pt may develop changes in the mental status (i.e confusion),focal
neurologic signs, seizures and multiorgan dysfunction, leading to death.
>Pts with suspected Rocky mountain spotted fever shd be treated empirically,without
waiting for the for confirmation of diagnosis,since the dealy in treatment is associatd
with a high mortality rate.
>What are the lab tests anyways?
=>Biopsy of the skin lesions that appear on 3rd-5th days of illness can confirm it.
=>Antibodies appear at 7-10 days of illness, for which serological tests used are

indirect fluorescence antibody testing, Enzyme immunoassay or complement


fixation.
=>Weil-Felix test was used in the past to detect cross reacting antibodies.Its a
non-specific test and not recommended anymore.
>Thrombocytopenia is one of the many fatal complications of this disease.
>Doxycycline is the treatment of choice in both children and adult pts with proven or
suspected Rocky mountain spotted fever.Its typically continued for atleast 3 days aftr
defervescence.
>Chloramphenicol is an alternative option for the treatment of pts with suspected
Rocky mountain spotted fever.It has a high incidence of side effects and is only
reserved for pregnant females and for pts who are unable to tolerate tetracycline.

254.Diphtheria:
>If there is a high suspicion, diphtheria antitoxin shd be administered as soon as possibl
to avoid complications such as myocarditis,neuritis or rarely neuritis.
>Diphtheria antitoxin is made with horse serum, thus the risk of hypersensitivity or
serum sickness is approximately 10%.There is also a risk of anaphylaxis.For these
reasons,epinephrine must be always available.

255.Isoniazid for nine months is the first line treatment for latent tuberculosis in both
children and adults.In cases of Isoniazid resistance , rifampin becomes the drug of
choice and is typically given for six months in children and four months in adults.

256.Remember, if the index pt is resistant to Isoniazid drug then the one who got latent

tuberculosis by being in contact with index pt will also be resistant to INH and
alternative regimens wud be required to treat LTBI then.

257.Since obtaining a sputum sample from children is frequently difficult, a morning


gastric aspirate is often used to obtain bacteriologic samples from sputum swallowd
during the night.

258.HIV PEP:
>Post exposure HIV prophylaxis with highly active anti retroviral therapy for 28 days
OR 4 weeks is recommended following high risk exposure to an HIV infcted individual.
>Triple drug therapy with tenofovir-emtricitabine-raltegravir is preferred due to low
side effect profile and few drug-to-drug interations.It shd be started within 72 Hrs of
exposure.
>However, 2 drug regimens may be considered for individual cases when better
tolerability or compliance is needed.
>In cases of low risk exposure ( e.g exposure to urine or saliva w/o visible blood ) OR
presentation >72 Hrs after exposure, the risks of PEP likely outweigh the potential
benefits and is not recommended.

259.Rabies:
>Acquisition of rabies from bats can occur from an unrecognized bite or scratch or
possibly by inhalation of aerosolized air particles.
>Hydrophobia is pathognomonic for rabies; water triggers pharyngeal spasms that
cause the pt to be frightened of drinking.The illness usually starts with a few days of

non-specific prodromal symptoms, including fever and malaise. Neurologic symptoms


(e.g confusion,lethargy,paralysis,aphasia) develop later in the course of disease.
>Bats are found in all states except Hawaii and spelunking ( cave exploration ) is a risk
factor for rabies acquisition from bats.
>Rabies is a universally fatal disease once pts are symptomatic and most of them suffer
from coma and death within weeks of illness onset.Treatment is primarily palliative.
>The Milwaukee protocol is an experimental approach to rabies treatment that consist
of therapeutic coma induction and antiviral therapy.Although a few pts with suspectd
rabies have survived with this protocol, several have died despite this investigative
therapy.Routine implementation of the protocol is not currently recommended.

260.Community acquired pneumonia in older pts with other coexisting medical


illnesses shd be ideally treated in the hospital

277.Gonococcal urethritis:
>Most common symptom is painful urtheral discharge
>Its diagnosed by the presence of >5 WBCs/hpf in urethral secretions, presence of
leucocyte esterase , or >10 WBCs on microscopy of first voided urine.
>The diagnosis of gonococcal infection is confirmed with a Gram stain smear, which if
negative shd be followed up with nucleic acid hybridization and amplification methds.
Nucleic acid hybridization and amplification methods can be used on cervical,vaginal,
urethral or urine specimens.
>Pts with gonococcal urethritis are frequently co-infected with Chlamydia trachomatis
as well.

>Treatment is Ceftriaxone 250 mg Intramuscular single dose to cover N.Gonorrhea &


Oral Doxycycline 100 mg twice daily for 7 days or Azithromycin 1 g as a single dose
to cover Chlamydia trachomatis as well as resistant strains of N.gonorrhea.

278.Enterovirus and Arbovirus and the most common cause of viral meningitis or
encephalitis in the pediatric population.The most common ones are Eastern euine
encephalitis,St.louis encephalitis,Colorado tick fever and California encephalitis.Thez
infections are most frequent during summer,late spring and early fall , especially in
the rural areas.Most arbovirus infections are zoonosis ( transmitted through animal
vectors), and for this reason, these infections are more common in rural areas.
Herpes simplex virus is the most common cause of viral meningitis in the adult
population, not in children.

279.Diagnosis of PCP infection shd not be assumed in HIV+ve pts.The entire clinical
picture must be considered.
Most common cause of common acquired pneumonia even in HIV pts is
S.pneumoniae.

280.Legionella infection causes community acquired pneumonia that is commonly


accompanied by extra-pulmonary symptoms such as diarrhea,nausea,vomiting &
neurologic findings such as headache and confusion.

281.Erysipelas:
>A specific form of cellulitis

>Lower extremeties are most common site but face is involved in 5-20% of pts.
>The lesion is usually red,painful,edematous,elevated and sharply demarcated from
the uninvolved skin and a butterfly pattern involving the cheeks and bridge of nose
may be seen.
>The onset is abrupt and accompanied by systemic symptoms i.e fever,chills,malaise
>Group A streptococcus is the typical etiologic agent of erysipelas , although sometimz
other beta hemolytic streptococci are involved.

282.Bacterial Meningitis:
>Antibiotic treatment prior to obtaining CSF levels may cause negative Gram stain &
culture.
>Antibiotic pretreatment can also cause higher CSF glucose and lower protein levels
in bacterial meningitis.
>High suspicion of bacterial meningitis, even with negative Gram stain and culture,
warrants a full course of antibiotic therapy.

283.Cryptococcal antigen test has a sensitivity of 93-100%.

284.Guillian barre syndrome: High protein, normal cell count.

285.Lyme disease:
>The most common complication of tick bites in most cases is local inflammation or
local infection ( if a wound is produced ).
>The risk of aquring lyme disease after being bitten by a tick is <1.5%.

>In order to be infected with lyme disease , the pt must have been exposed to the tick
for more than 36-48 Hrs , becx the transmission of the infectious agent-Borrelia
Burgforferi-takes place only after the tick is firmly attached to the skin and has
suctioned a certain amount of blood which gives it an engorged appearance.
( engorged->suspect lyme disease wud occur now )
(non-engorged->most probably disease not transmitted )
>If a tick is found and it is not engorged with blood,there is no risk of lyme disease,as
ticks take atleast 24 Hrs to firmly attach to their victims.
>The tick that transmits lyme disease is brown. ( the one transmitting RMSF is black ).
>Majority of pts have a transitory skin reaction in the first 24-72 Hrs on the site of
contact with the tick;this lesion must not be confounded with erythema migrans rash.

286.Pregnancy and Hepatitis-B:


>All pregnant women shd be screened for Hepatitis B during their first prenatal visit.
>Women with unknown status or those having risk factors ( IV drug abuse, multiple
sexual partners, concomitant Hepatitis-C, concomitant HIV infection ) shd be screened
again near the time of delivery for Hepatitis-B.Women with risk factors shd be
vaccinated against Hepatitis-B regardless of their risk factors.
>Pregnant women with active Hepatits-B infection shd be treated with antiviral
therapy as well as shd receive Hepatitis-B vaccine.
>Infants born to Hepatitis-B+ve mothers shd receive Hepatitis-B immune globulin as
well as vaccine within 12 Hrs of birth.

287.Oropharyngeal lesions in children:


A.Herpangina=>Caused by Coxsackie group-A virus.It is usually seen during the summer
in children aged 3-10, who present with fever,drooling,sore throat,
decreased appetite,headache and painful posterior pharyngeal vesicles
The lesions can evolve rapidly from erythematous macules to vesicles
on soft palate and tonsillar pillars,creating a painful pharyngitis.
Diagnosis is based clinically on the enanthem.Therapy is directed at
symptomatic relief with saline gargles,analgesics and antipyretics.Cold,
non-acidic fluids or popsicles can be soothing and provide hydration.
Antiviral therapy is not indicated in this self limited illness, which typicaly
resolves within a week.Hand washing by children and their close contcts
can prevent spread during outbreaks.
B.Apthous stomitis=>Lesions are often larger and are ulcers located on anterior oral
/ Canker sores

mucosa ( lips,cheeks,mouth floor,ventrum of tongue ).There are


no fever or systemic symptoms.

C.Group-A strep. Pharyngitis=> Tonsillar exudates,fever, anterior cervical lymphadnpthy


D.Herpes gingivostomatitis=> These pts tend to be febrile and ill appearing with vesicles
on the anterior oral mucosa,gingivae and perioral skin.
E.Infectious mononucleosis=>Adolesecent pts present as classic triad of fever,diffuse
bilateral cervical lymphadenopathy and exudative
pharyngitis.The spleen is sometimes enlarged.

288.HIV esophagitis:
>Characterized by painful swallowing ( odynophagia ).

>The initial treatment is fluconazole.


>Failure of antifungal treatment shd make u suspect other causative agents ( aside
from candida ).
>Giant ulcers with no virus found on biopsy suggests apthous ulcers which is treated
with prednisone.

289.Invasive aspergillosis:
>Is common in bone marrow transplant recepients especially allogenic transplantation
(vs autologous transplantation).Other risk factors include old age,acute graft vs host
disease, and corticosteroid therapy.
>It causes a combination of Systemic symptoms + pulmonary symptoms + nasal/Sinus
symptoms.

290.CMV can cause the following in bone marrow recepients:


A.Most commonly;
=>fever of unknown origin
=>intersitital pneumonitis
=>enteritis
B.Less oftenly;
=>retinitis
=>Encephalitis
=>Hepatitis
=>Bone marrow suppression
Ganciclovir prophylaxis significantly reduces the risk of CMV infection in bone marrow

recepients.
They also give Fluconazole prophylaxis to prevent candidiasis.

291.Herpes zoster:
>Due to reactivation of latent varicella zoster in sensory or dorsal nerve root ganglion
>Greatest risk group is elderly and immunosuppressed
>Prodrome of fever,headache,malaise and localized pain followed by the development
of painful vesicular rash in the distribution of spinal nerve roorts, typically the thoracic
or lumbar dermatomes.The vesicles envolve into pustules before crusting in about
7-10 days.Pain is the most consistent symptom and can persist for days to months
after the rash resolves.
>Mainstay of treatment is oral acyclovir.Its a clinical diagnosis and diagnostic workup
is not necessary.For maximum efficacy, start the treatment in 48 hours, so as to
improve the acute pain,promote early healoing and prevent the development of
postherpetic neuralgia.
>All pts with disseminated herpes zoster shd be maintained on contact and airborne
isolation until all the skin lesions are dry and crusted.All healthcare workers w/o a
history of varicella zoster virus infection are strongly encouraged to avoid contact
with pts with varicella or herpes zoster infection.
>Contact precautions are indicated in hospitalized pts with localized herpes zoster but
these are not currently recommended in a community setting.

292.Complications of Varicella ( chickenpox ) can be bacterial superinfection,varicella


pneumonia,cerebellar ataxia, meningoencephalitis and death.

293.HIV:
>In pts with clinical presentations suggestive of primary HIV infection and negative or
indeterminate HIV ELISA tests ( a scenario that can occur when the test is performed
after exposure and before seroconversion ), the diagnosis can be definitively confirmd
with an HIV RNA PCR assay or a test for the p24 antigen.
>HIV+ve pts not already on HAART shd have their CD4 counts and viral load measured
every 3-4 months to determine the optimal time to initiate medications.A similar
schedule is recommended for those pts on HAART to evaluate the efficacy of treatmnt
>CD4 lymphocyte count is an indicator of current level of immunosuppression and is
referred by some authors as the immunological damage that has already occurred
>Viral load is a good marker of disease activity or the potential for future damage to
immune system.It is referred to as the damage that is about to occur.Therefore
plasma viral load has prognostic significance at any level of CD4 count in pts with HIV
infection.

294.Syphilis:
>HIV infected pts with syphilis of unknown duration or late latent syphilis ( syphilis
acquired more than a year earlier ) with neurologic symotoms shd have CSF examined
before treatment to know if there is neurosyphilis.Pts with CD4 count </= 350 / uL and
Rapid plasma regain test (RPR) titres >/= 1:32 have a higher risk of neurosyphilis.
>Rx:
=>Primary , secondary and early latent syphilis ( <1 yr duration ) shd receive Benzathne
Penicillin G Intramuscular single dose.
=>Pts with late latent syphilis ( >1 yr duration ), syphilis of unknown duration,syphilis

with gummatous lesions and cardiovascular syphilis shd receive Benzathine


Penicillin-G intramuscular once weekly for 3 weeks.
=>Pts with neurosyphilis or syphilitic ocular disease shd receive Aqueous Penicillin-G
intravenous for 2 weeks/10-14 days. Ceftriaxone or tetracyclines may be considered
as alternate RX for penicillin allergics , however penicillin is always preferred Rx.
=>Pts with congenital syphilis shd receive aqueous penicillin-G intravenous for 10 days.
>An acute febrile reaction that develops within 24 Hrs after the initiation of treatment
for spirochetal infection ( e.g syphilis,leptospirosis,tick borne spirochetes ) z commonly
referred to as Jaresh-Herxheimer-reaction.Fever is usually accompanied by malaise,
chills,headache and myalgias.The etiology of the reaction is thought to be due to an
innate immunological reaction to the lysis of spirochetes.No effective prevention is
available.Pts shd be informed of the possible signs and symptoms and advised to
contact their clinicians if a severe reaction occurs.

295.Chronic prostatitis:
>Presents as lower urinary tract symptoms,perineal discomfort and prostatic
tenderness.However it can be asymptomatic and have no findings on physical
examination.
>Urine cultures are usually positive in this entity.
>Rx:
Antibiotic therapy is given for 6-12 weeks.Not all antibiotics have an adequate
penetration when the infection is chronic.
=>Quinolones are preferred and the most widely used from this group are
Ciprofloxacin and Levofloxacin which have a cure rate of about 70%.

=>A second alternative is TMP-SMX which is given as a double strength tablet


every 12 Hrs.

296.Human Bite:
>Can be complicated by a polymicrobial soft tissue infection.Associated with aerobic
(alfa-hemolytic streptococci including viridans group & Staph aureus) and anaerobic
bacteria.Eikenella corrodens, a gram negative anaerobe, is a common finding in
infections due to human bites.
>Depending upon the depth and severity of bite, the infection can involve soft tissues,
bone and joints.Initial evaluation in includes a thorough skin and musculoskeletal
examination;imaging may be needed if deep infection is suspected.
>Rx:
=>Local wound care and irrigation
=>With the exception of facial injuries ( where cosmetic outcome is important and
infection risk is relatively low), primary closure shd be avoided.Most wounds are
left open to heal by secondary intention.
=>Tetanus prophylaxis shd be considered depending upon the pts vaccination status
and severity of the bite.
=>Oral Amoxicillin-clavulanic acid ( or ampicillin-sulbactam if parenteral therapy is
needed ) is the drug of choice for significant wound infections due to mammalian
bites,including human bites.It provides coverage against oral aerobic and anaerobic
flora, including Eikenella corrodens.

297.Contact with saliva is considered an extremely rare cause of HIV transmission.Bite

wounds are generally considered low risk exposures unless the biter has oral bleed
at the time of bite.

298.Hepatitis-C:
In pts with positive ELISA anti-HCV antibodies, there are several potential explanations;
persistent Hepatitis-C infection,cleared infection or false positive ELISA result.The
diagnosis shd be confirmed by high specificity testing ( usually HCV RNA ). The positive
predictive value of ELISA anti-HCV in a pt with a low risk profile and normal AST/ALT is
low.

299.Lactational Mastitis:
>Typically occurs during the first 2 months postpartum as the mother and infant are
establishing breastfeeding.Difficulties with breastfeeding can lead to prolonged
engorgement, inadequate milk drainage, and clogged milk ducts ( tender palpable
lump or cord ).Bacteria from the skin can enter the milk ducts and proliferate in
stagnant milk, resulting in infection.The most common organism is Staph aureus.
>Diagnosis is based on the clinical presentation and workup is unnecessary.
Manifestations include fever, local breast pain, erythema and swelling.
>The antibiotics of choice include oral Dicloxacillin ( anti-staphylococcal penicillin )
and Cephalexin ( first generation cephalosporin ).
>In addition to antibiotics and analgesia,treatment of lactational mastitis requires
effective and frequent milk drainage.Milk ducts are most efficiently drained by
direct breast feeding.The first step in management is to evaluate breastfeeding
technique and positioning.This is best achieved by observing a feed.The correct

position for optimal milk intake involves the infant forming a tight seal around most of
the areola.However, a common cause of severe pain during latching occurs when the
infant only suckles on the nipple.This improper position can cause nipple blistering.
>The characteristic finding of an abscess is a focal area of fluctuance ( wavelike motion
on palpation indicates pus collection ). Induration ( skin thickening from edema and
inflammation ) can occur in both mastitis and abscess. Ultrasound is the gold standrd
modality to differentiate these conditions.If a fluid collection is identified, ultrasound
is used to guide needle aspiration under local anesthesia.
>Incision and drainage is the preferred treatment for most skin abscesses.However,
incision and drainage of a breast abscess has increased risk of complications (e.g milk
fistulas), slower recovery time (can negatively impact milk supply and breastfeeding)
and a less desirable cosmetic outcome.Its generally performed after an unsuccessful
needle aspiration.

300.IV Vancomycin is reserved for pts with MRSA risk factors ( e.g recurrent hospitalizns
/antibiotics, incarceration ), hemodynamic instability or failed outpatient therapy.

301.Cold compresses provide relief from pain and swelling.

302.TMP-SMX shd be avoided during breast-feeding as small amounts can enter the
breast milk.This medication can potentially displace bilirubin from albumin and can
cause neonatal kernicterus.

303.A breast infection that is refractory to treatment shd prompt investigation for
inflammatory breast cancer.A malignant mass is a rare but possible obstacle to milk
drainage.On examination, accompanying symptoms typically include skin thickening,
peau dorange and axillary lymphadenopathy.MRI is the imaging of choice.
Palpation guided fine needle aspiration is the usual biopsy method for suspected
cancer.

304.Mammograms are not recommended in women aged <40 becx younger women
tend to have denser breasts which makes mammograms harder to interpret.In
addition its associated with high rate of false positives in women aged <40.

305.Lyme arthritis:
Early symptoms (e.g erythema migrans , non specific constitutional symptoms) can be
overlooked or misdiagnosed and many pts do not remember the tick bite.Untreated dx
can progress to more serious manifestations,such as the inflammatory monoarticular
arthritis.The knee is the most commonly affected joint in Lyme arthritis.
Late lyme disease is one of the most common causes of pediatric arthritis in Lymeendemic areas.In contrast to other forms of infectious arthritis, pts with Lyme arthritis
tend to be well appearing and can bear weight on the affected joint.They are also less
likely to have fever, pain and constitutional symptoms.However,because findings often
overlap with potentially destructive septic arthritis and Lyme serology can take several
days to return,needle aspiration of synovial fluid ( arthrocentesis ) shd be performed
as soon as possible.In Lyme arthritis,the synovial fluid typically shows a non bloody

inflammatory profile with an average leucocyte count of 25,000 cells/uL.The primary


diagnostic test for Lyme arthritis is serum serology as it is difficult to isolate Borrelia
Burgdorferri from synovial fluid.Enzyme linked immunosorbent assay screening shd be
obtained , followed by confirmatory Western Blot testing.IgG serologies to Borrelia
Burgdorferri would be positive in all pts with Lyme arthritis.
If clinical suspicion of Lyme arthritis is high, antibiotic therapy can be started while the
results are pending as serology can take several days to return.The prognosis of Lyme
arthritis is good and a 28-day course of oral Doxycycline or Amoxicillin is usually
curative.Most pts are disease free within 6-12 months.

306.Infectious mononucleosis:
>Amoxicillin or Ampicillin administration in a pt with infectious mononucleosis
classically causes a generalized maculopapular rash.The mechanism is likely
immune-complex mediated by circulating antibodies towards penicillin-derivatives.
>Supportive treatment and observation are the mainstays of treatment for individuals
with infectious monoculeosis.This includes acetaminophen and NSAIDs for fever
and sore throat as well as adequate nutrition,fluids and rest.
>Sports shd be avoided for >3 weeks due to the risk of splenic rupture.
>Antibiotic use is associated with a generalized rash.

307.Intestinal parasitosis/Helminthis infections are very common in developing nations


including much of Latin America and shd be considered in pts who have recently
immigrated to the United States from a country with high rate of parasitic infection.
Many of these infections result in chronic diarrhea and weight loss.Irritation of the

gastrointestinal tract results in chronic blood loss and subsequent iron deficiency
anemia.Eosinophilia is common and can be helpful distinguishing feature from othr
causes of chronic diarrhea.The prognosis is excellent with adequate therapy.
However reinfection is common if inadequate hygiene measures cause family
members to get infected and then subsequently reinfect the child.Family members
shd be tested to determine if they are infected and appropriate therapy shd be givn
to them as well.
Anemia secondary to parasitosis typically does not improve acutely once the infect
resolves.Rather,improvement will depend on the start of iron replacement therapy
to replace the depleted iron stores.

308.Trichomoniasis:
>Presents with malodorous , frothy yellow green vaginal discharge, vulvar & vaginal
pruritis and dyspareunia from vaginal inflammation.
>Oral Metronidazole for the pt and the partner.Metronidazole is known to enter
breast milk in significant concentrations but few adverse effects have been reportd
in infants.Therefore, most physicians recommend that lactating women receive
metronidazole 2 g by mouth for 1 dose and that breastfeeding be discontinued for
12-24 hrs only.Although 500 mg twice daily metronidazole by mouth for 7 days is
equally efficacious as a single 2 g dose, single dose therapy is preferred for minimal
disruption of breastfeeding.

309.Recommended vaccines for adults:


>The tetanus-diphtheria (Td) booster vaccine every 10 years to all adults after age 18
with Tdap as a one-time substitute.
>For adults age > 65 , PCV 13 initally followed by PPSV 23 6-12 months later.
>Sequential PCV 13 and PPSV 23 also recommended for age <65 in certain high risk
conditions such as CSF leaks,Cochlear implants,Sickle cell disease,Congenital/acquired
asplenia,immunocompromise,chronic renal failure.
>PPSV 23 alone is recommended to adults aged <65 with other chronic medical
conditions incuding smoking,diabetes,heart or lung disease and chronic liver disease.
>Annual seasonal influenza vaccine for all adults regardless of the age and esp for
those over 65 or with significant comorbidity.

310.PCP:
>Atovaquone is used in pts with mild-moderate PCP who are unable to tolerate
TMP-SMX.
>Intravenous trimetrexate is used in pts with PCP who are intolerant of TMP-SMX or
IV pentamidine.

311.PML:
>I think when u seen an HIV+ve pt with neurologic deficits/seizures/aphasia issues,
think of PML.There is no evidence of increased intracranial pressure.
>Brain biopsy reveals intranuclear inclusions in oligodendrocytes,demyeilination
and astrocytosis.
>Although there is no specific treatment for PML,regressions of well over two years

have been documented in HIV+ve pts treated with HAART. HAART is not specific
treatment but YES it prolongs life in PML pts , so start it asap if not started before.
>Pts with CD4 counts >100/uL at baseline and an ability to sustain an HIV viral load
<500 copies/mL have a better prognosis.
>Without treatment with HAART, the majority of pts stricken with PML will die within
3-6 months onset.Usage of HAART in this pt population may prolong survival for
more than 2 years.

312.Pelvic actinomyces:
Actinomyces is a gram positive anaerobic bacillus that is common in normal vaginal
flora.There are several case reports of endometritis, pelvic inflammatory disease,
pelvic abscess and retroperitoneal fibrosis associated with Actinomyces in intrauterine ( IUD ) users.However the incidental identification of actinomyces in the vagina
or cervix is not diagnostic of disease and is not predictive of future disease developmnt
Approximately, 7% of women with an intrauterine device have actinomyces-like
organisms on Pap smear.Asymptomatic pts with incidental colonization by actinomyces
are at extremely low risk of pelvic actinomycosis.Antibiotic therapy for these pts is not
needed.
In these pts,current guidelines recommend continued IUD use and education about the
small risk of actinomycosis.If a pt shows signs and symptoms of pelvic infection
(e.g pelvic mass,pain, cervical or uterine tenderness), the IUD shd be removed and sent
for culture and antibiotics shd be administered.

313.Pts with severe closdtridium difficile colitis need to be treated with oral vancomycin
>Temperature > 38.3 C (100.9F)
>WBC count >15,000/uL.
>Creatinine >1.5x baseline
>Serum albumin <2.5 g/dL

312.Prevention and control of Clostridium difficile in institutional settings:


Clostridium difficile is the most common cause of nosocomial diarrhea.Health care
facilities can take significant steps to prevent and control the infection.Initial measurz
shd include infection rate monitoring in the facility,hand hygiene,and contact
precatuions.All pts shd be placed under contact isolation for the duration of diarrhea.
Health care providers shd wear proper gowns and gloves when examining pts and also
disinfect stethoscopes or use disposable ones.
Alcohol-based hand sanitizers are usually recommended over soap and water in most
infection control cases.However, alcohol based sanitizers do not effectively eliminate
C.difficile spores from the hands of healthcare workers.As a result, hand washing with
soap and water is preferred for pts with C.difficile colitis.Other measures include
effectively cleaning the pts environment ( e.g room, medcal equipment ) and limiting
the use of certain antibiotics (e.g clindamycin, fluoroquinolones,cephalosporins).

313.Antidiarrheal agents shd be avoided in C.difficile colitis due to the increased risk of
complications (e.g colonic dilation).

314.Some studies have shown increased risk for C.dificile diarrhea with proton pump
inhibitors.

315.Anal abscesses:
Anal abscesses arise when one or more of the several glands that encircle the anus
become blocked and the bacteria within grow unchecked.Pts with anal abscesses
typically present with severe,constant pain that may be accompanied by fever or
malaise.Physical examination commonly reveals erythematous, indurated skin or a
fluctuant mass over the perianal or ishiorectal space.Purulent material may be seen if
the abscess has begun to drain spontaneously.It is widely agreed that the most imprtnt
aspect of treatment is prompt incision and drainage of the abscess.Perianal and small
ischiorectal abscesses are often drained in the office,but larger ischiorectal abscesses
typically require surgical intervention.Antibiotics shd also be prescribed in those pts
who have diabetes mellitus, immunosuppression,extensive cellulitis or valvular heart dx.
Fifty percent of pts with anal abscesses will go on to develop a chronic fistula from the
involved anal gland to the overlying skin.Pts with fistulas typically present with anal
abscesses that persist after incision and drainage or with a pustule-like lesion in the
perianal or ischiorectal area that continually drains.Surgical repair is usually necessary to
eliminate the fistula while preserving fecal continence.

NERVOUS SYSTEM
1.Myelomeningoceles:
>Bladder dysfunction is a universal complication of lumbosacral myelomeningoceles
(excluding neurological deficits)
>Second most common extraneural complications occur that of lower GI tract
>Lower extremity fractures occur in approximately 30% of pts w/o a known hx of
trauma.These fractures may be associated with vigorous physical therapy.
>Children with S2-S3 involvement can have external anal sphincter dysfunction that can
lead to fecal incontinence.

2.Late Neurosyphilis:
Typically manifests years after Treponema Pallidum infection and can be characterized
by tabes dorsalis and Argyl Robertson pupils
Tabes dorsalis is a neurodegenerative condition that involves the posterior spinal
column & nerve roots.Posterior spinal column involvement results in impaired vibration
and proprioception , sensory ataxia and instability during the Rombergs test.Nerve root
involvement can contribute to diminished pain & temperature sensation and areflexia.
Pts might complain of lancinating pains, described as brief shooting or burning pains in
the face, back or extremeties.Tabes dorsalis can be associated with Argyl Robertson
pupils which are characterized by normal pupillary constriction to accommodation but
not with light.

3.Vit B12 defeciency can lead to subacute combined degeneration that is a myelopathy
characterized by both posterior spinal column disease and lateral corticospinal tract
disease, the latter of which causes spastic paresis and hyperreflexia.

4.Pseudotumor cerebri:
Aka idiopathic intracranial HTN arises from chronically elevated intracranial pressure.
Papilledema is evident on examination and may lead to progressive optic aptrophy &
blindness.This condition is most common in obese premenopausal women but is also
thought to occur secondary to endocrine disorders (e.g hypoparathyrodism, hypothyroidism, adrenal insufficiency, cushing disease) and the usage of certain medications
(e.g isotretinoin, all-trans retinoic acid, minocycline, tetracyclines, cimetidine, corticosteroids, danazol, tamoxifen, levothyroxine, lithium, nitrofurantoin)
The pt would present with headaches, pulsating sounds in ears and ophthalmologic
ex. would reveal papilledema, pericapillary flame hemorrhages, venous engorgement,
and hard exudates.

5.Chronic tension headaches are linked with contraction of scalp and neck muscles and
may recur daily.

6.Cluster headaches more common in young men.

7.Migraine headaches are recurrent throbbing or pulsatile headaches often associated


with a prodrome of nausea, vomiting, photophobia, & phonophobia.When they occur,
the prodromes are characterized by visual scintillations, scotomas, dizziness or tinnitus

8.Following parathyroidectomy, serum calcium levels fall and produce symptoms of


hypocalcemia such as perioral numbness, muscle cramps, carpopedal spasm, positive
Chvosteks sign and positive Trosseaus sign. Severe hypocalcemia can cause mental
status changes and seizures.The fall of serum calcium levels after parathyroidectomy
is by relative hypoparathyroidism which results from the suppression of normal

parathyroid glands by high calcium levels, which fail to respond to low calcium levels
following surgery.The suppression is transient and occurs within a few Hours following
surgery and most pts recover in few days.
Typically the signs of hypocalcemia are bilaterally symmetrical.Unilateral signs suggest
an alternate diagnosis.

9.Hungry bone syndrome :


Occurs 2-4 days following surgery and is caused by sudden withdrawal of parathyroid
hormone in pts with severe hyperparathyroidism, causing an increased influx of calcium
from the circulation into the bone.Preoperative risk factors for hungry bone syndrome
are severe hyperparathyroidism, severe bone disease and Vit-D deficiency.All these
conditions have a very high bone turn over.Postoperatively, when parathyroid hormone
levels fall, the dynamics of bone turnover shifts from the net efflux of calcium from bone
to net influx of calcium into the bone.

10.Almost 90% of right handed pts and 60% of left handed pts have a left hemispheric
dominance for speech and language functions.

11.Construction apraxia:
Seen with the involvement of parietal lobes and is much more pronounced with lesions
of the right parietal hemisphere (non dominant parietal lobe).Pts have marked difficulty
in copying single line drawings.They may also experience difficulty in wearing clothes &
appear to struggle while attempting to get into a coat or pants.This is known as dressing
apraxia. Confusion is also seen in pts with lesions of the nondominant parietal lobe.

12.Gerstmann syndrome:
Caused by damage to dominant parietal lobe, especially the inferior parietal lobe.
These pts have difficulty in performing simple arithmetic tasks (acalculia), inability to
name individual fingers (finger agnosia), impaired writing (agraphia) , and right/left
confusion (difficulty in identifying the right or left side of the body).
CANNOT WRITE, CANNOT MATH, CANNOT NAME FINGERS, CANNOT NAME SIDE.

13.Lesions of the non-dominant temporal lobe can cause visual diorders ( upper
quadrantanopsia ) and impaired proprioception of complex sounds (auditory
agnosia).

14.Dominant temporal lobe involvement causes homonymous upper quandrantanopsia


and wernickes aphasia, which is characterized by the impairment in comprehension
of spoken or written language.Pts have difficulty in expressing their thoughts in a
meaningful manner.

15.Paradoxical emboli are those that originate in the venous system & enter the arterial
system via an intracardiac shunt (e.g foramen ovale, ASD ). They are more common
cause of stroke in young man than elderly.They can be diagnosd with echocardiogram
and bubble study.

16.Thorough history taking in a pt with a fall is crucial in order to determine if the


underlying cause is syncope , mechanical or seizure. Thorough history taking,
physical examination (including orthostasis), and EKG reveal the cause of syncope
in about 50% of pts.

17.Tunnel vision in vasovagal syncope.

18.Meningovascular syphilis:
Its a condition in which low grade infection in the subarachnoid space can affect the
intracranial vessels and potentially result in stroke.Confusion, headache and stiff neck
are seen less frequent than in the more common meningitis form.Serum RPR and CSF
VDRL testing shd be performed to confrm the diagnosis of syphilis.Intravenous penicillin
is the appropriate treatment.

19.Dementia:
Memory loss accompanied by impairment in one of the following cognitive domains;
reasoning/planning, language, handling of complex motor tasks (praxis), visual-spatial
disorientation, or object and facial recognition.Clock drawing is difficult for many
dementia pts ( constructional apraxia ) & serves as a useful test when a full mini-mental
status examination cannot be performed.

20.Parkinsonism in Lew body dementia pts most commonly manifests as bradykinesia,


rigidity and gait abnormalities ; tremor z rare but common with idiopathic Parkinson
disease.

21.Lewy body dementia:


Is the second most common cause of degenerative dementia after Alzheimers disease.
To make the diagnosis, pts must have dementia and atleast 2 of the 3 core clinical
features: cognitive fluctuations (means sometimes they are fine and totally okay and on
other occasions they would be confused and disoriented), visual hallucinations (2/3rd of
pts) and parkinsonism.

Approximately half of the pts would have neuroleptic hypersensitivity ( worsening


parkinsonism, confusion and autonomic dysfunction ). They might have REM sleep
problems ( e.g vivid dreams ) and repeated falls. Syncope or near-syncope events
support the diagnosis.
BOTTOM LINE: Lewy body dementia is the second most common degenerative cause
of dementia after Alzheimer disease and is usually characterized by the
presence of progressive dementia, visual hallucinations, parkinsonism
features, early gait dysfunction and neuroleptic hypersensitivity.
Treatment of Lewy body dementia (LBD) overalps with that of both Alzheimer disease
and Parkinson dementia.LBD is treated with cholinesterase inhibitors (also used for AD)
and with antiparkinon medications such as levodopa and dopamine agonists (also used
for PD).
Pts with LBD are prone to visual hallucinations , which may be exacerbated by dopamine
agonist therapy.The hallucinations in LBD can be simple or complex & are often underreported by pts.These visual hallucinations usually regress when the dopamine agonists
are withdrawn.

22.Frontotemporal dementia presents earlier (pts are usually in their 50s) & manifests
as personality changes that initially predominate over cognitive function. Pts typically
will present with disinhibiton, personality changes, extreme agitation and urinary
incontinence.

23.Classic triad of normal pressure hydrocephalus (NPH) consists of dementia, urinary


incontinence & abnormal gait. Its a reversible cause of dementia that can be detected
on CT scan of the brain.Gait dysfunction in pts with NPH (magnetic gait) can appear
similar to a parkinsonism gait (shuffling).

24.Vascular dementia tends to present as progressive cognitive dysfunction. Memory


impairment in vascular dementia is frequently mild. Focal neurologic signs consistent
with stroke are frequently present (with or without a history of stroke).Parkinsonism
features may be seen,although rigidity & bradykinesia are typically more predominant
in the lower extremeties.

25.Atypical depression is characterized by leaden paralysis and interpersonal rejection


hypersensitivity.

26.Possible presentations of diabetic erectile dysfunction in the diabetic male include


diminished testicular sensation, bladder dysfunction and inability to masturbate.

27.The cremasteric reflex is regulated at L1-L2 level of the spinal cord.This reflex can be
diminished or lost secondary to diabetic autonomic neuropathy.
If the trauma will cause it, then we will have some additional features as well i.e loss
of hip flexion and adduction becox hip flexion and adduction are under the control of
L1-L2.

28.Symptoms of cauda equine syndrome include severe lower back pain, urinary or
bowel incontinence, motor weakness, or sensory loss in the legs bilaterally and
saddle anesthesia.

29.L5-S2-> Dorsiflexion and plantar flexion.

30.Vit-B12 defeciency:
Dementia & subacute combined degeneration (SCD) are potential complications of B12
deficiency that are reversible with vitamin supplementation in the majority of cases.
The dementia is of progressive nature that interferes with daily activities & dependency
on others.The pts have signs of dorsal spinal column dysfunction ( e.g impaired vibration
, positive Romberg sign ) & lateral corticospinal tract abnormalities (e.g spastic paresis,
hyperreflexia ) which are fairly specific for SCD.

31.Ineffective erythropoiesis is a well known phenomenon that occurs in pts with


Vit-B12 defeciency.The mechanism that leads to this phenomenon involves defective
DNA synthesis with megaloblastic transformation of bone marrow and intramedullary
hemolysis.Although intense erythroid hyperplasia occurs , the erythroid cells do not
mature normally and subsequently die in the bone marrow.Markers of hemolytic
anemia (e.g elevated LDH, indirect hyperbilirubinemia, low haptoglobin) may become
evident but reticulocyte response is typically absent in such pts.

32.Gilberts syndrome-> impaired hepatic conjugation.

33.Wernickes encephalopathy:
Characterized by the triad of confusion, ataxia & nystagmus (leading to opthalmoplegia),
though all 3 features may not be present always.It occurs due to thiamine deficiency &
is most commonly seen in alcoholics. Its a medical emergency and requires urgent
administration of thiamine either intravenously or intramuscularly.
The first step in the treatment of any alcoholic pt is the administration of thiamine,
( before glucose ) to prevent the onset or progression of Wernickes encephalopathy.

34.Benzodiazepines and beta blockers are beneficial for alcohol withdrawal

35.Korsakoffs psychosis:
Characteristic feature of this syndrome is confabulation. These pts confabulate in an
attempt to fill in the gaps in memory that they experience. Both anterograde and retrograde amnesia can occur, though anterograde amnesia is more prominent.
Korsakoffs psychosis is a common & preventable sequel of Wernickes encephalopathy
Thiamine, if given during the stage of Wernickes encephalopathy, can prevent the onset
of Korsakoffs psychosis. The administration of glucose prior to thiamine can precipate
Korsakoffs syndrome.In such pts, brain MRI frequently shows abnormal enhancement
of the mammillary bodies.

36.The involved area of the brain in pts with Wenicke-korsakoff syndrome is the
diencephalon, particularly the mammillary bodies and the thalamus.

37.Dementia is a progressive illness characterized by functional decline and impairment


across several cognitive domains. Medications can slow the progression but there is
no cure.

37.Alzheimers dementia:
Is the most common form of dementia. Onset occurs after age 60 and initially affects
memory and language predominantly.Pts will often attempt to cover these deficits with
confabulations or manifest irritability due to frustration with testing. Agitation, paranoid
delusions and disorientation can occur.Cognitive enhancing medications can sometimes
slow the progression of dementia but there is no cure.

38.Delusions of items being stolen ( often due to the pt forgetting where the item was
placed ) are common in moderate to severe dementia.

39.Symptoms of apathy, memory difficulty and apparent lack of interest seen in


Alzheimer disease can resemble depression.

40.Tuberous sclerosis: (TSC)


Is caused by mutation in 1 of 2 proteins , hamartin (TSC-1 gene) or tuberin (TSC-2 gene).
These proteins form a complex that plays a role in cell division and differentiation.
Mutation in these genes can be inherited in an autosomal dominant fashion or they can
be a de-novo mutation (>80% cases).
Affected pts can develop benign tumors throughout the body, including the skin (e.g
ash leaf spots <hypopigmented patches> , angiofibromas, shagreen patches), Central
nervous system (e.g glioneuronal hamartomas-the tubers of TSC), heart (e.g cardiac
rhabdomyomas), and kidney (e.g angiomyolipomas).Mild developmental delays can also
occur.
The initial evaluation of a pt with suspected TSC shd include a thorough cutaneous
examination, fundoscopy, and a brain MRI to evaluate for hamartomas.An EEG is also
indicated , as seizures are a predominant symptom that generally worsens over time.
Seizures can be focal, general tonic-clonic, or subclinical (e.g behavioural changes).
Normal EEGs serve as baselines for future testing , while abnormal EEGs require antiepileptic therapy.Abdominal imaging to evaluate for renal lesions is recommended.If
the diagnosis is uncertain, genetic testing for mutations in TSC-1 and TSC-2 can be
performed.
The natural history of Tuberous sclerosis complex is progressive and neurologic
impairment is the leading cause of death in pts with TSC.Pts usually have significant

developmental delays and uncontrollable seizures.Death may be due directly to tumors


(mass effect frm subependymal tumors-obstructive hydrocephalus) or from impairment
caused by tumors (e.g status epilepticus,aspiration pneumonia).Optimal seizures control
is strongly associated with prolonged life span.For these reasons, brain MRI & EEG shd
be obtained when TSC is initially diagnosed.
Renal involvement (obstructive angiomyolipoma-renal failure) is the second most
common cause of death after neurologic impairment.For this reason, abdominal imagng
is part of the initial evaluation for suspected TSC.
Rhabdomyomas develop in utero, asymptomatic and resolve in infancy. They dont
require routine Echocardiograms.

41.Perinauds syndrome:
A tumor of the pineal gland characteristically causes Perinauds syndrome, which is
characterized by the loss of pupillary reaction, vertical gaze paralysis, the loss of
optokinetic nystagmus and ataxia.The headache is a prominent feature and is due to
obstructive hydrocephalus.Some pineal tumors are germinatous and secrete HCG ,
which can cause precocious puberty in prepubertal males.

42.Craniopharyngioma is classically associated with diabetes insipidus and a deficiency


of one or more anterior pituitary hormones.

43.Frontal lobe tumors can cause loss of inhibition that cause the release of primitive
reflexes such as glabellar tap, grasp and palmomental reflexes.Frontal lobe tumors
can be associated with Foster Kennedy syndrome (optic atrophy on the side of the
tumor and papilledema on the contralateral side).

44.Multiple sclerosis:
>Mostly in young females.
>There are neurological deficits disseminated in time and space.Relapsing-remitting
is the most common pattern and is characterized by neurologic recovery in between
the episodes of acute exacerbations.
>50% of pts develop optic neuritis which typically presents with monocular visual loss
accompanied by eye pain that worsens with eye movement.
>Pts can develop transverse myelitis resulting in upper motor neuron signs and sensory
loss below the level of spinal involvement.
>Cerebellar involvement can also occur.
>Diagnosis is supported with MRI revealing hyperintense lesions in the brain and/or
spinal cord on T-2 weighted images.Ovoid-shaped periventricular white matter lesions
can be seen in 23-75% of pts.
>Evoked potentials, nerve conduction studies and EMGs have limited diagnostic utility.
>Lumbar puncture is not required for diagnosis & is usually performed in atypical cases
to confirm the diagnosis.Oligoclonal bands and elevated IgG index can be seen in
>90% of cases.
>Acute exacerbation;
=>Both oral and IV steroids are considered equally efficacious in hastening recovery
=>Pts with optic neuritis shd receive IV steroids as oral agents may be associated with
an increased risk of recurrence
=>Plasmapheresis shd be considered in refractory cases
>Beta-interferon or glatiramer acetate are indicated for chronic maintenance therapy in
pts with relapsing-remitting multiple sclerosis as they can decrease the frequency of
relapses and reduce the development of brain lesions.

>Pregnancy considerations of Multiple sclerosis;


=>Treatment is similar as those of non-pregnant pts
=>IV steroids are used acute exacerbations as they are generally well tolerated & not
associated with teratogenicity.
=>Majority of drugs used for maintenance therapy for MS are category-B (no evidence
of risk in available studz) or C (risk cannot be ruled out), so they can be used safely.
Teriflunomide and mitroxantrone are exceptions to this rule as they are both known
teratogens that are contraindicated in pregnancy.
=>Pregnant women with MS usually have lower disease activity during pregnancy, and
higher disease activity in the post-partum period.
=>Infants born to MS pts warrant special consideration as they have an increased risk
of developing MS (e.g 3-23% of MS cases are familial) and can be born with lower
birth weights.
=>Pregnant women with MS have a modest increase in C-section and assisted delivery
rates compared to those w/o MS.
>Muscle spasticity is one of the most common disabling symptom associated with MS.
It frequently involves the lower extremities and may result in pain/stiffness of the leg,
esp when rising out of bed or a chair.First-line treatment for severe spasticity includes
oral muscle relaxants such as baclofen or tizanidine, as these have similar efficacy and
tolerability.Non-pharmacological measures such as physical therapy and stretching
exercises may also be helpful.
>Amantadine is used in the treatment of MS-related fatigue.
>Gabapentin or duloxetine can be used for the management of neurpathic pains of MS
>Timed voiding, Fluid restriction of <2 L/day and anticholinergic medications (e.g
oxybutyinin/tolterodine) used for management of urge incontinence.

45.Acute dystonic reactions can occur in pts receiving dopamine antagonist therapy
(e.g antipsychotics, metoclopramide) and are characterized by sustained muscle
contractions resulting in twisting or abnormal postures (e.g torticollis).Benztropine
is an anti-cholinergic medication that can treat drug-induced dystonia.

46.Familial tremors/Benign essential tremors:


Its the most common cause of postural tremor and its incidence generally increases
with age.The familial cases usually have an autosomal dominant pattern and presents
at a relatively younger age.
The tremor is usually present in the distal upper extremities and becomes much more
pronounced with outstretching of the arm.It also increases at the end of an activity or
movement.All pts with essential tremors typically do not have any other neurological
signs. The absence of resting tremor (which worsens at rest but improves with voluntary
activity), rigidity (increased resistance to passive movements), bradykinesia ( slowing of
voluntary movements) or gait difficulty differentiates this condition from the early onset
of Parkinsons disease. The presence of a head tremor also suggests essential tremors.
Benign essential or familial tremors usually do not cause any significant disability & the
pt can expect to have a normal life expectancy.There are no associated neurological
symptoms in pts with benign essential tremors.
Beta-blockers (propranolol) are the first-line agents for symptomatic treatment of pts
with benign essential tremors.Medications only have a limited effect on symptom
control.They do not cause a complete resolution of the symptoms.

47.To correctly diagnose and treat pts with chronic headaches, the characteristics of the
headaches must be clarified. Such pts shd be instructed to make a headched diary for
one week & this shd include the headaches frequency, duration, intensity, associated

symptoms and medications used.

48. Cluster Headaches:


>Cluster headaches occur in clusters.Characteristic symptoms are severe retroorbital
pain, lacrimation, conjunctival injection, rhinorrhea, sweating, and pallor. Cluster
headaches can present as Horners syndrome.
>The acute management of cluster headaches usually begins with 100% oxygen
inhalation which is surprisingly very effective in the majority of the pts.
Another option for abortive therapy of cluster headaches is Sumatriptan,either
subcutaneous or intranasal (avoid in coronary artery dx pts).Ergot preparations &
NSAIDs have also been reported to be effective.
>Verapamil z the drug of choice for prevention of cluster headaches becx it is relatively
well tolerated with few side effects and has been evaluated in a randomized controlld
trial.Given the cardiac effects of Verapamil, a screening EKG shd be done at higher
level doses to assess for heart block or bradycardia.
>Prednisone and lithium are both alternative agents.

49.Temporal arteritis occurs in older individuals.

50.Picks dementia:
Is a slowly progressive frontal lobe dementia characterized by speech abnormalities
(e.g logorrhea, echolalia, aphasia, mutism), impaired executive functioning (e.g initiation
,goal setting , & planning), irritable mood, hyper oral behavior & disinhibition.Cognitive
function is largely normal.Neuroimaging studies reveal the classic finding of prominent
symmetrical atrophy of the frontal and/or temporal lobes.The condition may present in

a fashion similar to Alzheimers disease and the diagnosis is sometimes not confirmed
until the autopsy. Microscopic findings include gliosis, neuronal loss & swollen neurons
that may contain Pick bodies, which are silver-staining cytoplasmic inclusions.

51.Hutington disease is a slowly progressive degenerative brain disorder characterized


by chorea & behavioural disturbances. The chorea usually includes frequent, abrupt
jerks of the limbs or trunk. Grimacing and grunting may occur , & the gait is disjointed
choreic.Memory is typically preserved until the later stages of disease, but executive
functning, attention & judgement may be impacted early on.Depression & irritability
are common.

52.Microscopic findings in lewy body dementia in the brainstem and cortex include the
presence of Lewy bodies, which are cytoplasmic inclusions.

53.Common in alcoholics with chronic thiamine deficiencies,Wernickes encephalopathy


is characterized by mental impairment, ophthalmoplegia, horizontal nystagmus, and
cerebellar ataxia.The addition of memory loss and confabulatory psychosis define
Wernickes-Korsakoff syndrome.

54.A CT-scan of the head is the first step in the management of a pt with sudden-onset,
severe headache, elevated blood pressure and vomiting.

55.CT-scan w/o contrast is the first step in management of Subarachnoid hemorrhage.


If CT-scan z negative/equivocal, then LP with CSF analysis shd be done.The presence
of xanthochromia on LP is diagnostic of SAH.

56.Opiate withdrawal:
Opiate withdrawal can present as rhinorrhea, lacrimation, mydriasis, yawning, insomnia
, abdominal cramping, nausea, vomiting, diarrhea, body aches, arthralgias, myalgias,
piloerection, raised pulse, raised blood pressure and diaphoresis.
Opiate withdrawal can be managed with opioid agonists ( methadone, buprenorphine )
or non-opioid medications such as Clonidine.
Methadone is preferred over buprenorphine due to potential worsening of withdrawal
from buprenorphine (from its partial opioid antagonist effects).
Due to their high level f required supervision,Opioid agonists shd be used only for detox
in supervised inpatient or outpatient settings.So, if supervision is not possible, then we
would rather use Clonidine.
Vital sign abnormalities can occur with opioid withdrawal and shd be treated with opioid
withdrawal medications, followed by traditional blood pressure medications if pressure
remains dangerously elevated.Common adjunctive treatment includes benzodiazepines,
and antiemetics and antidiarrheals for symptomatic relief.

57.Amyotrophic lateral sclerosis:


Is the most common form of progressive motor neuron dx.Its a relentlessly progressive
disorder that involves both the lower motor neurons (consisting of anterior horn cells
in the spinal cord and brainstem neurons innervating the bulbar muscles) and upper or
corticospinal motor neurons.At its onset, ALS may involve selective loss of function of
only upper or lower motor neurons , but it ultimately causes progressive loss of both
upper and lower motor neurons.
The initial sign of the disease with lower motor neuron involvement is an insidiously
developing asymmetric weakness, usually first evident distally in one of the limbs.Pts
may disclose a history of recent development of cramping with volitional movements

that typically occur in early morning hours (e.g while stretching in the bed).Weakness is
associated with progressive wasting, atrophy of the muscles and spontaneous twitching
or fasciculations of motor units.Involvemnt of the bulbar muscles leads to difficutly with
chewing and swallowing , as well as fasciculations of the face and tongue.
ALS with prominent corticospinal involvement is characterized by hyperreactivity of
muscle-stretch reflexes (tendon jerks) & frequent spastic resistance to passive
movements of the affected limbs.
Ocular motility, sensory, bowel, bladder, & cognitive functions are preserved, even
with advanced disease.
Riluzole is a glutamate inhibitor that is currently approved for the management of
ALS. Although it cannot arrest the underlying pathological process, it may prolong
survival and delay the need for a trachestomy.

58.The decline in the level of cognition in vascular dementia is relatively abrupt and
progresses in a step wise fashion.

59.Binswangers disease is a type of vascular dementia that involves white matter


infarcts.Pts with this disease usually present with apathy, agitation and bilateral
corticospinal or bulbar signs.

60.Fibrinolytic therapy with tPA shd be considered for pts with symptoms of acute
stroke that have been present <3-4.5 Hrs. A rapid assessment must be performed
for bleeding risk ( and especially intracranial hemorrhage ), which is a serious concern
and a common side effect of tPA administratn.Several conditions including presence
of active bleeding and hypodensity in >33% of an arterial on CT scan , are absolute
contraindications to tPA administration.Other contraindications r bleeding diathesis

(e.g platelets <1,00,000/uL), intracranial surgery in the last 3 months & B.P >185/110.

61.All pts with acute stroke shd have an evaluation of the heart and neck vessels to rule
out the possible embolic sources.
>Duplex U/S can evaluate the carotids alone.Magnetic resonance angiography or CT
angiography can evaluate the carotids and intracranial vasculature.
>Electrocardiogram (ECG) is helpful to evaluate myocardial ischemia or arrhythmia
that may result in intracardiac thrombus formation and resultant embolus.
>Transthoracic echocardiogram is generally recommended to evaluate for the prznc
of an intracardiac thrombus.Transesophageal echocardiogram is more sensitive but
is invasive and more difficult to perform in pts with acute stroke.Cardiovascular MRI
for evaluation of intracardiac thrombus may be an option for some pts.

62.Subarachnoid hemorrhage is most often due to a ruptured saccular aneurysm.SAH


typically presents with sudden onset of a severe headache and possible nausea,
vomiting, brief loss of consciousness, focal neurologic deficits or meningismus.Pts
usually develop a suddenly worsening headache different from their typical headache
pattern, or experience a new headache described as the worst headache of my life.
The headache is typically lateralized to the side of aneurysm & can correlate to rapid
spread of bleeding.Non-contrast head CT is the initial preferred test for diagnosis,
followed by lumbar puncture to document xanthochromia if head CT is negative.

63.Head CT performed within 6-12 Hours of onset of SAH is nearly 100% sensitive for
diagnosing SAH, but this rapidly decreases with increased time from SAH onset and
smaller hemorrhages.

64.The typical CSF finding on LP in SAH pts include an elevated opening pressure,
xanthochromia (pink/yellow tint due to hemoglobin degradation products) or
consistently elevated RBCs count in the 4 CSF tubes.An elevated RBC count in
the first tube , followed by declining number of RBCs with each successive tube
suggests a traumatic tap.Retained CSF blood can take upto 6 hours to degrade and
cause xanthochromia ; for this reason, LP is most sensitive >6 hours from SAH onset.

65.Acetaminophen combined with aspirin and caffeine can effectively treat tension
headaches refractory to initial NSAIDS drugs.

66.Headaches can be divided into primary causes (e.g migraine, cluster headaches,
tensions headaches) and secondary causes (e.g infection , tumor, vascular). Pts
w/o red flag symptoms may be treated for primary causes but pts with positive
red flag signs and symptoms , such as papilledema , shd be evaluated for secndry
causes.
Increasing severity, red flags and papilledema prompts the consideration of a mass
lesion and neuroimaging.

67.Papilledema can occur in the following cases;


>Mass lesions
>Cerebral edema
>Increased CSF production
>Decreased CSF outflow (venous thrombosis)
>Idiopathic intracranial HTN (pseudotumor cerebri)
As the ICP remains elevated, optic cup margins become obscured and cotton wool spots
and splinter hemorrhages develop. Over time, the disc becomes pale, the hemorrhages

and exudates eventually resolve, and the cup margins remain permanently obscured.

68.Papilledema consequences;
=>Obscuration of disc margins
=>Obscuration of vessels
=>Splinter hemorrhages
=>Serpentine engorgement of small veins

69.Pseudotumor cerebri:
Aka idiopathic intracranial HTN, is a potential cause of headaches typically seen in
overweight women of childbearing age.Papilledema is classically present on eye
examination.A negative neuroimaging study (except empty sella) and an elevated
opening pressure on lumbar puncture (>20-25 cmH2O) can help to confirm the
diagnosis.
Suspicion->funduscopic ex. for papilledema->Neuroimaging (-ve)-> LP (inc openg press)

70.Carotid artery dissection:


A unilateral headache presenting with associated Horner syndrome shd be considered
a carotid artery dissection until proven otherwise.
The sympathetic nerves travel along the carotid artery; as a result, carotid artery disctn
is one of the important causes of Hornery syndrome.Carotid artery dissection refers to
the separation of the carotid artery wall layers & is seen with trauma, connective tissue
dx, smoking, neck manipulation, HTN, or 3-point restraint seatbelts in motor vehicle
accidents.Anhidrosis may be absent in internal carotid artery dissection (partial Horner
syndrome) becx sweat nerve fibers travel along the external carotid artery.
Head and neck vasculature imaging is the initial diagnostic modality of choice.CT

angiography is the reasonable 1st step; if results are negative but carotid dissection z
still suspected, either magnetic resonance angiography or catheter angiography (gold
standard) may be performed.
Management consists of antithrombotic therapy such as antiplatelet agents or anticoagulation.Stroke is an important potential complication of carotid artery dissection.

71.Tension headaches:
Is the most common headache syndrome.Characterized by non-throbbing headache wth
bilateral localization.There are no associated nausea and vision problems,physical ex. is
normal & there are no neurological abnormalities.They tend to occur on the week days
mostly and this may be associated with stress and overstrain at work.

72.Migraine headaches are generally throbbing in nature, family hx is usually +ve,


accompanied by aura, nausea, vomiting and photophobia.

73.Cluster headaches are always unilateral, and typically characterize in the periorbital
region.

74.Brain tumor headaches;


>bilateral but worse ipsilaterally
>Morning headaches
>nausea and vomiting
>headaches worsened by bending
>night awakenings
>neurological findings

75.Benign paroxysmal positional vertigo: (BPPV)


BPPV z defined as abnormal feeling of motion triggered by certain provocative positions.
The condition is most often attributed to canalithiasis, or the presence of calcium rocks
within the posterior semicircular canal.If there is substantial debris in the canal, then
linear accelerations (e.g gravity) my cause the endolymph to move inappropriately.This
results in the inaccurate sensation of spinning subsequent to movement of the head.
Although the condition is often idiopathic, it can also be associated with head trauma
or vestibular pathologies.
Imaging of the head is not indicated, as BPPV is a clinical diagnosis.
BPPV shd first be treated with the canalith repositioning procedure, which is a series of
maneuvers that move particles out of the posterior semicircular canal and into the
utricle.
Plugging of the posterior canal is a surgical procedure used in pts with with intractable
symptoms of BPPV.In most cases, the procedure reduces the functionality of posterior
semicircular canal w/o affecting hearing.

76.A number of nerve injuries can occur during carotid endarterectomy. Inadvertent
retraction or transection of the hypoglossal nerve causes tongue deviation towards
the site of injury.

77.Brain death:
>A brain death diagnosis requires a central nervous system catastrophe of known
etiology with an absence of confounding factors (e.g endocrine or electrolyte distrbncz
,drug intoxication) and absence of hypothermia.(pts with hypothermia require a
warming blanket to increase the body temperature to >36 C <96.8F> )
>Neurological ex. typically shows irreversible absence of cerebral and brainstem reflexes

including papillary, oculocephalic , oculovestibular (caloric), corneal , gag, sucking,


swallowing and extensor posturing.
>A positive apnea test can confirm brain death by documenting an absent respiratory
response off the ventilator for 8-10 minutes with a PaCO2 >60 mmHg (or >20 mmHg
from baseline) and a final arterial pH <7.28
>EEG shows electrocerebral silence or absence of somatosensory or brainstem evoked
potentials.
>Transcranial Doppler shows the absence of intracranial blood flow.
>Pts with brain death can have spontaneous movements originating from peripheral
nerves or the spinal cord.Possible reflexes include finger flexion, truncal movements
(e.g superficial and deep abdominal muscles), triple flexion response (flexion at the
hip, knee, and ankle with foot stimulation), plantar reflexes (Babinski sign), limb
movements to painful stimuli, and alternating flexion-extension of the toes.
>Absent cough on tracheal suctioning in brean dead pts.

78.Prolactinomas:
Dopaminergic receptor agonists (bromocriptine and cabergoline) are the first line
treatment for prolactinomas, including large prolactinomas. With such treatment ,
the tumor size decreases within days and the pts vision improves even before the
decrease in tumor size is appreciated on MRI. Once treatment with an agent is started,
the pt will require close monitoring of his clinical symptoms, prolactin levels, visual field
assessments and MRI of his pituitary gland.
Surgery is rarely required.

79.Parkinson disease:
>Its a clinical diagnosis that depends on the ability to recognize characteristic symptoms

and physical signs.


>The diagnosis requires the presence of bradykinesia in addition to tremor or rigidity
>A significant improvement in symptoms with dopaminergic medications is also highly
supportive and improves diagnostic accuracy.
>Other features that are suggestive of the disease include a unilateral onset and
persistent asymmetry of symptoms.
>Conventional brain MRI is of limited value in making a diagnosis of PD.However, it shd
be performed to evaluate for alternate conditions such as a stroke and hydrocephalus.
>The striatal dopamine transporter scan (DaTSCAN) is nuclear medicine scan that uses
ioflupane-123, a radioisotope with a high affinity for dopamine transporters located in
in the striatal region of the brain.Low uptake in striatal region has diagnostic accuracy
equal to that of physical examination and is typically reserved for pts in which the
diagnosis is equivocal.
>Dopamine agonists (Pramipexole, Bromocriptine) are preferred for younger pts <65
with less severe symptoms to save levodopa treatment for a later time when the pts
symptoms are more severe.
>Levodopa is only reserved for pts >65 who have severe symptoms affecting their
quality of life becx there are potential concerns regarding its prolonged use.Levodopa
may hasten the destruction of substantia nigra cells and worsen the symptoms in long
term.There might also be a limited response to levodopa and a higher incidence of
levodopa-related dyskinesia in pts with early onset PD.

80.Propranolol and primidone have similar efficacy in the treatment of essential tremor
and are considered first-line medications.

81.Myasthenia gravis:
Its a disease of the neuromuscular junction.The classic feature is fluctuating muscle
weakness that worsens throughout the course of the day.Most pts first notice double
vision and ptosis that worsens at the end of the day, which is caused by weakness in the
extraocular muscles.The next most common muscles are the bulbar muscles resulting in
dysphagia and dysarthria.Proximal limb weakness is found less frequently.Fatiguability
of muscle function is the key to differentiating myasthenia gravis from other similar
disorders.
Myasthenia gravis is an autoimmune disease caused by antibodies to acetylcholine
receptors.There is a known association between thymoma and MG.

82.Cerebral palsy: (CP)


The etiology is multifactorial and includes prematurity, birth asphyxia, and intrauterine
infections.CP is classified by the pts tone-spastic, dyskinetic, mixed or hypotonic and
further categorized based on which limbs are affected-hemiplegic, diplegic or quadriplegic.Spastic CP, characterized by spasticity, an upgoing Babinski reflex and hyperreflexia, is most common.
The typical history of a CP pt includes a premature birth, gross motor delay within the
first year of life and early hand preference.Physical ex. in infancy reveals hypotonia but
later progresses to spasticity.Hyperreflexia and persistence of primitive reflexes may
also be seen.Spastic diplegia is also characterized by a scissoring gait and toe walking .
Mental retardation and seizures may also be associated with CP.
CP is often diagnosed clinically based on the hx and physical ex.MRI of the brain is the
imaging of choice to look for abnormalities and a possible etiology for the symptoms.
MRI will show brain abnormalities in about 60-80% of infants with cerebral palsy.

83.Diaphragmatic paralysis can lead to orthopnea (shortness of breath that is worse in


the supine position) which can be easily confused with congestive heart failure.The
difficulty with lying supine can lead to poor sleep, daytime fatigue, and morning
headaches.Pulmonary auscultation is typically non-specific but findings of atelectasis
in the lower lobes can occur.One helpful clue on physical examination suggestive of
diaphragmatic paralysis is paradoxical abdominal wall retraction during inspiration
when the pt. is lying supine, which occurs becx the diaphragm is not contracting.A
sniff test using fluoroscopy can be helpful in confirming the diagnosis.
The most common cause of bilateral diaphragmatc paralysis are neurological diseases
such as ALS.Therefore, in pts with this presumed diagnosis,the presence of neurologic
signs and symptoms shd be sought elsewhere in the body.

84.Dementia;
=>Pts cannot care for themselves
=>They have changes in personality
=>Cannot draw a clock
=>Learning and memory impairment
=>Reasoning ability is impaired
=>Spatial ability and orientation is impaired
=>Language is impaired

85.In order to diagnose dementia, reversible causes must first be ruled out, including
hypothyroidism and B12 defeciency.Thus, thyroid function tests shd be one of the
steps in the evaluation of dementia.In alcoholics & nutritionally deficient populations
, vitamin B1 deficiency is another common cause of reversible dementia.

86.Periodic sharp wave complexes observed on EEG and the presence of a 14-3-3
protein in the CSF is helpful for the diagnosis of Creutz-feldt Jacob disease.

87.Alzheimers disease:
Pts with Alzheimer dx have decreased levels of acetylcholine due to degeneration of the
basal nucleus of Meynert in the forebrain and diffuse deficiency of choline acetyl
transferase, which is responsible for the synthesis of acetyl choline. Acetylcholinesterase
inhibitors circumvent this problem by inhibiting the counterregulatory breakdown of
acetylcholine, thus increasing the level of acetylcholine. Tacrine and Donepezil are
acetylcholinesterase inhibitors indicated for the treatment of Alzheimers dementia.
Donepezil offers advantages over Tacrine in that it is once per day dosing and has more
improvement in behavioural and cognitive domains as well as fewer side effects.

88.Simple febrile seizure:


Simple febrile seizures are generalized, brief (lasting less than 15 minutes) and occur in
children aged 6 months-5 years.These seizures r associated with elevated temperature
greater than 38C in children that are otherwise neurologically normal and display no
evidence of meningitis or encephalitis.Febrile seizures often occur on the first day of a
mild viral illness as the childs temperature is rising and may be the first manifestation
of illness.Family history can be positive.
Generally laboratory studies are not necessary for the evaluation of the simple febrile
seizure , although it may be appropriate to explore the cause of fever in children with
other symptoms (e.g severe diarrhea).

89.If the pt receives tPA for ischemic stroke;


=>They shd not receive antiplatelet therapy (aspirin) in the first 24 Hrs but could be

started after 24-48 Hrs if the pt is stable.


=>They shd not receive anti-coagulation with heparin/warfarin which is actually
contraindicated in such case in the first 24 Hrs
=>Blood pressure shd be kept below 185/105 mmHg and if its higher than that then
intravenous (not oral) medications such as labetolol, nicardipine and sodium
nitroprusside to avoid the risk of hemorrhagic transformation.
=>HTN upto a blood pressure of 220/120 mmHg z generally permitted in pts wth acute
ischemic stroke who did not receive thrombolytic therapy.
=>During the first 24 Hrs there shd be no invasive testing or procedures & the pt needs
frequent neurologic checkups.

90.Childhood absence epilepsy: (CAE)


The typical age of onset is 4-8 years of age.There would be no neurological signs.There
would be no myoclonic activity.The typical EEG pattern is 3/second generalized spike &
wave activity.A decline in school performance may be the initial presentation and
sometimes , these can manifest as behavioral changes.The history of staring spells is
typical.
Ethosuximide is the first line agent for the treatment of childhood absence seizures.
Valproate has a broad anti-epileptic spectrum and is usually reserved for pts with
absence seizures who also demonstrate other types of seizures.
Traditional anticonvulsants wont work for it infact they can exacerbate it (e.g
gabapentin).
The prognosis is generally good, especially if there are no associated generalized tonicclonic seizures.Staring episodes will disappear in teenage years.The risk of persistence z
higher in pts who develop generazlied tonic clonic sezures.

91.Unlike CAE, juvenile myoclonic epilepsy (JME) z characterizd by late onset of absence
seizures with myoclonic activity and is associated with life long seizures.

92.The causes of excessive daytime sleepiness are narcolepsy, sleep apnea syndrome,
periodic limb movements of sleep, insufficient sleep, or the effects of sedating meds.
Arriving at a specific diagnosis may be difficult if no associated clue (e.g history of
snoring or cataplexy) is present. Overnight polysomnography (PSG) seems to be the
best approach in managing pts with an impartial history.PSG evaluates the quality of
sleep and the recorded tracings can suggest the presence of obstructive sleep apnea,
periodic limb movements of sleep, or REM behavior disorder.

93.The management of pts with narcolepsy involves initial evaluation wth PSG, followed
by multiple sleep latency test.

94.Vertebrobasilar insufficiency:
Due to reduced blood flow to the base of the brain typically secondary to emboli,
thrombi, or arterial dissection.The labyrinth and brainstem are commonly affected and
symptoms may include vertigo, dizziness, dysarthria, diplopia and numbness.Risk factors
include diabetes mellitus, HTN, hypercholesterolemia, arrhythmia, coronary artery dx,
circulatory problems and a hx of smoking cigarettes.

95.Labyrinthitis is a mild , often self-limited condition characterized by vertigo , tinnitus,


nausea, and a loss of balance.The disorder often follows a viral illness (e.g influenza).
Labyrnthitis may also be cauzd by trauma, bacterial infection, allergies, benign tumors
and certain medications.

96.Nystagmus is commonly seen in BPPV.

97.Intentional head trauma: (IHT)


One of the most common forms of IHT is shaken baby syndrome (SBS).In SBS, violent
shaking of the infant causes his brain to suffer acceleration-deceleration injuries as it
collides against the inside of the cranium.As a result, retinal hemorrhages,often bilateral
and flame shaped, are found in 85% of pts with SBS.Note that newborn infants can have
retinal hemorrhages that are not caused by IHT but these usually disappear by 4-6 wks
age.Subdural hematomas can be present in both IHT and accidental head trauma.Other
findings, such as bruising, rib/metaphyseal fractures, or grab marks, may or may not be
present.Babies who are fussy and who are born to first time parents with no close family
support are most at risk for IHT.The parents involved often provide a questionable hx.
There would be a hx of multiple ER visits.Lethargy, vomiting and apnea can occur due to
increased intracranial pressure secondary to IHT.
Infants are particularly prone to dangerous sequelae from violent head shaking for
several reasons.First , they often have a large head to body ratio and weak neck musclz,
making their heads unstable and vulnerable to the whipping back and forth motion of
SBS.The infant brain is also incompletely myeilinized , making it more susceptible to
shearing forces during shaking.
CT of the head is a fast and cost effective method for evaluating infants with intentional
head trama to look for subdural hematomas which usually have a mixed-density pattern
on CT, in contrast to the homogenous pattern seen in accidental head trauma.Subdural
hematomas are crescent shaped overlying the brain on imaging & are the most common
type of intracranial bleed in this setting.

98.Status epilepticus:
Is defined as a single continuous seizure for >30 minutes or multiple seizures occurring
in a short period wth no return to baseline mental status.Common factrz that decrease
seizure threshold include medication noncompliance, intercurrent illness, electrolye
abnormalities, sleep deprivation and head injury.
An actively seizing pt shd undergo immediate assessment of the airway , breathing and
circulation. An intravenous line shd be placed in anticipation of medication administratn
and lab testing (e.g serum glucose to rule out hypoglycemia as the etiology).Abortive
therapy is recommended for seizures lasting >5 minutes, as delayed treatment can reslt
in increased risk of complications (e.g hypoxemia, academia). First line treatment is a
benzodiazepine (e.g lorazepam, diazepam) and shd not be delayed, even if there is
difficulty obtaining intravenous access.If sezure activity persists, fosphenytoin should be
administered.

99.Brain death:
Brain death requires clinical evidence of Central nervous system catastrophe of known
etiology with absence of confounding factors (e.g endocrine or electrolyte abnormalities
,drug intoxication) and hemodynamic instability (core body temperature >36C <97F>,
systolic blood pressure >100 mmHg)
I
Pts with positive clinical criteria shd undergo neurologic ex. to document absent cortical
and brainstem function (e.g coma,absent motor response to pain, absent pupillary light
reflex,absent corneal and oculocephalic reflexes, and absent cough reflex with tracheal
suctioning).
I
Pts with positive clinical criteria and abnormal neurological findings shd undergo apnea

testing to confirm brainstem failure.


I
Brain death is diagnosed in pts with a positive apnea test if they also meet local legal
requirements (e.g repeat neurologic examination in 24 Hrs, another neurologists
assessment to confirm brain death).

100.Facial palsy:
>Rapid onset of unilateral upper and lower facial weakness is consistent with Bells palsy
,idiopathic neuropathy of cranial nerve VII.Common findings of affected side include
inability to raise the eyebrow or close the eye, drooping of the mouth corner and
disappearance of the nasolabial fold.Due to stiffness/pulling of the face to one side, the
pt can report a funny feeling.
Corticosteroids within 3 days shd be started to have complete recovery.Eye care in the
form of eye patching and artificial tears is very important.Most pts wud recover within
1-6 months of onset.
>If the forehead or eye brows are spared, then intracranial lesion is suspected and brain
imaging is warranted in such cases.
>Lyme disease workup (serum ELISA) is required in cases of pts travel to endemic area,
bilateral involvement or other concerning features of Lyme disease.

101.Approximately, 20% of cases of dementia are due to potentially reversible conditnz.


In particular, hypothyroidism, vit B12 defeciency, and depression shd be ruled out in
the initial evaluation of dementia.
Hypothyroidism in elderly pts can present with hyponatremia, macrocytosis,
bradypsychia (slowed thought process) and slowed speech.

102.Meralgia paresthetica:
Is a very common syndrome that is caused by the entrapment of the lateral femoral
cutaneous nerve, which is a small, purely sensory nerve that is a direct branch of the
lumbar plexus.It can be compressed as it courses from the lumbar plexus, through the
abdominal cavity, under the inguinal ligament and into the subcutaneous tissue of the
thigh.Physical ex. typically reveals an area of decreased sensation over the anterolateral
thigh w/o any muscle weakness or deep tendon reflex abnormalities.
Its mostly a clinica diagnosis that is based on the typical hx and physical findings.Some
of the common causes are obesity and use of tight garments around the waist.Mngmnt
includes reassurance of the patient , weight loss in obese pts and avoidance of tight
fitting garments to reduce the pressure on the nerve entrapment area.

103.Femoral nerve;
=>anterior and anteromedial thigh paresthesia
=>Quadriceps muscle weakness
=>Decreased knee jerk reflex

104.Obturator nerve;
=>Sensory loss over the medial thigh
=>Weakness in leg adduction

105.L5-S1 radiculopathies are typically accompanied by back pain that radiates down
the lateral or posterior aspect of the leg.Weakness of the muscles supplied by the
corresponding segment is usually present.

106.Amaurosis fugax:
Is a sudden and transient monocular blindness due to an acute ischemic event involving
retinal artery.Ischemia will manifest as disc paleness. Even though its reversible, it z a
marker of carotid artery atherosclerotic dx that z usually advanced.Physical examination
will often reveal a carotid bruit.Carotid Doppler evaluation is necessary to evaluate the
extent of the disease and to assess the need for a carotid endarterectomy.

107.Pseudopapilledema may be seen in established nerve damage or in optic neuiritis

108.Acute retinal vein thrombosis will be seen as extensive hemorrhage in the retina on
funduscopic examination.

109.Children with hemophilia and other bleeding diathesis who are symptomatic at the
time of presentation are at incrzd risk of intracranial hemorrhage after even minor
head trauma.A head CT without contrast shd be performed emergently.Factor VIII
or Factor IX shd be replaced asap following the traumatic injury.

110.Do an emergent Head CT scan in cases of signs of basilar skull fracture;


=>CSF rhinorrhea
=>CSF otorrhea
=>Raccoon eyes ( orbital echymoses)
=>Battle sign (mastoid echymoses)

111.In the absence of trauma, elderly pts who require medical attention and report
multiple falls or have gait abnormalities shd be evaluated with a comprehensive &
multifactorial approach that includes evaluation of medication use, alcohol use,

chronic diseases, environmental factors, blood pressure and postural stability.

112.Elderly pts shd be asked annually about a history of falls.Pts who report a fall should
undergo a comprehensive evaluation including atleast one postural stability test (e.g
Get up and Go test).Pts who perfrm poorly or have recurrent falls may need more
extensive workup.

113.Guillian barre syndrome:


Is an acute inflammatory demyeilinating ascending polyneuropathy characterized by
progressive flaccid paralysis and areflexia.The pathogenesis is immunoligcally mediated
,with approximately 2/3rd of the pts presenting with a hx of upper respiratory or GI
infection in the preceding 2-4 weeks (campylobacter jejuni, CMV,EBV, or HSV infections)
The pt usually presents with progressive lower extremity weakness that may ascend
rapidly over days to involve the upper extremity, cranial nerves, and eventually, the
muscles of respiration.The physical examination reveals bilateral symmetric weakness or
flaccid paralysis and absent deep tendon reflexes in the extremeties.Approximately,
25-30% of pts with GBS will eventually develop neuromuscular respiratory failure and
need respiratory support with mechanical ventilation.
All pts with suspected GBS and clinical signs and symptoms of progressive respiratory
compromise shd be hospitalized immediately and closely monitored for signs of
respiratory failure.This includes continuous pulse oximetry monitoring, bedside
spirometry, arterial blood gas measurement and frequent clinical evaluation for signs of
respiratory fatigue.Measurements of bedside vital capacity provide the most useful
information regarding the degree of respiratory impairment.Respiratory failure is an
important and common cause of death in these pts.Urgent intubation & mechanical
ventilation shd be considered for pts with rapidly progressive respiratory compromise to

decrease the risk of complications and mortality.


Most pts with GBS have spontaneous remission; they either recover completely or have
only minor neurological deficits.Some of the factors associated with a poor outcome
include older age, rapidly progressive disease, prolonged mechanical ventilation and the
presence of associated co-morbidity.
After the airway is secured, specific therapy with either plasmapheresis or IVIGs shd be
instituted.
The indications for plasmapheresis are : severe flaccid paralysis, bulbar palsy, progressve
respiratory failure and pts on mechanical ventilation.Its most effective when it z started
within seven days of the onset of symptoms, however, some improvement still occurs if
it started late in the course of disease.Plasmapheresis is not usually indicated for mild or
non progressive dx in the ambulatory pts.
Daily administration of IVIGs for two weeks may also be used to treat pts with GBS becx
it has been shown to be as effective as plasmapheresis.
Corticosteroids have no role in the treatment of GBS.

114.Tick paralysis:
Caused by the attachment of a neurotoxin-secreting tick.Although this disease has been
reported worldwide, most cases occur in North America and Australia. The tick species
that cause most cases in the United States and Canada are Dermacentor andersoni (the
Rocky mountain wood tick) and D.Variabilis (the American dog tick). Symptoms develop
5-6 days after a female tick attaches to the patient. The typical presentatn z progressive
ascending paralysis that occurs over a matter of hours to days.Complete paralysis may
develop in severe cases.Fever is typically not present in tick-borne paralysis.Pupillary
abnormalities are uncommon in tick paralysis.If pt has a history of fever or prodromal
illness , the diagnosis is unlikely.Finding the attached tick on the skin is the most imp

diagnostic measure.The D/Ds for the tick paralysis are GBS,myasthenia gravis & botulism
In most cases of tick paralysis, removal of the tick will cause a substantial improvement
of the paresis within several hours.Extra vigilance is required when searching following
areas : the scalp, the axillae, the ears, buttocks and interdigital spaces.
Sometimes, the paralysis may worsen for 24-48 hours after the removal of the tick
(typically I.Holocyclus).In such cases, careful observation and supportive therapy shd be
provided.
Note: In GBS;
=>Paralysis occurs over a matter of days to weeks
=>CSF would show albuminocytological dissociation.

115.Botulism usually causes a descending paralysis, cranial nerves are affected early in
the course of disease and pupillary abnormalities are common.

116.A flaccid paralysis and paresthesias can point towards both GBS and myelopathy.
However, a sensory level and bladder/bowels dysfunction would make myelopathy
as the diagnosis.Myelopathy occurring after a URI would make it transverse myeilitis
These pts shd be evaluated immediately with an MRI.Steroids shd be administered
after the diagnosis is established with MRI. Steroids are given empirically if suspicion
for compressive myelopathy is high, as in malignancy.

117.Plain X rays of the vertebrae is indicated only in pts with low back pain without
neurologic deficits and who are at high risk of vertebral fractures (e.g trauma,
prolonged glucocorticoid use, osteoporosis , age >70).

118.Post-concussive syndrome:
Traumatic brain injury ( TBI ) of any severity can lead to post-concussive syndrome a few
hours to days later which includes the following constellation of symptoms: headaches,
confusion, amnesia, difficulty in concentrating, or with multi-tasking, vertigo, sleep
disturbances, hypersensitivity to various stimuli, mood alteration and anxiety. These
symptoms typically resolve with symptomatic treatment within a few weeks to months
following TBI; however, some pts may have persistent symptoms lasting >6 months.

119.Cannabis withdrawal involves atleast 1 physiological symptom (e.g abdominal pain,


sweating, shakiness, fever, chills, headache) and 2 or more of the following symptmz
developing after cessation of heavy and prolonged use : irritability, insomnia, decrzd
appetite, restlessness, depressed mood.

120.Blepharospasm:
Is a form of focal dystonia.There z typical history of periodic involuntary eye closure that
can be provoked by external stimuli (e.g bright light or irritants).Older women are esp.
predisposed to this condition.Potential explaination z that dry eyes in postmenopausal
women may serve as a trigger for blepharospasm.Botulinum toxin injections are the
mainstay of treatment.They can be used for many years without any loss of efficacy or
significant side effects.

121.Lesions of the thalamus or cortex are associated with sensory loss in contralateral
face and body.
Brainstem lesions typically involve the cranial nerves and impair sensory perception
over the ipsilateral face and contralateral body.

122.Wallen burg syndrome:


Occurs due to lateral medullary infarction from the occlusion of intracranial vertebral
artery.

Sensory findings include the loss of pain and temperature in the ipsilateral face and
contralateral trunk/limbs.There might be abnormal facial sensations and pain (early
symptom)

Vestibulocerebellar symptoms are most devastating.Pts often report difficult feeding


themselves.They often overshoot the targets due to ipsilateral limb ataxia.Pts report
vertigo (dizziness), falling towards the side of the lesion, difficult sitting upright w/o
support.They might have diplopia and nystagmus, both horizontal and vertical.

Ipsilateral bulbar muscle weakness can lead to dysphagia ( & aspiration), dysarthria,
dysphonia, & hoarseness (due to ipsilateral vocal cord paralysis), diminished gag reflex.

Autonomic dysfunction can occur and can lead to Horners syndrome (ptosis, miosis,
anhidrosis), intractable hiccups and lack of automatic respirations especially during
sleep.

Motor function of the face and body is typically spared.

123.Medial medullary syndrome (alternating hypoglossal hemiplegia) presents with


contralateral hemiparesis and contralateral loss of tactile, vibratory and position
sense AND ipsilateral tongue paralysis with deviation towards the side of lesion.
Its typically due to branch occlusion of vertebral or anterior spinal artery.

123.Lateral mid-pontine lesions affect the motor and principal sensory nuclei of the
ipsilateral trigeminal nerve, causing weakness of the muscles of mastication,
diminished jaw jerk reflex, and impaired tactile and position sensation.

124.Medial mid-pontine infarction presents with contralateral ataxia and hemiparesis


of the face, trunk and limbs (i.e ataxic hemiparesis).

125. Lateral medulla -> wallenburg syndrome


Medial medulla->Alternating hypoglossal hemiplegia
Lateral mid-pontine->5th C.N
Medial mid-pontine->Ataxic hemiparesis

126.Carbamazepine:
>Can lead to bone suppression leading to the development of neutropenia, aplastic
anemia and thrombocytopenia.Pts shd be made aware of symptoms such as fever,
mouth ulcers, easy bruising and petechiae.
>Elderly pts are at risk of SIADH.
>Becx of some mild anticholinergic effects, there is a risk of glaucoma, urinary
retention or constipation.

127.Psychogenic coma:
A normal reaction to caloric stimulation of the external auditory canal (oculovestibular
reflex) strongly suggests psychogenic coma.
Caloric stimulation of the vestibular apparatus is performed by irrigation of the external
auditory canal with cold water.A normal response is characterized by a transient,

conjugate, slow deviation of gaze to the side of the stimulus (brainstem mediated) ,
followed by saccadic correction to the middle (cortical correction).A caloric response
cannot be voluntarily suppressed; therefore normal oculovestibular reaction strongly
suggests psychogenic coma.

128.Acoustic neuromas result from the profileration of schwann cells (schwannomas)

129.Normal pressure hydrocephalus:


Is characterized by the classic triad of gait disturbance followed by impaired cognition &
urinary incontinence.Gait is usually wide based and can be described as being magnetic
due to the appearance of the feet being stuck to the floor.Cognitive impairment usually
follows gait dysfunction and can include early psychomotor slowing and problems with
executive function (e.g financial planning).Urinary incontinence is typically a late
manifestation.Typical findings on brain MRI include Ventriculomegaly out of proportion
to sulcal enlargement and the absence of obstruction.
Pts with a supportive therapy and ventriculomegaly on neuroimaging shd first undego
lumbar drainage of CSF. The lumbar tap test (Miller Fisher test) evaluates parameters
such as gait speed, stride length, verbal memory, and visual attention before and after
removal of 30-50 mL of CSF. Pts with improvement in any of the parameters have a high
probability of responding to definitive treatment with ventriculoperitoneal (VP) shunt
placement.If there is no improvement with CSF drainage, VP shunting may not be very
helpful.

130.Transverse myelitis :
Rapidly progressive weakness of the lower extremities following an upper respiratory
infection, accompanied by sensory loss and urinary retention, is characteristic for

transverse myelitis. Dull back pain may be present. Neurologic examination initially
reveals muscle flaccidity and hyporeflexia , but spasticity and hyperreflexia develop
subsequently. In pts presenting with acute transverse myelopathy, other causes (esp.
compressive lesions) shd be ruled out by obtaining a careful history and performing the
appropriate imaging (e.g MRI).

131.Traumatic Lumbar Puncture:


Traumatic LP results from accidental damage of a blood vessel during the procedure.A
high RBC count w/o xanthochromia is characteristic.An RBC count >6000/mm3 may
indicate traumatic LP.WBC elevation in traumatic elevation is commonly explained by
the blood leak if approximately one WBC is present per 750-1000 RBCs.The protein level
is elevated in the presence of traumatic LP and the glucose level is typically normal.
Xanthchromia is the discoloration of centrifuged CSF due to hemoglobin breakdown.
These are characteristic for SAH and appear 2-4 Hrs after RBCs enter the subarachnoid
space.These are present in more than 90% of pts within 12 Hrs of SAH.

132.WBC/RBC ratio not exceeding 0.01 has almost 100% negative predictive value for
meningitis.

133.Common tumors that metastasize to the spinal cord include prostate, lung, breast
,renal cell carcinoma and non-hodgkins lymphoma.

134.Epidural spinal cord compression:


Presents as weakness and sensory changes in the lower extremities.Bladder or bowel
dysfunction usually develop late in the disease but may present early in those with
compression at the level of conus medullaris.
High dose steroids shd be administered immediately to decrease edema and swelling
around the tumor tissue.
MRI shd be obtained emergently to obtain the diagnosis.Its also needed to assess the
spinal stability and plan definitive therapy.Generally, pts with an unstable spine and
radioresistant tumors shd undergo urgent surgical evaluation for decompression.Pts
with a stable spine & minimal neurologic symptoms or those with highly radiosensitive
tumors, such as lymphoma, may be treated with radiotherapy alone.

135.Lumbar puncture may be performed in pts with myelopathy of undetermined


etiology

136.Febrile seizures:
Children who experience febrile seizures are at ~30% risk of recurrence and at
silght increased risk (<5%) for subsequent development of epilepsy (e.g afebrile
seizures) compared to the general population.Febrile seizures have no effect on
intellectual outcome.
Diagnostic workup for simple febrile is not recommended due to the likelihood of
normal findings, lack of cost-effectiveness and potential to exacerbate caregiver
anxiety.

137.Narcolepsy:
The typical age of onset is late teens to early 20s.Narcolepsy shd be considered in a
young pt with excessive day time sleepiness, especially if the pt also experiences
falling asleep suddenly at inappropriate times, hypnagogic hallucinations ( hallucinations
with falling asleep e.g feeling of people walking around, hearing ones name called on
falling asleep) or cataplexy ( brief episodes of sudden bilateral muscle tone loss
precipitated by emotions such as laughter or joking OR can also present as spontaneous,
abnormal facial movements w/o emotional triggers ). There can be multiple awakenings
during sleep ( fragmented sleep). CSF analysis in Narcolepsy reveals Hypocretin-1
defeciency.

Polysomnography can diagnose narcolepsy; characteristic findings include multiple


spontaneous awakenings and reduced sleep efficiency (total sleep time divided by total
recording time) & latency of REM sleep (REM sleep latency z <15 minutes in narcolepsy)
Polysomnography can also exclude other causes of Excessive daytime sleepiness (e.g
sleep apnea).

Modafinil z generally preferred over older stimulants (methylphenidate, amphetamines)


due to lower abuse potential and low side effect profile, for treatment of narcolepsy.

Pts with cataplexy wud require serotonin-norepinephrine reuptake inhibitors (e.g


venlafaxine) or selective serotonin reuptake inhibitor or a tricyclic anti-depressant.

Sodium oxybate (salt form of gamma-hydroxy butyrate) can improve nocturnal sleep,
relieve excessive daytime sleepiness and has anti-cataplectic properties; however, it
z rarely used due to abuse potential and restrictive regulations.

Behavioural modifications like scheduled naps during the day and good sleep hygiene
are also important.

138.A sleep diary can document a pts sleep pattern over several weeks and is helpful
for evaluating circadian rhythm disorders (e.g delayed sleep phase, advanced sleep
phase)
Melatonin and light therapy are useful in such pts.

139.Identifying underlying depression in pts with Parkinsons disease is often difficult.An


empiric trial of SSRI anti-depressants could be helpful in uncertain cases before any
changes in medication for Parkinsons disease are considered, because depression
affects upto 50% of pts with Parkinsons disease.

140.Atypical neruoleptics can be used to treat hallucinations in pts with Parkinsons dx.

141.Gait types:
>Shuffling gait;
Its seen in Parkinsons disease.The pt takes small steps and almost never raises hz feet
from the floor while walking.When the pt stops walking or runs into an object, he
appears hesitant and his gait freezes.In more advanced cases of Parkinsons disease,
the shuffling gait is more severe and it occurs with both forward and backward
movements and is accompanied by postural instability.
>Senile gait;
Described as walking on ice.The pt has his feet wide apart, with his knees and hips
flexed and his arms flexed and extended, as if he were expecting to fall (also known as

cautious gait)
>Scissoring gait;
Occurs in spastic paraparesis.The pt drags the legs forward with every step.The gait is
slow , with hips flexed, legs straight, and with no flexion at the knees.It is called
scissoring gait due to its characteristic circular leg movements.
>Drunken sailor gait;
Is seen is cerebellar ataxia and is described as a jerky gait.Pt is hesitant and walks in a
zigzag pattern, wherein the rhythm of the steps is irregular.
>Steppage gait;
Occurs in distal lower motor neuron disease wth foot drop.There z excessive elevation
of the legs during walking. (toes touch the floor before heels).

142.Spinal Epidural Abscess:


It can be caused by hematogenous spread of bacteria frm a distant source, continguous
tissue infection (e.g vertebral osteomyelitis), or direct inoculation (e.g steroid injections,
epidural anesthesia). Other risk factors include IV drug abuse, and immunocompromised
pts such as diabetes, alcoholism and HIV. Staph. aureus accounts for about 60% of cases
of infections caused by pyogenic bacteria.
Presents with fever, focal spinal tenderness/back pain, and neurologic deficits like lower
extremity weakness and flaccid paralysis, sensory deficits and bladder/bowel dysfunctn.
Evaluation includes urgent MRI, complete blood count, blood cultures, & inflammatory
markers (ESR and CRP).
CT-guided aspiration and culture to guide the therapy and antibiotics are needed.
Emergency surgical decompression (preferably within 24 Hrs) & drainage of the abscess
are recommended for most pts.
(IV steroids are NOT recommended. They are for malignant spinal compression actually)

143.Phenytoin toixicity:
The earliest sign is the presence of nystagmus on far lateral gaze.Some other effects
include blurred vision, diplopia, ataxia, slurred speech, dizziness, lethargy and decreased
mentation, which progresses to coma.
Usual therapeutic range is between 10-20 mcg/mL and most pts will experience adverse
dose related neurotoxic effects with levels greater than 20 mcg/mL.
The first step in the management of side effects due to higher drug levels is to reduce
the dose or alter the treatment schedule to minimize the peak drug levels.

144.Abrupt cessation of an antiepileptic drug is not recommended.Any antiepileptic


drug that needs to be withdrawn due to intolerable side effects shd be tapered
gradually (over days to weeks) to prevent seizure relapse.

145.STROKE:
All stroke pts need swallowing evaluation screen before they can be given anything
by mouth. Stroke pts, particularly those with symptoms of dysarthria are at risk for
oropharyngeal dysphagia.
A quick swallowing screen evaluation (e.g water swallow test, Torronto bedside
swallowing screening test) can be performed at the bedside by nurses or other
trained providers.A more formal evaluation by a speech and language pathology
specialist can be conducted early in hospitalization as indicated and then followed
by a barium swallow study, if needed.
Deep venous thrombosis and subsequent pulmonary embolism are common causes
of morbidity and mortality in pts with acute stroke , particularly those with dense
hemiparesis.Low-dose Heparin or low-molecular weight heparin shd be used for
prophylaxis in most pts with acute ischemic stroke.

In hemorrhagc stroke,mechanical DVT prophylaxis (pneumatic compression devices)


would genereally be used initially.

146.Subarachonid hemorrhage involving the posterior communicating artery aneurysm


will cause CN-III palsy leading to ptosis, dilated pupil (aniscoria) and downward and
outward deviation of the eyeball.

147.Juvenile myoclonic epilepsy: (JME)


It usually affects healthy adolescents and often begins with myoclonic jerks of the upper
extremeties within the first hour of waking.Generalized tonic clonic seizures occur later
in disease course in almost all pts. Up to 50% of pts can have a concomitant psychiatric
diagnosis such as anxiety disorder.

EEG classically demonstrates bilateral polyspike and slow discharge during the interictal
period.Valproic acid is first line recommended treatment as it successfully suppresses
seizures in 80% of pts.It shd be used catiously in women of childbearing age due to
potential teratogenic effects.
Gabapentin is contraindicated in JME as it can exacerbate seizures.

148.Alzheimers dementia is a diagnosis of exclusion.That means to first rule out the


reversible causes namely a nutritional or other anemia, hyperglycemia, electrolytes
disturbances, hyper- or hypocalcemia, renal failure, and hypothyroidism.
Drugs for alzheimers dementia (tacrine/rivastigmine/galantamine/donepezil) would
take weeks before therapeutic effects are obtained.
There are some cases of Alzheimer dementia for whom neuroleptic drug (major
tranquilizer) may be prescribed for episodes of aberrant and/or socially disruptive
behavior that can occur.During such episodes, family members may also require
support and a sedative.

150.It is inappropriate to arrange institutional chronic care for or to treat (with an


acetylcholinesterase inhibitor) a confused elderly pt with impaired memory and
judgement until a dementia work-up has been completed to rule out reversible
causes.

PREGNANCY, CHILDBIRTH AND PUERPERIUM:


1.Group-B-streptococcus (GBS) is a common colonizing organism in the human GIT and
genital tracts and the most common cause of neonatal infection in the first week of
life. The most effective strategy to prevent intrapartum maternal-neonatal transmissn
z to perform GBS screening at 35-37 weeks of gestation & then administer intrapartum
antibiotic prophylaxis (IAP) during labor to those women with GBS colonization.
Women who r GBS negative do not need antibiotic prophylaxis for prolonged rupture
of membranes as over-treatment may result in antibiotic resistance or emergence of
other neonatal problems.
Women who miss screening and have an unknown status shd be treated in labor only
if they r at <37 weeks of gestation, develop an intrapartum fever or have rupture of
membranes for > 18 hours; otherwise their risk of transmission is low.
A pts GBS positive status from a prior pregnancy is not predictive of colonization in
the current pregnancy and is not an indication for IAP. The exceptions to screening
are GBS bacteriuria at any point during the current pregnancy or invasive early-onset
GBS disease in a prior child. These women shd receive IAP as their urogenital tract
colonization is more likely to persist and spread to the newborn.
Penicillin is the antibiotic of choice; it reaches therapeutic levels in the amniotic fluid
and fetus without toxicity and has a narrow spectrum of coverage, which minimizes
the risk of resistance. Prophylaxis shd be given 4 hours before delivery.
Indications of GBS treatment;

Prior birth to an affected infant with early onset GBS disease.


GBS bacteriuria or GBS UTI anytime during current pregnancy
GBS positive at 35-37 weeks of gestation
treat the unknown GBS if <37 weeks OR Intrapartum fever OR rupture of
membranes for >18 hours

2.Genital Herpes in pregnancy:


C-section is the mode of choice delivery in pts with genital herpes or prodromal
symptoms to prevent neonatal herpes. It does not completely eliminate the risk
of neonatal herpes but it does bring It down to 1.5% contrary to 7% with vaginal
delivery.
If a pregnant lady has a history of genital herpes, then even if she is
asymptomatic, she is still a candidate for prophylactic antiviral therapy against
herpes from 36 weeks gestation to delivery. The reason is to prevent the
outbreak of genital herpes again and hence prevent the need for C-section.

3.Exercise in pregnancy:
All healthy pregnant women with uncomplicated pregnancies are encouraged to
exercise for 30 minutes daily at a moderate intensity that allows the mother to
carry on conversation while exercising.
While water-immersion exercise is regarded as an excellent choice for pregnant
women, scuba diving is specifically prohibited in pregnancy due to the risk of
decompression injury and gas emboli in the fetus.

4.Anti-D immune globulin shd be administered intramuscularly at 28 weeks gestation to


all RH (D) negative women if the anti-D antibody screen is negative and the father is
not known with certainty to be Rh (D) negative.

5.Causes of raised serum AFP;


-> improper fetal dating
-> Multiple gestation
-> Neural tube defects such as spina bifida and anencephaly
-> Congenital nephrosis
-> Ventral wall defects
-> Dermatologic disorders and some tumors.

6.Serum AFP is done at 16-20 weeks gestation and is said to be raised when it is more
than 2.5 MoMs. The positive predictive value of this for neural tube defects z very low.
Pts with raised serum AFP shd undergo an ultrasound to check for neural tube defects,
which is done at 20 weeks of gestation in most centers of the world.
Amniocentesis would only be considered if ultrasound fails to show any neural tube
defects. Amniotic fluid AFP levels can then be measured (and also AchE levels) , with
elevated values being considered diagnostic. However, because of the high sensitivity
of ultrasound fr neural tube defcts, the use of amniocentesis after a normal ultrasound
is controversial.
Raised AFP -> Ultrasound -> If negative, go for amniocentesis.

7.The risk of transverse limb abnormality, a complication of chorionic villus sampling, is


greatest when the age of gestation is less than 9 weeks and lowest when the age of
gestation is greater than 11 weeks.

8.Superior Sagittal sinus thrombosis:


This is a rare condition that can be associated with trauma, infection, hypercoagulation,
vasculitis, nephrotic syndrome, severe dehydration, and pregnancy. The diagnosis is
suggested by history (severe headache, associated conditions), physical findings
(hemiparesis, papilledema, seizure) and imaging tests. MRI imaging and MR venography
are very useful for establishing the final diagnosis, which shows thrombus in superior
sagittal sinus.
The management of pts with venous sinus thrombosis typically includes adequate anticoagulation with Heparin, even if an area of hemorrhagic infarction is demonstrated on
CT scan. It is important to understand that the hemorrhagic foci that occur in this case
are secondary to venous hypertension.

9.Anti-epileptics and pregnancy:


Although anti-epileptic use is associated with increased risk of congenital
anomalies, 90% of the pregnancies go normal with anti-epileptic use.
We do not know as to which anti-epileptic is most teratogenic, therefore its
recommended to continue the current anti-epileptics that the pt is taking for her
epilepsy.
When it comes to Valproate, things are different. If the pt is not pregnant yet,
then change her valproate to some other anti-epileptic 6 months before a
planned pregnancy. But if she is already pregnant now, then dont change it and
just carry on with continued valproate use.
Discontinuation of or switching of anti-epileptics to other anti-epileptics is not
recommended during the pregnancy because it has minimum possible benefit,
can precipitate a seizure in the pt and also carries risk for the fetus becx of the
overlapping effects of multiple drugs.
For pts taking carbamazepine or valproate, high dose folic acid shd be started
ideally before conception. For other anti-epileptics, the standard dose of folic
acid shd be used. In pts taking anti-epileptics and not started folic acid yet, start it
asap.
Although anti-epileptics do get excreted into breast milk but taking anti-epileptic
medications is not a contraindication to breastfeeding.

10.Pts who have a first degree relative with spina bifida are at increased risk of having a
child with the same disease. The risk is markedly decreased by the administration of
folic acid prior to conception and during the first four weeks of pregnancy.
P.S : CVS is indicated for diagnosis of genetic disorders such as Downs syndrome but
not neural tube effects becx it does not measure AFP levels.

11.Early amniocentesis (performed before 15 weeks of age) is associated with a greater


risk of fetal loss, equinovarus foot and amniotic fluid loss.

12.Myelomeningocele:
Most pts are identified prenatally using ultrasound or MRI, although in some cases, the
diagnosis can be unsuspected until the child is born. The majority of the pts with
myelomeningocele also have hydrocephalus and Chiari-II malformations. An urgent and
thorough neurologic evaluation is needed. Surgical closure of the neural tube defects
must be done during the first 24-48 hours after birth in order to prevent the occurrence
of infections in the central nervous system. Spinal imaging, particularly with MRI, is
typically used in the presurgical evaluation of most spinal conditions. However,
myelomeningocele is an exception to this practice. Imaging is rarely performed given
the urgency of the condition, difficulties in performing an MRI on a potentially unstable
newborn, and ability to make the diagnosis based on physical examination. Imaging is
used in the follow up examination after the myelomeningocele is repaired.

13.Antenatal corticosteroid therapy has been proven to be effective in reducing the


perinatal morbidity and mortality associated with preterm labor. It reduces the risk
of infant respiratory distress syndrome & intraventricular hemorrhage. Intramuscular
administration of steroids provides stable and predictable concentration of the drug
in the blood that is required to achieve the desired fetal effects.

14.Most of the breech presentations assume a cephalic presentation by 36 weeks of


gestation. After 36 weeks, only 6% of the fetuses will be in breech position and
approximately 1/3rd will spontaneously convert to the cephalic position before the
delivery.
External cephalic version shd not be attempted until after 37 weeks of gestation. At
that time, version shd only be attempted if there are no contraindications to vaginal
delivery and fetal well-being has been established.

15. Some indications for caesarean delivery are lage fetus, hyperextended head, footling
breech and fetal distress.

16. Caput succedaneum and Cephalhematoma:


Caput succedaneum is a swelling of the scalp due to subcutaneous hemorrhage that
crosses the suture lines. It can cause blue discoloration of the scalp. Caput succedaneum
tends to both appear sooner after birth and resolve sooner than cephalhematoma.
Cephalhematoma is a scalp hemorrhage that is limited by the periosteal attachments to
a single cranial plate and therefore does not cross suture lines. It is associated with an
underlying skull fracture in 10-25% of the cases. It does not cause discoloration of the
scalp. The swelling is not visible for several hours after birth due to slow bleeding. It
resolves by 2 weeks to 3 months.
Both caput succedaneum and cephalhematoma can result from birth trauma & require
no treatmnt unless the hemorrhages are significant enough to cause hyperbilirubinemia.

17.HELLP syndrome (Hemolytic anemia, Elevated liver enzymes and Low platelets) is
a severe form of preeclampsia. Magnesium sulfate infusion is indicated for pts with
severe preeclampsia and HELLP syndrome becx it significantly reduces the risk of
seizure.

18.Although 50% of all cases of eclampsia occur prior to the term but the risk is
significant within first 24-48 hours after delivery.

19.Retroverted Uterus:
Retroverted uterus is a common condition found in 11% of the female population. It is
not an etiologic cause of abortion, regardless of the trimester of pregnancy. If this
condition is secondary to PID or endometriosis, it can be associated with infertility, but
not abortion. Retroverted uterus is also related to dyspareunia, low back pain and
dysmenorrhea. Retroverted uterus will reposition into an anterior position between
12-16 weeks of pregnancy but repositioning does not happen in 1.4% of cases. In such
cases, physicians have to manually reposition the uterus to avoid uterine incarceration
in the sacral region, which later could jeopardize the lives of the mother and the fetus
and subsequently require surgery. Otherwise surgery is recommended only for pts with
dyspareunia related to a retroverted uterus.

20. Hyperthyroidism management in pregnancy:


Methimazole is a potential teratogen and can cause scalp defects, choanal atresia and
tracheoesophageal fistula.
During the first trimester, pregnant pts shd be switched to propylthiouracil, which
carries less risk of birth defects. For the second and third trimesters, pts are often
switched back to methimazole becx propylthiouracil carries a risk of liver failure.
Surgery and radioiodine shd be avoided in pregnancy.

21.Postpartum endometritis:
It is a polymicrobial infection of decidua (the pregnancy endometrium) characterized by
fever, uterine tenderness, foul smelling vaginal discharge, and leukocytosis. As the
infection is often produced by both aerobes and anaerobes from the genital tract, any
treatment regimen must include broad spectrum antibiotics that also cover beta
lactamase producing anaerobes. The gold standard of treatment for endometritis is

clindamycin and gentamicin.


The most important risk factors in the development of endometritis is the route of
delivery. Endometritis occurs after 3% of vaginal births but after 15-30% of caesarean
deliveries. Several causes contribute to this finding, including contamination of the
uterine cavity, prolonged rupture of membranes and presence of sutures or other
foreign bodies.

22.Metronidazole is contraindicated in breast feeding mothers.

23.Antenatal corticosteroid therapy shd be given to any pregnant woman from 24 to 34


weeks of gestation with intact membranes at high risk for preterm delivery.
Betamethasone (and sometimes dexamethasone) is used for antenatal corticosteroid
therapy.

24.Bacterial vaginosis:
It is an imbalance in the normal vaginal bacterial flora associated with increased
Gardnerella vaginalis, Mycoplasma hominis and anaerobic bacteria, along with a
corresponding decrease in hydrogen peroxide-producing lactobacilli. BV is diagnosed
when 3 of the following criteria are met:

Homogenous vaginal discharge


Vaginal pH >4.5
Amine odor with application of KOH
Presence of clue cells on microscopic examination of a wet mount.

Although many women are asymptomatic, BV can cause excessive white/gray vaginal
discharge with a foul fishy odor, especially after unprotected intercourse. A course
of oral metronidazole or clindamycin will improve the microflora imbalance but
recurrence is common. Pts shd be informed that metronidazole crosses the placenta

but has no known teratogenic effects.


Aymptomatic pts do not need to undergo screening.
Bacterial vaginosis during pregnancy is associated with complications like preterm
delivery, premature rupture of membranes and spontaneous abortion. Treatment of
bacterial vaginosis with metronidazole/clindamycin is effective in relieving the symptmz,
however it has not been shown to reduce the risk of preterm delivery. So thats why,
pregnant pts with abnormal vaginal discharge shd be tested for BV and then treated just
to relieve symptoms and not for sake of reducing these complications. This is also the
reason we dont screen asymptomatic pts for BV, simply becx we cant reduce the
complications from BV through our treatment. All we do with our treatment is to relieve
the symptoms of pts.
Sexual activity is a risk factor for BV but otherwise BV is not an STD. and besides that,
if the lady gets BV, we just need to treat her and not the male partner becx the males
do not experience symptoms and the partners treatment does not seem to affect the
pts recovery course.

25. Subchorionic hematoma:


Ultrasonography is the best means of discerning bleeding between the endometrium
and the gestational sac in the pregnant woman, a condition termed subchorionic
hematoma. Subchorionic hematomas appear on sonography as crescent shaped
hypoechoic regions adjacent to the gestational sac & are the most commonly identified
source of first trimester bleeding. Women with this condition are managed expectantly
until the symptoms resolve or additional findings develop. Once a subchorionic
hematoma has been diagnosed, it shd be re-evaluated with a repeat ultrasound one
week later.
There are no known therapeutic interventions for this condition.

Pregnant women who have subchorionic hematomas r at increased risk for experiencing
spontaneous abortions. There appears to be a correlation between this risk and larger
hematoma size, increased maternal age and earlier gestational age.

26. Kleihauer-Betke test is used to measure fetal cells in maternal circulation. This
test is indicated when there is a large antepartum bleed in a mother who is Rh -ve.

27. First trimester bleeds are associated with increased risk of preterm births,
premature rupture of membranes, and growth restriction of fetus.

28. Transverse limb anomaly occurs in children with the rare autosomal dominant
disorder called Adams-Oliver syndrome.

29.Down syndrome:
Trisomy 21. Increased risk with increased maternal age i.e >35. Most common cause of
mental retardation in children.
The integrated test is the best screening test for Down syndrome. It is comprised of
ultrasound measurement of nuchal translucency thickness at 10 weeks and the
measurement of serum markers from the first trimester (PAPP-A) and second trimester
(AFP, hCG, unconjugated estriol and dimeric inhibin-A).
Actual definitive diagnosis of Down syndrome is made by examining the fetal karyotype
, which requires chorionic villus sampling or, more commonly amniocentesis.

30.Polycystic ovarian syndrome:


Anovulation is the principal cause of infertility in these pts. Weight reduction in
overweight or obese pts with this syndrome is associated with restoration of ovulation

, decrease in androgen production and pregnancy.


If weight reduction fails to restore the ovulation, clomiphene citrate can be tried.

31. Severe preeclampsia presents with blood pressure >160/100 mmHg, headaches and
blurred vision. Pt may have edema of hands and feet. Hyperreflexia is an ominous
sign and usually heralds eclampsia.
Pts with severe preeclampsia & near to term shd be stabilized with anti-hypertensive
(hydralazine/labetolol) and seizure prophylaxis (MgSO4). Once they are stable,
delivery is performed afterwards. The mode of delivery depends upon the individual
case.
Pts with severe preeclampsia and remote from term are best managed in a tertiary
care center or by referring them to obstetricians who specialize in the management
of high risk pregnancies.

32. All pregnant drivers should wear seat belts with both lap and shoulder straps.

33.A hx of preeclampsia in the past increases the risk of developing this complication
during a subsequent pregnancy. The risk is higher if the preeclampsia presented
earlier (age of delivery was less than 32-33 weeks) or if the pt has renal disease or
chronic hypertension.

34. Folic acid:


All women who wish to get pregnant shd take 0.4 mg of folic acid daily for > 1
month before the projected date of conception.
High risk women such as those taking anti-eplileptics and those who previously
had a baby with neural tube defects shd take 4 mg of folic acid for > 1 month
before pregnancy.

35.Pregnancy may be delayed for > 1 month after discontinuation of OCPs becx the
pt may continue to have anovulatory cycles.

36.Septic abortion:
Septic abortion presents wth fever, chills, lower abdominal pain, bloody, purulent
vaginal discharge, and a dilated cervix.
The emergency management includes intravenous fluids, broad spectrum antibiotics
(e.g monotherapy with piperacillin-tazobactam or imipenem, triple therapy with
clindamycin, gentamicin and ampicillin), and blood and endometrial cultures.
After the pt is stabilized with intravenous fluids and broad spectrum antibiotics,
uterus must be evacuated promptly by suction curettage.
An emergency hysterectomy may be necessary if the pt fails to respond to antibiotics
and suction curettage. (Other indications for hysterectomy include the development
of a pelvic abscess or signs of clostridial sepsis i.e crepitus of pelvic tissue, radiographic
evidence of air in the uterine wall).
Untreated septic abortion can lead to salpingitis, peritonitis, septic shock and death.

37. Bipolar mania in pregnancy:


Haloperidol is safe and effective treatment for severe bipolar mania during pregnancy.
Second line treatments include lithium, 2nd generation antipsychotics (e.g quetiapine,
risperidone) and electroconvulsive therapy. Treatment decisions vary from case to case
depending on acuity, trimester and patient preference.

38.Cervical insufficiency:
Cervical insufficiency is the inability of the cervix to hold a pregnancy in the absence of
labor and places a woman at jeopardy for preterm birth. Risk factors for cervical
insufficiency include collagen abnormalities (e.g Ehlers Danlos syndrome), Uterine
anomalies (e.g septate uterus, bicornuate uterus), prior obstetric trauma (e.g cervical
laceration from delivery), prior mechanical cervical dilation (e.g dilation and curettage,
pregnancy termination), and treatment of cervical intraepithelial neoplasia by
conization or loop electrosurgical excision procedure.
High risk pts shd undergo serial ultrasound for cervical length and evaluation for
cerclage placement during 16-24 weeks gestation.
Diagnosis can be made by history of painless dilation and a second trimester loss,
detection of cervical dilation on physical examination in the second trimester or a
combination of history of preterm birth and current cervical length <25 mm between
16-24 weeks gestation.

39.Postpartum hemorrhage: (PPH)


Uterine atony is the cause in 80% of cases, so it shd be presumed to be the cause
whenever there is PPH.
Fundal massage helps stimulate uterine contractions in an atonic uterus and shd be
started while other interventions are being prepared.
Oxytocin, a uterotonic, shd be started right away if not started before. If it was being
given before during labor, then the dose shd be increased.
Other uterotonic medications (methylergonovine, carboprost and misoprostol) shd be
reserved as alternatives.
If initial measures are unsuccessful, alternative etiologies shd be considered like
perineal lacerations, retained products of conception, coagulation defects and uterine

rupture. Pelvic examination and uterine ultrasound shd be performed to check for these
causes.
For retained products of conception, the ultrasound wud show the presence of an
echogenic mass or absence of normal endometrial stripe (less consistent finding).
Curettage is the treatment of choice for retained products of conception.

Bottom line: The majority of PPH cases are caused uterine atony. Fundal massage,
uterotonic medications, fluid resuscitation, and transfusion as necessary
are the primary interventions in PPH. Other etiologies (e.g perineal
lacerations, uterine rupture, retained products of conception, and
coagulation defects) shd be considered in pts in whom initial measures at
controlling PPH are unsuccessful.

40.Group-B-streptocouccs = Strpetococcus agalactiae.

41.Asymptomatic bacteriuria in the pregnant female must be promptly treated with


antibiotics. A positive urine culture for GBS in a pregnant female shd be treated
immediately with 10-day course of antibiotics. The antibiotic may either be
penicillin-G or cephalexin, both antibiotics are considered safe for the fetus.
Delays in treatment allow the bacteriuria to progress to cystitis or pyelonephritis,
conditions associated with preterm labor and premature birth.

42.Shoulder dystocia:
Difficulty in delivering the anterior shoulder is called shoulder dystocia. The incidence
is <2% and is associated with multiple neonatal morbidities (e.g clavicular fracture,
brachial plexus palsies). Maternal factors that increase the risk of shoulder dystocia
include diabetes, post term pregnancy, history of shoulder dystocia in a prior pregnancy
, protracted active phase and prolonged second stage.
In managing shoulder dystocia, first ask the pt to not to push. The pts legs should then
be placed in the knee/chest position i.e sharp hip flexion while in supine position. This
is called McRoberts maneuver and this will straighten the lumbosacral angle and rotate
the symphysis pubis anteriorly. Another assistant then applies suprapubic pressure
downward and laterally on the babys anterior shoulder to dislodge it.
If the anterior shoulder does not deliver despite the McRoberts maneuver, then the
birth attendant shd consider cutting an episiotomy to facilitate additional maneuvers.
Following maneuvers may be applied then;
Delivery of posterior arm
Pressure against babys posterior shoulder either anteriorly or posteriorly &
anterior rotation (Woods corkscrew or Rubin maneuver)
Placing the mother on all fours- Gaskin maneuver
Replacement of babys head on vagina followed by Caesarean deliveryZavanilli maneuver- is the last resort.

43.Pruritis in pregnancy:
Pruritus is a very common complaint during the pregnancy. It is reported to affect up to
20% of pregnant women. The symptom may be a manifestation of pregnancy associated
dermatosis, although there is usually no pathologic process present in most cases.
Common pruritic locations are the scalp, anus, vulva and abdomen (during the third
trimester). Pregnancy-induced pruritus may be related to dermographism or urticaria,

which is common in the last half of pregnancy. Topical steroids, antihistamines, oatmeal
baths, emollients and UVB are used to treat the condition.

44.Papular urticarial papules and plaques of pregnancy (PUPP) is a relatively common


pregnancy associated dermatosis that is characterized by pruritic erythematous
papules within the striae gravidarum. The lesions may spread to involve the
extremities.

45.Herpes gestationis:
Herpes gestationis (also called pemphigoid gestationis) is an uncommon blistering
dermatosis that is associated with pregnancy. It typically starts during the second or
third trimester or post-partum period. It may first manifest as abdominal pruritus,
a relatively common benign condition during the pregnancy, with later development
of a rash. The rash is localized around the umbilicus and is characterized by papules,
urticarial plaques and vesicles, although bullae may also form. Contrary to its name,
herpes gestationis is not caused by a viral infection and is believed to be an autoimmune
disorder.
Bottom line: The rash of herpes gestationis is localized around the umbilicus and is
characterized by papules, urticarial plaques and vesicles.
In herpes gestationis, autoantibodies can be detected in the skin as well as circulation.
Corticosteroids are the mainstay of therapy. In early and mild cases, topical mid-potency
steroids (e.g triamcinolone) are used. In more advanced cases or if topical steroids are
not effective, systemic steroids are employed.
Oral antihistamines can be added to the treatment regimen to alleviate pruritic
symptoms.

46. important point !!!!!


Topical steroids & antihistamines are widely used to treat many pregnancy associated
dermatoses, including papular urticarial papules and plaques of pregnancy (PUPP);
therefore, these are typically the correct choices for questions asking about treatment
of pregnancy associated dermatoses !

47.Intrahepatic cholestasis of pregnancy-> Severe generalized pruritus->


Ursodeoxycholic acid.

48.Neonatal breast hypertrophy is a common finding in up to 70% of male & female


neonates. High levels of maternal estrogen cross the placenta during the third
trimester, but the level in the newborns blood decreases after delivery, stimulating
the neonatal pituitary to produce prolactin. Prolactin stimulation causes unilateral
or bilateral breast enlargement in the neonate. Breast tissue is palpable and milk may
be expressed when manipulated (galactorrhea). Neonatal breast hypertrophy and
galactorrhea are self limited and usually resolve within 6 months. In newborn girls,
other effects from maternal estrogen include labial swelling, leukorrhea & uterine
withdrawal bleeding. Parents shd be discouraged from expressing the milk, which
can prolong galactorrhea by stimulating prolactin and oxytocin release from the
pituitary. Inappropriate breast manipulation can also predispose to infection or
galactocele formation.

49. Isoummunization:
Events that are associated with feto-maternal hemorrhage (such as placental abruption)
may require adjustments in the dosage of anti-D immune globulin. Therefore, the
presence and amount of feto-maternal transfusion shd be determined in such cases.

The rosette test is a qualitative test that helps determine the presence of feto-maternal
hemorrhage. If -ve , the standard dose of anti-D immune globulin shd be administered.
If positive, the amount of hemorrhage can be evaluated using Kleihauer-Betke test or
fetal red cell stain using flow cytometry, and the dose of anti-D immune globulin shd be
corrected accordingly.
Failure to adjust the dose of anti-D immune globulin after events that are associated
with excessive feto-maternal hemorrhage may result in maternal alloimmunization.
If the anti-D antibody levels are > 1:16 , then the mother is said to be sensitized and we
can say, that the dose of anti-D immune globulin given to her was inadequate.

50.Diaphragmatic paralysis of newborn:


Diaphragmatic paralysis of the newborn usually results from a phrenic nerve injury. The
two most common causes of phrenic nerve injury are birth trauma and cardiothoracic
injury. The birth trauma of phrenic nerve is typically accompanied by the signs of
brachial plexus injury, such as Erbs palsy (I think both brachial plexus and phrenic nerve
come from the same roots, so thats why both get injured together).

51.Ipsilateral and less frequently, contralateral hypoplastic lung is characteristic for


diaphragmatic hernia. Mediastinal shift to the unaffected side typically occurs.

52. Sacral agenesis is a rare anomaly observed in children of diabetic mothers.

53.Pregnancy in Eisenmenger syndrome pts and other cyanotic heart disease pts:
Pts with ASD/VSD/PDA have left to right shunting. As pulmonary vascular resistance
(pulmonary hypertension) gradually increases, the shunt direction reverses, resulting
in permanent right-to-left shunting and cyanosis.

Pregnancy in the setting of Eisenmenger physiology, pulmonary hypertension or other


congenital heart diseases is associated with a significant risk of maternal and fetal
morbidity and mortality (up to 50%). During pregnancy, normal physiologic changes
include expansion of the intravascular volume, increased cardiac output, and decreased
systemic vascular resistance. The combination of these changes can exacerbate an
existing right-to-left shunt. Most maternal deaths occur during first week postpartum
as the sudden drop in systemic vascular resistance after delivery increases the
right-to-left shunt and worsens the hypoxemia. Elective termination shd be discussed
due to the markedly high risk of death to the woman and fetus. Elective termination is
safest if performed in a hospital during the first trimester of pregnancy.
Pts with cyanotic heart diseases shd be educated about these risks and counselled on
contraception options. Hysteroscopy is a safe method for permanent sterilization.
Pts who prefer reversible contraception can be offered a progestin implant, injection
or pill. Combination estrogen/progestin OCPs are contraindicated due to the potential
for thromboembolism.
Bottom line: Eisenmenger syndrome and other cyanotic heart diseases in pregnancy are
associated with high maternal and fetal mortality. Hysteroscopic sterilizatn
or progestin contraceptives shd be offered to prevent pregnancy. Estrogen
containing contraception is contraindicated due to thromboembolic risk.
Elective termination shd be discussed with pregnant pts who have severe
cardiac disease.

54.Pregnant pts whose partners have a known history of Herpes, type-specific antibody
testing for HSV-1 & HSV-2 is the most appropriate method to determine if they have
been infected. Testing will identify infected pts with a sensitivity of 96-100% and
specificity of 97-98%. Pts with positive serology then can be prescribed prophylactic

antiviral therapy beginning at 36 weeks.

55. Pregnancy outcomes in Adolescents ( < 20 age ):


Adolescents r at increased risk of adverse pregnancy outcomes, which include perinatal
mortality, preterm delivery and premature and low birth weight infants. It is unclear
whether these outcomes are due to socioeconomic status (e.g low education and low
income) or biologic immaturity. Moreover, the children of adolescent mothers may be
at increased risk of severe socioeconomic outcomes, including future cognitive disordrz.
Adolescent pregnancy does not increase the risk of congenital malformations.

56.Twin pregnancy:
In case of dichorionic/diamniotic or monochorionic/diamniotic twins, go for
vaginal delivery and expectant management if the first twin is vertex
presentation and the second is also vertex, provided the fetal heart rate tracing
is reassuring (e.g category 1).
When do you want to go for caesarean in twin pregnancy;
Monochorionic/Monoamniotic
first twin is in breech presentation (Breech/vertex OR Breech/Breech).
Vertex/Breech presentation if the non-presenting twin i.e Breech one is either
<1500 g or > 20% estimated fetal weight of the presenting twin
Non reassuring fetal heart rate tracing of either twin.

57.Indications for forceps delivery;


Prolonged or arrested 2nd stage of labor
Non reassuring fetal status
Maternal cardiac/neurologic disease.
Forceps delivery is contraindicated if the amniotic sac is intact.

58.Internal podalic version is a set of maneuvers used prior to delivering a non-vertex

second twin who is still a candidate for vaginal delivery. It entails manipulating the
fetus inside the uterus from a transverse or oblique lie to a breech presentation to
grasp the feet and deliver it breech per vagina.

59.Diamniotic twins who are in a vertex/vertex presentation are ideal candidates for
vaginal delivery. If the second twin is in vertex presentation and has a normal heart
tracing, labor of the second twin may be managed similarly to a singleton pregnancy.

60.GBS colonization:
Intrapartum antibiotic prophylaxis (IAP) is considered adequate if ampicillin,
penicillin or cefazolin is administered > 4 hours before delivery. Vancomycin,
clindamycin or other antibiotics are not considered adequate IAP due to
increasing bacterial resistance and slower distribution into the tissue.
All infants born to mothers with indications for GBS prophylaxis should be
observed in the nursery for > 48 hours to monitor for signs of infection (e.g
lethargy, poor feeding, temperature instability) regardless of whether
adequate IAP was administered.
A complete blood count and a blood culture should be obtained if the infant is
preterm (<37 weeks gestation) or if there was prolonged rupture of
membranes (> 18 hours).

61.Second stage arrest in labor:


For nulliparous women, second stage arrest of labor is defined as lack of progress
towards delivery of the baby after > 3 hours of pushing w/o epidural anesthesia or
>4 hours of pushing with epidural anesthesia.
For multiparous women, diagnosis is made by 1 hour earlier in the presence or absence
of epidural analgesia ( > 2 hours w/o an epidural and > 3 hours with an epidural).
Cephalopelvic disproportion is the most common etiology of arrest disorders.
Cesarean delivery is the treatment of choice for arrest disorders.

Operative vaginal delivery with forceps/vacuum is contraindicated when cephalopelvic


disproportion is suspected.

62.Screening in pregnancy:
=>Recommended tests in first trimester;

Blood type and antibody screen


Rh type
Hb and Hct (CBC)
Rubella status test
Screening for syphilis
Screening for chlamydia
Screening for HIV
HBsAg test
Urine analysis and culture for screening for asymptomatic bacteriuria
Pap smear

=>Recommended tests in third trimester;

Blood pressure
Weight
Uterine fundal height
Fetal heart tones
Fetal presentation and activity
Urine glucose and protein for screening for glucosuria and proteinuria

63. Gestational thrombocytopenia:


Gestational thrombocytopenia is a benign condition defined by the presence of five
distinct criteria: mild and asymptomatic thrombocytopenia, development late in
pregnancy, no history of thrombocytopenia outside of pregnancy, no evidence of fetal
thrombocytopenia, and spontaneous resolution once the child is delivered. Although
the condition is not entirely understood, it is thought to have an immune etiology and
may be a mild, transient form of idiopathic thrombocytopenic purpura (ITP).

Management is conservative & includes follow up to ensure that the thrombocytopenia


resolves after delivery.
Gestational thrombocytopenia is a benign condition that is responsible for most
instances of thrombocytopenia in pregnancy. It is therefore not necessary to repeat
the platelet count in an otherwise asymptomatic pt until after delivery.

64.Thrombocytopenia is a common finding in HIV pts and may be classified as primary


or secondary. Primary HIV-associated thrombocytopenia is the most common cause
of low platelet counts in HIV pts & is clinically similar to ITP. Secondary HIV associated
thrombocytopenia is caused by infections, malignancy, medications, secondary
hypersplenism, TTP or DIC.

65.Pregnancy can trigger and exacerbate SLE symptoms.

66.Pseudothrombocytopenia is defined as a spuriously low platelet count secondary to


laboratory artifacts. Platelet clumping associated with the anticoagulant used in
purple or lavender-top tubes is the most common cause.

67. Exercise in pregnancy:


Pregnant pts shd be encouraged to participate in aerobic exercises like swimming,
walking or cycling.
Exercises that shd be avoided in pregnancy include scuba diving (becx of increased
risk of decompression sickness in the fetus) as well as those sports that place undue
stress on pregnant womens joints (e.g distance running) or those that significantly
increase her risk of falling (e.g skiing).

68.Adnexal cyst:
For non pregnant women, cysts less than 10 cm shd be observed for spontaneous
resolution.
For pregnant women, surgical removal of the cyst is indicated if the cyst is greater
than 5cm due to higher chances of rupture, hemorrhage and torsion & ultimately
preterm labor and delivery. The surgical removal is performed only during the
second trimester.

69.Erbs palsy:
It is the most common form of obsetetrical brachial plexus injury and involves the upper
roots (C5, C6 and sometimes C7) of the plexus, resulting in an adducted and internally
rotated right arm with forearm pronation and flexed wrist. A serious complication is
diaphragmatic paralysis due to phrenic nerve involvement. The prognosis of Erbs palsy
is typically good, with an 80% chance of full or near-full recovery.

70.Arrest of labor in the first stage:


Arrest of labor in the first stage is defined as > 6 cm cervical dilation, ruptured
membranes and one of the following:
No cervical change for > 4 hours despite adequate contractions OR
No cervical change for > 6 hours with inadequate contractions.
So, with cervix dilated 6 cm or more and there is no progress to labor, then first make
sure to check the adequacy of uterine contractions in montevido units with an
intrauterine pressure monitor. Adequate contractions mount to 200 to 250 montevido
units.
Most common cause of arrest in first stage of labor in nulliparous women r inadequate
contractions. other potential causes of labor arrest include a contracted pelvis, fetal
malpresentation (e.g occiput posterior) or fetal macrosomia.

71.

72. Condyloma accuminata:


Condyloma accuminata is the dermatologic manifestation of an infection with the HPV
virus, with over 90% of such condylomas arising from HPV subtypes 6 and 11. HPV is
primarily transmitted through sexual contact, and the areas affected include the penis,
vulva, vagina, cervix, perineum and perianal region. Less frequently, HPV may be found
in the oropharynx, larynx, or trachea secondary to oral-genital contact or secondary to
vertical transmission from mother to infant during childbirth. One relatively common
benign laryngeal tumor in children, recurrent respiratory papillomatosis, is caused by
the acquisition of HPV during passage through the vaginal canal.
Condyloma accuminata is not considered a contraindication to vaginal delivery in the
pregnant women.

73.Preeclampsia:
Preeclampsia is characterized by elevated blood pressure ( equal to or greater than
140/120) with proteinuria (defined as >300 mg/24 hours, protein/creatinine ration of
>0.3 or dipstick of >1+ ) OR evidence of end-organ injury in pregnant pts at > 20 weeks

gestation without pre-existing hypertension or renal disease. Onset of preeclampsia at


<34 weeksgestation is a significant risk factor for developing severe features and
maternaland fetal morbidity. Pts at early gestational age shd be hospitalized as
preeclampsia can progress rapidly. Admission allows for monitoring the development of
severe features by frequent blood pressure checkups and lab testing. If preeclampsia
progresses, treatment can be provided rapidly and includes the use of magnesium
sulfate for seizure prophylaxis and anti-hypetensive drugs for hypertensive emergency.
In addition, corticosteroids can be administered to accelerate the fetal lung maturity
in anticipation of preterm delivery.
Bottom line: Preeclampsia is characterized by elevated blood pressure with proteinuria
or evidence of end organ damage in pregnant pts at > 20 weeks gestation
without pre-existing hypertension or renal disease. Pts with new diagnosis
of pre-eclampsia at <34 weeks gestation shd be admitted to the hospital as
they are at increased risk for developing preeclampsia with severe features.

74. Delivery is definitive treatment to prevent maternal complications such as seizures,


stroke, renal or hepatic damage, and death. It is recommended for pts at <37 weeks
gestation who develop features of severe preeclampsia or fetal compromise
(e.g fetal growth restriction).

75.Anti hypertensives are given in pregnancy when the blood pressure is equal to or
greater than 160/110 mmHg to prevent stroke and other vascular complications.

76.The accidental usage of oral contraceptives during the first trimester of pregnancy
is not associated with increased risk of fetal malformation.

77.Although DES has been linked to adenocarcinoma chance in babies, but the risk is
only 1 case for every 1000-2000 women.

FEMALE REPRODUCTIVE SYSTEM AND BREAST:


1.Primary amenorrhea is usually due to chromosomal abnormalities and physiologic
delay of puberty (45% and 20% of causes, respectively). In both cases, development
of secondary sexual characteristics is delayed.

2.Imperforate hymen:
Presents as cyclic, pelvic or abdominal pain with primary amenorrhea. Sometimes, a
small suprapubic mass (uterus containing retained menstrual blood) can be palpated.
Perineal exam will reveal a bulging, bluish membrane between the labia. This bluish or
violaceous discoloration is due to blood sequestration behind the imperforate hymen &
is also known as hematocolpos. Secondary sexual characteristics are not delayed in this
case of primary amenorrhea.

3.Palpable breast mass:


In women over 30 years of age, both mammography and ultrasound shd be performed.
In women under 30, ultrasound alone is generally performed becx the breasts r denser,
which limits the sensitivity of mammography. If the ultrasound shows a concerning
abnormality, then mammogram is also performed. Simple cysts are generally classified
as benign but can be aspirated if they are uncomfortable. Solid, uncircumscribed masses
usually represent fibroadenomas and can be followed with repeat ultrasound and
mammogram in 6 months, but pts generally requst biopsy fr confirmation. Complicated
cysts or suspicious solid lesions generally require a core needle biopsy.
MRI is generally reserved for certain diagnostic dilemmas but is not recommended for
the evaluation of palpable abnormalities.

4.Fibrocystic disease presents with pain and cystic changes that become worse prior to
menstruation. Pts are reevaluated after a menstrual period.

5.Sexual history shd be obtained routinely in health visits. Physicians shd normalize
sexual health concerns and provide pts with the opportunity to openly discuss these
issues.

6. In pursuit of exploring the cause of sexual problems, all postmenopausal women shd
be asked about vaginal dryness and dyspareunia, so as to rule this out first as the
cause of sexual problems.
Other than that, proper exploration of physiological, psychological, relationship factors
and medication adverse effects shd also be sorted out as the possible causes.

7.Combined hormonal contraception:


Estrogen component increases the risk of thrombosis. Hence do not give
combined OCPs to those having or at increased risk of thromboembolic diseases.
Migraine headaches with aura is an absolute contraindication to the use of
combined OCPs as migraine increases the risk of ischemic stroke compared to the
general population.
Pts with known vascular disease or with blood pressure 160/100 mmHg or more
should not receive combined OCPs due to the risk of myocardial infarction or
stroke.
Age in itself is not a contraindication. However, age 35 or more combined with
smoking 15 or more cigarettes a day is an absolute contraindication to the use of
combined OCPs.
Such pts with above conditions shd be offered progestin only methods (implants,
injections) or copper intrauterine devices.

8.Athlete females:
Amenorrhea is thought to occur in female athletes when there is a relative caloric
deficiency secondary to inadequate nutritional intake as compared to the amount of
energy expended. Women athletes with this condition have been shown to have
decreased levels of LH and GNRH, resulting in estrogen deficiency. These amenorrheic
women are therefore at increased risk for all conditions associated with estrogen
deficiency, including infertility, vaginal atrophy, breast atrophy and osteopenia.

9. Breast Cancer screening:


Mammography is recommended every 2 years starting at the age of 50 or as per
aggressive recommendations to start at the age of 40.
When to do genetic testing for breast cancer ?

Two first degree relatives with breast cancer with one being at the age <50.
Three first degree or second degree relatives with breast cancer
One first degree or second degree relative with breast and ovarian cancer
One first degree relative with bilateral breast cancer
Breast cancer in a male relative
Ashkenazi Jewish women with any 1st or 2nd degree relative with breast or
ovarian cancer.

10.Pap smear:
All ladies 21 or more age shd undergo pap smear.
The risk factors for cervical intraepithelial neoplasia and malignancy include
smoking and infection with HPV 16 and 18.
How do you manage high grade squamous epithelial lesions on pap smear?
It needs prompt evaluation with colposcopy and biopsy.
CIN-1 is not concerning as it has low potential for malignant transformation.
CIN-2 and CIN-3 have an increased risk for cervical malignancy.
If the pt is 25 or more age, then we do excision or ablation for CIN-2 and CIN-3.
If the pt is less than 25 or if they are pregnant or if they are young and want to be

pregnant, then we dont go for excision/ablation. Actually those less than 25 have
a high rate for spontaneous regression & due to the risks fr cervical insufficiency
or stenosis from treatment, we prefer to observe and follow up such pts.
To be more specific, among CIN-2 and CIN-3, we prefer to observe the CIN-2 with
cytology and colposcopy every 6 months for one year. AND for CIN-3, we prefer
treatment with excision/ablation.
Bottom line: CIN-2 in pregnant pts or in young women who desire childbearing shd
be managed by observation due to the risk for cervical insufficiency that
may result from treatment. All other pts with high grade lesions should
undergo ablation or excision due to risk for malignant transformation.

11.Anovulatory uterine bleeding in menopausal transition:


Menopause is the permanent cessation of menses after the loss of ovarian function.
The median age of menopause is 51 and the diagnosis is made after > 1 year of
amenorrhea. Most women experience symptoms of menopause (e.g hot flashes,
irregular menses, sleep difficulties) for a few years until the cessation of menses. This
period is called menopausal transition (peri-menopause). After age 40, the absence
of ovulation (anovulation) is physiologic as gradual oocyte depletion and abnormal
follicular development lead to failure of progesterone secretion. However, the
endometrium may continue to proliferate due to estrogen, which leads to
inconsistent menstrual bleeding. As the risk of endometrial hyperplasia and cancer
increases with age and long periods of anovulation, any woman age > 45 with
suspected anovulatory bleeding should be evaluated with endometrial biopsy.
Endometrial hyperplasia and cancer cannot be distinguished clinically as bleeding
can range from spotty to heavy.
After the endometrial carcinoma is ruled out, the treatment options for anovulatory
bleeding in menopausal transition are cyclic progestin therapy, low dose oral

contraceptive pill, levonorgestrel intrauterine device, and cyclic hormonal therapy.

12.In postmenopausal women with vaginal bleeding, ultrasound measurement of


endometrial thickness can be used to evaluate for endometrial cancer. If
endometrial thickness is < 4 mm, the risk of endometrial cancer is very low and
endometrial biopsy is not needed. In a peri-menopausal woman, ultrasound for
endometrial thickness is not useful as hormonal activity is still present and
endometrial thickness is variable.

13.We do FSH test to confirm the menopause only in uncertain cases and NOT in
classic cases (hot flashes, irregular bleedings, sleep difficulties). FSH is elevated
in menopause.

14.Efficacy of different contraceptions:


Pregnancy rates are higher when the pts must use a condom with every sexual act,
return to the office for a shot or rely on memory to take a pill daily. Long-acting
reversible methods, such as the intrauterine device and implant, have equivalent
perfect and typical use with >99% effectiveness.
So most effective to least effective are following;
Intrauterine device, Implants > Injections > Pills/patch/ring > Condom > withdrawal.
So, Intrauterine device and implants are MOST effective with both perfect & typical
use.

15.Breast self examination (BSE) is not recommended at any age.

16.Women who have sex with women (WSW):


WSW women have an increased risk of bacterial vaginosis through exchange of
vaginal secretions.
WSW womens risk of acquiring HPV infection and cervical cancer is similar to
heterosexual women. However they have lower rates of screening due to
misconceptions regarding risk of infection. Routine cervical cancer screening is
therefore recommended in this patient population.
WSW women shd receive age appropriate vaccines, including HPV & Hepatitis-B,
if not previously immunized.
Routine screening for STIs in this pt population shd follow the general guidelines.

17.HPV vaccination is recommended at age 9-26.


HPV vaccination is NOT indicated in pregnant women.

18.Hepatic adenomas & OCPs:


Small (<5 cm) and asymptomatic hepatic adenomas are usually managed with
discontinuation of OCPs. Malignancy should be suspected if the tumor increases in
size even after the discontinuation of OCPs or if the AFP level is increased.
Symptomatic pts and pts with bigger lesions shd undergo surgical resection.
Liver biopsy and fine aspiration are usually not indicated becx the hepatic adenomas
have a tendency to bleed following biopsy.
The two main complications of hepatic adenomas are sudden rupture with intraabdominal bleeding and malignant transformation.

19.Lobular carcinoma in situ (LCIS):


Lobular carcinoma in situ (LCIS) z a non-malignant lesion that arises in the terminal ducts
and lobules of the breast. The lesion itself has no malignant potential but signifies an
increased risk of developing invasive lobular or ductal carcinoma in either breast

(roughly 7 to 8 times the general population). If LCIS is detected on needle biopsy,


excisional biopsy is recommended since a significant percentage of cases are upstaged
to either invasive cancer or ductal carcinoma-in-situ (DCIS). After the lesion has been
excised, the management is more controversial. The most common management course
is follow up surveillance. Chemoprevention with SERMS, such as tamoxifen or raloxifene
, is an alternative option. Some pts may request prophylactic bilateral mastectomy whch
is also a viable alternative.

20.Pap smear:
A pap smear is considered unsatisfactory if it is unlabeled, contains <5000 wellvisualized squamous cells, or has obscuring inflammation or blood.
Transformation zone (TZ) is the junction b/w squamous and glandular cells at the
external cervical os and is at greatest risk for neoplasia.
Pap smears containing metaplastic and endocervical cells (EC) is considered to be
a good sample of the transformation zone (TZ) of the cervix.
Pap smears can still be considered satisfactory even without EC/TZ component.
The cytology is negative for intraepithelial lesions with an absent or insufficient
EC/TZ zone;
>If the pt is 21-29 years age, then repeating the cytology and pap smear would
show no increase in cellular abnormalities compared to other women. For this
patient population, continuing the routine screening is recommended. Also, the
HPV infection in younger pts is transient and malignant transformation is rare.
Therefore, HPV testing is also not recommended.
>If the pt is 30 or above age, then HPV testing is recommended. Those with -ve
HPV test can be followed with routine screening. While those with positive
HPV test require further follow-up.

21.Uterine prolapse:
Uterine prolapse is typically seen in multiparous, postmenopausal women with history
of multiple vaginal deliveries. The injury to the pelvic ligaments and loss of estrogen
weakens the endopelvic fascia. The uterus and cervix descend down the vaginal canal

towards the vaginal orifice (introitus). Pts usually complain of a sensation of pressure or
heaviness in the pelvic area, which is relieved by lying down & aggravated by prolonged
standing or exertion. Some pts may complain of low back pain, dyspareunia, or a visible
mass at the introitus. In chronic cases, pts may hve bleeding or discharge frm ulcerative
, superficial epithelium. All symptomatic pts (constant sensation of heaviness, pain, or
bleeding) shd have surgical correction of the defect in the pelvic support. The aim of
surgical treatment z to completely relieve the symptoms and prevent any future relapse.
Manual repositioning does not correct the underlying defect in the pelvic support and is
not recommended.

22.Tamoxifen is indicated for estrogen receptor positive breast cancers and for
prevention of breast cancers in women with high risk of breast cancer.

23.Asymptomatic pts on tamoxifen do not need additional screening for endometrial


cancer other than an annual gynecologic examination with a complete history and
routine pap smears.
Symptomatic pts with vaginal bleeding or other symptoms warrant further
investigation with transvaginal ultrasound (TVUS) or endometrial biopsy.

23.Breast cysts
>Simple breast cyst:
Simple cysts are well circumscribed on ultrasound, can range from thin to thick
consistency with varying color and usually disappear with aspiration of the fluid.
Unless blood is present, no further intervention is required initially except for serial
ultrasounds.
Bottom line: Breast cysts that disappear with aspiration are consistent with a simple

cyst. Simple breast cysts with benign features on imaging can be managed
with fine needle aspiration (FNA) and serial routine follow up with U/S to
to document resolution.
>Solid, circumscribed masses:
Solid, circumscribed masses usually represent fibroadenomas and can be followed with
repeat ultrasound and mammogram in six months, but pts generally request biopsy for
confirmation.
>Complex cysts:
Have thick walls on ultrasound and may contain both cystic and solid components.
These lesions require core needle biopsy.

24.Pap smear:
A.The pap smear results are ASCUS/LSIL
>If pt is 21-24 age;
We repeat the pap smear in 12 months. If the results are again ASCUS/LSIL, then
we repeat it again in 12 months. If again the results are ASCUS, go for colposcopy.
OR
We repeated the pap smear in 12 months and it turned out to be negative. Then
repeat it again in 12 months. If the results are negative, then again repeat it at 12
months for the 3rd time. If results are negative again, switch to routine screening.
>If the pt is 25 or above age;
Go for HPV testing. If the HPV test is positive, go for colposcopy. If the HPV
results are negative, go for HPV testing PLUS repeat Pap smear in 3 years.
B.The pap smear results are ASC-H or AGC or HSIL;
Go for colposcopy.

ASCUS-> Atypical squamous cells of undetermined significance

ASC-H-> Atypical squamous cells, cannot rule out high-grade squamous intraepithelial
lesion.
AGC-> Atypical glandular cells
HSIL-> High grade squamous intraepithelial lesion.

Bottom Line: For women aged 21-24 with findings of atypical squamous cells of
undetermined significance (ASC-US) or low grade squamous intraepithelial
lesion, repeat cytology at 1 year z recommended. Colposcopy is performed
only if the pt has 3 consecutive Pap smears with ASC-US on cytology or a
finding of atypical squamous cells, cannot rule out high-grade squamous
intraepithelial lesion. Women aged 25 or more shd have reflex human
papillomavirus testing.

25.Breast mass:
The initial test of choice in women under the age of 30 with a breast mass on clinical
exam is an ultrasound. Solid masses seen on ultrasound with benign features shd be
evaluated with a mammogram while masses with atypical features may also require
a core needle biopsy. If a simple cyst is noted on ultrasonogram, a simple needle
aspiration is sufficient if the pt desires or can be followed up with routine clinical exams
without further testing.

26.Condyloma accuminata:
These can manifest with pruritis, bleeding, burning, tenderness, vaginal discharge, and
pain, depending on the location. Large lesions can interfere with defecation, vaginal
intercourse and delivery. Small lesions are frequently accidental findings on physical
examination or can be minimally symptomatic. Visual inspection with application of

acetic acid ( lesions should turn white ) is usually sufficient for making the correct
diagnosis.
Condyloma accuminata are caused by HPV and represent the most common viral
sexually transmitted disease in the US.
Pts with HIV infection have a higher incidence of anogenital warts and their lesions
tend to be larger.
The three major modalities employed in pts with condyloma accuminata are chemical
or physical destruction, immunotherapy and surgical excision.
Chemical destruction by trichloroacetic acid is the initial approach preferred by many
physicians. Repeated applications are necessary as the clearance rate is not very much
high. (Podophyllin has similar effects as trichloroacetic acid but is not indicated for
internal use in pts with condyloma accuminata & is also contraindicated in
pregnancy).
Systemic or topical interferon is a form of immunotherapy that has been proven
effective in pts with anogenital warts.
Ablative or surgical procedures are considered in pts in whom medical therapy is not
effective.

27. Ruptured ectopic pregnancy:


The clinical presentation of an acute abdomen (tenderness with rebound and guarding)
with a positive pregnancy test and no intrauterine pregnancy, is a surgical emergency.
The most likely diagnosis z rupturd ectopic pregnancy wth intraabdominal hemorrhage.
Initial evaluation of suspected ectopic pregnancy with transvaginal ultrasound (TVUS)
may show an adnexal mass with empty uterus in a patient with an unruptured ectopic
pregnancy. This mass can have an increased vascularity (ring of fire sign), fetal pole or
fetal heartbeat.

In a ruptured ectopic pregnancy, TVUS may show echogenic fluid (blood) in cul-de-sac,
around the adnexa and in the upper abdomen (Morrisons pouch). The posterior
cul-de-sac (rectouterine space) is the most dependent position of the pelvis and is the
primary place that bleeding of pelvic organ will accumulate. Morrisons pouch is the
hepatorenal space; blood seen on U/S in it suggests that >500 cc of free blood is present
in the abdomen. The blood loss in ruptured ectopic pregnancy typically comes from a
hole in the fallopian tube. Essentially, the pregnancy implants and grows in the tube
until there is no more physical space, at which point the tube bulges, stretches and
eventually breaks open. The pregnancy (which earlier may have been visible on TVUS
as a bulge/cyst in the adnexa) is expelled into the peritoneal space along with the blood.
If there is enough blood to reach Morrisons pouch, the pregnancy will generally float
in this blood away from the adnexa. and an adnexal mass will no longer be seen on
TVUS. The bleeding must be identified and stopped surgically as the tube continues to
receive blood flow from branches of the uterine and ovarian artery and therefore will
continue to bleed.

28.Premenstrual syndrome:
Premenstrual syndrome (PMS) is characterized by emotional & physical symptoms that
occur in the second half of the menstrual cycle and resolve promptly with the onset of
menses. Typical symptoms include depression, anxiety, irritability, bloating and breast
tenderness. Premenstrual dysphoric disorder (PMDD) is a severe form of PMS
characterized by predominant anger and irritability.
SSRIs are first line therapy for PMS/PMDD. If the frist SSRI is ineffective, another SSRI
may be tried.
If SSRIs fail & the pt does not wish to become pregnant, oral contraceptives containing
estrogen & progestin are a reasonable 2nd line therapy (progestin-only contraceptives

have not been proven effective).


Benzodiazepines and GNRH agonists (leuprolide) can also be effective but have more
significant adverse effects.
The lifetime risk of psychiatric disorders in pts with PMS approaches 80%. These include
primarily mood and anxiety disorders.

29.Contraception:
What is the most appropriate strategy in most states to decrease the pts risk for
preventing pregnancy and STDs ?
Condoms-> Would prevent STDs.
OCPs-> To prevent unwanted pregnancy. (most pts do not feel comfortable with
implants or intrauterine devices inserted in their bodies, though implants and
IUDs are most effective).

30. Vulvovaginitis:
Most of the symptoms are nonspecific and self diagnosis and treatment by the pt is
unreliable. Even a diagnosis by history and physical examination by a physician are
not reliable and shd always be confirmed in the office by microscopy. This is important
to avoid frequent misdiagnosis and inappropriate treatment. The most commonly used
techniques are saline and KOH preparation for microscopy. The vaginal secretions have
characteristic pseudohyphae in candidiasis. Bacterial vaginosis is confirmed by the
presence of clue cells in the smear. Mobile trichomonads and an abundance of PMNs
are found in pts with trichomoniasis.

31.Bleeding is the most common short term complication of conization (cone biopsy),
both cold knife conization and LEEP.

32.Endometriosis may cause intestinal obstruction.

33.Pts with unilateral, spontaneous & guaiac positive or grossly bloody nipple discharge
shd be evaluated with a mammogram and ultrasound. The most common cause of
pathologic nipple discharge z a papillary tumor that arises from the breast duct lining.

34.SILICONE BREAST IMPLANTS:


Silicone breast implants do not cause any major rheumatologic, autoimmune,
neurologic disorders, or breast cancer risk.
Most morbidity from silicone breast implants is associated with local
complications such as capsular contracture (resulting in pain), distortion of
shape, implant deflation and rupture.
There are reports that women with silicone breast implants are at an increased,
albeit extremely low, risk of developing anaplastic T-cell lymphoma compared to
the general population.
Silicone breast implants do not cause any disease or defects in the developing
fetus.
There is no evidence of any harmful effects in babies who are breastfed by
mothers with silicone breast implants.
Women with breast implants shd continue to have screening mammograms at
regular intervals.

35.Endometriosis:
Typically presents with dysmenorrhea, dyspareunia and infertility.
Visualization of endometrial implants, typically through laproscopy, is required for the
diagnosis of endometriosis & is recommended for before developing a therapeutic plan
in those pts with severe symptoms.
Preferred medical treatmnt options include NSAIDs, GnRH analogs, danazol (a synthetic
androgen) and OCPs.
Surgical treatment options include removal of lesions through bipolar coagulation or

laser vaporization, while severe cases may require hysterectomy and bilateral salpingooophorectomy.

36.Pelvic inflammatory disease (PID):


Fever, chills, lower abdominal pain, cervical discharge, cervical & adnexal tenderness.
Among all the risk factors, the one associated with highest increase in risk for PID is
multiple sexual partners. Other risk factors are lack of barrier contraception, age <35
years, a history of previous episodes of PID and , possibly, vaginal douching.
There is only minimally increase in the risk with the use of intrauterine devices,
especially the modern IUDs.

37.Emergency contraception:
>Oral;
Ulipristal is the most effective oral contraceptive for up to 5 days after unprotected
course, followed by levonorgestrel & then combined hormonal contraceptives (OCPs)
Ulipristal is an anti-progestin that prevents pregnancy by inhibiting or delaying the
ovulation. Levonorgestrel primarily delays the ovulation.
>The copper intrauterine device is more effective than emergency oral contraceptives
but is contraindicated in women with active infection.
emergency contraception;

Copper intrauterine device


Ulipristal pill
Levonorgestrel pill
OCPs

38.Individuals who have experienced sexual assault shd be treated empirically for
chlamydia (azithromycin), gonorrhea (ceftriaxone) and Trichomonas vaginalis

(Metronidazole). It is not necessary to diagnose T. vaginalis, chlamydia, or


gonorrhea prior to prophylaxis as many individuals may not return for results
and miss their opportunity for treatment.
Postexposure prophylaxis for hepatitis-B and HIV depends on immunity and risk
factors, respectively
postexposure prophylaxis for sexual assault;

HIV
Hepatitis B
Chlamydia
Gonorrhea
Trichomonas vaginalis

39.Indications fr HIV postexposure prophylaxis include unprotected vaginal intercourse


within 72 hours with someone at high risk of being HIV positive (user of injection
drugs) and evidence of vaginal trauma.

40.The most common cause of secondary amenorrhea is pregnancy. Drugs that induce
the cytochrome P450 system can reduce oral contraceptive efficacy. The most
commonly used anti-seizure medicatns (e.g phenytoin, carbamazepine, ethosuximide
, phenobarbital, topiramate) are a good example. Alternative anti-seizure medicatns
that do not decrease OCPs efficacy include gabapentin and valproate.

41.Sickle cell disease and contraception:


Pregnancy increases the risk of complications in pts with sickle cell disease such as acute
pain crises, thromboembolic events, blood transfusions, pulmonary and urinary tract
infections and preterm delivery.
The most effective form of contraception are the intrauterine devices and implants.

Progestin only methods are first-line for women with sickle cell disease as they have
lower thromboembolic risk compared to estrogen-containing methods and generally
do not worsen anemia. The progestin-releasing IUD is the best choice as it gradually
decreases monthly blood loss.
Do not copper IUDs as they are associated with heavy menses and dysmenorrhea.
Do not use estrogen containing contraceptives as they increase the thromboembolic risk

42.Contraception side effects:


Breakthrough bleeding is the most common side effect of OCPs.
When first initiated, the OCPs may also cause breast tenderness, nausea,
headaches, and moodiness from exogenous estrogen, however, these symptoms
resolve over time.
OCPs do NOT cause weight gain or infertility.
DMPA causes temporary, reversible, bone loss in young adolescents but its use
shd not be limited in young people becx of this. Daily calcium and Vit-D intake
and regular weight bearing exercises can overcome this effect.
Pelvic examination shd be performed before inserting an IUD but is not necessary
with most other forms of contraception.
Contemporary IUDs do not cause infections. They may increase the likelihood of
cervical or uterine infections at the time of insertion in someone with a sexually
transmitted infection (e.g chlamydia, gonorrhea) as the insertion involves dilating
the cervix and introducing the device into the uterus. IUDs do not increase the
likelihood of infection beyond 3 weeks following insertion.

43.Ectopic pregnancy:
A complication that occurs with risk factors such as prior ectopic pregnancy, tubal
dysfunction, PID, or in utero exposure to DES. The most common location for ectopic
pregnancy is ampullary portion of fallopian tube. Symptoms occur about 6-8 weeks
after the last menstrual period and include amenorrhea, abdominal pain, and possible
vaginal bleeding. Ruptured ectopic pregnancy may cause hemodynamic instability and

significant intraabdominal hemorrhage.


Initial evaluation of suspected ectopic pregnancy includz transvaginal ultrasound (TVUS)
and serum beta-HCG concentration. The discriminatory zone in the serum HCG level
,above which a gestational sac can reliably be visualized by TVUS if an intrauterine
pregnancy is present, is HCG >1500 IU/L.
If HCG is <1500 IU/L, then repeat HCG in 48-72 Hrs.
If HCG is >1500 IU/L, and TVUS does not confirm an intrauterine pregnancy, then the
pregnancy is not in the uterus. In such cases, ectopic pregnancy is confirmed if a mass
(i.e gestational/yolk sac, embryo or cardiac activity) is identified in the adnexa.

Bottom line: The discriminatory zone (>1500 IU/L) is the serum beta HCG concentration
above which a TVUS shd be able to identify an intrauterine pregnancy. An
HCG level above the discriminatory zone, without TVUS indication of an
intrauterine pregnancy, is most likely from an ectopic pregnancy.

44.Implantation bleeding is physiologic bleeding that occurs approximately 14 days after


fertilization due to attachment of the fertilized egg to the uterus lining.

45.An ovarian cyst during early pregnancy is generally a corpus luteum, which develops
after ovulation & produces progesterone to maintain a pregnancy until the placenta
can take over this function.

46.Women aged <30 shd not be screened for HPV testing as the infection in this
population is transient and typically does not lead to cervical cancer. HPV testing
in younger women also leads to unnecessary colposcopies.

47.Pap smear:
HGSIL revealed on Pap smear indicates a 1-2% probability of already having invasive
cervical cancer & a 20% probability of acquiring invasive cervical cancer if left untreated.
All women with HGSIL shd have either immediate referral for colposcopy of the cervix
and vaginal fornices (with endocervical curettage of all visible lesions) or loop electrosurgical excision (in non-pregnant adult women). If the colposcopy is unsatisfactory, the
next step is a diagnostic excisional procedure (i.e conization or LEEP).
Bottom line: If a pap smear reveals a high grade squamous intraepithelial lesion,
immediate referral for colposcopy or a diagnostic excisional procedure is
indicated.

48.Sepate uterus:
Septate uterus is the uterine abnormality most predisposed to recurrent pregnancy loss,
with a miscarriage rate that might be as high as 50%. While the embryo can implant on
the uterine septum, there is an altered blood supply to the septum that can result in
abnormal implantation, which can lead to pregnancy loss. Septate uterus is associated
with miscarriage in both the first and and second trimesters, although first trimester
miscarriages are more common. Metroplasty performed via hysteroscopy (endoscopic
procedure) is the treatment of choice & results in a dramatic decrease in the miscarriage
rate. Surgical treatmnt of septate uterus z not typically done on pts who r asymptomatic
or unable to conceive. It is important to differentiate septate uterus from bicornuate
uterus. They have a similar appearance, but laparotomy is needed for bicornuate uterus.

Bottom line: Septate uterus is associated with recurrent miscarriages but not an inability
to conceive. The embryo can usually implant but might implant abnormally.
Hysteroscopic metroplasty is the treatment of choice for septate uterus.

49.Acute dysfunctional uterine bleeding:


Dysfunctional uterine bleeding is typically the result of ovulation. In the absence of
ovulation, there is no progesterone influence on the endometrium, so the normal
signal for cyclic endometrial sloughing (menstrual bleeding) is absent. Estrogen, as
a trophic hormone on the endometrium, causes endometrium overgrowth that
ultimately outgrows its blood supply resulting in irregular sloughing that may be
significant (hemorrhage). The treatment of dysfunctional uterine bleeding involves
hormonal therapy to stabilize the endometrium. Estrogen shd be used in all pts who
are actively bleeding as it promotes hemostasis. High dose estrogen followed by
progestin is the treatment of choice. For less severe cases, oral contraceptives in doses
3 to 4 times the regular dose can be used as an alternative.
Intravenous conjugated estrogen is used in pts who cannot tolerate oral medications
and in unstable pts with severe bleeding as it can induce hemostasis rapidly.

Bottom line: The treatment of choice for acute anovulatory uterine bleeding is medical
therapy with oral contraceptives containing high dose estrogen to stabilize
the endometrium and stop bleeding. Intravenous estrogen can be used to
induce rapid hemostasis in pts with severe bleeding.

50.Rectocele:
Rectocele is a relatively common condition in older women and is characterized by the
displacement of the rectum through posterior vaginal wall defect (s). The condition is
typically caused by damage to rectovaginal septum incurred during vaginal childbirth
and is exacerbated by periodic increases in intraabdominal pressure (e.g when laughing
or coughing) and the effects of gravity.

Surgical repair with posterior colporrhaphy, is an appropriate recommendation for


women with symptomatic rectoceles.
Women with symptomatic rectoceles who are poor surgical candidates may be treated
with pessaries, which are structures designed to support the vaginal wall. Pessaries
should only be used in conjunction with vaginal estrogen; without it, these can cause
chronic discharge and bleeding secondary to injuryof the vaginal tissues.
Appropriate recommendations fr asymptomatic women include regular pelvic exercises
, avoidance of activities related to increased intraabdominal pressure and regular usage
of intravaginal estrogen to prevent tissue atrophy.

51.Ovarian/ Adnexal torsion:


A gynecologic emergency that most commonly affects women of reproductive age.
Ovarian torsion is partial or complete rotation of the ovary around the infundibulopelvic
(suspensory ligament of the ovary) and utero-ovarian ligaments. Torsion can also affect
ovarian blood supply as the ovarian vessels travel in the ligament. Adnexal torsion refers
to the fallopian tube also twisting along with the ovary. Right sided torsion is more
common due to the longer length of the right utero-ovarian ligament and because the
rectosigmoid colon occupies the space around the left ovary. Risk factors for torsion
include pregnancy, ovulation induction during infertility treatment and ovarian masses
(especially >5 cm).
Pts usually develop acute onset of moderate to severe lower abdominal pain, nausea
and vomiting. Fever and leukocytosis indicate possible adnexal necrosis. However,
vaginal bleeding is uncommon. Pelvic examination may show a palpable and tender
adnexal mass. Initial evaluation includes complete blood count, electrolytes and serum
HCG test (to exclude ectopic pregnancy). The diagnosis is confirmed on ultrasound
(abdominal and transvaginal) with color Doppler which will reveal an enlarged and

edematous ovary with localized tenderness and signs of impaired ovarian blood flow on
Doppler.
Management includes detorsion with emergency laproscopic surgery.
Salpingo-oophrectomy is reserved for obvious adnexal necrosis or suspected ovarian
malignancy. Untreated torsion may lead to chronic pelvic pain, infertility, hemorrhage,
or peritonitis and sepsis.
Bottom line: Ovarian & adnexal torsion occurs mst commonly in women of reproductive
age, often associated with pre-existing ovarian enlargement (e.g pregnancy
, tumors). Pts develop sudden onset of unilateral lower abdominal pain
(usually right sided), nausea and vomiting. Ultrasound (abdominal and
transvaginal) using color Doppler is preferred for diagnosis and typically
shows an enlarged, edematous ovary with decreased blood flow.

52.Primary dysmenorrhea:
Is due to excess production of Prostaglandin-F2a. Primary dysmenorrhea typically
presents within 6-12 months of menarche. Other common symptoms include nausea,
vomiting, back pain and bloating in the setting of a normal physical examination.
The first line management of primary dysmenorrhea is a 2-4 month trial of an NSAID
such as Naproxen. NSAIDs treat the underlying etiology of primary dysmenorrhea as
they are prostaglandin inhibitors. The medication shd be taken 2-3 days prior to the
onset of menses and continued throughout the menstrual cycle. If the pain does not
subside, OCPs shd be added.

MALE REPRODUCTIVE SYSTEM


1.Leriche syndrome is the triad of lower extremity claudication (buttock and thigh pain),
absent or diminished femoral pulses and erectile dysfunction and is caused by aortoiliac occlusion. Look for multiple atherosclerotic factors such as cigarette smoking,
diabetes mellitus & hyperlipidemia.
Pts with erectile dysfunction and atherosclerotic risk factors shd receive the screening
for peripheral arterial disease such as Ankle brachial index.
Pts with ED are also at risk for coronary artery disease and hence should receive
appropriate diagnostic testing such as exercise/pharmacological stress testing.

2.Pts with testosterone deficiency can usually achieve nocturnal penile erections.

3.Retrograde ejaculation is the most frequent complication of Transurethral resection of


prostate (TURP) , which is an invasive procedure used in the management of benign
prostatic hyperplasia (BPH).
Other complications are urinary tract infections and urethral strictures.

4.Pain in the testis can be caused by either epididymitis or testicular torsion.


Differentiate the two by means of cremasteric reflex.
Epidydimitis-> cremasteric reflex present. Another clue to epidydmitis is Prehn sign
i.e relief of pain with elevation of testis. Do urinalysis in all cases and do
urethral swab microscopy in cases of sexually transmitted diseases
mediated epididymitis.
Testicular torsion-> cremasteric reflex absent. Confirm the diagnosis by Doppler U/S or
nuclear scan. Prompt orchiopexy is needed.

5. Cremasteric reflex absent in two conditions;


=> Testicular torsion
=> Boys under the age of 6 months.

6.Swelling of the testis;


=> Hydrocele
=> Varicocele
=> Spermatocele
=> Testicular cancer

7.Hydrocele is a cystic fluid collection between the visceral and parietal layers of the
tunica vaginalis that can communicate (increased size during the day and with the
Valsalva maneuver) or not communicate (fixed size and not reducible) with peritoneal
cavity. Transillumination test is positive.

8.A painless scrotal mass in young male under age of 35 with negative transillumination
is most suggestive of testicular cancer.
Scrotal ultrasound is initial test of choice to evaluate a testicular mass to differentiate
between testicular cancer and non-malignant causes. If the U/S is suggestive, pts shd
subsequently get a CT scan of the abdomen and pelvis ( metastatic disease would first
appear in retroperitoneal lymph nodes) , Chest X ray ( and if abnormal, CT scan of
the chest ) and tests for the tumor markers. Radical inguinal orchiectomy is the
definitive procedure for histologic diagnosis.

9.Seminomas-> elevation of beta HCG only


Non-Seminomas-> elevation of AFP and beta HCG both.

10.Semen analysis is performed early in the evaluation of infertile couple and is usually
the initial screening test.

11. Anovulation-> check prolactin, serum progesterone (mid-luteal phase), basal body
temperature, endometrial sampling
Tubal and uterine abnormalities-> Hysterosalpingography.
Pelvic pathology and endometriosis-> Laproscopy.

12.Pearly penile papules are considered a normal variant. They are not spread by sexual
contact & activity. They harbor no malignant potential. These asymptomatic lesions
are more common in uncircumscribed males & are thought to occur in a substantial
proportion of the male population. They typically appear as one or multiple rows of
small, flesh colored, dome topped or filiform papules positioned circumferentially
around the corona or sulcus of the glans penis. Treatment is not necessary.

13.Condyloma accuminata-> HPV infection, Veruccous growths.

14.Cryptorchidism:
Presents with a hypoplastic, hypopigmented, poorly rugated and empty scrotum. Risk
factrs include prematurity, small for gestational age, low birth weight, in utero exposure
to DES and pesticides and neural tube defects.
Most undescended testis descend spontaneously by age of 6 mnths. Testis that have not
descended by the age of 6 months rarely descend, hence elective orchiopexy is perfrmd.
Orchiopexy improves the fertility and testicular growth (spermatogonia degeneration
begins in the undescended testis at the age of 6 months), decreases the risk of testicular
torsion, and decreases but not eliminates the risk of cancer. However, cancer detection

is improved as abnormal masses are more easily palpated in the scrotum compared to
the groin or abdomen.

15.Signs of testicular torsion;


tenderness, swelling, discoloration

16.Viagra in pilots:
Viagra can affect blue green color vision. 3% of pts taking recommended doses can
experience blue haze. Furthermore, taking higher than recommended doses has been
associated with a difficulty in blue green discoloration. Pilots taking Viagra may thus
experience an inability to distinguish between the blues and greens found in cockpit
instruments and runway lights. For these reasons, US Federal Aviation Authority (FAA)
has recommended all pilots to wait for a period of atleast six hours after taking Viagra
before flying a plane. This recommendation is more known to pilots as six hours from
Viagra to throttle .

17.Scrotal trauma:
Mild trauma (with minimal pain and swelling and a normal physical examination)
does not require testing or surgical consultation. It can be managed with bed rest
, ice packs, supportive underwear (briefs) and NSAIDS, with follow up in 48 hours
(or earlier if symptoms worsen)
Moderate trauma (with moderate pain and swelling) requires further evaluation
by scrotal ultrasound. If the ultrasound is normal, management is as with mild
trauma. If ultrasound is abnormal, surgical consultation z indicated. Decreased
blood flow is an indication for surgical intervention.
Severe trauma (with severe pain and swelling or a difficult/abnormal physical
examination) or suspicion of specific diagnoses (scrotal hematoma with testicular
compression, hematocele or testicular rupture, fracture, torsion, or avulsion)
requires immediate ultrasound by an experienced operator and expert surgical

consultation to determine the need for surgical exploration.


18. Management of penetrating scrotal trauma depends upon the extent of injury.
Superficial wounds require only local wound care and closure; wounds that extend
into or through the dartos layer require surgical exploration.

19.Retrograde urethrography is useful when urethral injury is suspected (e.g blood at


the meatus).

20.Prostatitis:
Pts present with perineal pain and testicular pain & dysuria as well. They have a tender
, edematous/boggy prostate on examination.
Pts shd be first evaluated with a urinalysis and urine culture.
Acute prostatitis is caused by the same organisms responsible for UTIs and urethritis,
with urine cultures identifying the responsible pathogen. Common symptoms include
high fevers, chills, dysuria, pelvic or perineal pain, cloudy urine and urinary frequency,
urgency, or dribbling. A tender prostate on examination confirms the diagnosis.
Chronic bacterial prostatitis presents with dysuria, increased urinary frequency, and
recurrent UTIs. Symptoms tend to be milder and prostate examination may even be
normal. Cultures of urine and prostatic secretions are usually positive.
Chronic non bacterial prostatitis is a common noninfectious inflammatory disorder
characterized by a history of pain in the lower abdomen or genitals and >20 leucocytes
per hpf in prostatic secretions. Cultures of these secretions are negative. Symptomatic
relief of nonbacterial prostatitis may be achieved with the use of Sitz baths and antiinflammatory medications. Transurethral microwave thermotherapy may also improve
symptoms and psychotherapy is recommended for pts experiencing sexual dysfunction.

Antibiotic use is of no help for chronic non bacterial prostatitis.

21.In cryptorchidism, early orchiopexy helps to prevent testicular torsion and possibly
infertility. Although the risk of malignant transformation may decreased a little after
the surgery, it remains higher than that of the general population.

22.Complications of undescended testis include decreased fertility, increased risk of


testicular torsion, inguinal hernia and malignant transformation.

23.TURP;
Retrograde ejaculation is the most complication and occurs in 70% of the pts. The
bladder neck fails to close after the procedure and enables the sperm to flow
backward to the bladder.
Hyponatremia is an acute complication that is caused by fluid absorption during
the procedure
Hematuria may be observed after the TURP but usually stops within 3 to 4 weeks
after the procedure.
Elevation of PSA can occur after the procedure but returns to normal in 3-4 wks

24.Prostate cancer:
Most prostate cancer detected by screening does not cause a problem during the pts
lifetime. In the majority of men with prostate cancer, the disease progresses so slowly
that they die of other causes.
PSA screening causes a high false positive rate and leads to unnecessary biopsies and
treatment. Despite these concerns, there is a small but real mortality benefit to PSA
screening such that men should be given all of the relevant information so that they
can make an informed decision about testing.
The possibility of being diagnosed with prostate cancer by screening is relatively high

but PSA screening has been shown to have a very little reduction in mortality.
Treatment modalities for prostate cancer consist of radical prostatectomy or radiation,
and both are frequently associated with complications. Radical prostatectomy has a
mortality rate of 0.5%. Either treatment can lead to erectile dysfunction, incontinence,
or bowel dysfunction in a significant percentage of pts.

25.Varicocele:
Venous drainage of the testes involves a complex network of veins called pampiniform
plexus, which are responsible for keeping the temperature of the scrotal sacs below the
normal body temperature. Dilatation of the pampiniform plexus (varicocele) results in
an increased temperature of the scrotal sacs, thereby causing testicular atrophy.
Varicoceles are more common on the left side becx the left spermatic vein enters the
left renal vein at a 90-degree angle. The right spermatic vein drains at a more obtuse
angle directly into the inferior vena cava, thereby facilitating a more continuous flow.
Processes that cause inferior vena caval obstruction (e.g clot, tumor) shd be ruled out
in pts who have bilateral varicocele, right varicocele or varicocele that does not
disappear in the supine position. Abdominal CT shd be done to look fr causes of inferior
cava obstruction.

26. TCAs can lead to urinary retention in elderly

26. Transient elevations of PSA can occur due to urinary retention, subclinical
inflammation, cystoscopy, digital rectal examinations or ejaculation.
Urinary retention leads to elevated PSA due to prostatic inflammation and swelling.
However, the rise in PSA normalizes over 2-6 weeks.

27. Persistent elevation of PSA occurs in BPH, prostate cancer and chronic prostatitis.

28. PSA can be very useful as a diagnostic test in ill pts with symptoms of possible
prostatic disease (e.g hematuria, urine retention, unexplained pelvic pain). It can
also be used to follow the course of established prostate cancer.

29.Epidydimitis:
Presents with mild to moderate pain accompanied by swelling and tenderness. No
irritative voiding symptoms are usually present and the urinalysis is normal. Most
common cause is Chlamydia trachomatis. Prehns sign (decrease in pain on testicular
elevation) is usually but not universally present.
P.S: Orchitis presents with sudden onset of fever, severe scrotal pain and swelling.

30. Testicular torsion is characterized by sudden development of moderate to severe


pain. High riding testis can usually be palpated on the affected side. The cremasteric
reflex is usually absent. In equivocal cases, color Doppler is helpful.

31. Varicocele presents as dull scrotal pain relieved by recumbency. A soft scrotal mass
can be palpated ( bag of worms ) that disappears in the recumbent position.

32.Erectile dysfunction:
Preserved nocturnal and early morning erections-> Psychosocial cause.
Loss of nocturnal and early morning erections-> Organic cause.
Although erectile dysfunction is common in older men, it is not part of normal ageing.

33. Priapism:
Can occur with thromboembolism (e.g sickle cell anemia), neurogenic dysfunction,
neoplasia, trauma, and medications ( psychotropics most commonly responsible).
Trazodone causes priapism through alpha blockade and serotonin receptor stimulation.
Priapism shd be first treated conservatively (e.g ice pack stimulation). If this fails, the
injection of an alpha agonist ( e.g phenylephrine or epinephrine ) is appropriate and
shd be performed within the first several hours to prevent long term damage.
Oral terbutaline is alternative and helps one-third of pts, but alpha agonists are given
first.
Angiographic embolization is for high flow priapism e.g arteriocavernosal fistulas.
IV fluids counteracts priapism in only Sickle cell pts.

34.TURP is frequently associated with hyponatremia secondary to addition of isosmotic


flushing solutions to the circulation. The serum sodium level can fall below 100
causing nausea, confusion, disorientation, twitching, seizures and hypotension.
Hyponatremia can be confirmed by the presence of an osmolal gap that can exceed
30-60 mosmol/kg.

35.Hyperkalemia;
increased release from cells;
pseudohyperkalemia, metabolic acidosis, hyperglycemia, increased tissue
catabolism, beta blockade, exercise.
reduced urinary excretion;
hypoaldosteronism, renal failure, volume depletion, renal tubular acidosis type-4
, ureterojejunostomy.

36. Hypernatremia;
secondary to unreplaced water loss;
insensible and sweat losses, GI losses, Diabetes insipidus, osmotic diuresis,
hypothalamic lesions.
Water loss into cells;
severe exercise, seizures.
Sodium overload;
intake of hypertonic solutions

37.Hypocalcemia:
Loss of calcium from the circulation;
hyperphosphatemia, acute pancreatitis, osteoblastic metastasis, intravascular
complexing with certain substances, acute respiratory alkalosis.
Other miscellaneous causes;
hypoparathyroidism, magnesium metabolism disorders, vit-D deficiencies, sepsis,
fluoride intoxication.

38.Hypokalemia:
Increased entry into cells;
extracellular pH increase, increased insulin, increased beta adrenergic activity,
hypothermia.
Miscellaneous causes;
decreased potassium intake, increased GI losses, increased urinary losses,
increased sweat losses, dialysis, plasmapheresis.

39. D/Ds of testicular swelling;


epidydimitis, hydrocele, testicular torsion, varicocele, hernia, hematoma.

40.Klinefilter syndrome:
Malignancy with a reported higher incidence in pts with klinefilter syndrome is breast
cancer.
Testicular cancer does not occur with an increased frequency in such pts; however, a
working diagnosis of testicular cancer shd be made in any pt with a history of painless
scrotal swelling, unless proven otherwise. If scrotal ultrasound and serological tests
confirm the suspicion, a radical inguinal orchiectomy is done to provide a histological
diagnosis.

CARDIOVASCULAR SYSTEM:
1.SINUS BRADYCARDIA:
Sinus bradycardia after an acute inferior wall MI is usually transient and generally
resolves within the first 24 Hrs.
Pts with symptomatic bradyarrhythmias in inferior MI should be treated initially with
intravenous atropine.
Temporary transvenous cardiac pacing is indicated in pts with persistent and
symptomatic bradyarrhythmias (e.g hypotension, dizziness, heart failure, syncope)
that are not responsive to atropine.

2.In contrast to pts with inferior MI, bradyarrhythmias associated with anterior MI are
commonly due to damage to the conduction system below the AV node. AV block in
the setting of anterior MI portends a bad prognosis.

3.Thrombolysis is contraindicated in the setting of recent surgery of < 2 weeks.

4.Compartment syndrome:
Rapidly worsening limb pain and swelling with paresthesias in a crush injury is highly
concerning for acute compartment syndrome.
Pts wth compartment syndrome may develop rhabdomyolysis and release of myoglobin.
Heme pigment from myoglobin is nephrotoxic (directly toxic to tubular cells, combines
with tamm-horsfall protein to form tubular casts, induces vasoconstriction resulting in
reduced medullary blood flow). Pts may develop acute renal failure, especially if they
are volume depleted.
Compression of the nerves in compartment syndrome can cause sensory and motor loss

below the compression site in the affected limb. (Not paraparesis, which occurs with
spinal cord damage).
Measurement of tissue pressures should be done. <20-30 mmHg indicates significant
compartment syndrme. Pressure >30 mmHg or delta pressure (diastolic blood pressure
minus compartment pressure) is diagnostic. Serial measurements shd be performed
even if initial pressures are within normal limits.
Signs and symptoms of compartment syndrome are unreliable and a high index of
suspicion is needed to make the diagnosis. Disease severity cannot be assessed based
on clinical features alone. Time to intervention is the most important factor predicting
complete functional recovery of the limb. All constrictive coverings (casts, dressings)
should be removed and supplemental oxygen given. Supportive measures to maintain
perfusion pressure includes keeping the limb at torso level and treating hypotension if
present. Fasciotomy is the definitive treatment of choice.

5.RBC transfusion would be indicated if Hb is below 7 g/dL. When Hb is 8-10 g/dL,


transfusion would be indicated in symptomatic pts or due to ongoing bleeding.

6.Pts with unstable angina & high risk for an acute coronary event, such as having risk
factors like HTN, Hyperlipidemia, Diabtes Mellitus etc, should be managed with early
coronary angiography, followed by percutaneous intervention (PCI) or CABG. Pts with
diabetes usually have extensive coronary artery disease, which progresses rapidly.

7.Acute mitral regurgitation:


Presents with rapid onset of pulmonary edema (bibasilar crackles), sudden onset of
of hypotension that can rapidly progress to cardiogenic shock, with poor tissue
perfusion & peripheral vasoconstriction. Cardiac examination shows a hyperdynamic

precordium and a soft, low pitched, decrescendo systolic murmur heard best at left
lower sternal border/cardiac apex. Signs of elevated right ventricular pressure such
as distended neck veins and tricuspid regurgitation are present. Pts may be diaphoretic
and having pallor on physical examination and cool extremities due to to poor tissue
perfusion.
Pts with acute MR typically have a more dramatic presentation (due to lack of time for
left heart adaptation) compared to those with chronic MR. Pts with acute, severe MR
have early equalization of left atrial and left ventricular pressures and up to 50% of pts
(especially with ischemic MR) may have no audible murmur (silent MR)
Diagnosis is typically confirmed by transthoracic or transesophageal echocardiography.
Bottom line: Actue mitral regurgitation (MR) presents with sudden onset of pulmonary
edema, hypotension, & hyperdynamic precordium with a systolic murmur.
The murmur can be soft and/or inaudible (silent MR) in up to 50% of pts
with acute, severe MR.

8.Ehlers Danlos syndrome:


Occurs due autosomal dominant mutations in COL5A1 and COL5A2 genetic mutations.
Ehlers Danlos syndrome is characterized by connective tissue abnormalities including
joint hypermobility, recurrent joint dislocations, velvety hyperextensible skin with
easy bruisability and delayed healing with atrophic scars. Pts may have associated
skeletal abnormalities, including thoracolumbar scoliosis and pes planus. Other features
include hernias, cervical insufficiency and uterine prolapse. Mitral valve prolapse with
myxomatous degeneration of the valvular apparatus is a complication of EDS and can
lead to rupture of chordae tendinae and acute mitral regurgitation.

9.Pts with graves disease can develop hyperpigmented, violaceous, and orange-peel

textured papules (infiltrative dermopathy).

10.Aortic stenosis:
Following findings indicate severe Aortic stenosis;
A soft, single S2.
A delayed and diminished carotid pulse (parvus and tardus).
Loud and late-peaking systolic murmur.
A normal second heart sound (S2) is characterized by increased splitting of the aortic
(A2) and pulmonic (P2) components during inspiration. The presence of a normal split
S2 indicates pliable aortic leaflets and is the most reliable finding for excluding severe
Aortic stenosis. By contrast, in severe AS, progressive calcification and stenosis of the
aortic leaflets lead to reduced mobility and delayed closure, with softening of the sound
and synchronization of A2 and P2.
Normal split S2-> Mild AS
No split and Single S2-> Severe AS
*Early peaking of murmur is suggestive of mild to moderate AS, however, this finding
is neither sensitive nor specific for ruling out severe AS.
*Loud systolic ejection murmur is highly specific for severe AS, however, low intensity
murmur does not rule out severe AS becx many pts with severe AS can have low
intensity murmur.
*Systemic HTN has no correlation with severity of AS

11.S4;
Hypertensive heart disease
AS
Hypertrophic cardiomyopathy

12.Infant of diabetic mother:


Maternal diabetes carries numerous risks for fetus including macrosomia, hypocalcemia
, hypoglycemia, hyperviscosity due to polycythemia, respiratory difficulties,
cardiomyopathy, and congestive heart failure. Most of these adverse effects can be
avoided with strict glycemic control.
Fetal cardiomyopathy and CHF result from excess glycogen deposition within the
myocardium resultng in hypertrophy of the fetal heart musculature. The interventricular
septum is most commonly affected resulting in ventricular outflow obstruction. The
treatment of this condition is carried out with observation and conservative medical
therapy as it would be for any other infant with heart failure due to outflow obstruction.
The defect will often correct spontaneously in the infant following birth as it will no
longer be exposed to the maternal hyperglycemia and the excessive glycogen within the
myocardium will be depleted by normal metabolic processes during fasting periods.

13.Paroxysmal supraventricular tachycardia: (PSVT)


>Includes different types of SVT, including atrioventricular nodal reentrant tachycardia
, AV reentrant tachycardia, and atrial tachycardia. The term PSVT excludes atrial fib.
and atrial flutter.
>ECG shows;
Narrow and regular QRS complexes
Unidentifiable P waves
>In hemodynamically stable pts, the next step in evaluation includes identifying the type
of SVT with the use of IV adenosine or vagal maneuvers (carotid sinus massage or
Valsalva).
>In hemodynamically unstable pts, urgent cardioversion is required.

ECG OF PSVT:

ECG OF ATRIAL FIBRILLATION:

ECG OF ATRIAL FLUTTER:

ECG OF SINUS TACHYCARDIA:

14.Wolff-Parkinson white syndrome:


Is due to an accessory conduction pathway that directly connects the atria & ventricles
, bypassing the atrioventricular node (AV node). The classic ECG findings are short PR
interval (<0.2 secs), slurred upstroke of the QRS complex (delta wave shown by the red
arrows) and widening of the QRS complex.

ECG OF WPW SYNDROME:

15.Mahaim and James fibers tachycardias are preexcitation forms that lack delta waves.

16.Coronary artery stent thrombosis:


Stent thrombosis is an uncommon but potentially fatal complication of coronary artery
stenting, with most events (Myocardial infarction or death) occurring within 30 days of
stent placement. The premature discontinuation of dual antiplatelet therapy with
aspirin & P2Y12 receptor inhibitors (i.e clopidogrel/prasugrel/ticagrelor) is the strongest
predictor of stent thrombosis after coronary stent placement. Pts should be aggressively
screened & counselled for medication compliance to reduce the risk of stent thrombosis

Cocaine use is associated with a small risk of early stent thrombosis.

17.Give statins to the following;

Age 40-75 with diabetes


LDL > 190 mg/dL
Clinical ASCVD
Calculated 10-year ASCVD risk >7.5%.

18.Fibrates are indicated for triglycerides level > 500 to 1000 mg/dL

19.Cocaine abuse:
Chest pain in pts who have recently taken cocaine is common and caused by several
factors, including increased myocardial oxygen demand, vasoconstriction, and a
prothrombotic state (secondary to platelets activation). Pts who have recently taken
cocaine have an increased rate of myocardial infarction and aortic dissection and the
risk of these complications is highest in the first hours after drug use.
For the most part, management of patient with cocaine related chest pain is similar to
the chest pain in the general population. One key difference is that cocaine related
chest pain pts shd be treated with early benzodiazepines which decrease the anxiety &
agitation associated with cocaine use. Becx the cardiovascular manifestations of
cocaine use are intimately associated with its neuropsychiatric effects, benzodiazepines
may indirectly decrease myocardial ischemia by resolving associated hypertension and
tachycardia.
If blood pressure does not decrease with benzodiazepines, then go for phentolamine,
an alpha antagonist, which is particularly effective and additionally decreases the
coronary artery vasospasm. Nitroprusside and nitroglycerine are reasonable alternative
options.

Beta blockers shd be avoided in cocaine related chest pain, becx they would lead to
unopposed alpha stimulation that would exacerbate the coronary vasospasm.

20. Infarction pericarditis:


Infarction pericarditis can occur within one to four days as a direct complication of a
transmural myocardial infarction. It presents as a chest pain that gets worse with
position changes and deep inspiration. It is usually a clinical diagnosis and is supported
by the presence of pericardial friction rub and EKG changes of pericarditis (sinus
tachycardia, diffuse ST segment elevations with PR segment depression). A pericardial
rub is usually heard over the left sternal border and can be present during any of the
phases of the cardiac cycle. It is heard as a superficial scratchy or grating sound, which
gets more pronounced when the patient leans forward. The EKG changes of pericarditis
are not always seen, but usually resemble that of an acute myocardial infarction.
Infarction pericarditis is seen less frequently in pts with early and complete reperfusion.
It is usually a transient episode and does not affect the management of acute MI, unless
it is complicated by a large pericardial effusion or tamponade.
Infarction pericarditis is usually transient (last for only a few days). It should be treated
with high dose aspirin (anti-inflammatory doses) that is effective in alleviating the pain
associated with acute pericardial inflammation. There are some concerns that other
NSAIDs may increase the risk of myocardial rupture after a transmural MI.

21.A left ventricular aneurysm usually occurs as a late complication of a transmural MI


(usually large anterior wall MI). It may result in heart failure, ventricular arrhythmias,
or peripheral arterial embolization due to formation of a left ventricular thrombus.
It develops over a longer period of time and is usually not associated with chest pain.
Persistent ST elevation can be present in these pts. There is increased incidence of

ventricular aneurysm with Corticosteroids.

22.Dresslers syndrome or post-cardiac injury syndrome occurs in pts with myocardial


infarction and after cardiac surgery. It is an autoimmune mediated syndrome, which
usually develops weeks to months after an acute MI. It usually presents with fever,
leukocytosis, pleuritic chest pain and a pericardial friction rub.
NSAIDs are the treatment of choice. Corticosteroids are used in refractory cases.

23.VSD:
Look for a step up in oxygen saturation from right atrium to right ventricle due to
left-to-right shunt. The left-to-right shunting allows the mixing of oxygenated blood
from the left ventricle with the deoxygenated blood in the right ventricle.
Pts with a VSD have a harsh holosystolic murmur with maximal intensity over the left
third and fourth intercostal spaces, often accompanied by a palpable thrill.

>Step up in oxygen saturation from superior/inferior vena cava to right atrium indicates
ASD. (fixed splitting of S2).
>Step in oxygen saturation from right ventricle to pulmonic artery indicates PDA
(continuous murmur in the left infraclavicular area).

24.Digital clubbing;
Bronchiectasis
Pulmonary neoplasms
Cyanotic congenital heart diseases

25.Both smoking and Hormone replacement therapy increase the risk of DVT. Estrogens
in oral contraceptives also increase ones risk for DVT, possibly becx it can cause
activated protein-C resistance. Obesity also increases the risk for DVT. Protein-C
and antithrombin-III deficiencies are congenital causes of hypercoagulability.

26.DVT:
Presents with lower extremity pain, swelling and warmth.
Either unfractionated or low molecular weight heparin is the initial treatment in pts with
DVT becx it has rapid anticoagulation effect to decrease propagation of clot. Warfarin is
started after the patient is adequately anticoagulated on heparin and then continued for
atleast three months and sometimes six months in pts with first episode of DVT. Lifelong
anticoagulation should be considered in pts with recurrent DVT.
HRT should be avoided in pts with a history of thromboembolic disease and stopped if
the pt is taking already and developed DVT.

27.Multiple studies have demonstrated that Tamoxifen use is also associated with
increased risk of venous thromboembolic disease.

28.Acute limb ischemia: (ALI)


Acute limb ischemia is defined as a sudden decrease in limb perfusion and is most often
caused by acute arterial occlusion due to systemic embolism from a cardiac source (e.g
atrial fibrillation, left ventricular thrombus, or septic emboli from infective endocarditis).
Pts typically have six Ps of acute ischemia: pain, pallor, paresthesia, pulselessness,
poikilothermia, and paralysis.
Pts with ALI with threatened viability of the limb hve severe pain (rest pain) wth sensory
and/or motor loss, delayed capillary refill, and inaudible arterial Doppler signals. All pts

with clinical signs and symptoms of ALI shd receive anticoagulation (i.e intravenous
Heparin bolus followed by continuous Heparin infusion) immediately while awaiting
further interventions. Anticoagulation prevents thrombus propagation and formation
of new distal thrombus due to stasis. Pts with an immediately threatened extremity are
at increased risk for irreversible myonecrosis within 4-6 Hrs & should have emergency
surgical revascularization.
*Pts with ischemic but viable extremity on examination (Intact arterial and venous
Doppler signals and no sensory or motor deficits) can be managed with catheter based
intra-arterial administration of thrombolytics or surgical revascularization.
*Transthoracic echocardiogram can be obtained later, when the pt is stable, as a part of
the comprehensive diagnostic evaluation.

Bottom line: Pts wth an immediately threatend extremity (severe pain, delayd capillary
refill, absent arterial Doppler signals, and sensory or motor deficits) are at
increased risk for irreversible myonecrosis within 4-6 Hrs and should have
anticoagulation and emergency surgical revascularization.

29.Stable angina:
A history of chest pain that z worsened by exertion & relieved by rest z highly suggestive
of stable angina.
Stress testing z most beneficial to risk stratify pts at intermediate risk of coronary artery
disease.
Pts with a high risk (risk factors +ve, ECG and ECHO showing +ve findings for example)
or underlying heart failure should proceed directly to coronary angiography.

30.Risk factors for coronary artery disease:


Pts with diabetes have a significant risk of atherosclerotic vascular disease and future
cardiovascular events, with all-cause mortality equal to pts with established coronary
heart disease (CHD) and prior myocardial infarction. As a result, diabetes is considered
a CHD risk equivalent. Diabetes has even more prognostic significance for women.
Chronic kidney disease is also considered a CHD risk equivalent.
Other risk factors like advanced age, male gender, family history of CHD, hypertension,
dyslipidemia, cigarette smoking, and obesity have synergistic effects and greatly
increase the risk.

31.The risk of cardiovascular events declines rapidly after smoking cessation and
approaches that of non smokers in 2-3 years.

32.Acute decompensated heart failure: (ADHF)


Presents with dyspnea, tachypnea and hypoxemia due to pulmonary edema. Chest X
ray would show the edema. Occurs as a result of a sudden rise in intracardiac filling
pressures and is seen most commonly in pts with left ventricular systolic and/or diastolic
dysfunction. Other causes include coronary ischemia or acute coronary syndrome, acute
valvular abnormalities (mitral or aortic regurgitation), severe hypertension, and severe
renal disease.
Pts with acute decompensated heart failure and evidence of fluid overload should be
treated wth intravenous diuretics and oxygenation as part of initial therapy. By reducng
intravascular volume, they provide symptomatic relief and improve oxygenation.
Intravenous vasodilators (nitroglycerine or nitroprusside) should be used in addition to
intravenous diuretics for rapid symptomatic relief in pts with acute pulmonary edema
and severe, uncontrolled hypertension. They are also recommended in pts with acute

decompensated heart failure who do not adequately respond to diuretics alone.


Nitroglycerin-induced vasodilation improves preload and afterload and reduces the
intracardiac filling pressures, leading to rapid symptomatic relief in pts with ADHF.
An echocardiogram should be obtained in all pts with pulmonary edema to evaluate
for potential precipitating cardiac causes of acute decompensated heart failure.

33.Statins:
Pts with symptomatic myopathy from statin use should discontinue therapy. In
asymptomatic pts, a CK level >10 times the upper limit of normal range is
considered an indication for discontinuation of statin therapy.
Statin therapy can potentiate muscle injury and elevation of CK levels following
prolonged and vigorous exercise. Most such pts should be restarted on statin
therapy after CK levels have normalized.

34.Cholesterol in children:
Children with a family history of high cholesterol or premature coronary artery disease
are at increased risk for developing coronary artery disease and should have their lipid
profile determined soon after they are two years old. Dont check it before two years
age, becx the diet is mostly composed of fat and cholesterol in children less than 2 yrs.
If a family member has a cholesterol level greater than 240, order random
cholesterol. If the latter is <170, then repeat in five years. If the it is >200, then
order FLP (fasting lipid panel).
If the family member has a history of premature coronary artery disease, order
FLP directly.
Cholesterol restricted diet should be started as soon as the lipid disorder is confirmed.

35.STEMI:
Percutaneous coronary intervention is recommended within 12 Hours of
symptom onset AND within 90 minutes of presentation at a PCI capable facility
(door to balloon time) OR within 120 minutes of at a non-PCI capable facility (to
allow time for transport to a PCI capable facility).
PCI is associated with lower rates of recurrent MI and intracranial hemorrhage
and improved survival with primary PCI compared to fibrinolytic therapy.
Fibrinolysis is recommended for STEMI pts arriving within 12 Hours of symptoms
onset who cannot undergo PCI. (e.g non PCI capable facility).

ECG OF STEMI: anterior wall MI in V1-V4 (occlusion of left descending artery).

ECG OF ACUTE PERICARDITIS:

36. Hypertrophic cardiomyopathy:


Implantable cardioverter defibrillator placement is indicated for preventing sudden
cardiac death (SCD) in pts who have hypertrophic cardiomyopathy and increased risk
for SCD due to:

Prior history of cardiac arrest or sustained, spontaneous ventricular tachycardia


Family history of sudden death
Recurrent or exertional syncope
Non sustained VT
Hypotension with exercise
Extreme left ventricular hypertrophy

*Beta blockers are considered first line treatment for hypertension in pts with HCM
*Alcohol septal ablation is usually reserved for pts with persistent symptoms refractory
to medical therapy.

37.Heart failure:
Echo results show low ejection fraction and cardiac dilatation. The most common cause
of heart failure is ischemic heart disease. In a new case of CHF with unknown etiology,
efforts must first be made to rule out the presence of coronary lesions which may be
corrected by angioplasty. Other important causes of CHF are hypertension, valvular and
renovascular disease.

38.Cocaine abuse:
Cocaine induced vasospasm may lead to coronary artery thrombosis, which cannot be
reversed by vasodilator agents. In pts with transmural ischemia (ST elevation), the initial
treatment regimen consists of nitrates (or calcium channel blockers), aspirin and
benzodiazepines.
If the patient does not promptly improve with these drugs, immediate coronary
angiography should be performed to check for any coronary thrombus.

Bottom line: Immediate coronary angiography is performed in pts with cocaine induced
transmural ischemia (ST elevation) to check for any coronary thrombus.

39.Renal parenchymal disease is an important cause of secondary hypertension in


children and young adults and may cause severe or resistant hypertension.
Focal segmental glomerulosceloris is an important cause of nephrotic syndrome in
in African American pts leading to renal parenchymal disease and hence hypertensn.
The pt can present with dyspnea and peripheral edema due to fluid retention. The
diagnosis is usually established by urinalysis and chemistry panel (proteinuria,
increased serum creatinine) and confirmed by renal biopsy.

40.Hyperaldosteronism and atherosclerotic renal artery stenosis are important causes


of secondary hypertension in middle aged and elderly pts.

41. Clinical clues suggesting secondary HTN are;

Acute rise in blood pressure in a patient with previously stable values


Age of onset prior to puberty
Age <30 in non-obese, non-black pts with no family history
Malignant HTN (i.e evidence of end-organ damage such as retinal hemorrhages,
heart failure).
Severe or resistant HTN (uncontrolled despite an appropriate 3-drug regimen).

42.Pts with inherited hypercholesterolemia may have elevated LDL cholesterol despite
maintaining a healthy weight. They are at increased risk for coronary events and
should have a close follow-up. Medical therapy with statins (e.g atorvastatin) is
recommended for pts with known atherosclerotic disease, LDL >190 mg/dL, age
40-75 with diabetes and a calculated 10-year atherosclerotic coronary heart disease
risk > 7.5%.
If the pt is not a candidate for statins, thn they should be advised to exercise regularly
and maintain dietary fat restriction.

43.Multifocal atrial tachycardia: (MAT)


Is characterized by presence of narrow QRS complexes, P waves of 3 or more different
morphologies, and variable PR segments and R-R intervals. The heart beat can reach
upto 200 beats per minute.
MAT is usually secondary to the following conditions;
Hypoxia
COPD
Hypokalemia

Hypomagnesemia
Coronary/Hypertensive/valvular disease
Medications ( i.e theophylline, aminophylline, isoproterenol ).
Hypoxia and COPD are the most common causes.
Rule out Hypoxia as the cause by measuring the arterial oxygen saturation.
Therapy of MAT z directed towards the correction of underlying etiology (e.g hypoxia,
hypokalemia, hypomagnesemia). If therapy is not effective and there are actually no
contraindications, beta blockers can be used successfully. In pts with asthma or COPD,
verapamil is the drug of choice.

44.Blunt cardiac injury: (BCI)


Pts with BCI can have varying degrees of severity, including transient dysrhythmias,
cardiac wall motion abnormalities, cardiogenic shock and free wall rupture. A patient
can either be asymptomatic or manifest various symptoms of cardiac injury ranging
from mild chest pain to severe heart failure and cardiovascular collapse. Cardiac
contusion (injury to the myocardium) is the most common injury but there is no gold
standard for diagnosis. The first step after initial stabilization of the patient is to do a
chest X ray to evaluate for fractures, pneumothorax, hemothorax and widened
mediastinum indicating possible aortic assessment with sonography in trauma (FAST)
examination or CT scan of the chest ( or transesophageal echocardiography) can be
done to evaluate for aortic injuries or hemopericardium.
All pts should have a 12 lead ECG, especially if they have anterior chest trauma (e.g
bruises on anterior chest wall), are elderly or have a history of coronary artery disease.
12-lead ECG is the single most important screening test for BCI. Pts with possible BCI
and normal ECG findings require no further treatment or investigation. Abnormal ECG
findings in BCI include a new bundle branch block, persistent sinus tachycardia, ST

depressions or elevations, and other arrhythmias.


Pts with hypotension or abnormal ECG findings should be evaluated with FAST
examination and echocardiography for tamponade, hemopericardium, valvular heart
abnormalities and thrombi. Transesophageal echocardiography is more sensitive than
is transthoracic echocardiography in providing optimal images of free wall and valvular
ruptures.
Bottom line: 12-lead ECG is the most important screening tool to help identify clinically
significant blunt cardiac injury.

45.Trastuzumab associated cardiotoxicity:


Trastuzumab is a monoclonal antibody that targets HER-2. Trastuzumab related
cardiotoxicity is due to loss of myocardial contractility (myocardial hibernation) leading
to a decrease in left ventricular ejection fraction (LVEF). The incidence of cardiotoxicity
is approximately 5% with trastuzumab monotherapy but it is 25% with trastuzumab
combined with anthracyclines (e.g doxorubicin) and cyclophosphamide. The decline in
LVEF is usually asymptomatic but may occasionally lead to overt clinical heart failure.
The cardiotoxicity associated with trastuzumab is reversible. There is complete recovery
after discontinuation of trastuzumab. In contrast, anthracyclines associatd cardiotoxicity
may not be reversible after treatmnt discontinuatn due to myocyte necrosis, destruction
and replacement by fibrous tissue.
Cardiotoxicity associated with anthracyclines is strongly related to cumulative doses but
trastuzumab associated cardiotoxicity is not dose related.
Trastuzumab should be withheld for 4 weeks if LVEF decreases by 16% or more from
baseline or by 10-15% from baseline to below the lower limits of normal It should be
completely discontinued if pts develop symptomatic heart failure.

Trastuzumab related cardiotoxicity responds well to standard therapy for heart failure
, including beta blockers, and ACEIs.

44.Hypertrophic cardiomyopathy:
Autosomal dominant disorder of cardiac sarcomere. Presents with fatigue, exertional
dyspnea, systolic murmur at left lower sternal border that accentuates with Valsalva
and standing, asymmetric septal hypertrophy, systolic anterior motion of mitral leaflets
and left ventricular outflow tract gradient. Pts can develop palpitations, presyncope and
syncope.
Pts with symptoms of heart failure and increased LVOT gradient should be treated with
negative inotropic agents (beta-blockers, verapamil or disopyramide) as the initial
medical therapy. Beta blockers r the most commonly usd agents fr initial monotherapy.
Verapamil or disopyramide can be added to beta blocker therapy in pts with persistent
symptoms.
Alcohol septal ablation is reservd for pts with persistent symptoms refractory to medical
therapy.
Vasodilators (such as CCBs), ACEIs, ARBs and nitrates reduce the systemic vascular
resistance with worsening of the LVOT gradient and symptoms. These should not be
used in pts with HCM.

45.Studies have shown that oral vitamin-K is as effective as intravenous Vit-K in


lowering the INR in 24 Hours. Since Intravenous vit-K has a risk of anaphylaxis,
oral vit-K is the preferred route.
Oral vit-K is the preferred route at any supratherapeutic INR unless there is serious
bleeding.

46.Warfarin and increased INR:

47.Constrictive Pericarditis:
Pericarditis is a common complication of coronary artery bypass graft surgery, and some
degree of postoperative pericardial effusion develops in almost 85% of pts. Most pts
are asymptomatic from small effusions. However, continued pericardial inflammation
occurring over months may lead to development of a thickened, fibrous pericardium
and constrictive pericarditis.
Constrictive pericarditis is an important cause of right heart failure. Pts typically have
peripheral edema, ascites & hepatic congestion with hepatomegaly, which can progress
to cirrhosis (cardiac cirrhosis). Other findings may include elevated JVP with prominent
Y descent and hepatojugular reflux, Kussmauls sign (increase or lack of decrease in

JVP on inspiration) and pericardial knock (mid-diastolic sound).


The ECG may show sinus tachycardia, or can show low voltage QRS complexes. Chest
radiograph often reveals calcified cardiac borders. Echocardiogram may show increased
pericardial thickening or calcification, atrial enlargement, and abnormal septal motion.
However, these findings are neither sensitive nor specific for the diagnosis. Treatment
involves supportive care (e.g anti-inflammatory agents) or pericardiectomy fr refractory
cases.
Bottom line: Constrictive pericarditis is an important cause of right heart failure. It
may occur following viral pericarditis or cardiac surgery. Characteristic
features include peripheral edema, ascites, elevated jugular venous
pressure, hepatojugular reflux, Kussmauls sign, pericardial knock, and
pericardial calcifications on chest radiograph.

48.Cardiac amyloidosis should be suspected in pts with unexplained congestive heart


failure (predominantly diastolic dysfunction), low voltage on ECG and echocardiogrm
showing increased ventricular wall thickness with normal left ventricular cavity
dimensions (especially in pts without hypertension).

49.Paroxysmal atrial fibrillation:


Pts with this condition may develop recurrent episodes of palpitations that resolve
spontaneously over time. However, may pts many remain asymptomatic and are
unaware of the paroxysmal AF. The 2 basic treatment approaches for recurrent
paroxysmal or persistent AF are rate control with atrioventricular nodal blocking
drugs (e.g beta blockers, calcium channel blockers, digoxin) or rhythm control with
antiarrhythmic drugs to maintain sinus rhythm. Clinical trials have found similar rates
of thromboembolic risk, morbidity and mortality between the 2 strategies.

Rate control is effective in most pts with paroxysmal AF. Rhythm control for maintaining
sinus rhythm should be considered in pts unable to achieve adequate heart rate control,
recurrent symptomatic episodes (e.g palpitations, lightheadedness, dyspnea, angina) or
heart failure symptoms in the setting of underlying left ventricular systolic dysfunction.
The choice of antiarrhythmic drug is based primarily on the presence or absence of
underlying structural heart disease, defined as coronary artery disease, LV hypertrophy
or heart failure due to ischemic or nonischemic cardiomyopathy.
Amiodarone z an option for most pts but is preferred for those with congestive heart
failure to ischemic cardiomyopathy

50.Acute decompensated heart failure: (ADHF)


Presents with dyspnea, orthopnea, paroxysmal nocturnal dyspnea, tachypnea,
tachycardia and bilateral lung crackles. There may be possibly an S3 gallop, jugular
venous distension and peripheral edema. The ADHF most commonly occurs due to
left ventricular systolic and/or diastolic dysfunction, with or without coexisting
valvular or coronary heart disease. A sudden increase in pulmonary capillary wedge
pressure (along with left atrial and ventricular filling pressures or LV preload) leads to
accumulation of fluid in pulmonary interstitial and alveolar spaces.
The goal of initial management in ADHF is to ensure hemodynamic stability, maintain
adequate ventilation and oxygenation and provide rapid symptomatic relief, along with
ongoing evaluation to identify the predisposing and/or precipitating causes. An urgent
pulse oximetry, chest radiograph and ECG should be obtained. The most appropriate
next step in improving symptoms of ADHF is to reduce the cardiac preload wth diuretics
and intravenous vasodilators (e.g nitroglycerine, nitroprusside, nesiritide) in selected
pts without hypotension.
Current guidelines do not recommend the use of morphine in pts with ADHF.

51.Cardiac tamponade:
Presents with hypotension, jugular venous distension and distant heart sounds. Cardiac
tamponade z a clinical diagnosis and immediate pericardiocentesis should be performed
in pts with tamponade & hemodynamic collapse. Removal of pericardial fluid
significantly relieves the elevated intrapericardial pressure and improves hemodynamic
status.
Several echocardiographic features are useful to diagnose early cardiac tamponade
including;
Right atrial and right ventricular collapse during diastole

Exaggerated respiratory variation of cardiac and venous flows


Inferior vena cava (IVC) plethora (dilation and <50% decrease in IVC size during
inspiration).

52.Pulsus paradoxus;

Cardiac tamponade
Asthma
COPD
Constrictive pericarditis
Obstructive sleep apnea
Marked obesity
Pulmonary embolism
Severe pectus excavatum

53.Diffuse left ventricular hypokinesis on Echo-> Dilated cardiomyopathy

54.Low voltage QRS complexes on ECG;

Pericardial effusion
Obesity
Severe hypothyroidism
Amyloidosis
Anasarca

55.Obtaining a detailed history and physical examination is the initial method of choice
to diagnose congestive heart failure prior to obtaining any confirmatory diagnostic
tests.

56.Bicuspid aortic valve: (BAV)


Bicuspid aortic valve is the most common congenital heart disease in adults, affecting
about 1% of the general population. It can occur as an isolated lesion or in association
with other congenital cardiovascular abnormalities, including coarctation of the aorta,
sub or supravalvular aortic stenosis, ventricular septal defect, and sinus of Valsalva
aneurysm. BAV is also risk factor for aortic dilation, with a reported incidence of 20-80%.
Progressive aortic dilation can occur in aortic sinuses, aortic root, or ascending aorta,
leading to aortic aneurysm formation and risk for aortic dissection. Pts with BAV should
have a thorough evaluation with echocardiogram, CT or MRI imaging to assess the aortic
root and ascending aorta for dilation.

57.Peripartum cardiomyopathy:
It is an uncommon heart failure that is defined by the following criteria;
Development of heart failure during the last month of pregnancy or within 5
months after delivery.
Left ventricular ejection fraction <45%.
Absence of other causes of heart failure
Absence of heart disease prior to the pregnancy
The potential risk of complications (significant decline in LVEF, recurrent heart failure,
death) with subsequent pregnancy is substantially higher in pts with persistent LV
dysfunction compared to pts whose LV function returns to normal. Risk in subsequent
pregnancy is also higher in those with severe LV dysfunction (LVEF < 25-30%) at the time
of initial diagnosis. Pts with history of peripartum cardiomyopathy who r contemplating
another pregnancy should be evaluated with transthoracic echocardiogram and those
with persistent LV dysfunction should be advised to avoid pregnancy.

58.Cardiogenic shock;
Low cardiac index
Elevated pulmonary capillary wedge pressure
Elevated systemic vascular resistance to maintain adequate tissue perfusion
pressure

59.Sepsis/Neurogenic shock;

Decreased systemic vascular resistance


Decreased PCWP
Increased cardiac index as a compensation
Increased MVO2 in Sepsis

60.Pulmonary embolism;

Increased right atrial pressure


Increased right ventricular pressure
Increased pulmonary artery pressure
Hyperdynamic left ventricular function with increased Cardiac output
PCWP and SVR are unchanged

61.Hypovolemia;
Decreased cardiac index
Increased systemic vascular resistance
Decreased PCWP.

62.Atrial fibrillation:
Beta blockers or non-dihydropyridine calcium channel blockers are the preferred first
line agents for ventricular rate control in pts with rapid atrial fibrillation. Beta blockers
are contraindicated or should be used with caution in pts with worsening congestive
heart failure, hypotension, bronchospasm and bradyarrhythmias.

63.Compartment syndrome is caused by increased tissue pressures in a confined space


leading to impaired perfusion. It is common following traumatic injuries (esp long
bone fractures) but can also occur in nontraumatic settings such as arterial occlusion
and reperfusion.

64.TCA poisoning:
Pts with TCA overdose can develop central nervous system findings, including mental
status changes (e.g drowsiness, delirium, coma), seizures, and respiratory depression.
Anticholinergic effects include dry mouth, blurred vision, dilated pupils, flushing,
hyperthermia, and urinary retention. Cardiotoxic effects (e.g sinus tachycardia,
hypotension, arrhythmias) cause most of the mortality in pts with TCA overdose.
TCAs inhibit fast sodium channels in the His-Purkinje tissue and the myocardium to
decrease conduction speed, increase phase-0 deplorization, and prolong the refractory
period. This can prolong the QRS interval (>100 msec), and cause arrhythmias (e.g
ventricular tachycardia, ventricular fibrillation). TCAs can also decrease calcium influx
into the myocardium and increase peripheral vasodilation, causing hypotension.
Finally, TCA overdose can also cause acidemia, which can mildly increase serum
potassium due to cellular exchange of hydrogen and potassium.
Sodium bicarbonate is the most effective agent for managing TCA-induced cardiotoxic
effects (QRS widening or ventricular arrhythmias). Sodium bicarbonate increases serum
pH and extracellular sodium. The increased pH (goal 7.5 to 7.55) modifies TCAs to their
neutral (non-ionized) form, making them less available to bind to the rapid sodium
channels. The elevated extracellular sodium increases the electrochemical gradient
across cardiac cells & also affects the ability of TCAs to bind to the fast sodium channels.
Pts who r refractory to sodium bicarbonate may respond to adjuvant magnesium or

lidocaine.
*TCAs inhibit central GABA receptors and can cause seizures. However, GABA agonists
(e.g benzodiazepines) are preferred to treat seizures.

65.Sodium bicarbonate can alkalinize the urine and hence enhances the excretion of
salicylates, thus counteracting the salicylate toxicity.

66.Bicuspid aortic valve:


Bicuspid aortic valve is one of the most common (about 1% of the population) types of
congenital heart disease. It cn occur sporadically or as an autosomal dominant inherited
disorder with incomplete penetrance. Pts with uncomplicated bicuspid aortic valves
typically have an ejection murmur and a sound or click best heard at the left lower
sternal border.
Chest X ray is usually unremarkable but can occasionally reveal AV calcification, aortic
enlargement (due to aneurysm), or rib notching (due to coarctation). Transthoracic
echocardiogram is recommended for diagnosing bicuspid aortic valve and follow up.
First degree relatives also should be screened for bicuspid AV to avoid complications
including severe regurgitation, ascending aorta or aortic root dilation and dissection. Pts
are at increased risk for endocarditis due to abnormal valve anatomy and function;
however, routine antibiotic prophylaxis is not recommended.
Coronary artery anomalies, including anomalous origin of the arteries, are also seen in
some pts. However, routine screening for coronary anomalies is not recommended in
pts with bicuspid AV.
Balloon valvuloplasty is indicated in symptomatic and asymptomatic (if they plan to
become pregnant or participate in competitive sports) young adults when the following
criteria are met:

Aortic stenosis
No significant AV calcification or aortic regurgitation
Peak gradient >50 mmHg

67.Nitroprusside toxicity:
Nitroprusside is a potent vasodilator that works on both arterial and venous circulation
and is used for hypertensive emergency management. It has rapid onset and offset of
action. The most important side effect is cyanide accumulation and toxicity.
Pts with chronic renal failure or those receiving a high dose or prolonged infusion of
sodium nitroprusside are at increased risk for cyanide toxicity. As a result, low infusion
rates (<2 ug/kg/min), short-term usage and close monitoring are recommended.
Toxicity should be suspected in all pts on nitroprusside infusion who have unexplained
metabolic acidosis and altered mental status.
Treatment involves cessation of nitroprusside and administration of sodium thiosulfate.

68.Pulmonary artery hypertension:


Pulmonary artery hypertension often difficult to distinguish frm congestive heart failure
or coronary artery disease based on history alone as pts typically have dyspnea or chest
pain on exertion due to an inability to increase cardiac output with exercise. Passive
hepatic congestion leading to abdominal pain or anorexia can help to suggest right sided
heart dysfunction. A loud S2, enlarged pulmonary arteries on chest imaging and signs of
right heart strain on EKG should all suggest pulmonary hypertension as the diagnosis.
Echocardiography is the test of choice to confirm this suspicion, which can give an
estimate of pulmonary artery pressure. Echocardiography is also able to evaluate for
right ventricular hypertrophy, right atrial size, findings of right ventricular failure and
valvular incompetence.

Pts with pulmonary artery hypertension need thorough evaluation for left sided heart
disease, chronic thromboembolic disease, pulmonary disease, obstructive sleep apnea
as the secondary causes of causing pulmonary artery hypertension. If all these are -ve,
then its said to be idiopathic (primary) pulmonary arterial hypertension.
Pts with idiopathic (primary) pulmonary arterial hypertension should first receive a
vasoreactivity test where the pulmonary arterial pressure response to a vasodilator is
measured with a right heart catheter; pts who respond favorably may be treated with
a calcium channel blocker. Pts without a favorable response on vasoreactivity testing
are either treated with a prostanoid medication such as epoprostenol, an endothelin
receptor antagonist such as bosentan, or a phosphodiesterase-5 inhibitor such as
sildenafil.
Endothelin-1 is a potent vasoconstrictor, the levels of which are elevated in pts with
IPAH. Endothelin receptor antagonists can increase exercise capacity, reduce dyspnea,
and improve cardiopulmonary hemodynamic variables in pts with idiopathic pulmonary
arterial hypertension.

69.The cause of the syncope can be established by careful history taking, physical
examination, simple lab tests and ECG in 70% of cases.

70.Antibiotic prohphylaxis against infective endocarditis:


Antibiotic prophylaxis to prevent infective endocarditis is not recommended in pts
undergoing gastrointestinal (GI) or genitourinary (GU) tract procedures. However,
pts with an established GI or GU infection must be adequately treated with antibiotics
prior to the procedure, including an agent active against enterococci.

Procedures that warrant antibiotic prophylaxis in pts with high risk cardiac conditions
include:
Dental procedures that involve manipulation of gingival tissue or periapical
region of teeth or perforation of the oral mucosa.
Respiratory tract procedures that involve incision or biopsy of the respiratory
mucosa (e.g tonsillectomy, adenoidectomy, bronchoscopy with biopsy).
GI or GU tract procedures in pts with an established infection or colonization.
Procedures on infected skin or musculoskeletal tissues.

71.Asymptomatic left ventricular dysnfucntion/Reduced Ejection fraction:


While left ventricular dysfunction commonly presents with signs and symptoms of
heart failure, a significant percentage of people with low ejection fraction are
asymptomatic.
The most important medication for pts with asymptomatic left ventricular dysfunction
is an ACE inhibitor, which can delay the onset of symptomatic heart failure & potentially
prolong survival. Initiation of a beta-blocker is also reasonable, although evidence for
beta blockers is not quite as strong as it is for ACE inhibitors.

72.Peripheral arterial disease: (PAD)


Look for symptoms of claudication and ankle-brachial index <0.9.
Pts should be advised on aggressive risk factors modification, with counselling for
smoking cessation, adequate blood pressure control and screening and treatment for
diabetes mellitus.
Such pts are candidates for statin therapy as PAD is an ASCVD equivalent.
Anti-platelet therapy with aspirin or clopidogrel is also recommended to prevent
ischemic stroke, MI, and cardiovascular death. (Clopidogrel was found more effective
than aspirin in preventing vascular events in pts with established cardiovascular dx

according to CAPRIE trial).


A supervised exercise program (30-45 minutes of exercise >3 times a week for >3
months) can reduce the symptoms and improve the maximum walking distance in pts
with intermittent claudication. Pts are started on a treadmill or track for 35 minutes
until they develop claudication. The sessions are gradually increased until the pt can
walk without symptoms for 50 minutes.
Pts with persistent symptoms despite antiplatelet therapy and an adequate exercise
program can also start cilostazol, which can significantly improve their walking
distances.
Percutaneous or surgical revascularization is generally reserved for pts who continue to
have symptoms despite exercise and pharmacologic therapy.

73.Recent MI:
Look for Q-waves and T-waves inversions in ECG. Such pts would present with exercise
intolerance & fatigue. The pts can have symptoms of indigestion. Such pts need further
evaluation with transthoracic echocardiogram to assess left ventricular function.
Such pts need;
Appropriate risk factor control
Antiplatelet therapy with aspirin and P2Y12 inhibitors (e.g clopidogrel, prasugrel,
ticagrelor).
Cardioselective beta blockers e.g Metoprolol/Atenolol/Bisoprolol etc. Beta
blockers would reduce the short term morbidity (recurrent symptoms or
reinfarction) and improve long term survival in pts with coronary artery disease
and prior MI and/or left ventricular dysfunction. In pts with Diabetes and recent
MI, beta blockers have been shown to have an overall benefit at least as great as
those without diabetes. Therefore, beta blocker therapy need not be withheld in
such pts due to concerns about masking hypoglycemia or worsening glycemic
control.
Statins therapy

ACEIs/ARBs
Aldosterone antagonists (eplerenone).

74.ECG OF RECENT MI:

75.Preoperative evaluation:
The preoperative assessment of a pts cardiac risk depends on the pts cardiac risk factrz
, the pts functional capacity, and the risk of the proposed surgery.
Low risk procedures do not need further evaluation. Most ambulatory and
superficial surgeries such as some endoscopic procedures are considered low
risk.
Intrathoracic, intraperitoneal and orthopedic surgeries are considered
intermediate risk surgeries.
Vascular surgeries are considered high risk.

76.Heart failure:
A pt is in NYHA-III heart failure and has been on optimal medical therapy (antiplatelets,

beta blockers, ACEIs, Statins, Aldosterone antagonists) and still has symptoms.
Go for cardiac resynchronization with biventricular pacing if there is following criteria;
Ejection fraction <35%
NYHA-II, III or IV heart failure symptoms
Left bundle branch block with QRS duration greater than 150 msec.
Cardiac resynchronization improves the exercise tolerance and reduces the NYHA
functional class and reduce the rate of recurrent hospitalization and overall mortality.

77.Atrial septal defect:


ASDs are the second most common congenital heart defect in adults (bicuspid aortic
valve is the most common).
Many pts may remain asymptomatic until adulthood. Those with significant shunt flow
may hve decreased exercise tolerance (i.e dyspnea and fatigue) and other complications
such as pulmonary hypertensn, rt. heart failure, stroke due to paradoxical embolization
and atrial arrhythmias (Atrial fibrillation and flutter).
Cardiac auscultation in pts with ASD (with large left to right shunt & normal pulmonary
artery pressure) typically reveals a characteristic wide and fixed splitting of the 2nd
heart sound. Pts may also have a mid-systolic ejection murmur over the left upper
sternal border resulting from increased flow across the pulmonic valve and a middiastolic rumble resulting from increased flow across the tricuspid valve.
Transthoracic echocardiogram can show the abrupt discontinuity of interatrial septum,
with right atrial & ventricular dilation (from volume overload due to left-to-right shunt).

78.Cardiac auscultation in pts with hypertrophic cardiomyopathy and LV outflow


tract obstruction reveals a harsh crescendo-decrescendo murmur at the left lower
sternal border.

79.Pts with VSD typically have a harsh holosystolic murmur with maximal intensity over
the left 3rd and 4th intercostal spaces, often accompanied by a palpable thrill.

80.Target specific oral anticoagulants: (TSOACS)


TSOACS (e.g Dabigatran, Rivaroxaban, Apixaban, Endoxaban) are approved for
use in pts with non-valvular atrial fibrillation to decrease the risk of thrombotic events.
TSOACS should not be used in following pts;

Mitral stenosis
Prosthetic heart valves (bioprosthetic and mechanical)
Severe decompensated valvular disease likely to require valve replacement
End-stage renal disease because TSOACS are renally excreted.

81.Post-MI and Sexual activity:


Sexual activity is associated with an increase in heart rate and blood pressure and a
small but measurable absolute increase in the risk of MI.
MI pts who have undergone successful revascularization or have no evidence of
ischemia on exercise testing can be considered low risk. Low risk pts can perform
light intensity exercise without symptoms. Examples include those with few CVD
risk factors, controlled hypertension, asymptomatic left ventricular dysfunction,
or successful revascularization of clinically significant lesions (>50-60%).
Low risk pts can safely resume sexual intercourse soon after MI within 3-4 weeks
and possibly as early as 1 week.
High risk pts should be referred for a detailed assessment prior to advising on
activity. Examples include those with refractory angina, NYHA-IV heart failure,
significant arrhythmias or severe valvular disease.
For intermediate/indeterminate risk pts, stress testing is recommended to
reclassify them as low- or high-risk and to help guide decisions.

82.It is not unusual for pts on dual antiplatelet therapy to experience a small amount of
bleeding when brushing their teeth or shaving; they can be counselled to use a soft

toothbrush or an electric razor.

83.Inferior wall MI:


Pts typically present with hypotension, jugular venous distension, and clear lung fields.
Pts with Right ventricular MI are treated similar to other pts with acute MI with
appropriate dual antiplatelet therapy, statins and anticoagulant therapy, as well as
emergency revascularization with thrombolytics or primary percutaneous intervention.
Drugs that slow the heart rate (beta blockers) or decrease the contractility (CCBs)
should be used with caution. Nitrates, diuretics and opioids should be avoided as
they decrease right ventricular preload, which in turn impairs the cardiac output and
can cause profound hypotension.
Pts with hypotension and normal or low JVP should be treated with a bolus of IV saline.
This generally improves right ventricular preload.

84.Wolff Parkinson White syndrome:


The classic ECG findings in a patient with wolff-parkinson-white syndrome are short
PR interval, delta wave and wide QRS complex. Pts become symptomatic with
tachyarrhythmias. Development of atrial fibrillation can be dangerous as it can result
in ventricular fibrillation and cardiac arrest. Advise the pt to avoid drinking, as it can
precipitate the atrial fibrillation.
Catheter ablation therapy is the preferred treatment of Wolff-Parkinson-White
syndrome in pts with symptomatic arrhythmias becx the risk of sudden cardiac death
is increased in pts with previous episodes of tachyarrhythmias.
Catheter ablation in asymptomatic pts with WPW pattern on EKG is controversial.

85. ECG OF WPW SYNDROME:

86.Vasovagal syncope, also known as neurocardiogenic syncope, can have many


triggering factors, such as stress and pain, and is caused by decreased sympathetic
drive (causing vasodilation) combined with increased parasympathetic response
(causing bradycardia).

87.Atrial fibrillation:
All pts with new onset atrial fibrillation should undergo CHA2DS2VASC score assessment
to estimate thromboembolic risk. Pts with a score of 0 are at the lowest risk of
thromboembolic complications and are usually managed without anticoagulation. Pts
with a score of 2 or more should be managed with anticoagulant therapy (e.g warfarin,

dabigatran, rivaroxaban, apixaban ).


Rate control of A-fib is done beta blockers/diltiazem/digoxin. Hemodynamically unstable
pts require emergency cardioversion. Rhythm control can be considered in pts unable to
achieve adequate rate control, with recurrent symptomatic episodes (e.g palpitations,
lightheadedness, dyspnea, angina) or with heart failure symptoms in the setting of
underlying left ventricular systolic dysfunction.

88.Coronary artery disease:


Coronary revascularization is indicated primarily for 2 groups of pts with stable angina;
Pts with refractory angina despite maximal medical therapy
Pts in whom revascularization will improve long term survival. This includes
those with left main coronary stenosis and those with multivessel CAD
(especially involving the proximal LAD) along with left ventricular systolic
dysfunction (i.e reduced ejection fraction).
In pts with diabetes and multivessel coronary artery disease, coronary artery bypass
graft (CABG) surgery is preferred over percutaneous coronary artery intervention due
to a lower rate of all-cause mortality and myocardial infarction with CABG.
*PCI with bare metal or drug eluting stents is an excellent revascularization option for
pts with refractory angina due to severe single or multivessel CAD not involving the
proximal LAD. However, CABG is superior to PCI in pts with multivessel CAD (especially
involving the proximal LAD) and left ventricular dysfunction, with lower rates of repeat
revascularization and improved clinical outcomes.

89.Chest pain:
Acute coronary syndrome is the most common life threatening cause of chest pain.
Pts without a clear etiology of chest discomfort should be assumed to have ACS
until proven otherwise. Repeat ECG & serum cardiac biomarkers (troponins and/or

CK-MB, with troponins preferred) are recommended for further evaluation in pts with
a low to intermediate risk of ACS and a non-diagnostic initial ECG.
*Dont go for stress testing in pts with ongoing chest pain.-> WRONG !!!!!!
*Ischemic chest pain is waxing and waning type & it does get relieved with nitroglycerin.
It can be provoked by acitivity, can be radiating and can be squeezing/crushing/burning
/elephant sitting on chest type of pain.

89.Hypertension Control:
Weight loss is the most effective nonpharmacological measure to decrease the blood
pressure in overweight individuals. Other effective measures include DASH diet (diet
low in sodium and saturated fat and high in potassium, calcium and dietary fiber),
moderation of alcohol intake (<3 drinks/day for men and <2 drinks/day for women),
regular moderate exercise and smoking cessation.

90.Diagnostic evaluation of stable angina:


All angina pts shd have first a resting ECG
Pts with typical angina symptoms with multiple CHD risk factors (e.g Diabetes,
HTN, family history) or uncontrolled angina symptoms on optimal medical
therapy are considered high risk pts. These pts shd be referred for coronary
angiography.
Pts without significant CHD risk factors or whose symptoms are controlled with
medical therapy are considered to have low to intermediate pretest possibility of
CHD. Such pts shd be referred for exercise treadmill stress testing (ETT).
Pts whose medical conditions limit their ability to exercise to a satisfactory level
should not undergo ETT. In addition, certain baseline ECG abnormalities (e.g left
ventricular hypertrophy, left bundle branch block, paced ventricular rhythm,
digitalis use, and >0.5 mm ST depression) can interfere with ECG interpretation
during exercise. These pts should undergo vasodilator stress imaging (e.g. single
photon emission CT, positron emission tomography) to improve the specificity of
the test.

91.Lightning strike:
Excellent cardiopulmonary resuscitation remains the most important component of
basic and advanced cardiac life support (ACLS) for pts with asystole following lightning
strike. Every effort shd be made not to interrupt CPR cycles. Chest compressions shd
be continued while administering vasopressors (epinephrine/vasopressin) & identifying
and treating reversible causes of cardiac arrest. Lightning injury can cause temporary
autonomic dysfunction with fixed & dilated pupils; this finding should not preclude
extended resuscitation efforts. Most victims of lightning injury r relatively young, have
few clinical comorbidities and may recover following a prolonged period of asystole if
they receive appropriate CPR.
Bottom line: Lightning injury can cause cardiac arrest with asystole or ventricular fib.
It shd be treated with prompt uninterrupted cardiopulmonary resuscitation
and use of vasopressors (epinephrine or vasopressin). Atropine is NOT
recommended for the treatment of asystole or pulseless electrical activity.

92.Down syndrome:
Endocardial cushion defect is the most common congenital heart lesion in patients with
Downs syndrome. Echocardiography is the most useful test for diagnostic evaluation of
this condition.
Other malformations in Down syndrome include Duodenal atresia, Hirschsprung dx,
atlantoaxial instability, and hypothyroidism.
Pts with Downs syndrome are at increased risk for developing acute leukemia later in
life. Other conditions that such pts are at increased risk of developing are Alzheimer-like
dementia, autism, ADHD, depression and seizure disorder.

93.The hyperoxia test measurz the PO2 levels in the preductal arterial with the neonate

breathing room air and then 100% oxygen. This test allows for the differentiation b/w
cyanotic heart disease and pulmonary disease.

93.Anomalous origin of coronary artery:


Its a common cause of sudden cardiac death (SCD) . The most common anomalies
associated with SCD are the origin of left main artery from the right sinus of vasalva
and the origin of right coronary artery from the left coronary sinus. Pts with coronary
artery anomaly may initially experience exertional chest pain, presyncope/syncope,
and SCD in a very young pt.
The ECG and Echo do not show any abnormalities (The ECG and Echo will be abnormal
in HOCM).

94.Brugada syndrome:
Characteristic ECG findings are right bundle branch block and ST elevations in V1-V3.
Associated with an increased risk for SCD due to ventricular arrhythmias.

95.Long QT syndrome:
It can be congenital or acquired & increases the risk of SCD due to Torsades-de-pointes
(a form of polymorhphic ventricular tachycardia). A QTc >450 msec in men & >470 msec
in women is considered a prolonged QT interval.

96.ECG OF LONG QT SYNDROME:

97.Vasovagal syncope/Neurocardiogenic syncope:


Vasovagal syncope is associated with prodromal symptoms of nausea, lightheadedness,
pallor and diaphoresis. It usually occurs when the pt is erect, and recovery occurs in the
supine position. Pts are usually young & otherwise healthy. Precipitating events include
prolonged standing, exertion, venipuncture, or a painful stimulus.

98.Cardiovascular syncope is due to arrhythmias, or obstructive lesions such as aortic

stenosis or HOCM. Syncope may be associated with prodrome of palpitations.


Arrhythmias usually occur without warning, with the patient in a supine or resting
position and lasts for a few seconds. Pts will have risk factors such as left ventricular
dysfunction, prior myocardial infarction or conduction system disease. Physical ex.
may show the murmur of aortic stenosis or HOCM. EKG/Holter monitoring may
detect an arrhythmia.

98.Steering wheel injuries:


Aortic injury is the most common cause of sudden death due to steering wheel injuries.
Pts typically die at the scene before ambulance arrives. Rapid deceleration produces a
shearing force along the aortic arch where the aorta is firmly attached. It is usually
observed in the area of ligamentum arteriosum, the aortic root, and the diaphragmatic
hiatus.

99.Atrial myxomas:
Left atrial myxomas are the most common cardiac tumors and can present with signs
and symptoms of mitral valve obstruction (e.g diastolic murmur and tumor plop), rapidly
worsening heart failure in otherwise healthy young individuals, and new onset atrial
fibrillation. Myxomas frequently embolize systemicaly, leading to acute arterial occlusn
in otherwise healthy pts.

100.Factor V leiden mutation leads to increased risk of venous thrombosis (not arterial)
such as DVT, cerebral thrombosis, mesenteric thrombosis, & portal vein thrombosis.

101.VSD:
Typically leads to a harsh, holosystolic murmur in the 4th left intercostal space and a
palpable thrill. VSD is most common congenital heart defect at birth, with spontaneous
closure in 40-60% of the pts during early childhood.
A small restrictive VSD is associated with a loud murmur, but a large nonrestrictive VSD
is associated wth a softer murmur which occurs early in systole due to early equalization
of right and left ventricular pressures. Adult pts who have persistent small restrictive
defect with small left-to-right shunting usually have favorable hemodynamics & normal
right sided heart pressures. These pts remain asymptomatic and this condition is often
detected during routine clinical evaluation due to harsh holosystolic murmur with
maximal intensity over the 3rd and 4th intercostal spaces along the left sternal border.

102.Mitral valve prolapse:


Pts with mitral valve prolapse typically have single or multiple non-ejection clicks
and/or mild to late systolic murmur of mitral regurgitation that is best heard at or just
medial to the cardiac apex. This systolic murmur of MVP starts earlier (and is longer and
softer) with standing, Valsalva and inhalation of amyl nitrate. Conversely, it is delayed
in onset (and shorter in duration) with squatting, leg elevation and handgrip.

103.TOF;

Right ventricular outflow tract obstruction


overriding aorta
right ventricular hypertrophy
VSD

104.Mitral stenosis:
The most common cause of mitral stenosis is rheumatic heart disease.
There is often a latency period of 10-20 years between the initial episode of rheumatic
fever & symptomatic MS, with most pts manifesting during the 4th or 5th decade of life.
Common symptoms include dyspnea (70% of pts), fatigue, atrial fibrillation & thromboembolism (e.g stroke). Hemoptysis (from pulmonary edema or pulmonary apoplexy)
and hoarseness (due to compression of recurrent laryngeal nerve by enlarged left
atrium) may occur. Pregnant women wth MS are at risk fr atrial fibrillation & pulmonary
edema due to physiologic hypervolemia and increased left atrial and pulmonary venous
pressure. Advanced disease may lead to right sided heart failure (e.g edema, pleural
effusions, hepatic congestion). Cardiac auscultation may show the classic murmur, a
low-pitched diastolic rumble most prominent at the apex. Other findings include loud
1st heart sound (S1), loud pulmonic (P2) component of second heart sound (due to
pulmonary hypertension) and normal apical impulse.
MS diagnosis is based on clinical presentation and typical demographics; it is further
confirmed by classic findings from physical examination and diagnostic evaluation (e.g
chest X ray, Echocardiogram).

105.Noonan syndrome:
Noonan syndrome is an autosomal dominant disorder characterized by short stature,
facial dysmorphism, and a spectrum of congenital heart defects. Cardiac involvement
is seen in up to 90%% of pts and includes pulmonic stenosis, atrial septal defects, and
hypertrophic cardiomyopathy.

106.Varicose veins:
Most of the pts with varicose veins are asymptomatic. Some pts may complain of leg
cramping, heaviness, fatigue and swelling. The symptoms are generally worse in the
evening with prolonged standing and improve with leg elevation.
Most pts with symptomatic varicose veins should initially be treated with conservative
measures such as leg elevation, and compression stockings. Compression stockings,
along with leg elevation and weight reduction, decreases the venous pressures in the
lower extremities by direct compression and improves the pts symptoms. Compression
stockings should not be used in pts with underlying arterial insufficiency.
Injection sclerotherapy with or without anesthetics is used in pts with symptomatic,
small, varicose veins who have failed at least three to six months of conservative
treatment. It involves the injection of a sclerosing agent into the affected vein. It causes
endothelial damage and sclerosis of the involved vein, thus preventing the further vein
filling.
Surgical ligation and stripping is used in pts with large, symptomatic varicose veins with
ulcers, bleeding, or recurrent thrombophlebitis of the veins.

107.Exercise ECG is the preferred initial test for diagnosis of coronary artery disease
in most pts with intermediate risk who have a normal baseline ECG and are able to
achieve adequate workload during the exercise.
Pts at high risk fr symptomatic CHD should proceed directly to coronary angiography
in most cases.
*Those who hve baseline ECG issues should proceed to exercise testing with myocardial
perfusion imaging.
*Those who cannot exercise properly due to osteoarthritis or peripheral vascular dx
shd proceed to Pharmacological myocardial perfusion stress testing with vasodilator

stress (e.g adenosine, regadenoson) as an alternative.

108.Adenosine use Is contraindicated in asthma or history of bronchospasm.

109.Pre- and perioperative medication management:


Withhold the SERMS ( Tamoxifen and raloxifene ) atleast 4 weeks prior to any
surgical procedure due to the increased risk of venous thromboembolism.
Withhold the ACEIs just the night before surgery i.e 10 hours prior to the surgery
to avoid causing hypotension. Do NOT withhold ACEIs in HEART FAILURE PTS !!!.
Thats an exception.
Withhold the diuretics just the morning before the surgery and not before.
Some experts suggest withholding metformin as well.
Do not withhold the beta blockers, because they help reduce the cardiovascular
mortality in the perioperative settings and an withdrawal can lead to significant
cardiovascular morbidity.
Do not withhold the statins, as they reduce the risk of cardiovascular events
Do not withhold the calcium channel blockers during the surgery. They help
reduce the chances of ischemia and atrial arrhythmias in pts undergoing
noncardiac surgeries.
Do not withhold the alpha-2-agonists i.e clonidine and continue during the
surgery.

110.Dihydropyridine CCBs:
Dihydropyridine calcium channel blockers (e.g amlodipine and nifedipine) cause
precapillary arteriolar dilatation of vessels and can lead to peripheral edema.
ACEIs lead to post-capillary venodilatation of small vessels and can counteract this
effect and hence combination of ACEIs and CCBs leads to decreased incidence of
peripheral edema.
Other major side effects of CCBs include headache, flushing and dizziness.

111.Glyburide can cause dermatologic side effects including photosensitivity reactions,


maculopapular eruptions, purpura, or urticaria.

112.The most common side effects of hydrochlorthiazide are electrolyte imbalances


(hyponatremia, hypokalemia), renal failure, hyperuricemia (may precipitate acute
gout) and elevated glucose and lipid levels.

113.AV block:
The initial management approach is to identify and correct any reversible causes of
depressed AV conduction (e.g myocardial ischemia, medications, enhanced vagal tone).
Pts with Mobitz-type II AV block have a high likelihood of progressing to 3rd degree
or complete AV block. In the absence of reversible causes, all pts with symptomatic
Mobtiz type-I and both asymptomatic/symptomatic Mobitz-II block should be managed
with implantation of a permanent pacemaker. Pts with severe symptoms and/or
hemodynamic instability with severe bradycardia or hypotension should be managed
initially with a temporary transvenous or transcutaneous pacemaker prior to a
permanent pacemaker.
*Certain ECG features and maneuvers (e.g atropine, exercise, carotid sinus massage)
are useful in differentiating b/w Mobitz-I and Mobitz-II blocks in asymptomatic pts
with 2:1 block. Use of intravenous atropine and exercise stress testing can worsen
Mobitz-II AV block; carotid sinus massage worsens Mobitz-I block and can paradoxically
improve Mobitz-II block.

ECG OF MOBITZ-II BLOCK:

114.It is recommended that warfarin dose be reduced by 25-50% to compensate for the
increase in serum concentration of warfarin

115.Pulseless electrical activity:


Pulseless electrical activity z defined as presence of an organized electrocardiographic
rhythm without sufficient cardiac output to produce a palpable pulse or measurable
blood pressure. Immediate cardiopulmonary resuscitation with chest compressions
should be started in an attempt to adequately perfuse the brain. Defibrillation or
cardioversion are not a component of resuscitation for PEA, although cardioversion is
the best choice in pts who have unstable tachycardia with a pulse.

116.Thrombolytic therapy in MI:


Pts with chest pain suggestive of an acute myocardial infarction are eligible for
thrombolytic therapy if they present within 12 hours of symptom onset and if an ECG

demonstrates ST elevation >1 mm in two contiguous leads. Additional candidates for


thrombolytic therapy includes individuals with persistent symptoms of new left bundle
branch block.
It is important to administer nitroglycerin in these pts before performing ECG to rule
out coronary vasospasm.
Contraindications to thrombolytic therapy include evidence of active bleeding, adverse
intracerebral history (history of hemorrhage, ischemic stroke, or neoplasm), systolic
blood pressure greater than 180 mmHg, recent trauma, or drug allergy.

117.ST depression >1mm is commonly caused by ischemia, strain, digitalis effect, &
hypokalemia and hypomagnesemia.

118.T wave inversion commonly occurs in myocardial infarction, angina and ventricular
hypertrophy.

119.Poor R wave progression occurs when the R wave in leads V1 through V4 remains
the same size or increases very little.

120.Mitral regurgitation:
In mitral regurgitation, some of the blood in the left ventricle z pumped forward through
the aortic valve (forward stroke volume), and some is forced backward through the
incompetent mitral valve (regurgitant stroke volume). Chronic MR causes volume
overload of the left ventricle and left ventricular chamber enlargement. This leads to an
increase in LV contractility and total stroke volume via Frank Starling mechanism. As a
result, pts with chronic MR who have normal ventricular function will actually have an
increased LV ejection fraction (LEVEF). Conversely, a normal LVEF in a pt with severe

MR suggests significant ventricular dysfunction.


Current guidelines advise valvular surgery in pts with severe MR in the presence of
symptoms (e.g exertional dyspnea, fatigue, overt heart failure) and LVEF <60%
regardless of symptoms.
*Chronic aortic regurgitation leads to an increase in LV end-diastolic volume and wall
stress, resulting in LV enlargement and hypertrophy. This in turn causes an increase
in total stroke volume, and cardiac output. However, in contrast to MR, pts wth chronic
aortic regurgitation and normal left ventricular function tend to maintain normal LVEF
and forward cardiac output.

121.Pts with carotid artery stenosis >70% should have carotid endarterectomy to reduce
the risk of stroke & death. Carotid artery stenting may be considerd as an alternate
procedure depending upon the availability and expertise of local specialists.
Medical management alone consisting of antiplatelet agents & control of risk factrz
for atherosclerosis, is recommended for pts with lesser degree of stenosis.

122.Acute coronary syndrome:


Includes STEMI, NSTEMI and UA (unstable angina).
UA and NSTEMI are distinguished by the absence (UA) and presence of elevated cardiac
biomarkers (NSTEMI).
Relief of acute chest pain with nitroglycerin points to ACS as the etiology.
In pts with acute chest pain and relief of pain nitroglycerin, a single negative cardiac
biomarkers test does not rule out NSTEMI. It takes 6 hours for cardiac enzymes to
start rising. Hence such pts should be admitted to the hospital or observed in the
emergency department for a longer period & evaluated with serial cardiac biomarkers
(troponin and/or creatinine kinase-MB). The typical rise and fall of cardiac biomarkers

along with symptoms of chest pain is considered specific for the diagnosis of ACS.
Pts with elevated cardiac enzymes (NSTEMI) are usually referred for cardiac
catheterization.
Pts with no elevation of cardiac enzymes on serial measurements (UA) would be
sent for further stress testing.
*Stress testing is contraindicated in the presence of acute MI.

Bottom line: Pts with suspected acute coronary syndrome should be admitted and
and evaluated with serial cardiac biomarkers (troponin and/or CK-MB)
to exclude myocardial infarction. Stress testing should be performed
only after myocardial infarction has been excluded with serial negative
troponins.

123. Cardiogenic syncope:


A pt has multiple risk factors for cardiac disease and he develops syncope at rest
w/o signs and symptoms of other types of syncope.
Such pts needs to be admitted to the hospital for 24 Hrs EKG monitoring in inpatient
setting. An acute coronary event should be ruled out with serial cardiac enzymes
and an echocardiogram is needed to evaluate left ventricular function. Coronary
revascularization is needed if evidence of coronary ischemia exists.
Bottom line: Pts with multiple risk factors for developing cardiac arrhythmias need
proper evaluation for possible implantable cardiac defibrillator placement
to prevent sudden cardiac death.

124.Vasovagal syncope:
Vasovagal syncope is usually a clinical diagnosis and needs no further testing if history
and physical examination strongly point towards the diagnosis & resting ECG is normal.
However, in uncertain cases, upright tilt table testing is used to establish the diagnosis.

125.In carotid sinus hypersensitivity, syncopal episodes are triggered by neck movemnts
or pressure on the carotid sinus by tight neck collars. The syndrome is a cause of
unexplained falls in older adults (>40). Carotid sinus massage z considerd diagnostic
If pts develop asystole >3 seconds, a fall in systolic blood pressure >50 mmHg, &/or
reproduction of symptoms (e.g syncope).

126.The pharmacological stress agents (adenosine and dipyridamole) act by producing


coronary vasodilatation and increasing the coronary flow rate and velocity. In
normal coronary vessels, the resulting vasodilation increases the blood flow;
But in areas with severe stenosis, there is already a compensatory microvascular
dilatation at rest to maintain normal blood flow, so no further increase in the flow
occurs. The resulting heterogenous blood flow due to the stenotic or occluded
region is detected by radionuclide imaging studies as a perfusion defect. After
establishing the presence of a defect, the next step is to localize the affected
myocardial region in order to identify the culprit vessel.

127.Left circumflex artery-----> lateral wall of the left ventricle


Left main coronary artery branches;
Left anterior descending artery---> Anterior wall of left ventricle
Right circumflex artery---> Right ventricle and inferoposterior walls of left
ventricle.

128.Aortic stenosis:
Aortic valve area of less than 1 cm sq. is considered severe stenosis. The onset of
symptoms markedly affects the prognosis in pts with aortic stenosis. Therefore,
prompt intervention is recommended in pts who have cardinal symptoms of aortic
stenosis (syncope, angina and CHF). Aortic valve replacement is associated with
marked reduction in symptoms and mortality in pts with symptomatic aortic stenosis
and is considered the treatment of choice.

129.Initial medical therapy for acute coronary syndrome includes oxygen, aspirin,
morphine and nitroglycerin for pain control, platelet P2Y12 receptor blocker
(e.g clopidogrel, ticagrelor), beta blocker, and anticoagulation (UFH/LMWH).
Intensive Statin therapy must be started as soon as possible.
Immediate release calcium channel blockers (e.g nifedipine) should not be used due
to association with hypotension, increased reflex sympathetic activity and worse
outcomes.

130.Mitral valve prolapse: (MVP)


Mitral valve prolapse (MVP) is found in 2-5% of adults and is the most common valvular
abnormality among the residents of industrialized nations. The conditions is typically
caused by redundant tissue at the valve ring (Barlows syndrome) and may be associated
with mild mitral regurgitation, though left ventricular function remains normal.
MVP causes a mid-to-late systolic click that may be followed by a late systolic murmur
most easily detected over the left ventricle.

131.Torsades-de-pointes: (TdP)
TdP refers to the form of polymorphic VT that occurs in the setting of acquired or
congenital QT prolongation and is characterized by cyclic alteration of QRS axis and/or
morphology.
TdP in pts with acquired LQTS is most commonly precipitated by premature ventricular
beats followed by a compensatory pause (short-long RR intervals) that initiates cycle
of VT. It can also be provoked in pts with bradyarrhythmias (sinus bradycardia or
frequent pauses), referred to as pause-dependent LQTS.
Other causes of acquired LQTS are electrolyte imbalances (e.g hypomagnesemia,
Hypokalemia & less commonly hypocalcemia), hypothermia & HIV infection. Common
medications implicated in LQTS include antipsychotics, antidepressants (SSRIs & TCAs),
certain antiarrhythmics (e.g amiodarone, sotalol and flecainide), macrolides,
fluoroquinolones and antifungals.
Structural heart disease, heart failure and left ventricular hypertrophy are associated
with an increased risk of developing drug-induced TdP.
TdP can be triggered by an adrenergic surge (exercise or sudden arousal) in some pts
with certain congenital forms of LQTS.
Intravenous magnesium sulfate is indicated as first line therapy for the treatment and
prevention of recurrent episodes of torsades-de-pointes, regardless of the pts baseline
serum magnesium levels i.e give it pts even if they have normal Mg levels. Recurrent
episodes of TdP can degenerate into ventricular fibrillation and death.
Temporary transvenous pacing in pts who do not respond to intravenous magnesium
sulfate.

132.Statin therapy is recommended fr all pts with known atherosclerotic cardiovascular


disease, regardless of baseline LDL levels. Therapy with non-statin lipid lowering

medications has not been shown to improve the clinical outcomes and is not usually
recommended.

133.Abdominal aortic aneurysms:


Most pts with abdominal aortic aneurysms are initially asymptomatic and the diagnosis
is made during an imaging study for an unrelated cause. Risk factors associated with
development of AAA include older age (> 60), male, cigarette smoking, family history
of AAA, white ethnicity and atherosclerotic disease. The natural history of AAA is
characterized by progressive expansion over time with an increasing risk of rupture. Risk
factors associated with aneurysm rupture include large diameter (20% risk in aneurysms
>6cm), rate of expansion (>0.5 cm in 6 months) and current cigarette smoking.
Current cigarette smoking is the most important modifiable risk factor and has been
associated with the highest rate of aneurysm expansion & rupture. The pathophysiology
is likely due to increased inflammation and degeneration of connective tissue in the
aortic wall. Therefore, smoking cessation is essential and has the greatest impact on
decreasing the likelihood of aneurysm expansion.
*Studies have shown a lower risk of AAA development and expansion in pts with
diabetes compared to those w/o diabetes.
*There is a very weak association b/w HTN and AAA formation, rate of expansion &
rupture.

134.CHF & Hyponatremia:


Hyponatremia in pts with CHF usually parallels the severity of heart failure and is an
independent predictor of adverse clinical outcomes. The presence of low cardiac
output and decreased perfusion pressure at the baroreceptors and renal afferent
arterioles leads to neurohumoral activation with the release of renin & norepinephrine

and secretion of ADH. ADH (vasopressin) binds to V2 receptors in the renal collecting
ducts and promotes the water reabsorption; renin (via angiotensin II) & norepinephrine
increase proximal sodium and water reabsorption and limit water delivery to the distal
tubules. These actions promote free water retention & lead to dilutional hyponatremia.
Angiotensin-II and the presence of low cardiac output also stimulate thirst, leading to
increased water intake and worsening hyponatremia. Water intake restriction is the
mainstay of initial therapy fr hyponatremia in pts with CHF. Correction of hyponatremia
does not improve the clinical outcomes associated with heart failure and is indicated
only in pts who develop symptoms due to hyponatremia or have severe asymptomatic
hyponatremia (serum sodium <120 mEq/L). Vasopressin receptor antagonists
(e.g tolvaptan) may be considered in pts with chronic heart failure and symptomatic
hyponatremia to raise the serum sodium above 120 mEq/L when other treatment
options have been unsuccessful.

135. Acute coronary syndrome:


Pts with acute chest discomfort with typical symptoms and the relief of pain with
nitroglycerin and ECG findings of ST-depression should be considered to be having ACS.
Such pts should be admitted for close monitoring and given the following therapy;
Dual antiplatelet therapy with aspirin and Platelet P2Y12 receptor blockers
(clopidogrel, prasugrel or ticagrelor).
Nitrates
Beta blockers
Statins
Anticoagulant therapy (unfractionated heparin, low molecular weight heparin,
bivalirudin or fondaparinux).
Dual antiplatelet therapy and anticoagulant therapy has been shown to significantly
reduce the risk of nonfatal MI and cardiovascular death in pts with ACS.

*Emergency cardiac catheterization and revascularization within 90 mins from the


first medical contact is the goal of treatment in pts witth STEMI.

ECG OF ST-DEPRESSION MI (NSTEMI) :

SVT:

136.Mitral stenosis:
Involvement of mitral valve z thought to occur in up to 90% of individuals wth rheumatic
heart disease.
The murmur of mitral stenosis is a soft, low pitched rumble heard best with the bell of
the stethoscope at the cardiac apex, which is located in the fifth intercostal space (b/w
the 5th and 6th ribs) at the left mid-clavicular line. It is often helpful If the pt exhales and
lies in the left lateral decubitus position.

*Tricuspid valve z best auscultated at the fifth intercostal space at the left sternal border
*Pulmonic valve is best auscultated at the second intercostal space at the left sternal
border
*Aortic valve z best auscultatd at the second intercostal space at the right sternal border

137.Marfan syndrome:
Aortic root disease with progressive aneurysmal dilation, aortic regurgitation, and
aortic dissection, is the leading cause of morbidity and mortality in pts with Marfan
syndrome (followed by heart failure secondary to mitral and/or aortic regurgitation).
Transthoracic echocardiography is recommended at the initial diagnosis and at 6-month
intervals to assess the aortic root and ascending aorta. Replacement of the aortic root
is necessary for pts with diameter >50 mm to reduce the risk of aortic dissection or
rupture.
*Acute pulmonary edema can occur as a complication of aortic regurg or worsening of

mitral regurg. due to abrupt spontaneous rupture of chordae tendinae.


*Mitral valve prolapse is a common finding in Marfan syndrome.
*Pts with Marfan syndrome can also have dilation of carotid and intracranial arteries,
with the risk of carotid dissection and stroke.

138.Alpha blockers are not recommended as first line treatment for hypertension.
However, they are effective in treating obstructive symptoms in pts with BPH
and may help to lower blood pressure in pts with comorbid hypertension.

139.ARBs to be given to pts who develop cough with ACEIs

140.Combination of ACEIs and dihydropyridine CCBs (e.g amlodipine) is especially


effective in reducing the risk of cardiovascular events and is recommended for pts
who need more than one drug to control blood pressure.

141.Aortic dissection:
Aortic dissection of the ascending aorta is a surgical emergency.
The goals of initial therapy are adequate pain control (morphine), lowering systolic
blood pressure to 100-120 mmHg and decreasing the left ventricular hypertrophy to
reduce the aortic wall stress.
Intravenous beta blockers such labetolol/propranolol/Esmolol are preferred to slow
the heart rate to <60 beats/min, lower the blood pressure and reduce left ventricular
contractility. Esmolol is preferred in acute settings due to its short half life.
Nitroprusside should be only used in addition to beta blockers if systolic blood pressure
remains above 100-120 mmHg after adequate beta blockade.

142.Inferior wall MI:


Sinus bradycardia can develop due to increase in vagal tone and sinus node ischemia
from right coronary blockade. Pts with symptomatic bradyarrhythmias resulting in
dizziness, confusion, syncope and worsening of ongoing ischemia should be treatd
with intravenous atropine.
Persistent hypotension after the treatment of bradycardia with atropine is suggestive
of volume depletion and/or right ventricular MI. These pts shd be managed with IV
fluids administration. Right ventricular MI is seen in 30-35% of those with acute inferior
wall MI due to occlusn of proximal rt. coronary artery before the origin of rt. ventricular
branches. Pts typically present with hypotension or shock, jugular vein distension and
clear lung fields in the setting of acute inferior wall MI.
Temporary cardiac pacing is indicated only in pts with persistent and symptomatic
bradyarrhythmias (with hypotension, dizziness and syncope) that are not responsive to
intravenous atropine.

143.Valve replacement and anti-thrombotic therapy:


Antithrombotic therapy with aspirin and warfarin is recommended for all pts with
mechanical aortic or mitral valve replacement to reduce the rates of systemic
thromboembolism.
Target INR for aortic valves without risk factors is 2.0 to 3.0.
Target INR for mechanical aortic valves with high risk features (e.g atrial fibrillation,
left ventricular dysfunction <LVEF <30%>, prior thromboembolism, presence of
hypercoagulable state) and mechanical mitral valves is 2.5-3.5.

DERMATOLOGY:
1.PREVENTING SKIN DAMAGE FROM SUN EXPOSURE:
Sun exposure or UV radiation-induced skin damage (sunburn) and risk of malignancy is
a common and preventable problem. Dehydration is another cause which can aggravate
skin damage by causing intracellular damage and cell death. All persons, especially those
at high risk of exposure, shd be counselled regarding the harmful effects of prolonged
sun exposure & the ways to prevent it. Such pts should be advised to use sun protective
behaviors, protective barriers (e.g clothing and sunscreens) and perform measures to
maintain adequate hydration.
Bottom line: Sun protective behaviors, sunscreens and maintaining adequate hydration
are the key factors to prevent skin damage from sun exposure.

2.ALOPECIA AREATA:
Alopecia areata is a type of non-scarring hair loss that can affect any hair-bearing area.
It is thought to be an autoimmune process that targets genetically predisposed
individuals. The classic clinical description is of a well-demarcated, often round, nonscarred patch of complete hair loss. The presence of so-called exclamation point hairs
(hairs which are tapered near the insertion into the scalp), especially at the periphery of
an alopecic plaque, z considerd pathognomonic of, but not necessary for, the diagnosis.
Areas of involvement may be single or multiple. Nail pitting is a commonly associated
finding. The affected areas of skin are typically asymptomatic, but the disease can be
emotionally traumatic and socially stigmatizing. Alopecia areata is often self-limited, but
may be relapsing and remitting or chronic and progressive. Pts with alopecia areata are
at inceased risk for other autoimmune conditions including autoimmume thyroid dx,
vitiligo and pernicious anemia.

Alopecia areata is characterized by discrete, smooth and circular areas of hair over the
scalp. There is no associated scaling, scarring or inflammation present in the areas of
hair loss. Most pts will have regrowth of the hair in the involved area over time.
Alopecia areata can commonly recur in pts even after successful treatment and normal
hair growth. Some factors associated with the higher chance of relapse include a longer
duration of the disease, a more extensive disease, involvement of the peripheral areas
and onset prior to puberty.
Topical or intralesional corticosteroids are the treatment of choice for pts with alopecia
areata. Patient education is also the key part of the treatment. Pts should be made
aware that 1. the disease is usually benign. 2. They can have multiple relapses in spite
of treatment. 3. Most pts can have normal hair growth within the next one to two years
even without treatment. They should also be informed that treatment with steroids,
either intralesional or topical, can restore normal hair growth but does not cure the
disease. After a steroid injection, new hair growth is usually seen in the next 4-6 weeks.
*pts with tinea capitis have an associated scaling and inflammation of the scalp in the
areas of hair loss.
*Areas of hair loss in 2ndary syphilis are not smooth and discrete. The lesions usually
have a moth-eaten appearance and are associated with significant scarring.
*Hair loss in androgenic alopecia starts from the front or temporal areas and progresses
gradually.

ALOPECIA AREATA:

3.MEDICATION INDUCED HAIR LOSS:


Certain medications may cause hair loss via a telogen or anagen effluvium. Medication
related hair loss typically manifests as diffuse, non-scarring hair thinning, rather than as
discrete bland patches. Telogen effluvium usually begins about 3 months after some
precipitating event, such as an illness, stressor or new medication. Beta blockers,
anti-coagulants, systemic retinoids, anti-convulsants, and anti-thyroid medications are
frequently implicated causes of telogen effluvium. Chemotherapeutics (e.g mitotic
inhibitors, alkylating agents, antimetabolites) can cause anagen effluvium, which usually
begins within 1-2 weeks of chemotherapy initiation.

4.TINEA CAPITIS:
Tinea capitis is a dermatophyte infection of scalp that affects children most commonly.
Trichophyton tonsurans and Mircosporum canis are the most common causes of tinea
capitis in USA. Clinically it consists of erythema, scaling and black dot alopecia (secondry
to breakage of hairs near the scalp), pustules or boggy plaques with posterior cervical
or posterior auricular lymphadenopathy. The inflammation and lymphadenopathy helps
to differentiate tinea capitis from alopecia areata.

5.Trichotillomania-> On physical ex., irregularly shaped areas of hair loss containing


hairs of many different lengths are typical.

6.PSORIASIS:
Psoriasis is characterized by well-demarcated erythematous plaques with silvery scale.
The extensor extremities (elbows and knees), periumbilical areas, gluteal cleft,
lumbosacral area, and scalp are among the areas most commonly involved. Hair loss is
usually minimal. Nail changes are common and includes pitting/onycholysis/thickening
and yellowing of the nail plate. Occasionally, Lisinopril can lead to aggravation of
psoriatic rash.
Psoriatic arthritis is seen in ~ 1/3rd of the pts and may even precede the skin lesions in
some pts. It is an inflammatory arthritis that presents with pain and stiffness in the
affected joints. Early morning stiffness occurs in the majority of the pts and is relieved
by physical activity. The common sites of involvement are the distal interphalangeal
joints and axial skeleton (causing spondyloarthropathy with sacroilitis or
spondylitis).
Pts with mild-to-moderate plaque psoriasis (<10% skin involvement) without associatd
arthritis are managed initially with topical high potency steroids (e.g betamethasone

0.05%) or vit-D derivatives (e.g calcipotriene). The lesions may be covered with an
occlusive barrier (e.g plastic wrap or occlusion tape).
Those with more extensive disease/joint involvement/psoriasis involving the nails
require systemic treatment (e.g methotrexate, cyclosporine or biologics).
Phototherpay with UVB radiation is only used for pts with extensive disease (>30%
of area affected). Severe lesions may be treated with a combination of coal tar
application and exposure to UVB light (Goeckerman regimen).
*Oral steroids are contraindicated as these may trigger pustular psoriasis.

PLAQUE PSORIASIS:

PSORIASIS:

PSORIASIS:

7.SEBORRHEIC DERMATITIS:
Popularly referred to as dandruff, seborrheic dermatitis is a common problem that is
characterized by erythematous & greasy scales secondary to hyperproliferation of
epidermis. Pruritus z usually mild. The dermatitis is distributed in areas rich in sebaceous
glands, including the scalp, face, and occasionally the upper trunk. Especially common is
involvement of the eyebrows, nasal bridge, lateral aspects of the nose, & the nasolabial
fold of the upper lip. Although the cause of the seborrheic dermatitis z uncertain, some
recent evidence suggests that skin saprophyte Malasezzia furfur (formely Pityrosporum
ovale) plays a causative role.
The scaling and pruritis of seborrheic dermatitis is best treated by daily shampooing of
the scalp with a medicated shampoo. Antiproliferative shampoos include tar-based
preparations, zinc pyrithione and selenium sulfide.
Physicians should consider the possibility of HIV infection in pts presenting with new
onset of severe seborrheic dermatitis.

SEBORRHEIC DERMATITIS:

8.ALLERGIC CONTACT DERMATITIS:


Allergic contact dermatitis is an antigen specific immune response to injury of the skin
by exogenous agents. This condition is typically categorized as the delayed type of
hypersensitivity. It is characterized by pruritis, erythema and edema, often followed by
the development of vesicles and bullae in the irritated region. If the insult is chronic,
lesions are typically dry, thickened and scaly.

9.PITYRIASIS ROSEA:
Is an acute inflammatory (presumed post-viral) rash characterized by an initial herald

patch, 2-6 cm annular lesion, followed days or weeks later by many smaller oval
pink/tan lesions, most commonly on the trunk in the Christmas tree distribution.

PITYRIASIS ROSEA:

10.SCABIES:
Scabies is manifested by intensely pruritic rash (often worse at night) with small papules
and vesicles, possibly with associated scale and thin linear burrows most commonly
located between the finger webs, lateral palms, flexor surfaces of wrists, ankle region
and lateral fingers. The rash is due to a delayed type-IV hypersensitivity reaction to the
mite (feces and ova included) and spreads through direct person to person contact.

Scabies can also involve other parts of the body (e.g elbows, axillary folds, nipples and
areola in women, scrotum and penis in men) but it is uncommon at the back and head
(except in children).
Examination usually shows excoriations with small, crusted, red papules scattered
around the region, often with linear burrows. Pts can also develop small vesicles,
pustules, wheals and excoriations that can obscure the classic burrows in the skin.
Diagnosis is confirmed by skin scrapings from lesions revealing mites, ova and feces
under light microscopy.
Lindane cream was formerly often prescribed for the treatment of scabies, but is rarely
used nowadays because of concerns about its serious adverse effects (e.g seizures,
aplastic anemia).
5% Permethrin cream is typically recommended in its place because of its superior
safety profile. Topical application over the whole body (excluding the head) is preferred
for eradicating the mites. The cream should be massaged thoroughly from the neck
down and then removed by thorough washing after 8-14 hrs. Common adverse
reactions of permethrin cream include local burning and pruritus.
Oral ivermectin is also a treatment option, especially for outbreaks in nursing homes
and other facilities where it may be difficult to use topical therapy.
Oral antihistamines can help decrease the dermatitis and associated itching and topical
steroids may be used for persistent dermatitis after mite eradication.
All family members and close contacts should be treated at the same time as the index
patient to prevent re-infestation. Bedding and clothing should be cleaned or placed in a
plastic bag for atleast 3 days as the mite can only live away from human skin for 2-3
days.

SCABIES:

SCABIES:

11.PORPHYRIA CUTANEA TARDA:


Porphyria cutanea tarda z a condition that arises from deficiency of uroporphyrinogen
decarboxylase, an enzyme in the heme synthesis pathway. Ths condition z characterized
painless blisters, an increased fragility of the skin on the dorsal surfaces of the hands, &
facial hypertrichosis and hyperpigmentation. It is often associated with Hepatitis-C
infection and can be triggered by the ingestion of certain substances (e.g ethanol,
estrogens), which should be discontinued if suspect. Diagnosis is confirmed with an
elevation of the urinary uroporphyrins. Phlebotomy or hydroxychloroquine may provide
relief, as can interferon alpha in those pts simultaneously infected with Hepatitis-C virus.

PORPHYRIA CUTANEA TARDA:

12.DERMATITIS HERPETIFORMIS:
Dermatitis Herpetiformis is characterized by pruritic erythematous and grouped papules
and vesicles that appear mainly on the elbows, knees, buttocks, back, posterior neck
and scalp. It is associated with celiac disease and a gluten-free diet is recommended for
such pts.
Skin biopsy is usually performed to differentiate DH from other autoimmune vesicular/
bullous skin diseases.

DERMATITIS HERPETIFORMIS:

DERMATITIS HERPETIFORMIS:

13.IMPETIGO:
Impetigo is characterized by pruritic and honey-colored macules, vesicles and bullae on
the face and other exposed parts. The condition is caused by Staphylococcus aureus or
group-A streptococcus.

14.TINEA VERSICOLOR: (TV)


Tinea Versicolor (TV) is a non-invasive fungal infection of the skin caused by Malassezia
species (Malasezzia furfur, Malassezia globose, Malassezia sympodialis). Clinical findings
are usually very characteristic and include multiple, often coalescing, small circular
maculae that may vary in color (white, pink or brown). The rash is typically more
prominent in the spring and summer as the organism inhibits pigment transfer to
keratinocytes and makes the affected skin paler than the unaffected skin. Lesions may

occasionally exhibit hyperpigmentation compared to surrounding skin due to a localized


mild inflammatory response.
Other than cosmetic effects, TV is usually asymptomatic, although mild pruritis may be
present. Typically, the lesions are located on the upper trunk, but the rash may also
involve the upper arms, neck and abdomen. TV diagnosis can be confirmed with a
KOH preparation showing hyphae and yeast.
The treatment of choice for Tinea versicolor is topical antifungal therapy (e.g selenium
sulfide, terbinafine, clotrimazole, ketoconazole). With extensive disease or recalcitrant
infection, oral antifungals (ketoconazole/itraconazole/fluconazole) are preferred.

TINEA VERSICOLOR:

15.ATOPIC DERMATITIS:
Atopic dermatitis typically affects the extremities and produces scaling and severe
pruritis, with excoriations and lichenification. Atopic dermatitis most often appears
in childhood and is frequently associated with other atopic conditions including
asthma and seasonal or perennial allergies.
The foundation of management includes avoidance of factors that contribute to
itching such as excessively hot or dry environments and harsh soaps and detergents.
Oral antihistamines are also useful. Regular use of emollients to maintain skin hydration
can also relieve pruritis.
Cotton clothing is generally recommended for pts with Atopic dermatitis.
Mitigation of environmental and food allergens has not been shown to have a significant
benefit for most pts with AD.
Additional interventions may be required if pts continue to have significant pruritis
despite these measures.
Most pts with AD will benefit from topical glucocorticoids. Low potency agents (e.g
hydrocortisone) may be adequate for mild symptoms. Topical glucocorticoids are
relatively contraindicated for AD on the face, eyelids or flexural areas. Calcineurin
inhibitors (e.g tacrolimus) may be considered for use in these areas.
More significant symptoms may warrant moderate-to-high potency glucocorticoids
(e.g triamcinolone, betamethasone).
Severe AD may require ultraviolet phototherapy or systemic immunosuppressants.

ATOPIC DERMATITIS:

ATOPIC DERMATITIS:

16.SECONDARY SYPHILIS:
Causes widespread mucocutaneous lesions. It has a highly variable appearance but
typical lesions are erythematous or pale pink macules that predominantly involve the
flexor surfaces of the extremities as well as the palms and soles.

SECONDARY SYPHILIS:

17.Intramuscular benzathine Penicillin-G is used for the treatment of primary, secondary


and early latent syphilis.

18.Eczema->Topical steroids.

19.TATTOOS:
The tattooing process involves repeated skin punctures using solid bore needles which
place the tattoo pigment into the dermis. The pigment is taken up by the macrophages
in the dermis which allows it to remain in the skin permanently.
The two most common techniques for removal of tattoos are Dermabrasion and
Laser removal.
Dermabrasion involves removal of the superficial layer of epidermis, thereby allowing
the pigment to leach out of the skin.
Laser removal involves the use of lasers of different wave lengths. It is thought that the
laser breaks up the pigments into smaller molecules, which are taken up and cleared by
the macrophages in the skin. Although it is a safe procedure, laser removal can leave
scar marks and can cause hypo- or hyper-pigmentation.

20.ACNE VULGARIS:
Contributing factrz to acne include incrzd sebum productn, follicular hyperkeratinization
, bacterial colonization (Propionibacterium acnes) and an inflammatory response in
some cases.
Life style interventions to manage acne include water-based skin products (rather than
oil based), PH neutral detergent cleansers; avoiding alkaline soaps and aggressive
scrubbing, using a low glycemic index diet and avoidance of saturated fats, dairy
products and refined carbohydrates.
For treatment of acne in women who may become pregnant, the preferred medications
include topical erythromycin, topical clindamycin (inflammatory acne), or azelaic acid
(comedonal acne), as these all are FDA-category-B.
Topical tretinoin and benzoyl peroxide should be avoided in pregnancy as these are
category-C.

Oral tetracyclines are category-D, so are contraindicated.


Tazarotene and isotretinoin are potent teratogens that are absolutely contraindicated
in pregnancy as these are category-X.
So, before prescribing anti-acne drugs, you have to do the pregnancy test or ask if the
lady wants to be pregnant.
Liver function tests and serum lipid levels should be performed regularly in pts treated
with oral (but not topical) retinoids.
Oral tetracycline & Doxycycline are photosensitizing agents & may cause exaggerated
sunburn in pts treated for acne. Benzoyl peroxide may also cause photosensitivity. Pts
taking minocycline have a lower risk for photosensitivity.
Side effects of Azelaic acid include local itching and irritation, depigmentation,
hypertrichosis and bronchospasm (rarely).
Isotretinoin is associated with numerous serious complications, includng hyperglycemia,
hypertriglyceridemia, hepatotoxicity, mucocutaneous reactions, blood dyscrasias, and
ocular toxicity. The hypertriglyceridemia if severe (>1000 mg/dL) can lead to pancreatitis
and hence the pts to be advised against excessive alcohol intake and also triglycerides
shd be monitored before administering isotretinoin & periodically during the treatment.
The drug shd be stopped if the triglyercides reach to >800 mg/dL. Isotretinoin is also a
potential teratogen and female pts of childbearing potential shd be tested fr pregnancy
and shd use 2 concurrent methods of contraception.
Isotretinoin has been associated with severe depression and suicidal ideation, although
it is not clear if the risk for suicide is higher in pts on isotretinoin than it is in other pts
with severe acne who r not using the medication. Pts on isotretinoin shd be monitored
for psychiatric symptoms and suicidality but prophylactic antidepressant therapy is not
warranted.
Other systemic treatments for acne, especially tetracyclines shd be discontinued in pts

initiating isotretinoin therapy due to the increased risk for adverse effects (e.g idiopathic
intracranial hypertension).
ACNE:

21.Doxycline;
Acne
Rosacea
Hidradenitis suppurativa

22.Tinea corporis;
Characterized by pink annular plaques with a scaly border and central clearing.
Topical clotrimazole is the treatment.

TINEA CORPORIS:

23.CHRONIC URTICARIA:
Chronic urticarial z defined as episodes of urticarial (welts or wheals) >6 weeks. Urticaria
is due to a trigger causing mast cell activation in the superficial dermis, which increases
release of multiple mediators (e.g histamine) that cause pruritus & localized swelling in
the upper layers of the skin. Up to 40% of pts can have associated angioedema due to
mast cell activation in the deeper dermal and subcutaneous tissues (e.g face, hands,

buttocks).
Chronic urticarial typically presents as episodes of well-circumscribed and raised
erythematous plaques with central pallor. The lesions can be oval, round or serpiginous
and up to several centimeters in diameter. Pts usually have intense pruritus that can
persist at night. Individual lesions appear and enlarge over minutes to hours before
disappearing within 24 hours.
Chronic urticarial can be due to physical stimuli (e.g cold, skin pressure), NSAIDS, stress
or systemic disorders (e.g autoimmune disease, vasculitis). However, up to 80-90% of
pts have no identifiable cause (idiopathic urticaria). The diagnosis is usually based on
characteristic clinical findings and does not require routine lab testing (e.g CBC). Further
testing is generally reserved for pts with systemic findings (e.g fever, weight loss,
arthralgias, arthritis, cold or heat sensitivity, abdominal pain, bone pain).
Chronic urticaria is usually treated in a stepwise approach, initially with a 2nd generation
antihistamine (e.g loratidine, cetirizine) in addition to avoidance of aggravating factors
(if identified).
Pts without improvement are then given one of the following: an increased dose of the
antihistamine, an additional first-generation H1 blocker (e.g hydroxyzine), a leukotriene
receptor antagonist (e.g monteleukast) or H2 blocker (e.g ranitidine) or a brief course
of oral steroids.
Pts with persistent symptoms can try a different option from the previous step or can
be referred to a subspecialist for alternate therapies (e.g hydroxychloroquine,
tacrolimus, omalizumab).
Chronic urticaria is usually a self-limited condition that resolves spontaneously within
2-5 years. Most studies have shown remission rates of nearly 30-50% at 1 year and up
to 70% by 5 years. Pts with chronic urticarial lasting >5 years also generally improve
and some studies have shown that up to 50% of those pts go into remission within the

next 5 years.
*Dietary changes usually do not improve chronic urticaria as Ig-E mediated food rxns
are not typically associated with chronic urticaria. However, Ig-E mediated food rxns
may cause acute urticaria and can improve with dietary modifications after identifying
the causative allergen.

URTICARIA:

24.INFANTILE HEMANGIOMAS: (STRAWBERRY HEMANGIOMAS)


Infantile/strawberry hemangiomas are the most common pediatric vascular lesion. Most
infantile hemangiomas are evident at birth and undergo a period of proliferation during
the first year of life followed by spontaneous involution by age 7-10. Small lesions with
no functional impairment or signs of ulceration are best managed with reassurance and
observation. Most pts require no further intervention.
Potentially disfiguring or complicated infantile hemangiomas require evaluation and
treatment. Complicated hemangiomas may involve the viscera, which shd be suspected

in infants wth multiple superficial lesions, hepatomegly, abdominal bruit, or tachycardia.


The risk of airway hemangioma z increased in an infant with facial or neck hemangiomas
, progressive hoarseness or stridor. In addition, periorbital hemangiomas can impair
vision. The first line treatment is propranolol, a non-selective beta blocker.
Large or rapidly growing infantile hemangiomas in areas prone to friction or pressure
(e.g axillae, perineum) are at risk for ulceration, bleeding, infection and scarring. A white
discoloration may be a sign of early ulceration; this is managed with gentle local wound
care and propranolol.
Bottom line: Most superficial infantile hemangiomas are small or clinically insignificant
and can be observed. Propranolol therapy should be considered for lesions
that are disfiguring or located at sites of potential functional impairment
(e.g eyelid, airway).

INFANTILE/STRAWBERRY HEMANGIOMA:

INFANTILE HEMANGIOMA:

25.IRRITANT CONTACT DERMATITIS: (ICD)


Three major causes of hand dermatitis are: irritant contact dermatitis, allergic contact
dermatitis and dyshidrotic eczema.
Allergic contact dermatitis is immune mediated.
Dyshidrotic eczema aka pompholyx is characterized by vesicular lesions involving the
palms and soles that is associated with pruritus, redness and scaling. The sides of the
digits may also be involved.
Irritant contct dermatitis z the cause of 80% of all cases of occupational hand dermatitis.
It is due to repeated exposure to irritants (e.g detergents, solvents, oxidizing agents)
over time and may occur after days, weeks or months of exposure to the irritant. ICD
is especially common in people with underlying atopic dermatitis and in health care
workers who use occlusive gloves frequently.
Clinical findings include pruritis, erythema and scaling of the skin. As symptoms become
chronic, hyperkeratosis and painful fissuring are also commonly seen. Symptoms are

often worst when the skin is the thinnest, at the dorsum of hands, fingertips and finger
webs. The diagnosis is primarily clinical but patch testing to rule out allergic contact
dermatitis may be useful on occasion.
All of these 3 kind of hand dermatitis are initially managed in similar fashion;

Identification and avoidance of the offending agents


Washing hands with lukewarm water and mild cleansers
Frequent use of emollients
Topical glucocorticoids.

IRRITANT CONTACT DERMATITIS:

IRRITANT CONTACT DERMATITIS:

ATOPIC HAND DERMATITIS:

ACUTE ALLERGIC CONTACT DERMATITIS:

ACUTE ALLERGIC CONTACT DERMATITIS:

DYSHIDROTIC ECZEMA:

DYSHIDROTIC ECZEMA:

26.PHOTOTOXIC DERMATITIS:
Can be induced by systemic medications such as diuretics, antibiotics, NSAIDs, and
various other drugs. Typically the phototoxic reaction manifests as exaggerated
sunburn response. The reaction consists of erythema, with or without bullae and
vesiculation over sun exposed areas such as the dorsal hands, forearms, upper chest
and face.
Generally the treatment for sunburn includes fluid replenishment and medications
for relief of pain and pruritis. NSAIDs (e.g indomethacin, ibuprofen) have been shown
to reduce the pain and erythema and can minimize damage to the epidermis if
administered immediately after sun exposure.
Oral antihistamines may be used for the relief of pruritis.

SUNBURN:

27.TINEA MANUUM:
Tinea manuum causes annular erythema with a trailing scale on the dorsal aspect or
ventral hands. Mild itching is usually present. Classically only one hand is involved,
although tinea pedis is often present as well.
Characteristic findings include erythema, cracking, scaling and pruritis.

TINEA MANUUM AND TINEA PEDIS:

TINEA MANUUM:

28.CHRONIC ALLERGIC CONTACT DERMATITIS:


Chronic allergic contact dermatitis is characterized by pruritic lichenified plaques. The
locations correlate with the causative allergen. Common triggers include a variety of
household and occupational allergens including nickel and other metals, cleaning
products, fragrances and various chemicals involved in rubber, plastic and leather
processing. Identifying a specific allergen is often difficult and may require patch testing.
For example, nickel allergy often presents near sites of frequently worn jewelry or
clothing fasteners, latex allergy often appears near clothing waistbands & leather allergy
typically appears on the feet.

Managemnt requirz careful avoidance of allergens. Topical high potency glucocorticoids


(e.g betamethasone, fluocinonide) are useful for treatmnt of flares in intermittent short
courses (<4 weeks). Topical tacrolimus is an alternative for areas where glucocorticoids
are contraindicated (e.g the face).

CHRONIC ALLERGIC CONTACT DERMATITIS: At the beltline.

29.LICHEN SIMPLEX CHRONICUS:


Lichen simplex chronicus (neurodermatitis) is characterized by thickened excoriated
plaques caused by persistent scratching and rubbing. It is associated with anxiety
disorders and typically occurs in areas that are easy to reach (e.g arms, legs, neck).

LICHEN SIMPLEX CHRONICUS:

30.CHERRY ANGIOMAS:
Cherry angioma is common benign vascular tumor seen in adults. Cherry angioma is
characterized by sharply circumscribed areas of congested capillaries and post-capillary
venules in the papillary dermis. Pts usually develop multiple small, vascular bright red
and dome shaped papules. CAs are most commonly found on the trunk and occur only
in skin and not in deep tissues or mucosa. The lesions usually blanch with pressure, but
some lesions can become fibrotic over time and will not blanch.
CAs usually do not regress spontaneously and are sometimes called senile hemangiomas

as they increase in number as the pt ages. They may occasionally bleed if disturbed but
are otherwise asymptomatic and do not need treatment other than for cosmetic reasnz.
Electrocauterization under local anesthesia can treat small lesions, while shave excision
with electrocauterization is required for larger lesions.
Bottom line: Cherry angiomas usually present as multiple small, vascular, bright red and
dome-shaped popular lesions in adults. CAs are benign lesions that increase
in number with age and do not regress spontaneously. Most lesions do not
require treatment.

CHERRY ANGIOMA/SENILE HEMANGIOMA:

31.KAPOSI SARCOMA:
Kaposi sarcoma is most oftens seen in pts who have coinfection with HIV and HHV-8.
It is considered as an AIDS-defining condition. KS usually appears as deep red, purple,
or brown macules on the trunk, extremities and face. The lesions can also present as
plaques, or nodules that are ovoid and linear in shape along skin tension lines.

KAPOSI SARCOMA:

KAPOSI SARCOMA:

KAPOSI SARCOMA:

31.SPIDER ANGIOMAS:
Usually present as bright red central papules surrounded by several outwardly radiating
vessels. They occur due to dilated central arterioles and associated superficial capillary
networks. Spider angiomas are estrogen dependent and are commonly found in pts with
cirrhosis.
SPIDER ANGIOMAS:

32.LICHEN PLANUS:
Lichen planus (LP) is an immunologically mediated skin disorder, affecting mostly
middle aged adults. It is thought to be caused by activated T-cells directed against
the basal keratinocytes of the epidermis.
It typically involves the skin, nails, mucous membranes of the mouth, and external
genitalia (vulva and penis). The classic skin lesions are shiny, discrete, intensely pruritic,
polygonal-shaped violaceous plaques and papules that are most frequent on the flexural
surfaces of the extremities. Wrists are a common site of skin involvement. Characteristic
whitish, lacy pattern, referred to as Wickham striae, is seen often on the lesion surfaces.
Mucosal lesions may appear as papular, atrophic, erosive lesions or may present solely
as lacelike, reticular Wickham striae, especially on the tongue and buccal mucoasa.
Genital LP presents usually with intensely pruritic violaceous papules on the glans penis
or vulva.
The diagnosis is mainly clinical, based on classic skin lesions in a characteristic distributn.
If it becomes necessary to confirm the diagnosis, a punch biopsy of the most prominent
lesions should be performed and sent for histopathologic examination.
Lichen planus often is associated with advanced liver disease, due to Hepatitis-C virus.
Screening for Hepatitis-C should be considered in pts with LP, especially those with
additional risk factors (e.g history of IV drug abuse).
Bottom line: Lichen planus is characterized by discrete, intensely pruritic, polygonal
shaped violaceous papules or plaques involving the flexural surfaces of
the extremities (most commonly wrists), buccal mucosa or external
genitalia. A skin biopsy may be required to confirm the diagnosis.
Lichen planus often is associated with advanced liver disease, due to
Hepatitis-C virus.

LICHEN PLANUS:

LICHEN PLANUS:

LICHEN PLANUS:

CUTANEOUS SARCOIDOSIS:

33.ACTINIC KERATOSIS:
Occurs in areas of chronic sun exposure such as face, scalp, ears, upper chest and dorsal
hands and forearms. AK lesions are characterized by small, rough, erythematous and
keratotic papules that are often easier to feel than they are to see. The skin upon which
they occur oftn shows signs of chronic photodamage such as dyspigmentation, wrinkling
, thinning and telangiectasia.
Untreated AK has up to a 20% risk of progression to squamous cell carcinoma. For this
reason, any AK lesions that are detected should be removed or destroyed. There are

numerous acceptable therapies for AK. Individual lesions can be destroyed with liquid
nitrogen cryosurgery or by surgical excision or curettage. However, treatment of
individual lesions is impractical when numerous small lesions are present. Field therapy
(an entire region of the body z treated at once) z employed instead. The most commonly
used field treatment is 5-fluorouracil cream; it is applied to the entire affected area for
a period of 3-6 weeks. Other options include imiquimod cream, topical diclofenac and
photodynamic therapy.
Biopsy is indicated for actinic keratosis when;

The diagnosis is unclear


Lesions >1 cm in diameter
Lesions are indurated
Ulceration is present
There is tenderness
Lesions are growing rapidly
Lesions fail to respond to appropriate therapy

Biopsy proven SCCs should be treated with either Mohs micrographic surgery or by an
excision with 4 mm margins.
*Cryodestruction is the most commonly employed method for treating solitary or few
AKs.

ACTINIC KERATOSIS:

ACTINIC KERATOSIS:

ACTINIC KERATOSIS:

ACTINIC KERATOSIS:

BUTTERFLY RASH OF SLE:

34.ACNE ROSACEA:
Chronic inflammatory disorder that presents with facial erythema and edema, and easy
flushing, often with telangiectasias, papules, pustules, and localized tissue hyperplasia.
The inflammatory lesions may appear similar to those found in acne, although
comedones are not present. It is most common in adults age 30-60 and occurs most
often in individuals with light skin, hair and eye color. Periods of exacerbations and
remission are expected.

Pts with only erythema and telangiectasias may be managed with topical brimonidine
and avoidance of factors that trigger flushing (e.g hot or spicy foods, alcohol, extremes
of temperature, emotional distress).
Pts with papular or pustular lesions are treated with topical metronidazole or azelaic
acid.
Oral antibiotics (e.g metronidazole, tetracyclines) are required for more severe or
refractory cases.
Therapy is usually successful in reducing the number & severity of the inflammatory
lesions but facial erythema may persist despite treatment.
Pts with rosacea often experience ocular symptoms. Manifestations of ocular rosacea
may include burning or foreign body sensations, blepharitis, keratitis, conjunctivitis,
episcleritis, corneal ulcers & recurrent ulcerations. The chalazion z characterizd by a
granulomatous inflammation of a Meibomian gland and presents as a painless pea-sized
nodule within the eyelid. Pts with ocular rosacea shd have ophthalmology consultation
to address the risk of more serious complications

PAPULOPUSTULAR ROSACEA:

ROSACEA:

ACNE ROSACEA:

CHALAZION:

35.PHOTOAGING:
Photoaging arises from the combination of intrinsic aging and damage caused by the UV
light. While intrinsic aging tends to result in fine wrinkles on an otherwise smooth skin
surface, photdamage tends to result in coarse, deep wrinkles on a rough skin surface.
Photoaged skin is often marked with actinic keratosis, telangiectasias & brown (liver)
spots.
Cigarette smokers have been shown to have more and deeper wrinkles than do nonsmokers due to harmful changes in blood flow or that circulating toxins from cigarette
smoke may alter the production of collagen or elastin.
Tretinoin is an emollient cream approved by the FDA for reduction of fine wrinkles,
mottled hyperpigmentation, and roughness of the facial skin. Tretinoin is also successful
in reducing the actinic keratosis and in improving the appearance of brown spots,
regardless of the etiology.

36.ERYTHRASMA:
Is an infection of the skin that occurs most often in intertriginous spaces and is caused
by Corynebacterium minutissimum (gram +ve bacillus). The appearance of erythrasma
includz a confluence of pruritic, reddish brown, finely wrinkled papules. The use of a
Wood lamp reveals acoral-red fluorescence caused by the Corynebacterium porphyrins.

ERYTHRASMA:

37.Mycosis fungoides; (cutaneous T-cell Lymphoma)


Has a highly variable appearance and may present as papules or plaques hyper
or hypopigmented patches, nonspecific erythema, or subcutaneous tumors.
Extradermal spread of the malignancy may cause regional lymphadenopathy,
infiltration of the lung, liver, or spleen and occasionally, there can be bone
marrow and CNS involvement.
Interferon alpha can be used

MYCOSIS FUNGOIDES:

38.POISON IVY CONTACT DERMATITIS:


Poison ivy contact dermatitis presents as a pruritic linear lesions, which are usually
papular. When the reaction is severe, the papules may evolve into vesicles, which can
exude a serous fluid. The lesions have a linear or curvilinear appearance, showing the
points of contact with the leaves or stems of the plant. New lesions can appear two to
three days later if the poison ivy resin was transported to other skin surfaces by the
patient before washing the primary lesion.

39.SEBORRHEIC KERATOSIS:
Seborrheic keratosis is a benign epidermal tumor due to proliferation of immature
keratinocytes, that are uncommon before age 30. SKs typically present after age 50 and
are more common in fair-skinned individuals. The pathogenesis is not well undersood.
Pts can develop single or multiple lesions that r swell-demarcated, pigmented, round or
oval, and have a dull or verrucous surface with stuck-on appearance. Typical locations
include the face, upper extremities and trunk.
SKs can be asymptomatic or cause pruritus, pain or bleeding (especially with friction
against clothing or jewelry). Explosive onset of multiple pruritic SKs (Leser-Trelat sign)
has been associated with malignancies (especially lung and GI tumors). Diagnosis is
actually based on clinical appearance of lesions but may require biopsy in uncertain
cases.
Treatment is not necessary unless the lesions are symptomatic or cause cosmetic
problems.
SEBORRHEIC KERATOSIS:

SEBORRHEIC KERATOSIS:

LESER-TRELAT SIGN:

CUTANEOUS WARTS: Salicylic acid is used for treating cutaneous warts.

40.DIAPER RASH:
Diaper dermatitis is a type of irritant contact dermatitis that is caused by a combination
of factors: overhydration, friction, maceration, and prolonged contact with excretions.
The appearance of rash after an episode of diarrhea is very characteristic. Management
includes keeping the diaper area of the skin as dry as possible. This involves frequent
changing of the diapers, avoid tight fitting of the diapers, exposing the skin to air, using
diapers with super absorbent surfaces and applying barrier creams such as zinc oxide
or petrolatum. An uncomplicated diaper rash resolves quickly if the abovementioned
measures are performed. Otherwise, a low-potency corticosteroid ointment may be
prescribed.

DIAPER DERMATITIS/RASH:

40.MALIGNANCY CRITERIA IN A PIGMENTED LESION:


The ABCDE criteria (asymmetric shape, border irregularities, color variegation, dia 6mm
or more, & evolving size, shape or color) r useful to assess the likelihood of malignancy
in a pigmented skin lesion.

If the ABCDE criteria is indeterminate or the clinician still has a persistent suspicion, then
ugly duckling sign may be used in which the likelihood of a malignancy of a pigmented
lesion may be judged with respect to its similarity or difference from other lesions on
the same patient.
Pts with skin lesions suspicious for melanoma should have excisional biopsy to include
the entire lesion with 1-3 mm margins of the surrounding skin and subcutaneous fat.

*Periodic surveillance (with or without photographic documentation) is appropriate


for pts at elevated risk for melanoma (e.g personal or family history of melanoma,
multiple atypical nevi).
*Sentinel lymph node biopsy or lymphatic mapping is indicated for pts with confirmed
melanoma who do not have clinical evidence of lymph node involvement but are at
increased risk (e.g tumor thickness 0.75 mm or more, tumor ulceration, evidence of
lymphatic invasion on biopsy).

NODULAR MALIGNANT MELANOMA: Occurs in fair skinned whites. Large numbers of


nevi and atypical nevi also correlate with the risk.

CAFE-AU-LAIT MACULES:

ACANTHOSIS NIGRICANS:

ACANTHOSIS NIGRICANS:

41.INVASIVE MUCORMYCOSIS:
Cerebral venous thrombosis can occur as a complication.
INVASIVE MUCORMYCOSIS:

42.TINEA CAPITIS:
Presents as scaly, erythematous patches of alopecia and cervical adenopathy. TC, often
referred to as ringworm of the scalp, is caused by a number of dermatophytes
including Trichophyton tonsurans, Microsporum canis, and Microsporum audouinii.
TC is most commonly diagnosed based on clinical features, though KOH examination of
epilated hair stubs or fungal culture are often helpful. TC lesions due to Microsporum

species may fluoresce green or blue when examined under Woods ultraviolet lamp.
However, TC due to T.tonsurans, which is most common in the United States, does not
fluoresce and Woods lamp examination is not usually performed.
First line treatmnt fr TC is usually oral griseofulvin, which is highly effective, inexpensive
and safe to use in children. Oral terbinafine is an acceptable alternative for TC due to
T tonsurans, but it is not recommended as empiric therapy in areas where Microsporum
species are common. Other effective treatments include oral itraconazole and oral
fluconazole. Topical antifungals are ineffective for eradication of the infection in pts
with involvement of the hair follicles, though some experts recommend them to reduce
transmission to household contacts during treatment.
*Alopecia areata is characterized by smooth areas of hair loss without any scaling

TINEA CAPITIS:

43.Head lice (Pediculus humanus capitis) infestation is common in children & may cause

itching of the scalp, neck and ears. Topical permethrin is indicated for the treatment.

44.DISCOID LUPUS ERYTHEMATOSUS:


Is characterized by inflammation and scarring of the hair follicles accompanied by
hypopigmented skin lesions and photosensitivity. Benefits from the use of topical
glucocorticoids.

DISCOID LUPUS ERYTHEMATOSUS:

45.PLANTAR WARTS:
Are a form of cutaneous warts usually due to HPV virus. Plantar warts most oftn appear
in young adults or in individuals with certain occupations (e.g meat, poultry, and fish
handlers). More extensive disease occurs in pts with atopic dermatitis and conditions
with decreased cellular immunity (e.g HIV, organ transplantation). Infection typically

occurs via skin-to-skin contact and can have an incubation period of 2-6 months after
exposure.
The diagnosis of plantar warts is usually made clinically with visualization of single
or multiple hyperkeratotic papules on the sole of the foot. Scrapings of hyperkeratotic
debris can confirm the diagnosis by showing thrombosed capillaries (also called seeds).
A shave or punch biopsy is rarely needed to confirm the diagnosis in uncertain cases.
Salicylic acid is preferred as initial treatment for plantar warts. Pts are instructed to
prepare the affected area by soaking it in warm water for 10-20 minutes. The salicylic
acid is then applied to the wart, taped in place with duct or athletic tape and then
kept dry for 48-72 hrs. The patient should then remove the patch, pare the area down
with a file or sandpaper, and then repeat the application. Liquid nitrogen cryotherapy
is a reasonable alternative but it can cause local burning and is usually avoided in
dark-skinned individuals due to possible hypopigmentation.
The initial response to salicylic acid treatment for plantar warts may take 2-3 weeks
to become apparent. Pts are usually assessed at 2-3 weeks intervals and treatment
is continued for 1-2 weeks after clinical resolution to eradicate the virus and prevent
recurrence.
*Topical imiquimod is used more commonly for anogenital warts.

PLANTAR WART:

46.FILIFORM CUTANEOUS WARTS:


Snip or shave excision is usually preferred for filiform cutaneous warts, which usually
appear as a solitary papule with finger like projections.

FILIFORM CUTANEOUS WARTS:

47.TINEA PEDIS:
Presents as chronic and progressive pruritic lesions or scales between the toes. Topical
miconazole cream is a treatment option.

TINEA PEDIS:

48.KELOIDS:
Keloids are benign fibrous growths that develop in scar tissue secondary to an
overproduction of extracellular matrix and dermal fibroblasts. Highly disfiguring and
painful. Clinical diagnosis and confirmatory biopsy is discouraged.
Intralesional glucocorticoids are the preferred means of treating most keloids.
Surgical excision is generally needed if glucocorticoids fail.
The earlier the lesion is treated, the greater the chance of improvement.
Treatment failure is common and later recurrence can occur even after surgical excision.
*if u do cryosurgery-> Hypopigmentation limits this modality in dark skinned pts.
*Silicone gel tx-> Local dermatitis is common, so limits the advantage of this modality
*Radiation-> Malignancy can occur, so limits it.

KELOIDS:

49.Thiazides are sulfonamides; therefore, hydrochlorothiazide use can cause


photosensitivity rash, and treatment of this rash includes discontinuation of the
thiazide, use of sunscreens and avoiding sun exposure.

50.Furosemide is not usually recommended for the management of HTN alone.


Furosemide can lead to steven jhonson type of reaction that is seen with use of
sulfonamide antibiotics.

51.DESMOID TUMOR:
Is a locally aggressive benign tumor arising from fibroblastic elements within the
muscle or fascial planes with very low potential for metastasis or differentiation.
Desmoid tumors are rare but increased in pts with familial adenomatous polyposis
(i.e Gardner syndrome) and thought to be due to abnormal wound healing or clonal

chromosomal abnormalities causing a neoplastic behavior. This tumor has a variable


clinical course, can range from slow to rapid growth and has varying sizes. Desmoid
tumors typically present as deeply seated painless or sometimes painful masses in the
trunk/extremity, intraabdominal bowel and mesentery and abdominal wall. They can
cause intestinal obstruction and bowel ischemia and have a high rate of recurrence,
even after aggressive surgery.
CT scan or MRI shd be done to evaluate the size of the mass, with biopsy for histologic
diagnosis. Surgery is the definitive therapy for desmoid tumors that are symptomatic,
cause risk to adjacent structures or cosmetic issues for the patient or are recurrent.
Pts who are not surgical candidates can be treated with radiation therapy. Close
observation is an alternative strategy in pts who are asymptomatic, have stable masses
or have intraabdominal masses seen in Gardner syndrome.
Bottom line: Desmoid tumors are slow growing and locally aggressive benign neoplasms
with a high rate of local recurrence, even after surgical excision.

52.Dermatofibroma is a benign proliferation of fibroblasts that usually occurs after


trauma or insect bite and can also be idiopathic. It is usually a firm hyper pigmented
nodule located on the lower extremities.

53.An epidermoid cyst is a discrete nodule located on the skin of extremities and occurs
as a result of normal epidermal keratin becoming lodged in the dermis. It resolves
spontaneously without treatment.

54.A pyogenic granuloma (granuloma telangiectaticum) is caused by capillary proliferatn


after trauma and usually presents as a dome-shaped papule with recurrent bleeding.
It is more commonly seen in pregnant women.

55.HERPES ZOSTER:
The diagnosis of Herpes zoster or varicella is made clinically, by the presence of
classic vesicles. No further lab tests are needed.
In immunosuppressed pts (e.g HIV), the rash can be atypical and can be confused with
herpes simplex. if the pt is critical and antiviral therapy needs to be started, lesions
can be scraped for PCR or immunofluorescence studies.
Any kind of stress on the body can precipitate reactivation. The hallmark feature is
grouped vesicles or bulla in a specific unilateral dermatomal pattern. Pain is another
very prominent feature and precede the onset of rash by a few days to weeks. Most
lesions crust over 7-10 days.
The treatment goal is to promote faster healing of the lesions and prevent long term
complications, especially post-herpetic neuralgia. Antiviral therapy with acyclovir or
valacyclovir for 7-10 days is the mainstay of therapy. If this is started within 72 hours of
the onset of rash, it can effectively hasten the resolution of the lesions and associated
pain.
Steroids ( neither topical nor oral ) have no proven benefit.
The incidence of Herpes zoster increases with advancing age, co-morbid conditions,
immunosuppression and significant physical stressors such as illness, surgery, trauma,
or chemotherapy.
Although it can recur, but not that likely. I guess only 5% chances. so unlikely to recur
repeatedly unless some stressor is present as enumerated above.
Immunization with the varicella zoster vaccine is recommended for adults age 60 or
more. The vaccine would reduce the risk of recurrence as well as the risk of postherpetic neuralgia.
If the pt is immunocompetent and the zoster is localized, then the transmission would
occur only with direct contact with open lesions and if the pts completely cover their

lesions with clothes or a dressing until the lesions have crusted, then the transmission is
minimal.
If a pt is immunocompromised & has a localized zoster OR if the zoster is disseminated,
then the risk of transmission is very high. Such pts need to be hospitalized and placed
in strict isolation until all of their lesions have crusted.
In contrast to zoster, primary varicella infection (chickenpox) is extremely contagious
and transmitted via contact and airborne droplets. Wait for the crusting of lesions, at
which time the pt is not contagious anymore. Complete crusting is hastened by antiviral
therapy but the pt who is still on therapy and crusting has not occurred, he/she is still
contagious.
TCAs, topical capsaicin cream, gabapentin & long-acting oxycodone are effective
treatment for Post-herpetic neuralgia, individually or in combination.
*Capsaicin cream increases the release of substance-P and then preventing its
reaccumulation in nerve fibers.

HERPES ZOSTER:

HERPES ZOSTER:

HEPRES ZOSTER:

SHINGLES:

56.BASAL CELL CARCINOMA:


A common skin malignancy with low metastatic potential. The typical appearance is
an enlarging fleshy nodule with ulceration and a pearly quality. BCC is associated with
sun exposure.
BCC:

BCC:

57.SKIN TAGS: (ACROCHORDONS)


Appear in the areas of friction (axillae, neck, inframammary, inguinal regions). There
is an association between skin tags and obesity, insulin resistance, over diabetes and
metabolic syndrome. Skin tags also occur commonly during pregnancy and in pts with
colonic polyps.

Diagnosis is based on clinical appearance. Skin tags do not usually require treatment,
although it may be considered if there is associated irritation or for cosmetic reasons.
Treatment options include snip excision or cryosurgery.

SKIN TAGS:

PERIANAL SKIN TAGS: Occur in Crohns disease.

PYODERMA GANGRENOSUM: Occurs in Crohns disease.

LEUKOCYTOCLASTIC VASCULITIS:

58.Hepatitis-C z asociatd wth lichen planus, porphyria cutanea tarda & cryoglobulinemia
causing leukocytoclastic vasculitis.

MOLLUSCUM CONTAGIOSUM:
Is a self limited, localized skin infection caused by a pox virus. It is characterized by small
skin colored papules with indented centers that may occur anywhere except the palms
and soles. The lesions may be accompanied by pruritus and surrounding dermatitis.
Children are most commonly affected, but adolescents, adults & immunocompromised
individuals can also develop MC. Transmission is through skin-to-skin contact or contact

with contaminated fomites. MC is often transmitted through sexual contact and may be
seen in association with other sexually transmitted diseases. It is also frequently seen
in pts with HIV and may be more widely transmitted and persistent in these pts. HIV
testing should be considered in pts with MC, especially for lesions that are widespread
or involve the face.
MC diagnosis is clinical and based on characteristic findings. The condition is typically
self limited but treatment may be considered to prevent further spread, reduce
symptoms or improve cosmesis. There are a number of techniques to remove lesions
including physical destruction vie curettage/cryotherapy or chemical removal with
topical agents (e.g podophyllotoxin).

59.SUN PROTECTION:
Excessive sun exposure is a risk factor for basal and squamous cell carcinoma and
malignant melanoma.
Pts should be counselled to apply sunscreen liberally 15-30 minutes prior to sun

exposure; sunscreen should be reapplied every 2 hours or after water exposure.


Sun protection factor (SPF) 15 is recommended for regular daily use and is often
concerned in facial lotions and cosmetics.
SPF >30 is recommended for outdoor work or recreation, as those participating in
outdoor acitvities tend to have less consistent sunscreen application.
Pts should be advised to avoid sun exposure between 10 PM to 4 AM whenever
possible. In addition, tightly woven, dark colored fabrics confer the greatest protection
from UV rays. However, these measures are often impractical for individuals who
participate in outdoor sports.
Pts should be discouraged from tanning bed and suntan lotion/oil use as these products
increase the exposure to UV light.

60.CHONDRODERMATITIS NODULARIS HELICIS:


Avoidance of pressure on affected ear is good.

61.ONYCHOMYCOSIS:
Onychomycosis z a fungal infectn of the toenails or fingernails. The most common form
of onychomycosis z a distal subungual onychomycosis, which z caused by dermatophyte
Trichophyton rubrum. Because nail dystrophies caused by other diseases (e.g psoriasis,
lichen planus, eczematous conditions) can mimic the appearance of onychomycosis,
confirmation with KOH preparation or periodic acid-Schiff staining of nail scrappings is
recommended. If KOH examination z negative, then evaluation by culture is appropriate.
Pts often request treatment of onychomycosis for cosmetic reason; such treatment
may be considered in carefully selected pts. However, pts should be advised about the
risks of treatment (primarily hepatotoxicity) and the potential of treatment failure and
recurrence. Medical indications for treatment of onychomycosis include significant pain
or functional limitation, history of cellulitis in the affected extremity or additional risk
factors for cellulitis (e.g diabetes). Oral terbinafine is considered first line therapy, and
oral itraconazole is an acceptable alternative. Infection of the fingernails requires 6 wks
of therapy and infection of the toenails requires 12 wks.

62.Allergic contact dermatitis is a common adverse effect of topical antibiotic use.


Topical antibiotics also increase the risk for antibiotic resistant organisms. The
manifestations of allergic contact dermatitis would be vesicles and erythema.
Topical antibiotics are useful for full thickness burns and partial thickness burns
with loss of epithelium. However, superficial burins with intact epithelium & most
minor cuts and scrapes do not require topical antibiotics as these would only increase

ACD risk and contribute to antibiotic resistance.

63.CELLULITIS:
Staph. aurerus is the cause
Oral cephalexin is the treatment

64.

65.MELASMA:
An acquired hyperpigmentation disorder that occurs on sun exposed areas of face.
Melasma is more common in women, especially during pregnancy. The risk increases
with subsequent pregnancies. Other risk factors include darker skin color, family hx,

thyroid dysfunction, medications (e.g antiepileptics, OCPs) and use of cosmetics.


Although the etiology is not completely understood, it is likely due to UV radiation
triggering melanocyte proliferation in sun exposed areas.
Pts typically develop irregularly shaped, hyperpigmented macules on the face with
varying color (light brown to dark brown or ash/blue). The rash occurs symmetrically.
Melasma during pregnancy typically resolves spontaneously within months after
delivery. Initial conservative measures involve avoiding sun exposure, wearing a wide
brimmed hat and using sunscreen that blocks both UVA and UVB sun radiation.

66.CUTANEOUS SQUAMOUS CELL CARCINOMA:


Well demarcated scaly patch, and plaque.

67.PRESSURE ULCERS:
Stage 1 ulcers are characterized by nonblanchable erythema of the intact skin
Stage 2 ulcers are superficial ulcers causing a partial thickness loss of the
epidermis, dermis or both.
Stage 3 ulcers are deeper ulcers causing a full thickness loss with damage to
subcutaneous tissue that may extend to, but not through any underlying fascia.
Stage 4 ulcers are very deep ulcers causing a full thickness loss with extensive
tissue destruction that may damage adjacent muscle, bone or supporting
structures.
Stage 3 or 4 pressure ulcers should be loosely packed with saline-moistened gauze to
preserve the moist wound environment.

68.Patch testing helps to differentiate between allergic and irritant contact dermatitis.

69.SQUAMOUS CELL CANCER:


Is the 2nd most common type of cancer in US.
The choice of treatment is surgical excision.
Alternative acceptable treatment options include cryotherapy, electrosurgery and
radiation therapy.

70.Vemurenafib is a BRAF kinase inhibitor that has been demonstrated to produce


significant tumor regression and improve survival in pts with metastatic melanoma
and a specific mutation that activates the MAP kinase pathway.

71.BULLOUS PEMPHIGOID:
Is an autoimmune disorder characterized by prodromal symptoms of itching & uritcaria
followed by tense bullae with erythematous urticarial lesions. It is more common in the
elderly populations and has been associated with a number of neurologic conditions
including dementia, bipolar disorder and multiple sclerosis.
Diagnosis is best established by skin biopsy of the margin of a bullous lesion & serum
assay for basement membrane antibodies.
Mild cases are managed wth high potency topical glucocorticoids but more severe cases
require treatment with systemic glucocorticoids and steroid-sparing agents (e.g
azathioprine, methotrexate) or combination of antibiotics (e.g tetracycline) with
nicotinamide.

BULLOUS PEMPHIGOID:

72.TINEA PEDIS:
Tinea pedis is characterized slowly progressive pruritus of the feet, erythema & scaling
between the toes, and extension to the soles and sides of the feet with a sharp border
between the involved and uninvolved skin. TP is the most common dermatophyte
infection and may be accompanied by infection of the hands (tinea manuum) or groin
(tinea cruris). It may also extend into the toenails as well (onychomycosis). Acute TP,
presenting with rapid onset vesicular or bullous lesions and severe pruritus, is usually
self-limiting. However, chronic TP is usually persistent or relapsing. The organisms
causing TP are widespread and it is most commonly seen following exposure to public

places while barefoot (e.g athletic activities, public swimming pools).


Mild TP usually responds to topical antifunglz (e.g terbinafine, miconazole, clotrimazole)
in 1-4 wks. More extensive disease or failure of topical agents may require oral therapy
(e.g terbinafine, itraconazole, fluconazole). Pts with onychomycosis also require oral
antifungals with a prolonged 12 week treatment course.

TINEA PEDIS:

TINEA CRURIS:
Caused by Trichophyton rubrum. The patch typically spreads with partial central
clearing and can be slightly elevated.

TINEA CRURIS:

BULLOUS TENIA: (TINEA PEDIS)

73.INTERTRIGO:
An inflammatory skin disorder that affects the intertriginous areas (inguinal, perineal,
gential, integluteal, axillary, or inframammary). Risk factors include conditions that
increase skin friction (e.g obesity), increase moisture (e.g tight clothing, hyperhidrosis),
or promote fungal overgrowth (e.g diabetes, immunosuppression). Candidal intertrigo
(primarily Candida albicans) is the most common etiology as the fungus grows well
under the warm, moist environment of the skinfolds.
Pts typically develop erythematous plaques and erosions, sometimes with satellite
papules (black arrows in figure below) and significant inflammation (white arrows in the
figure below). The lesions can be pruritic or painful if there is significant skin breakdown.
The diagnosis is made clinically but can be confirmed with KOH examination.
Topical antifungals (e.g miconazole, nystatin, terbinafine) are the preferred first line
treatment for intertrigo.
After antifungal treatment, use of skin drying agents and correction of any underlying
predisposing conditions can reduce the risk of recurrence

INTERTRIGO:

INTERTRIGO:

74.HIDRADENITIS SUPPURATIVA:
Aka acne inversa is a chronic inflammatory disorder characterized by occlusion of skin
follicles. It is most commonly seen in intertriginous areas (e.g axillary, inguinal, gential,
perianal, perineal) but can occur in any part of the skin that contains hair follicles. Risk
factors include family hx, smoking, obesity, and mechanical stress on the skin (e.g
friction, patient manipulation). Pts typically develop an initial solitary and painful
inflamed nodule in an intertriginous area that can last for several days to months before
progressing to an abscess with purulent or serosanguinous drainage. The nodules & pain
typically improve after drainage. Multiple recurrent nodules can lead to sinus tracts,
comedones and significant scarring.
General measures for all pts include weight loss, smoking cessation and daily skin
cleansing of the affected area.

Mild disease (Hurley Stage I) typically responds to topical clindamycin, but some
pts may require intralesional steroids or oral antibiotics during flare ups.
Moderate disease (Hurley Stage II) presents with inflammatory nodules, sinus
tracts, and scarring. These pts require oral antibiotics (tetracyclines preferred) for
several weeks to reduce the inflammation and improve the infection. OR oral
clindamycin + rifampin in refractory cases.
Severe and refractory cases (Hurley Stage III) usually requires medical therapy
with biologic TNF alpha inhibitors (e.g infliximab) and/or oral retinoids (e.g
acitretin) and occasionally surgical excision.

75.CANDIDAL DIAPER RASH:


Tomato red plaques and satellite papules are characteristic of candida intertrigo and
perineal infection. The infection is common in infants who have recently received

antibiotic therapy because the decrease in the normal bacterial flora favors yeast
proliferation. The condition can be confounded with diaper dermatitis, which affects
the same area but usually spares the crural folds.
Treatment of candidal diaper rash involves the application of topical antimycotic cream
such as clotrimazole cream or nystatin.
*Zinc oxide cream and petrolatum ointment are beneficial for diaper dermatitis.

76.Acne;
retinoids (salicylic acid/glycolic acid/azelaic acid)-> Benzoyl peroxide -> topical
antibiotics (erythromycin/clindamycin)-> oral antibiotics (erythromycin/clindamycin)->
oral isotretinoin.

77.Initial treatment for inflammatory acne includes topical retinoids and benzoyl
peroxide. Moderate or moderate-to-severe acne cases will benefit from the addition
of topical antibiotics (e.g erythromycin/clindamycin). Oral antibioitcs (erythromycin
/clindamycin) are usually reserved for severe cases. Oral antibiotics are also reserved
for widespread acne (e.g on the back or upper arms) that makes topical therapy
impractical.

78.ATOPIC DERMATITIS/ECZEMA IN CHILDREN:


Is very common in children. Infants with eczema typically have pruritus, erythema, and
scaly/crusted lesions on the trunk, extensor surfaces of extremities, cheeks and scalp;
diaper areas r usually spared. The pathogenesis involves genetic defects in protein and
lipids that normally act to keep the epidermis hydrated and protected. However,
disruption of this barrier reslts in an inflammatory reaction to triggers such as excessive
bathing, irritating detergents, and extreme temperatures.

Management requires compulsive elimination of triggers and frequent emollients to


provide a barrier and restore hydration. If emollients are inadequate, topical steroid
creams/ointments may be applied on the body. Low potency steroids are preferred
on the face and skin folds due to risk of skin atrophy.
Eczema is a risk factor for impetigo (honey crusted lesions) and cellulitis (focal erythema
, warmth, swelling).
Eczema is a clinical diagnosis based on Hx and physical exam.
Atopic dermatitis clears in nearly 40% of individuals by adulthood.

ATOPIC DERMATITIS/ECZEMA IN CHILD:

GOTTRONS PAPULES:

SPOROTRICHOSIS:

79.Cutaneous signs of insulin resistance include skin tags, acanthosis nigricans, and
xanthelasma.

XANTHELASMA:

80.Contagious ecthyma (orf) is caused by a poxvirus of sheep and goats and begins as
a single erythematous papule on the hand. Milers nodules are maculopapular/
vesicular lesions caused by a parapoxvirus of cows. Both of these conditions are self
limited and very uncommon.

Você também pode gostar